SOLUTIONS MANUAL for Financial Accounting in an Economic Context 8e Jamie Pratt

Page 1


CHAPTER 1 FINANCIAL ACCOUNTING IN AN ECONOMIC CONTEXT ISSUES FOR DISCUSSION ID1–1 Security analysts and stockholders: These users would use financial statements to try to estimate the future earnings and cash flow potential of the company, which would be used to project a value for the company’s stock. Bank loan officers: These users would use the financial statements to determine the ability of a company to repay loans to the bank. A company’s customers and suppliers: These users would use financial statements to determine whether to extend credit to the company (suppliers) or whether to rely upon the company to be a supplier (customers). Both suppliers and customers would also use the financial statements to monitor the company’s profit margins. Public utilities: This group would use the financial statements to determine the company’s growth rate and how that might impact upon the company’s utility needs. Also, they would evaluate the company’s ability to pay its bills. Labor unions: These groups would use the financial statements to monitor the profitability of the company to help determine the amount of pay raises and benefits that it will negotiate for from the company. A company’s managers: The company’s managers will use the financial statements to assess the overall financial health of the company. This could impact the managers in a number of ways: raises, promotion opportunities, performance of other departments, etc.

ID1–2 The board of directors serves various functions for a company. One is to represent and protect the interests of the stockholders who are not on the board. Another is to provide oversight and input to management. The managers are involved in running the business on a day to day basis whereas the board is more focused on the bigger, long term picture. A weak board may not ask probing questions of management but instead may take everything at face value and believe anything that management says to them. A healthy management team would want a strong board that delivers valuable input. A management team that wants a weak board of directors may be trying to hide something (management fraud). Auditors are concerned with management fraud because, if there is a problem, in many cases the auditors will be sued by the stockholders on the basis that the auditors should have detected the fraud. 1


ID1–3 The function of the audit committee is to provide a channel whereby the auditors report their findings and concerns, if any, to the board of directors. Typically there are outside members of the board that are on the audit committee so that if the auditors have concerns about management’s financial statements or activities, then the auditors have a way to speak directly to the board of directors. The auditors are in a sensitive position because the financial statements and activities that they are auditing are prepared by the same people who hire and pay the auditors. Therefore, they may be reluctant to jeopardize their relationship with the company by being too negative.

ID1–4 Banks make loans to customers and depend on those customers to repay the loans (called the “principal”) plus interest for the banks to earn a profit. If customers are not able to pay the interest, the banks cannot make a profit; further, if the customers are not able to repay the principal the banks will show a loss that reduces the equity on the balance sheet. Banks look at a number of factors, both “macro” and “micro” in nature. Banks will look at the overall strength of the economy and the likelihood for future growth; these are the macro issues a bank considers. Banks will also examine the specifics of a company’s individual performance within the economy; these micro issues often are seen in the financial statements of companies. Issues such as the amount of debt, the level of profits, the amount of cash on hand and the amount of cash generated by the business, and the quality and size of the assets can all be seen from the financial reporting system. Banks require borrowers to submit financial statements to show these performance measures. During the 2008-2009 recession and related credit crunch, banks were concerned about the macro issues shown in general economic data, as well as the micro issues shown in companies’ individual financial reports. The reluctance of banks to lend has been cited as one of the reasons for the length of the economic downturn.

ID1–5 Sales for Home Depot decreased because of the economic conditions during the years shown. One of the hardest hit sectors of the economy was housing, which means that builders purchased less materials from Home Depot to construct houses; existing homeowners were hurt during the recession and had less money to purchase goods from the company for home improvements. Profits decreased because the drop in sales was not offset with an equal drop in expenses. Home Depot was not able to reduce its fixed and variable expenses as quickly as the company saw its revenue drop. Assets decreased as the company depreciated existing stores without adding new locations—again, due to the weakness in the economy. Equity remained flat because the company kept (“retained”) more of its profits by lowering its dividends paid to shareholders. Finally, the cash balance slightly increased due to less cash outflows for investments in new stores; due to the economy, the company did not invest nearly as much cash in building new stores, keeping that cash in the business for the time being.

ID1–6 Creditors would impose these types of restrictions on Continental Airlines so that the creditors would be protected for their loans. These types of restrictions are fairly common and act as a trip wire to warn the creditors that business may not be going well. The cash restriction would force Continental to have enough cash to pay the interest on the debt, the minimum stockholders’ equity makes sure that there are assets to act as collateral for the loans, and the restriction of dividends insures that management doesn’t distribute cash or assets out to the stockholders and not leave assets in the company to satisfy the creditors.

2


These restrictions act as trip wires in that as soon as a restriction is violated the creditors can call the debt and force the company to pay back the loans. What is more typical is for the loans to be restructured. This usually means higher interest rates and fees to do the restructuring. These all put the creditors in a better position to protect their loans.

ID1–7 Companies would usually engage in this type of behavior to try to improve their stock price. By showing higher revenues or lower expenses investors are more likely to reward the company with a higher stock price. Companies that have negative cash flow are under a lot of pressure to maintain a high stock price since selling stock is the only way to fund the business. This type of incentive can lead to questionable behavior. The ethical implications are significant because if investors lose faith in the financial statements that are reported this would severely impact the stock market. A strong driver to a robust economy is access to capital (stock markets). If this source is reduced because investors don’t believe the numbers that are reported, a very bad impact on the overall economy would result.

ID1–8

This is the normal statement that an auditor would make about a company whose books it had audited and found no significant problems. This would be part of what is called a “non-qualified opinion”. If there was a particular item that the auditors did not agree with they would issue a “qualified opinion” – they would agree with everything except the qualified item that would be identified. “In our opinion”, shows that the statement represents the auditor’s opinion and not a fact; “fairly, in all material respects” means that the auditors can not say that every single number is exactly accurate to the penny but that the numbers are generally accurate. This reflects the concept of materiality; the auditors believe that all material items have been presented accurately. Finally, “in conformity with accounting principles generally accepted in the United States of America” means that the financial statements have been compiled in a way that meets all of the accounting principles that are called GAAP in the U.S but not necessarily in conformance with international standards. ID1–9 Corporate governance describes the relationship among the stakeholders of a company, mainly : the shareholders, the Board of Directors, management and the company’s auditors. Corporate governance mechanisms encourage management and the Board of Directors to act in the best interest of the shareholders and to provide the shareholders with accurate and timely financial information. The Sarbanes-Oxley Act was passed to upgrade the financial transparency of corporate operations, 3


requiring increased financial disclosures and management responsibilities for the intergrity of the financial statements. Improved information provided to shareholders and other providers of capital will strenghten the confidence in the financial system, ultimately benefitting both providers and users of capital.

ID1–10 Management is charged with the responsibility to benefit the shareholders’ investment in the company. Choosing investments that will boost the short-term results of the company in lieu of long-term gains does not meet this requirement. While satisfying the needs of Wall Street analysts for short term results, a management decision to forego larger long term returns violates the relationship between the owners of the company and the management of the company. Many observers feel that short term profit pressures from analysts have caused management to ignore its responsibility to work for the long term benefit of the shareholder. ID1–11 Financial analysts are charged with the task of following companies in specified industries and evaluating the past financial performance of those companies, as well as providing guidance for expectations for future financial performance. Until financial reporting is consistent across global lines, analysts must be in a position to understand, interpret, analyze and forecast financial performance using different financial systems. An analyst following the pharmaceutical industry needs to understand how companies compare against each other, how Novartis stacks up against Johnson & Johnson. Now, not only does an analyst have to understand two sets of financial reporting systems (IFRS and GAAP), but that analyst then has to perform some type of conversion, so the companies can be compared under the same (“apples to apples”) basis. Fluency in GAAP is not sufficient; an analyst must also speak the language of IFRS and be able to translate back and forth between the two systems. ID1–12 Accounting guidelines that are established based on a number of general principles have the advantage of being simplified and easier to understand and document. On the other hand, guidelines that are principle-based are more ripe to be exploited by companies desiring to present their financial statements in good light. Guidelines that are based on a detailed set of rules are, by definition, lengthy and complicated, trying to anticipate every possible business situation. However, some argue that the more detailed rules-based approach allows the users of financial statements to review companies’ financial performance from a consistent perspective.

ID1–13 Management accounting is the accounting system that generates information that is used exclusively by the managers of the company. Financial accounting refers to the financial statements that are prepared and distributed outside of the company. So in many cases management accounting information is the operational information used by the managers of the company. This can be very 4


proprietary to the company and so is not made public. Management accounting numbers are not subject to audit and therefore are prepared in whatever form is helpful to the manager.

Financial accounting information is audited and therefore has to follow GAAP. Its primary purpose is to be used by people outside of the company.

ID1-14 a. Nike is a manufacturing company, primarily engaged in the manufacture and distribution of athletic footwear and apparel. b. The firm of PriceWaterhouseCoopers audits the financial statements of Nike. The audit report states what years and financial statements were audited and therefore being commented upon by the auditor. The second paragraph explains what an audit is intended to do and how the company has gone about doing this audit. The company’s internal control procedures are discussed. Finally, the report states the auditors’ opinion regarding the financial statements that have been audited. The auditors do not evaluate the financial strength of the company; the auditor states that the financial statements “present fairly” the position of Nike; it is up to the user of the financial statements to analyze the company’s performance. c.

Net income in 2007 was $1,491,500,000, for 2008 net income was $1,883,400,000 and for 2009 net income was $1,486,700,000.

d. The amounts shown below are in millions:

Total liabilities Total assets TL/TA (%)

2009 $4,556.5 $13,249.6 34.39%

2008 . $4,617.4 $12,442.7 37.11%

Total liabilities include both Current and Long Term liabilities. From 2008 to 2009 Nike decreased the percentage of its assets that were financed by liabilities. This fact, of course, means that the company increased the percentage of its assets that were financed by equity. e. Cash from operating activities was $1,878,700,000 in 2007, in 2008 it was $1,936,300,000 and in 2009 it was $1,736,100,000. f.

Nike decreased its profitability (in both raw dollars and as a percentage of revenue) and decreased the amount of cash it generated from operating its core businesses. The company, however, reduced liabilities (in both raw dollars and as a percentage of total assets) and increased its shareholder equity. The economy certainly affected Nike, but the company is quite strong and well-positioned for the future.

5


CHAPTER 2 THE FINANCIAL STATEMENTS BRIEF EXERCISES BE2–1 2008 2008 Beginning Retained Earnings

+

2008 Revenues

$28.2

+

$43.3 X

2008 Expenses

$38.2

=

$2.7

2008 Dividends

=

2008 Ending Retained Earnings

X

=

$30.6

2008 Dividends as a percentage of 2008 net income: 2008 Dividends 2008 Net income ($43.3-$38.2)

=

$ 2.7 $ 5.1

= 52.9%

BE2–2 (1)

Current Liabilities financed $32 billion of the assets. Current Liabilities divided by Total assets = $32/$59 = 54.2%

(2)

Long-term debt financed $18 billion of the assets. Long-term debt divided by total assets = $18/$59 = 30.5%

(3)

Stockholders’ equity financed $9 billion of the assets. Stockholders’ equity divided by total assets = $9/$59 = 15.3%

BE2–3 (a)

Working capital = current assets – current liabilities. Boeing’s current assets total $27 billion, less $32 billion of current liabilities, gives the company negative working capital of $5 billion. Another measure of solvency would be the current ratio. The current ratio is current assets divided by current liabilities or $27 billion divided by $32 billion = 0.84. Both measures indicate that Boeing appears to have a solvency problem. Current assets are not sufficient to cover current liabilities. Under existing circumstances the Company will have to look to other sources to pay its current obligations.

(b)

No, Boeing has $15 billion of liquid current assets (cash, short term investments, and accounts receivable) but it has $32 billion of current liabilities.

1


(c)

Boeing would be more solvent if accounts receivable were $9.6 billion and inventory was $5.7 billion. Accounts receivable are closer to cash than inventory. This means that accounts receivable are expected to be converted to cash in a shorter period of time than inventory.

BE2-4 2008

2007

2006

Net cash flow from operating activities ......................... Net cash flow from investing activities .......................... Net cash flow from financing activities ..........................

$ 33,656 (29,143) (4,691)

$ 34,242 (18,616) (16,074)

$ 15,688 (8,366) (6,128)

Net change in cash .........................................................

$ ( 178) $

Cash at beginning of period ........................................... Cash at end of period………………………………. ...................

1,970 $ 1,792

(448)$ 1,194 $

2,418 1,224 1,970 $ 2,418

AT & T’s cash management activities over the three-year period of 2006 - 2008 appear to be extremely strong. The company is generating significant amounts of cash flow from operating activities, with 2007 and 2008 at roughly twice the level of 2006. AT & T is then able to reinvest substantial amounts in its asset base. At the same time AT & T is also able to fund its financing activities from its operating cash flow. The large amount of funds being used in investing activities indicates that AT & T is growing its capital-intensive business.

BE2–5 IFRS Format Non-current assets Current assets Less: Current liabilities Total

154,073 115,397 (94,384) 175,086

Non-current liabilities Equity Total

49,118 125,968 175,086

GAAP Format Non-current assets Current assets Total

154,073 115,397 269,470

Current liabilities Non-current liabilities Equity Total

94,384 49,118 125,968 269,470

Many non-US companies begin with non-current assets, add current assets, and then subtract current liabilities to reflect the resources available to generate revenues and profits. The IFRS balance sheet then


lists non-current liabilities and shareholders’ equity, which represent the financing sources of company resources; this amount is often labeled “capital employed.” GAAP balance sheets, on the other hand, list all assets owned (current and long-term) and then categorizes the financing sources (current and long-term liabilities, as well as shareholder equity) for those assets.

EXERCISES E2–1

1 2 3 4 5 6 7 8

Operating, Investing, or Financing Financing Operating Operating Investing Financing Financing Investing Operating

Balance Sheet

Income Statement

Statement of Cash Flows

Yes Yes Yes Yes Yes Yes Yes Yes

No Yes Yes No No No No No

Yes Cannot tell Yes Cannot tell Yes Yes Yes Yes

Operating, Investing, or Financing Financing Operating Operating Operating Investing Investing Financing Operating

Balance Sheet

Income Statement

Statement of Cash Flows

Yes Yes Yes Yes Yes Yes Yes Yes

No No Yes Yes No Cannot tell No No

Yes No Yes No Yes Yes Yes Yes

Statement of Stockholders Equity Yes Yes Yes No No Yes No No

E2–2

1 2 3 4 5 6 7 8

Statement of Shareholders Equity No No Yes Yes No Cannot tell No No

E2–3 a. b. c. d. e. f.

Balance sheet Income statement Balance sheet Income statement Balance sheet Income statement

g. h. i. j. k. l.

Balance sheet Balance sheet Balance sheet Balance sheet Income statement Income statement

m. n. o. p. q. r.

Balance sheet Balance sheet Balance sheet Income statement Balance sheet Balance sheet


E2–4 1. Statement of Stockholders’ Equity (Retained Earnings); Statement of Cash Flow, Income Statement 2. Income Statement 3. Balance Sheet 4. Statement of Cash Flow, Balance Sheet 5. Statement of Stockholders’ Equity; Statement of Cash Flow 6. Income Statement, Balance Sheet 7. Income Statement 8. Balance Sheet, Income Statement

E2–5 2006 2006 Beginning Retained Earnings

+

2006 Revenues

$5.3

+

$10.6 X

2006 Expenses

$8.7

=

$1.5

2006 Dividends

=

2006 Ending Retained Earnings

X

=

$5.7

2007* 2007 Beginning Retained Earnings

+

$5.7

+

2008 2008 Beginning Retained Earnings X

+ +

2007 Revenues

2007 Expenses

$12.7 X

– =

X $11.2

2008 Revenues $13.2 X

– – =

2008 Expenses 11.8 $1.4

2007 Dividends

=

2007 Ending Retained Earnings

$5.8

=

$1.4

= =

2008 Ending Retained Earnings $1.6

– –

2008 Dividends $1.2

*you must calculate the 2008 equation before you can calculate the 2007 equation

Sales growth ($) Sales growth (%) Profits ($)

2008 $0.5 3.9% $1.4

2007 $2.1 19.8% $1.5

2006 N/A N/A $1.9


Profits (% of sales) 10.6% 11.8% 17.9% Dividends (% of net income) 85.7% 386.7% 78.9%

The company saw significant sales growth, but profits were relatively flat (meaning that profits as a percentage of sales decreased). Dividends are a consistently high percentage of profits (well above profits in 2007), which is common in the utility industry.

E2–6 2007 2007 Ending Retained Earnings or 2008Beginning Retained Earnings

($523) X

= =

($499) $1,407

=

2007 Beginning Retained Earnings + Revenues for 2007 – Expenses for 2007 – Dividends for 2007

+

$1,383

X

$0

+

$1,522

$1,608

X

+

X

$1,550

$5

Expenses for 2007 are $1,407. 2008 ($758) X

=

($523)

=

$149

Dividends declared for 2008 are $149.

2009 ($596) X

= =

($758) $1,717

Revenue for 2009 is $1,717. 2007

2008

2009

Sales growth (%) .......................................................... Profits ............................................................................

N/A ($24)

10.0% ($86)

12.8% $ 167

Profits as a percentage of sales...................................... Dividends........................................................................ Dividends as a percentage of net income ......................

(1.7%) $ 0 N/A

(5.7%) $ 149 N/A

9.7% $ 5 3.0%

The advertising agency had modest sales growth from 2007 to 2009. However, from 2008 to 2009, the Company was able to go from losses to a profit. Even though the Company had a loss in 2008 the Company paid a healthy dividend. Then in 2009, when the Company showed a profit, it virtually eliminated the dividend. There is reason to be optimistic going forward. In 2009 the Company was able to show a nice growth in its sales while at the same time showing a reduction in its expenses.


E2–7 Solvency primarily indicates a company’s ability to meet its debt payments as they come due. Current liabilities are obligations that will be settled within one year or the company’s operating cycle, whichever is longer, through the use of current assets or the creation of new current liabilities. Current assets are those assets that will be consumed or converted to cash within one year or the company’s operating cycle, whichever is longer. Consequently, comparing current assets to current liabilities provides an indication of a company’s ability to meet its short-term debts. In this case, current assets were 2.76 ($348/$126) and 2.60 ($427/$164) times greater than current liabilities as of December 31, 2009 and December 31, 2008, respectively. Although comparing current assets to current liabilities provides a measure of a company’s solvency, this measure is not perfect. A true test of a company’s short-term solvency would be to compare the cash value of its current assets to the cash value of its current liabilities. For current liabilities, the book value is usually a good approximation of the cash value, since a company cannot, from a legal viewpoint, unilaterally change its debts. The situation is different for current assets though. The book value may or may not bear any relation to the cash value. Consequently, comparing the book value of current assets to current liabilities may not give an accurate measure of a company’s solvency.

E2–8 Method 1 Working capital as of 12/31/2009 ($348 – $126) ................................................................. Impact of method on current assets .............................. Impact of method on current liabilities ......................... New working capital as of January 2010 ........................

$

222

$

0 (200) 22

Method 2 $

222

$

0 0 222

It seems that only the second method would be acceptable to the company in terms of maintaining compliance with the minimum working capital covenant.

E2–9 2009

2008

2007

Beginning cash balance .................................................. Net cash flow from operating activities ......................... Net cash flow from investing activities .......................... Net cash flow from financing activities .......................... Ending cash balance .......................................................

$

5,191 9,897 (9,959) X $ 5,718

$

Y* X (4,193) (6,433) $ 5,191

$

X equals

589

12,089

$ (8,342)

................................................................. $

$

3,297 10,104 X (1,331) $ 3,728

*Beginning cash balance for 2008 = Ending cash balance for 2007. Cisco Systems’ cash management activities over the three-year period of 2007, 2008, and 2009 appear to be strong. The Company is generating a significant amount of net cash flow from operations each year (with


2008 being especially strong) and then is investing in its business. Financing activities (including dividends and/or share repurchases) reduced cash in 2007 and 2008, but turned positive in 2009.

E2–10 2008 Beginning cash balance .................................................. Net cash flow from operating activities ......................... Net cash flow from investing activities .......................... Net cash flow from financing activities .......................... Ending cash balance .......................................................

$

X equals

(978)

................................................................. $

$

2,213**$ (1,521) X 1,654 1,368

2007

2006

1,390 $ 2,845 (1,529) X $ 2,213$

X

$

2,280

(493)$

1,406 (1,495) (801) 1,390*

*2007 Beginning balance = 2006 Ending balance **2008 Beginning balance = 2007 Ending balance. Southwest Airlines’ cash management activities appear to be very good for the years 2006 and 2007, but significantly deteriorated in 2008 (due to the economic recession). The company generated a net cash inflow from its operating activities for the first two years shown, but dropped to a negative operating cash flow in 2008. A look at its investing activities reveals that the company is expanding its asset base, but had to curtail the amount spent in the third year due to the downturn. During 2008, the company had a cash inflow due to financing activities, possibly in response to a need for cash due to the poor operations. Overall, Southwest Airlines is a strong company experiencing a difficult time in its cyclical business.

E2–11 Lana & Son Statement of Cash Flows For the Year Ended Cash flows from operating activities: Cash collection from services provided.................................................. Cash payment for expenses ................................................................... Net cash increase (decrease) from operating activities .................. Cash flows from investing activities: Purchase of machinery ........................................................................... Net cash increase (decrease) from investing activities ................... Cash flows from financing activities: Proceeds from stockholders’ contributions ........................................... Payment of dividends ............................................................................. Net cash increase (decrease) from financing activities ................... Increase (decrease) in cash balance.............................................................. Beginning cash balance ................................................................................. Ending cash balance ......................................................................................

$4,000 (3,000) $1,000 $(3,000) (3,000) $7,000 (1,500) 5,500 3,500 13,000 $ 16,500 $

Based on just one year’s statement of cash flows it is difficult to comment adequately on Lana & Son’s cash management activities. However, one can observe that most of the cash during the year was generated as a


result of issuing equity. The company seems to be investing in its asset base. That will certainly help it grow in the future. Cash flows from operations is positive, which certainly is a good sign.

E2–12 Emory Inc. Statement of Cash Flows For the Year Ended Cash flows from operating activities: Cash collection from services provided.................................................. Cash payment for expenses ................................................................... Net cash increase (decrease) from operating activities .................. Cash flows from investing activities: Purchase of equipment .......................................................................... Net cash increase (decrease) from investing activities ................... Cash flows from financing activities: Proceeds from the bank loan ................................................................. Payment of dividends Net cash increase (decrease) from financing activities ................... Increase (decrease) in cash balance.............................................................. Beginning cash balance ................................................................................. Ending cash balance ......................................................................................

$40,000 (23,000) $17,000 $(23,000) (23,000) $30,000 (24,000) 6,000 0 25,000 $ 25,000 $

Based on just one year’s statement of cash flows, it is difficult to comment adequately on Emory’s cash management activities. However, it seems that the company is generating a substantial portion of its cash flows from operating activities. The company is taking some loans to finance its asset base which would be helpful in the future. Return on total assets and return on equity would probably increase.

E2–13 George’s Business Income Statement For the Year Ended Lease revenue................................................................................................... Expenses ........................................................................................................... Net income .......................................................................................................

George’s Business Statement of Stockholders’ Equity For the Year Ended Contributed Capital

Retained Earnings

$3,000 2,500 $ 500


Beginning Balance Stock Issue Net Income Cash Dividends Ending Balance

E2–13

$ 0 6,000 _____ $6,000

$

0

500 (800) $ (300)

Concluded George’s Business Balance Sheet As of Assets Cash ................................................................................................................ Land ................................................................................................................ Total assets ....................................................................................................... Liabilities & Stockholders’ Equity Note payable .................................................................................................... Contributed capital ........................................................................................... Retained earnings ............................................................................................. Total liabilities & stockholders’ equity .............................................................

$

2,700 8,000 $ 10,700 $

5,000 6,000 (300) $ 10,700

George’s Business Statement of Cash Flows For the Year Ended Cash flows from operating activities: Cash collections from customers .................................................. Cash payments for expenses ......................................................... Net cash flow from operating activities ................................... Cash flows from investing activities: Purchase of land ............................................................................ Net cash flow from investing activities .................................... Cash flows from financing activities: Proceeds from equity investor ...................................................... Proceeds from borrowing ............................................................. Cash payments for dividends ........................................................ Net cash flow from financing activities .................................... Increase in cash .................................................................................. Beginning cash balance ...................................................................... Ending cash balance ...........................................................................

$

3,000 (2,500) $

500

$ (8,000) (8,000) $

6,000 5,000 (800) 10,200 2,700 0 $ 2,700 $

Upon examining George’s financial statements the bank would certainly be concerned because George paid out more in dividends than the net income he realized during the year. George’s statement of retained


earnings shows a negative balance, which means that the payment to equity investors which was disguised as return on capital was in fact a return of capital. Generally, dividend payments cannot exceed the Retained Earnings balance.

E2–14 Mary’s Business Income Statement For the Year Ended Lease revenue................................................................................................... Expenses ........................................................................................................... Net income .......................................................................................................

$ 12,000 14,000 $ (2,000)

Mary’s Business Statement of Stockholders’ Equity For the Year Ended

Beginning Balance Stock Issue Net Income (Loss) Cash Dividends Ending Balance

Contributed Capital $ 0 30,000 ______ $30,000

Retained Earnings $ 0 (2,000) (1,000) $ (3,000)

Mary’s Business Balance Sheet As of Assets Cash ................................................................................................................ Land ................................................................................................................ Total assets ....................................................................................................... Liabilities & Stockholders’ Equity Note payable .................................................................................................... Contributed capital ........................................................................................... Retained earnings ............................................................................................. Total liabilities & stockholders’ equity .............................................................

$

2,000 40,000 $ 42,000 $ 15,000 30,000 (3,000) $ 42,000


E2–14

Concluded Mary’s Business Statement of Cash Flows For the Year Ended Cash flows from operating activities: Cash collections from customers .................................................. Cash payments for expenses ......................................................... Net cash flow from operating activities ................................... Cash flows from investing activities: Purchase of land ............................................................................ Net cash flow from investing activities .................................... Cash flows from financing activities: Proceeds from equity investor ...................................................... Proceeds from borrowing ............................................................. Cash payments for dividends ........................................................ Net cash flow from financing activities .................................... Increase in cash .................................................................................. Beginning cash balance ...................................................................... Ending cash balance ...........................................................................

$ 12,000 (14,000) $ (2,000) $ (40,000) (40,000) $ 30,000 15,000 (1,000) 44,000 2,000 0 $ 2,000 $

Mary should not have paid a cash dividend of $1,000 because of her dwindling cash position and negative earnings during the year. The dividend was a return of capital rather than a return on capital.


PROBLEMS P2–1 1. 2. 3. 4. 5. 6. 7. 8.

e e a a g c f c

9. 10. 11. 12. 13. 14. 15. 16.

a a c d c b e a

17. 18. 19. 20. 21. 22. 23.

c a d b e e e

X Company Balance Sheet (Date) Assets Current assets: Cash ................................................................................... Short-term investments .................................................... Accounts receivable .......................................................... Less: Allowance for uncollectible accounts ....................... Inventory ........................................................................... Prepaid rent ....................................................................... Total current assets ...................................................... Long-term investments: Land held for investment .................................................. Investment fund for plant expansion ................................ Total long-term investments ........................................ Property, plant, & equipment: Property............................................................................. Building .............................................................................. Less: Accumulated depreciation (building) ....................... Net book value of building ................................................ Machinery and equipment ................................................ Less: Accumulated depreciation (machinery & equipment) ................................................................... Net book value of machinery and equipment................... Total property, plant, & equipment ............................. Intangible assets: Patents............................................................................... Trademarks........................................................................ Total intangible assets .................................................. Total assets .............................................................................

$XX XX $XX XX

XX XX XX $XX $XX XX XX $XX

$XX XX XX $XX XX XX XX $XX XX XX $XX


P2–1

Concluded Liabilities and Stockholders' Equity Current liabilities: Accounts payable .............................................................. Wages payable .................................................................. Dividend payable ............................................................... Short-term notes payable ................................................. Current portion due of long-term debt ............................ Payments received in advance .......................................... Total current liabilities ................................................. Long-term liabilities: Bonds payable ................................................................... Total long-term liabilities ............................................. Stockholders' equity: Capital stock ...................................................................... Retained earnings.............................................................. Total stockholders' equity ............................................ Total liabilities and stockholders' equity ................................

P2–2 1. 2. 3. 4. 5.

e b e a e

6. 7. 8. 9. 10.

e e f c c

11. 12. 13. 14. 15.

e f f d c

$XX XX XX XX XX XX $XX $XX XX $XX XX XX $XX


P2–2 Concluded X Company Income Statement For the Period Ended Revenues: Sales................................................................................... Fees earned ....................................................................... Interest income ................................................................. Dividend income................................................................ Gain on sale of short-term investments............................ Total revenues .............................................................. Expenses: Cost of goods sold ............................................................. Operating expenses: Office salary expense ................................................... Insurance expense........................................................ Salesmen commission expense .................................... Depreciation expense................................................... Office supplies expense................................................ Advertising expense ..................................................... Total operating expenses ........................................ Other expenses: Interest expense ........................................................... Loss on sale of equipment............................................ Loss on sale of building ................................................ Total other expenses ............................................... Total expenses ................................................................... Net income .............................................................................

$XX XX XX XX XX $XX $XX $XX XX XX XX XX XX XX $XX XX XX XX XX $XX


P2–3 Nimmo Brothers Corporation Statement of Cash Flows Balance Sheet for the year ending 12/31/2011 as of 12/31/2011 Cash-Operating 275 Cash Cash-Investing (200) Other Current Assets Cash-Financing 330 Long-term Assets ∆ in Cash 405 Total Assets Cash-12/31/10 420 Cash-12/31/11 825

825 1,550 1,600 3,975

Income Statement Current Liabilities 995 for the year ending 12/31/2011 Long-term Liabilities 1,200 Revenue 4,200 Contributed Capital 1,200 Expenses 4,050 Retained Earnings 580 Net Income 150 Total 3,975

Statement of Stockholders’ Equity for the year ending 12/31/2011 Contributed Retained Capital Earnings 12/31/10 1,000 500 Net Income 150 Dividends (70) Stock Issuance 200 ___ 12/31/11 1,200 580


P2–4 Johnson Company Balance Sheet December 31, 2011 Assets Current assets: Cash ................................................................................................ Short-term investments .................................................................. Accounts receivable ........................................................................ Less: Allowance for uncollectible accounts .................................... Net accounts receivable ........................................................... Inventory ......................................................................................... Total current assets .................................................................. Property, plant, & equipment: Buildings .......................................................................................... Less: Accumulated depreciation ..................................................... Total property, plant, & equipment ......................................... Total assets ........................................................................................... Liabilities & Stockholders' Equity Current liabilities: Accounts payable ............................................................................ Taxes payable .................................................................................. Total current liabilities.............................................................. Long-term notes payable ....................................................................... Stockholders' equity: Contributed capital.......................................................................... Retained earnings............................................................................ Total stockholders' equity ........................................................ Total liabilities & stockholders' equity ...................................................

$ 8,000 40,000 $125,000 2,400 122,600 161,000a $331,600 $ 35,000 8,000 27,000 $358,600

$110,000 29,400 $139,400 79,100 $100,000b 40,100c 140,100 $358,600

__________________ a Inventory is reported at the lower of its cost or its market value. b $100,000 = $12,500 shares  $8 per share. c $40,100 = $65,000 cumulative earnings – $24,900 cumulative declared dividends. Based on only one year’s balance sheet it is a very difficult question to answer. This fact proves the point that (1) all the financial statements must be interpreted as a whole, and (2) that the information should be analyzed over a number of years to draw any meaningful conclusions.


However, based on what we have, I would not invest in this company. The current ratio is 2.379 but debt/equity ratio is 1.560, which is a cause for concern in the long term. Further, the company seems to be paying approximately 38% of its retained earnings beginning balance in dividends, which is good for the investors who are looking for short-term return on their capital.

P2–4 Concluded Johnson Company Balance Sheet December 31, 2011 Property, plant, & equipment: Buildings .......................................................................................... Less: Accumulated depreciation ..................................................... Total property, plant, & equipment ......................................... Current assets: Cash ................................................................................................ $ 8,000 Short-term investments .................................................................. Accounts receivable ........................................................................ $125,000 Less: Allowance for uncollectible accounts .................................... 2,400 Net accounts receivable ........................................................... Inventory ......................................................................................... Total current assets .................................................................. Less: Current liabilities: Accounts payable ............................................................................ Taxes payable .................................................................................. Total current liabilities.............................................................. Total…………………………………………………………………… $219,200

$ 35,000 8,000 $ 27,000

40,000

122,600 161,000a 331,600

$110,000 29,400 139,400

Capital Employed: Long-term notes payable ....................................................................... Stockholders' equity: Contributed capital.......................................................................... Retained earnings............................................................................ Total stockholders' equity ........................................................ Total ...........................................................................................

79,100

$100,000 40,100 140,100 $219,200

Many non-US companies begin with non-current assets, add current assets, and then subtract current liabilities to reflect the resources available to generate revenues and profits. The IFRS balance sheet then


lists non-current liabilities and shareholders’ equity, which represent the financing sources of company resources; this amount is often labeled “capital employed.” GAAP balance sheets, on the other hand, list all assets owned (current and long-term) and then categorizes the financing sources (current and long-term liabilities, as well as shareholder equity) for those assets.


P2–5 2008 Contributed Capital: Total assets = Total liabilities + Total stockholders' equity ($300 + $200 + $500 + $100 + $700) = ($200 + $500) + (Contributed cap. + $400) Contributed capital = $700 Net Income: Net income = Sales – Expenses = $1,000 – $400 = $600 Dividends: Ending retained earnings = Beginning retained earnings + Net income – Dividends $400 = $0 + $600 – Dividends Dividends = $200 2009 Inventory: Total assets = Total liabilities + Total stockholders' equity ($300 + $300 + Inventory + $200 + $600) = ($300 + $600) + ($400 + $800) Inventory = $700 Expenses: Net income = Sales – Expenses $400 = $1,100 – Expenses Expenses = $700 Dividends: Ending retained earnings = Beginning retained earnings + Net income – Dividends $800 = $400 + $400 – Dividends Dividends = $0 2010 Accounts Receivable: Total assets ($200 + Accts. rec. + $400 + $400 + $700) Accounts receivable

= Total liab. + Total stockholders' equity = ($500 + $800) + ($600 + $300) = $500

Expenses: Net income = Sales – Expenses ($100) = $700 – Expenses Expenses = $800 Dividends: Ending retained earnings = Beginning retained earnings + Net income – Dividends $300 = $800 + ($100) – Dividends Dividends = $400


P2–5

Concluded

2011 Accounts Payable: Total assets = Total liabilities + Total stockholders' equity ($500 + $700 + $400 + $400 + $800) = (Accts. pay. + $700) + ($600 + $600) Accounts payable = $900 Net income: Ending retained earnings = Beginning retained earnings + Net income – Dividends $600 = $300 + Net income – $200 Net income = $500 Sales: Net income = Sales – Expenses $500 = Sales – $600 Sales = $1,100 In order to assess the financial performance of this company, we need to calculate the measures of solvency and earning power. Respective measures are computed as follows: Measures of Solvency

2008

2009

2010

2011

Current Ratio: Working Capital: Debt/Equity Ratio:

5 $800 .64

4.33 $1,000 .75

2.20 $600 1.44

1.78 $700 1.33

The only measure of earning power that we can compute for this company is Return on Equity. The other measures, such as EPS and P/E Ratio, cannot be computed since the relevant information is not available. Measures of Earning Power

2008

2009

2010

2011

Return on Equity:

.55

.33

—*

.42

*No return on stockholder’s equity during 2010 since the company suffered a loss of $100. Overall, looking at the measures of solvency and earning power, one can safely conclude that the financial performance and position of the company has deteriorated since its inception in 2008. The current ratio has continued to decline and working capital has also gone down. While the company has taken more debt, it has been unable to leverage against the interest of the stockholders, since the return on equity has declined considerably. In one year, 2010, the company even suffered a loss. The company paid dividends even during the year of loss, indicating a poorly devised dividend policy.


P2–6 Kroger Balance Sheet December 31, 2009, 2008 2009 Assets Cash ............................................................................................. Accounts receivable ....................................................................... Inventory ........................................................................................ Property, plant, and equipment (net) ............................................ Other assets ................................................................................... Total assets .................................................................................$ Liabilities and Stockholders’ Equity Accounts payable ........................................................................... Other short-term debts .................................................................. Long term debt............................................................................... Stockholders’ Equity....................................................................... Total liabilities and stockholders’ equity........................................

2008

$

263 944 4,859 13,161 3,984 23,211 $

$

$

$

3,822 3,807 10,406 5,176 $ 23,211

242 786 4,849 12,498 3,918 22,293 3,867 4,816 8,696 4,914 $ 22,293

Supervalu Income Statement For the Years Ended December 31, 2009, 2008 Sales ............................................................................................. Expenses......................................................................................... Net income .....................................................................................

2009 $ 76,000 74,751 $ 1,249

2008 $ 70,235 69,054 $ 1,181

Solvency refers to a company’s ability to pay its obligations as they come due. The current ratio provides a measure of solvency by comparing those obligations that are coming due in the near future against those assets that the company expects to convert into cash or consume in the near future. Based on its current ratio, Kroger does not have sufficient current assets to cover its existing current liabilities in either year. In 2009 the current ratio was 0.80 ($6,066/$7,629), while it was 0.68 ($5,877/$8,683) in 2008. The Company’s solvency has improved in the time period shown. Earning power refers to a company’s ability to generate net assets through operations. Income has been fairly constant, as measured in terms of dollars and as a percentage of sales. Margins are thin in the company’s industry, but Kroger has shown consistent earnings in the time period.

P2–7 a. Assets are, for the most part, recorded at original cost. Over a period of time, the value of an item will change. For instance, the value of Eat and Run's property, plant, and equipment will most likely change as the items become older. Consequently, over time the cost of an item may have no relation to the item's market value. Since the cash received from selling an asset is based on the asset's market value, the asset's book value is not an accurate indicator of a company's value. b. The value of the firm would equal the sum of the fair market value of the assets less the sum of liabilities. The value of Eat and Run would, therefore, be as follows:


Market Value Cash ................................................................................... Short-term investments .................................................... Accounts receivable........................................................... Inventory ........................................................................... Prepaid insurance .............................................................. Property, plant, & equipment ........................................... Other assets ....................................................................... Total market value of assets.............................................. Less: Total liabilities .......................................................... Value of Eat and Run ......................................................... c.

$

25,000 19,000 25,000 33,000 0 100,000 0 $ 202,000 196,000 $ 6,000

If Eat and Run were to go bankrupt, the stockholders would receive anything left after all the assets were sold and the creditors were paid. In this case the fair market value of the assets exceeds the total liabilities, so the stockholders would receive the residual, which would be $6,000. As a practical matter, Eat and Run might have to hire lawyers and accountants for the bankruptcy proceedings. If this were the case, the lawyers and accountants would have to be paid before the stockholders received anything. So in this particular case, there may be nothing left for the stockholders once the creditors, lawyers, and accountants are paid.

P2–8 First, let us compute some relevant ratios that would help to evaluate the financial statements submitted by Romney Heights in support of its loan application to Acme Bank. Ratios 2011

2010

Liquidity Current Ratio (Current Assets ÷ Current Liabilities) Working Capital (Current Assets – Current Liabilities)

2.00

2.00

$7,000

$6,000

1.06

0.96

0.45

0.33

Long-Term Debt Paying Ability Debt/Equity Ratio (Total Liabilities ÷ Stockholders’ Equity) Operating Cash Flow to Total Debt


(Operating Cash Flow ÷ Total Debt) Ratios 2011

2010

Profitability Net Profit Margin (Net Income ÷ Sales)

0.34

0.19

Total Asset Turnover (Sales ÷ Total Assets)

0.55

0.58

Return on Assets (Net Income ÷ Total Assets)

0.19

0.11

Return on Assets (Net Profit Margin  Total Asset Turnover)

0.187

.110

Return on Equity (Net Income ÷ Stockholders’ Equity)

0.39

0.21

A thorough review of the various ratios reveals that Romney Heights is worth the risk. The bank should consider its loan application, at least for a short-term loan. The short-term solvency position is reasonably good. Working capital is positive and the current assets are twice the current liabilities. Regarding long-term debt paying ability the company seems to be heavily leveraged. The debt to equity ratio is more than 1 and has increased from 2010 to 2011. However, the concern is somewhat mitigated by a substantial increase in the proportion of operating cash flows to the total debt held by the company. The overall profitability of the company is on the rise, but the asset utilization is poor and flat. Since the return on equity has almost doubled, the company seems to be able to effectively leverage the increment in its debt to the advantage of its stockholders. Regarding the statement of cash flows, the company seems to be doing fine. Net cash flow from operating activities is positive. The company is investing in its asset base, probably intending to expand in the future by supplementing its cash flow from operating activities with financing either from bank loans or from equity.


P2–9 First, let us compute some relevant ratios that would help us evaluate the financial statements of Ted Tooney. Ratios 2011

2010

Liquidity Current Ratio (Current Assets ÷ Current Liabilities)

1.29

2.00

Working Capital (Current Assets – Current Liabilities)

$2,000

$4,000

Debt/Equity Ratio (Total Liabilities ÷ Stockholders’ Equity)

1.45

0.92

Operating Cash Flow to Total Debt (Operating Cash Flow ÷ Total Debt)

0.75

1.36

Net Profit Margin (Net Income ÷ Sales)

0.15

0.19

Total Asset Turnover (Sales ÷ Total Assets)

3.41

3.87

Return on Assets (Net Income ÷ Total Assets)

0.52

0.74

Return on Assets (Net Profit Margin  Total Asset Turnover)

0.51

0.74

Return on Equity (Net Income ÷ Stockholders’ Equity)

1.27

1.42

Long-Term Debt Paying Ability

Profitability

Looking at the declining trends of all financial indicators, it would be safe to decline Ted’s request for an equity investment in his company. The short-term liquidity of the company is going down. The working capital as well as the current ratio has declined. The company is becoming highly leveraged and the amount of operating cash flow as a percentage of total debt has considerably declined. This all indicates a worsening position. The profitability and return on assets are mediocre and declining. The return on equity has also declined as the company is not able to leverage its debt to the advantage of its stockholders. Even though the overall liquidity position is not that serious, the trend is towards the decline. In summary, a loan position may be taken with the company, but certainly not an equity position.


P2–10 a. As of 12/31/11 the current asset balance of Ellington Industries is 1.33 times the current liability balance. Since the debt covenant requires this balance to be 2 times the current liability balance, Ellington Industries must have current assets of at least $18,000. Since, it already has $12,000 invested in current assets, it will need to invest an additional $6,000 out of the long-term borrowing of $40,000 to comply with the debt covenant. That would leave $34,000 ($40,000 – $6,000) for additional investment in the land. b. Ellington Industries Balance Sheet January 1, 2012 Assets Current assets ......................................................................... Land investment ..................................................................... Total assets ............................................................................. Liabilities & Stockholders’ Equity Accounts payable ................................................................... Long-term liabilities ................................................................ Stockholders’ equity ............................................................... Total liabilities and stockholders’ equity ................................

$

18,000 89,000 $ 107,000 $

9,000 70,000 28,000 $ 107,000

Ratios Current assets/Current liabilities = $18,000/$9,000 = 2 Total liabilities/Total assets = $79,000/$107,000 ........ = .74 c. Ellington Industries Balance Sheet December 31, 2012 Assets Current assets ......................................................................... Land investment ..................................................................... Total assets ............................................................................. Liabilities & Stockholders’ Equity Accounts payable ................................................................... Long-term liabilities ................................................................ Stockholders’ equity ............................................................... Total liabilities and stockholders’ equity ................................

$

36,000 89,000 $ 125,000

$

7,000 70,000 48,000 $ 125,000

Since the dividend has to be paid in cash, it will come out of the current assets. According to the restrictions imposed by the debt covenant, the current assets must be twice the current liabilities, i.e., at least $14,000. This would result in an excess of $22,000 ($36,000 – $14,000) in the current assets.


Therefore, the company can pay a maximum of $22,000 in dividends without violating the debt covenant. If the company declares and pays $22,000 in dividends, then total liabilities/total assets would be equal to .75 ($77,000/$103,000).

ISSUES FOR DISCUSSION ID2–1 a. Net income represents the change in net assets (i.e., assets less liabilities) generated during the year from operating activities. Alternatively, cash flows from operating activities is the amount of cash the company generated during the year from operating activities. Since cash is simply one of many assets a company has, it is obvious that net income and cash flows from operating activities are not the same. Thus, it is quite possible for a company to have an increase in net assets from operating activities (i.e., net income) and at the same time have negative cash flows from operating activities. The ability of a company to pay dividends is a function of how much cash the company has available. A company could generate negative cash flows from operating activities but have large cash reserves from generating cash from operating activities in prior years. Alternatively, a company may have obtained enough cash to pay a dividend by borrowing the money or by selling assets. Remember, companies can generate cash from investing activities and financing activities in addition to cash from operating activities. b. A company could not continue generating negative cash flows from operating activities and expect to continue in business. A company cannot borrow money or issue stock indefinitely. At some point the creditors will demand to be repaid and the owners will demand some return on their investment. Sooner or later the company will have to generate cash from its operations to repay the creditors. Paying out dividends while generating negative cash flows from operating activities will only increase the company's cash problems.

ID2–2 The income statement of Disney would have shown increases in service revenue due to the movie Wild Hogs and increases in sales revenue from the products tied in to the movie Cars. The income statement would not yet reflect the upcoming two movies, as revenue cannot be recognized until the earnings process is substantially completed (which will not occur until the movies are released). Assuming the ESPN mobile phone licensing agreement has started, Disney would show an increase in service revenue (which would help to offset the softer service revenue from the theme parks). Finally, on the expenses portion of the income statement, the increase in programming costs for events shown on ESPN would be reflected as higher operating expenses (which would, of course, result in lower profitability).

ID2–3 a. The excerpt indicates that the Cummins Engine Company's creditors have imposed restrictions on Cummins as part of the borrowing agreement. The covenants restrict Cummins' abilities to pay dividends and borrow money and the relative amount of its current assets and current liabilities. If Cummins fails to comply with the covenants, its creditors could require Cummins to repay the loans immediately.


b. A bank or other creditor would impose such restrictions to protect itself from a loan default. That is, creditors impose restrictions on borrowers, such as the amount of cash that can be paid out for dividends, that increase the probability that the borrower will have sufficient resources to be able to make the interest and principal payments required under the borrowing agreement. c.

Debt covenants are often explicitly based on financial accounting numbers. For example, the current ratio is based on the amount of current assets and current liabilities reported on Cummins' balance sheet. Similarly, compliance with the dividend restriction can be assessed by examining the amount of dividends declared reported in the statement of retained earnings.

ID2–4 Hewlett Packard – HP is able to generate strong cash flow from its current operations ($14,591). The company appears to invest heavily ($13,711) in long term assets, probably for acquisitions and growth in its manufacturing operations. The company also spends significant funds ($2,020) by either retiring debt or returning money to the shareholders (in dividends and stock repurchases). The company’s strong cash flow from operations fuels its investments and its outflow of cash for financing. Southwest Airlines – Southwest operates in a very cyclical business that suffered greatly from the economic downturn. Cash from its operations (-$1,521) was negative, meaning the company lost cash by operating its core business. Due to the capital-intensive nature of its business, Southwest must continually purchase and upgrade aircraft to remain competitive, thus the large outflow (-$978) even in a difficult year. Southwest has raised cash from its financing activities, either from debt or equity proceeds, necessitated by the weak operating year. Boeing – Boeing suffered from the same macro economic problems that affected Southwest Airlines; the company generated negative cash from operations (-$401). Boeing differed from Southwest in that it was actively selling long term assets instead of purchasing additional ones. The positive cash from investing ($1,888) indicates the company is selling more long term assets than it is purchasing. Perhaps the company was downsizing its operations due to the recession. Finally, the company heavily used cash in financing, for debt repayments and/or share repurchases and dividend payments.

ID2–5 From the data given about the Goodrich Corporation it can be surmised that Goodrich has done a good job of generating positive operating cash flow, and that the amounts generated are increasing substantially over the time period shown. Given the fact that the company is a defense contractor and that defense spending has been heavy in the years shown, the increasing operational cash flow is an understandable trend. Strategically, the large investment in long term assets shown by the company’s cash from investing activities implies that the company is investing in long term assets such as equipment that will be used in the operations of the business. The performance of cash from financing activities shows that Goodrich has been able to return cash to shareholders and/or pay down debt, with the amounts increasing over the period shown (as the company’s operations throw off more cash). Overall, cash balances have remained relatively constant, as the strong cash inflows from operations have been used for the outflows of investing and financing.


ID2–6

A U.S. GAAP balance sheet shows the most liquid accounts first and then lists accounts in the order that they are convertible to cash. Those accounts being closest to cash are listed first. Secondly, liabilities are not shown in parentheses. Finally, some of the equity accounts carry slightly different titles. GlaxoSmithKline Consolidated Balance Sheet As of 12/31/2008 2008

2007

ASSETS: Cash Short term investments Accounts receivable Inventory Other assets Current assets

5,623 1,247 6,265 4,056 78 17,269

3,379 1,628 5,495 3,062 62 13,626

Non-Current Assets Investment in Affiliates Property, Plant & Equipment Other Investments Total

552 9,678 3,924 14,154

329 7,821 3,401 11,551

7,970

5,826

Goodwill & Other

Total Assets

LIABILITIES Loans Accounts payable Other current liabilities Current liabilities Long term liabilities Loans Long term payables Long term liabilities Shareholders’ Equity Common stock Additional paid in capital

39,393

31,003

956 6,075 2,986 10,017

3,504 4,861 1,980 10,345

15,231

7,067

5,827 21,058

1,415 1,326

3,681 10,748

1,503 1,266


Minority Interest Retained earnings Total shareholder’s equity

387 5,190 8,318

Total Liabilities and Shareholders’ Equity ID2–7

39,393

307 6,834 9,910

31,003

Earnings according to GAAP are accrual numbers, meaning that they don’t represent cash. For example, net income is derived by subtracting expenses from revenue, but revenue can be recognized even if the company has yet to receive the cash (accounts receivable are booked). If the accounts receivable, which represent a promise from a customer to pay cash, never convert into cash, the accrual net income figure is an overstatement of the company’s earnings power. Investors, therefore, look at net income in conjunction with operating cash flow to determine if the various components of accrual net income are supported by cash flows.

ID2–8 Both GE and Comcast are interested in focusing efforts on core business activities: for GE, running a television network did not fit in with its manufacturing and financial businesses, while Comcast saw a television network as a logical vertical extension of its core business of providing cable television services to consumers. The NBC transaction was completed simultaneously, with NBC’s ownership switching from GE/Vivendi to Comcast/GE. From GE’s perspective, it saw a net cash inflow (cash from investing activities decreased to purchase Vivendi’s 20% share and then increased when the 51% stake was sold to Comcast), while its balance sheet ultimately showed a decrease in NBC-related assets (from a consolidation of all NBC assets to a line item investment in NBC).

ID2–9 An analyst following both Nike (GAAP) and Adidas (IFRS) would not be pleased with the SEC decision. An analyst would like to review the financial results of the companies in a side-by-side, “apples-to-apples” comparison. With the previous requirement, the analyst could take the reconciliation prepared by Adidas and compare its net income and stockholders’ equity to those of Nike. Once the requirement was dropped, the analyst (with the same need for industry peer comparison) would effectively have to do the reconciliation by herself. The analyst would therefore need to be an expert in both GAAP and IFRS in order to compare the results of the two footwear and athletic apparel firms.

ID2–10 a. Sales Cost of sales

2009 $ 19,176 $ 18,627 10,572 55.1%

2008

2007

.

$ 16,326 10,240 55.0%

9,165 56.1%


S G & A expenses Interest expense* Taxes Net income

6,150 32.1% 40 0.2% 470 2.5% $ 1,487 7.8%

5,954 32.0% 39 0.2% 620 3.3% $ 1,883 10.1%

5,029 30.8% 50 0.3% 708 4.3% $ 1,492 9.1%

From 2007 to 2009 gross margin (the profit after cost of sales have been deducted from sales) improved by the same amount that operating margin (after overhead expenses have been deducted) deteriorated, leaving margins relatively constant; the dollar amount of profits increased due to the sales growth. In 2009, however, a series of impairment and restructuring expenses (one-time charges) decreased profits relative to prior years. *see Footnote #1 re: Interest Income and Interest Income, net b. Current assets Noncurrent assets Total assets

2009 $ 9,734 73.5% 3,516 26.5% $13,250

2008 . $ 8,839 71.0% 3,604 29.0% $12,443

From 2008 to 2009 there has been a slight change in the allocation between current and non-current assets. The company has marginally increased its share of shorter term assets. c. Current liabilities Long-term liabilities Total assets

2009 $3,277 1,279 $13,250

2008 . 24.7% $3,322 26.7% 9.7% 1,296 10.4% $12,443

From 2008 to 2009 Nike decreased the percentage of assets financed with current and long-term liabilities, meaning it increased the percentage of assets financed with equity. d. Nike is continuing to grow its business by investing over $400 million in property, plant and equipment annually and by acquiring another business in 2008. The main source of this spending was cash flow from operations ($1.7 billion in 2009, down slightly from prior years). The company’s financing operations represent a use of cash, mainly for dividend payments and share repurchases (combining for over $1 billion annually), forcing reliance on operations to fund its long-term investments. e. 2009 Net income Dividends paid Dividends as a percentage of net income

2008

$1,487 $ 467

$ 413

31.4%

21.9%

2007 . $1,883

$1,492 $344 23.1%

Nike also returns cash to shareholders by repurchasing their shares of stock ($649 million in 2009 and $1.2 billion in 2008, for example).


CHAPTER 3 THE MEASUREMENT FUNDAMENTALS OF FINANCIAL ACCOUNTING BRIEF EXERCISE BE3–1 1. Fiscal period 2. Economic entity 3. Conservatism 4. Consistency 5. Revenue recognition

6. Materiality 7. Matching 8. Objectivity 9. Objectivity 10. Stable dollar

EXERCISES E3–1 At the beginning of the period, $6 billion would allow the corporation to buy a "basket of goods." Due to the increase in the general price level, the same basket of goods would cost more than $6 billion at the end of the period. Therefore, the corporation would have less purchasing power at the end of the period than at the beginning of the period. The decrease in purchasing power would be computed as follows: 1. Compute the cost of the basket of goods at the end of the period: = $6,000,000,000 (1 + inflation rate) = $6,000,000,000  1.02 = $6,120,000,000 2. Compute change in cost of the basket of goods for the period: = $6,120,000,000 – $6,000,000,000 = $120,000,000 This decrease in purchasing power would not be reflected in the corporation's financial statements. Accountants adhere to the stable dollar assumption, which means that changes in the general price level are ignored when determining the value of assets and liabilities. This assumption allows users of financial statements to compare financial statements from different points in time. Boeing would want to keep its cash balance as low as possible because money sitting as cash makes very little to no investment income. Most companies would try to maximize its interest income by investing most of its cash. At the same time the company would not reduce its cash balance to zero because it has to have cash on hand to pay bills as they come due everyday.

E3–2 a. Each land acquisition would be recorded at its original cost of $15,000, for a total of $30,000. b. No, the company could not purchase the same basket of goods for $15,000 in 2011 as in 1993. To purchase the same basket of goods in 2011, the company would need $24,000 [$15,000  (1 + 60%)]. Therefore, cash held by the company from 1993 to 2011 would be subject to an economic loss of $9,000 ($24,000 – $15,000).

1


c.

There are two alternatives for reporting the value of the land if the stable dollar assumption is ignored. The first alternative is to report both pieces of land at 1993 dollars. The second alternative is to report both pieces of land at 2011 dollars. The two alternatives are shown below.

E3–2 Concluded 1993 land in 1993 dollars 2011 land in 1993 dollars Total land in 1993 dollars a b

$ $

15,000 9,375a 24,375

2011 land in 2011 dollars 1993 land in 2011 dollars Total land in 2011 dollars

$ $

15,000 24,000b 39,000

$9,375 = $15,000 cost of land in 2011 ÷ 1.6 $24,000= $15,000 cost of land in 1993  1.6

E3–3 Original Cost Cash Short-Term Investments Inventories Prepaid Expenses Long-Term Investments Notes Receivable Machinery Equipment Land Intangible Assets Short-Term Payables Long-Term Payables 1 2 3 4

2 X 3

Fair Market Value

Present Value

X 1 2 3

Replacement Cost

2 3 X

4 4 X 4 X X

Short-term investments are recorded at fair market value. Inventory is reported at the lower of original cost or market value, where market value can be based on fair market value or replacement cost. Long-term investments are carried on the books at original cost, fair market value, or amortized value, depending upon the type of investment. Long-lived assets, such as machinery, equipment, and intangible assets, are reported on the balance sheet at net book value, which equals original cost less the portion of original cost amortized to date.

E3–4 a. If Cisco were to determine that a portion of its inventory were obsolete, the company would lower the value of the inventory (an asset on the balance sheet) and would book an expense on the current income statement (which would ultimately lower stockholder equity on the balance sheet). b. Ultimately, the management of Cisco is responsible for forecasting future demand for its products (currently held in inventory) and the valuation to be used in the financial statements. Ultimately, therefore, it is management who controls the value of the asset and any associated writedown expense on the income statement.

2


c.

Driving the valuation of inventory is the aim to not overstate the value of items yet to be sold. We do not want financial statements to list the value of inventory at its cost if market forces have changed to the point that the company could only sell the inventory for a price below its cost. In this sense, conservatism rules the day, requiring companies to list their inventory at the lower of the cost of the inventory or the inventory’s value in the market. In equivalent terms, an owner of a gas station should not list his gasoline inventory at his cost of $2.50 per gallon if market forces have pushed down the selling price (the value in the market) of gasoline to $2.25 per gallon. We need, however, the company’s management to be able to back up the market values used with objective data (such as the published current selling prices of gasoline) in order to prevent manipulation of the financial statements. If a company uses a subjective measure of market value to book a loss in one reporting period (due to the writedown expense) and then generates a larger profit the following period (when the marked-down inventory is sold in the market at typical market prices), the financial statements have not provided the reader a clear understanding of the business performance.

E3–5 a. The most common point at which a company would recognize revenue is at the time of delivery. So in this case McKey and Company would recognize revenue in February. b. The four criteria for recognizing revenue are (1) the company has completed a significant portion of the production and sales effort, (2) the amount of revenue can be objectively measured, (3) the company has incurred the majority of costs, and remaining costs can be reasonably estimated, and (4) cash collection is reasonably assured. Presumably McKey and Company is reasonably assured that Cascades Enterprises will eventually be able to pay the $40,000, or McKey would not have entered into the agreement with Cascades Enterprises. Since the production and sales effort was not really complete until McKey shipped the brackets on February 9, February 9 appears to be the appropriate date to recognize the revenue. c.

Under the appropriate conditions, revenue can be recognized at several points in time. Revenue could be recognized (1) during production, (2) at the completion of production, (3) at the point of delivery, or (4) when the cash is collected. Case 1 normally arises in long-term construction projects such as office buildings, bridges, and so forth. Case 2 arises where goods are manufactured to the exact specifications of a customer, and the goods cannot be sold to another party. Case 3 is the most common point of revenue recognition. Case 4 arises when cash collection is not reasonably assured.

d. McKey's managers could be interested in the timing of revenue recognition due to incentives provided by contracts. For example, the managers may be paid a bonus based upon accounting income. A manager who is trying to maximize his or her bonus might prefer recognizing revenue in a particular period rather than in a different period. Another contract that might influence the actions of managers would be a debt covenant. If a debt covenant stipulates a maximum debt-to-equity ratio, and the company is nearing the ratio, the managers could improve the ratio by increasing stockholders' equity. One way to increase stockholders' equity is to increase net income. Consequently, speeding up the recognition of revenue (called front loading) might prevent the company from violating a debt covenant.

E3–6 a. (1) Revenue recognized at the end of the project. Lahmont Bridge Builders Income Statement For the Period Ended

Revenues from long-term contracts 3

Period 1

Period 2

$0

$600,000


Construction expenses Net income

E3–6

0 $0

400,000 $200,000

Period 1

Period 2

$ 450,000a 300,000 $ 150,000

$ 150,000b 100,000 $ 50,000

Concluded

(2) Revenue recognized during production. Lahmont Bridge Builders Income Statement For the Period Ended

Revenues from long-term contracts Construction expenses Net income

a $450,000 = ($300,000 ÷ $400,000)  $600,000 b $150,000 = $600,000 – $450,000, or ($100,000 ÷ $400,000)  $600,000

(3) Revenue recognized when payments are received. Lahmont Bridge Builders Income Statement For the Period Ended Period 1 $ 400,000 300,000 $ 100,000

Revenues from long-term contracts Construction expenses Net income Note:

In all three cases, costs are recognized as expenses in accordance with the matching principle. That is, the costs are not expensed until the costs have helped generate a benefit in the form of revenue.

b. Assumption (1) (2) (3)

Period 2 $200,000 100,000 $100,000

Period 1 Income $

Period 2 Income

0 150,000 100,000

$200,000 50,000 100,000

4

Total Income $200,000 200,000 200,000


E3–7 a. Original cost Depreciation expense Accumulated depreciation Net book value

2011

2012

2013

2014

2015

$25,000 5,000 5,000 20,000

$25,000 5,000 10,000 15,000

$25,000 5,000 15,000 10,000

$25,000 5000 20,000 5,000

$25,000 5,000 25,000 0

b. Since the truck has an estimated useful life of five years, it is assumed that RDP and Brothers will receive a benefit from using the truck in each of the five years. Consequently, RDP and Brothers expect to receive benefits from the truck in the future. According to the matching principle, costs should be matched against the benefits the costs help generate. Since the benefits from the truck will not be realized until future periods, the cost of the truck should be capitalized. In addition, an asset, by definition, is something that a company controls that is expected to provide benefits in the future. In order to receive the future benefits from using the truck, RDP and Brothers must continue to exist. In other words, RDP and Brothers is assumed to be a going concern. If accountants did not use the going concern assumption, it would be inappropriate to capitalize costs because the company may not exist when the expected benefits resulting from the cost are to be realized. c.

It is assumed that the truck will help generate a benefit (i.e., revenue) in each year of its useful life. Under the matching principle, the cost of an item should be allocated to the period(s) in which the cost helps generate a benefit for the company. In this particular case, the truck is expected to provide a benefit for five years. If the entire cost was expensed in 2011, then an improper matching of costs with the related benefits would arise in 2011–2015. However, by capitalizing the cost of the truck in 2011 and then allocating a portion of the cost to each of the next five years, RDP and Brothers is able to match the cost of the truck with each period in which the truck is expected to provide a benefit to the company.

E3–8 a. Costs that are expected to provide future benefits to a company are, by definition, assets. Hence, all such costs should be capitalized. As these costs help generate benefits, such as revenue, the costs are recognized as expenses and matched against the corresponding benefits. b. Capitalizing expenditures and subsequently amortizing these costs are not costless activities. A company incurs costs, such as bookkeepers' salaries, supplies, and so forth, when engaging in such activities. In certain instances, these bookkeeping costs may exceed the benefits derived from properly capitalizing and amortizing expenditures. This situation is most likely to arise when the amount of an expenditure is very small in relation to some criteria such as total expenditures, net income, or total assets. Such an expenditure is so small that users of financial statements would not care whether the expenditure was capitalized or immediately expensed. The financial statement users' decision process would not be influenced by the accounting treatment given such expenditures. In these cases, a company would apply the concept of materiality to decide whether an expenditure should be capitalized or expensed.

5


E3–9 a. (1) During 2010 the company changed depreciation methods. This change resulted in an increase of the book value of the assets versus if no change in accounting method had occurred. In other words, the depreciation expense went down by the same amount, i.e., $5,000. A decrease in the depreciation expenses would increase the net income by the same amount, i.e., $5,000. (2) During 2012 the company changed its method of inventory valuation, which also increased the book value of the inventory. Since the cost of inventory is allocated either to the cost of goods sold account or to the ending inventory account, this change implies that the Cost of Goods Sold decreased by $9,000. This would also increase the net income by $9,000. Overall it seems the company is having a bad year and is attempting to use liberal accounting policies to paint a “rosy” picture of the operations. b. Net income as reported Effect of depreciation change Effect of inventory change Adjusted net income

2009

2010

2011

2012

$ 21,000 0 0 $ 21,000

$ 24,000 (5,000) 0 $ 19,000

$ 23,000 (5,000) 0 $ 18,000

$ 29,000 (5,000) (9,000) $ 15,000

The adjusted net income figures indicate that if the company had not changed accounting methods, it would have reported declining profits. In fact, the company would have reported net income of only $15,000 in 2012. The reported net income figures have been enhanced with accounting techniques rather than by sound economic health. Consequently, the company's performance would be viewed less positively.

6


E3–9 c.

Concluded

Companies should adhere to the principle of consistency. This principle states that a company should use the same accounting principles and methods from year to year. Such a practice promotes the comparability of the company's financial statements over time and also promotes user confidence in the financial statements. If a company was free to switch accounting principles and methods at will, financial statement users would place very little faith in the statements. Under certain conditions, companies may switch accounting principles. The primary condition that must be met before a company may switch methods is the approval of the company's auditors. The company must convince its auditors that the environment it faces has changed sufficiently so that the new accounting principle, rather than the old principle, more appropriately reflects the company's financial position and performance.

E3–10 a. Under U.S. GAAP, conservatism and objectivity are important concepts in the valuation of assets such as inventory. GAAP statements are going to list inventory at its historic cost (an objective number), unless it can be documented objectively that market value has dropped below cost, in which case the inventory will be carried at the lower (more conservative) market value figure. IFRS statements, on the other hand, are going to first look to market value to determine carrying amounts. If management determines (without necessarily providing objective verification) that market value is different than the current carrying cost, the balance sheet value will be changed— and it may be changed higher or lower, depending on the move in market prices. (Inventory changes in GAAP will only be write-downs, never write-ups.) b. It is possible that year-end adjustments for inventory at Adidas will be positive (that is, the carrying amount of inventory will increase), due to management’s belief that its current ending inventory is more valuable than previously thought. If, for example, Adidas has a shoe line that is very popular in current youth fashion and the company is able to sell the shoes for a higher price (due to the demand from its customers), the company could write up the value of the ending inventory to its higher market value. Nike, on the other hand, follows U.S. GAAP, which would preclude such a move; Nike could have a hugely popular line of shoes (which may indeed prove to sell at higher market prices than previously thought), but the company would have to leave its ending inventory at its (objective) historic cost. Any increase in value in Nike’s inventory would have to be recorded only when the company actually sells the inventory at that higher price; GAAP does not allow the higher market value to be figured until the asset is sold in the market.

7


PROBLEMS P3–1 a. The company would report a gain of $10,000. b. No. During 2011 the purchasing power of money decreased by 10%. On December 31, 2011, it would require $1,100 [$1,100  (1 + 10%)] to purchase the same basket of goods that $1,000 would have purchased on January 1, 2011. The difference in purchasing power gives rise to an economic loss of $100. Therefore, $20,000 would not allow someone to purchase twice as many goods and services on December 31, 2011 as on January 1, 2011. To be able to purchase the same amount of goods, an individual would need $11,000 [$10,000  (1 + 10%)], which implies that to be able to purchase twice as many goods and services, an individual would need $22,000. c.

The $10,000 gain can be broken down into two components: a gain due to the increase in the value of the property and a gain due to general inflation. Since the inflation rate during 2011 is 10%, the value of the land would be expected to increase during 2011 by 10%, or $1,000. The remaining $9,000 of the gain is due to an increase in the value of the property, which represents an economic gain. Accountants ignore the effects of inflation due to the stable dollar assumption. This assumption allows financial statement users to compare financial statements from different points in time. Further, the stable dollar assumption gives rise to more objective financial statements. In order to adjust for the effects of inflation, the inflation rate must be known. Should the adjustment be based on wholesale, retail, global, national, state, industry, or company-specific inflation rates? Company-specific rates are probably the most relevant rate, yet they are probably the most subjective. The other rates may not be relevant for some companies. If managers were allowed to select the appropriate rate for their companies, they could manipulate the financial statements. On the other hand, if the FASB or SEC mandated the use of a particular inflation rate, the rate would not be relevant for many companies. Consequently, the choice of an inflation rate would be arbitrary and could lead to distortions in the financial statements. The use of original costs is arbitrary and also leads to distortions in the financial statements. However, the use of original costs has at least two advantages over the inflation-adjusted amounts. First, the use of original costs eliminates a potential source of manipulation of financial statements by managers. Second, users may disagree on the appropriate inflation rate for a company, and original-cost financial statements allow users to individually adjust the financial statements for their perceptions of inflation.

8


P3–2 a. The Banking Corporation will recognize interest revenue of $240. The amount of cash given to Bush Enterprises was $4,760 and in exchange Banking Corporation received a note receivable for $5,000. The difference is the amount of interest revenue that Banking Corporation will recognize in its books on December 31. b. Banking Corporation is better off at the end of the year than if the company had not invested the $4,760 on January 1. Overall, however, the company is worse off financially on December 31 than on January 1. To purchase the same basket of goods on December 31 as it could purchase for $4,760 on January 1, Banking Corporation would need $5,236 [$4,760  (1 + 10%)]. In other words, the company would need an additional $476. By loaning the money to Bush Enterprises during the year, Banking Corporation acquired $240. Hence, during the year the company became economically worse off by $236 ($476 – $240). Just to maintain its purchasing power, Banking Corporation would have to loan money at the inflation rate. To improve its purchasing power, Banking Corporation would have to loan money at a rate that exceeds the inflation rate. c.

As indicated in Part (b), Banking Corporation actually lost $236 of purchasing power during the year. On the other hand, Bush Enterprises gained purchasing power during the year. Bush could have invested the $4,760 it borrowed in a basket of goods on January 1. On December 31, Bush could sell the basket of goods for $5,236, repay Banking Corporation $5,000, and still have $236 left over. Consequently, Bush Enterprises ended up with the better deal. Whenever the interest rate on a loan is less than the inflation rate, the borrower has an advantage. Since accountants adhere to the stable dollar assumption, inflation is not reflected in financial statements. Consequently, the financial statements of Banking Corporation would indicate the company is better off by the amount of interest revenue, while the financial statements of Bush Enterprises would indicate the company is worse off by the amount of interest expense.

P3–3 a. Cash Inflows From Sale Asset A: Option 1 Option 2 Option 3

$1,500 1,500 0

Asset B: Option 1 Option 2 Option 3

500 500 0

Asset C: Option 1 Option 2 Option 3

3,000 3,000 0

Cash Outflow for Replacement $

0 (1,000) 0

Future Cash Flows 0 5,000 2,500

$1,500 5,500 2,500

0 (2,000) 0

0 3,500 2,500

500 2,000 2,500

0 (3,500) 0

0 5,000 2,500

3,000 4,500 2,500

9

$

Total Cash Flows


P3–3

Concluded

Kathy made the correct decision with respect to Assets B and C, but not to Asset A. As demonstrated above, Option 3 (i.e., retaining the asset) yields the highest net cash flows for Asset B. For Asset C, Option 2 (i.e., selling and replacing the asset) yields the highest net cash flows. However, the best option for Asset A is Option 2. If Kathy had selected this option, she would expect to generate a total of $5,500 in net cash inflows, an increase of $3,000 over the net cash inflows that are expected under the option she selected. b. The original cost information should not be used in evaluating Kathy’s decisions. Original costs represent sunk costs, and sunk costs should not be considered in future decisions. In evaluating the performance of a manager, we are interested in the cash flows generated by the manager. If the cash flow information is not available, then proxies for the cash flows must be used in the evaluation process. One such proxy is original cost data, which may be helpful in computing net income. However, if the cash flow information is available, then this information should be used in evaluating the performance. Since the cash flow information is available in this case, the original cost data can and should be ignored. c.

Under generally accepted accounting principles, assets should be carried on the balance sheet at original cost. Assuming that Kathy proceeds with her decision and keeps Assets A and B and replaces Asset C, the company should report the following amounts for each asset. Asset A Asset B Asset C

$4,000 1,500 3,500

The company is applying the principle of objectivity, which states that financial accounting information must be verifiable and reliable and that the value of transactions be objective. In many cases, original cost is the most objective of the potential valuation bases.

P3–4 a. Real sales did not actually increase by 9% from 2006 to 2008. To compute the real percentage change in sales, inflation must be considered. Converting 2008 sales to 2006 dollars reveals that 2008 real sales were actually $6.79 = [$7.2 ÷ (1 + 6%)]. Consequently, sales increased from 2006 to 2008 by $192 million, which is only a 2.9% increase in sales. b. (1) 2008 sales in 2006 dollars = $7.2 ÷ (1 + 10%) = $6.55 (2) Real change in sales = 2008 sales in 2006 dollars – 2006 sales in 2006 dollars = $6.55 – $6.6 = ($0.05) (3) Real percentage change in sales

= Real change in sales ÷ 2006 sales in 2006 dollar = ($0.05) ÷ $6.6 = (0.76%)

10


P3–4 c.

Concluded The stable dollar assumption assumes that inflation does not exist. So under this assumption, sales actually increased by 9%. However, once one realizes that the stable dollar assumption is simply an assumption that promotes the comparability of financial statements from different points in time and that it does not accurately reflect reality, one must consider price changes when comparing financial data from different points in time.

P3–5 a.

The first step in a comparison across currencies is to convert the different statements into one currency, using the latest available exchange rate. Converting the pounds of GlaxoSmithKline into U.S. dollars is shown below: Sales 24.3 pounds x $1/.69 pounds = $35.2 Assets 39.3 pounds x $1/.69 pounds = $57.0 Equity 7.9 pounds x $1/.69 pounds = $11.4 Converting the Euros of Sanofi-Aventis into U.S. dollars is shown below: Sales 27.5 Euros x $1/.71 Euros = $ 38.7 Assets 71.9 Euros x $1/.71 Euros = $101.3 Equity 45.1 Euros x $1/.71 Euros = $ 63.5

From a Sales and an Asset perspective, Pfizer is the largest of the three companies. From an Equity perspective, Sanofi-Aventis is the largest firm. b. If exchange rates move drastically, the comparison might yield a different answer. For example, if the exchange rate between the dollar and the Euro changed from .71 to .55 (due to macroeconomic events, such as a global debt crisis) then the Sales of Sanofi-Aventis would be larger (compared U.S. dollar to U.S. dollar) than those of Pfizer.

P3–6 a. In this case, the purchase price should equal the stream of future cash flows discounted to reflect the time value of money. The purchase price would be calculated as follows. Purchase price

= Present value of future cash flows = $219  Present value of an ordinary annuity factor for r = 12% and n = 10 = $219  5.65022 (from Table 5 in Appendix) = $1,237.4

b. Book value represents the residual ownership interest in the company based upon the financial statement values. This residual interest is, by definition, total stockholders' equity. Therefore, the book value of Manpower, Inc. is $2,484 ($1,283 of common stock + $1,201 of retained earnings). Using the accounting equation, the book value can also be calculated as total assets less total liabilities. c.

The purchase value of a company can be different from the book value of the company because the fair market value of individual assets and liabilities may be different from the book value of individual assets 11


and liabilities. The book values of many assets are largely based upon original costs, which ordinarily do not reflect fair market values.

P3–7 a. The book value of the building equals the value of the building according to Barry Smith's company's financial records. Long-lived assets are initially recorded at their cost, and then over time the assets are reported at net book value, which is original cost less the portion of the asset's cost amortized to date. In this case, Barry Smith paid $90,000 for the apartment building, and as of January 1, 2011, none of the cost had been amortized. Thus, the book value of the building on January 1, 2011 is $90,000. The economic value of the building is equal to the present value of the cash inflows the building will generate in the future less the present value of cash outflows the building will require in the future. In this particular case, there are two different types of cash inflows: the annual rental amounts of $65,000, which would be an annuity, and the expected proceeds of $40,000 from selling the building. The net present values of the cash flows are calculated below. Annual net cash flows Present value = ($65,000 cash inflow – $45,000 cash outflow)  Present value of an ordinary annuity factor for i = 10% and n = 10 = $20,000  6.14457 (from Table 5) = $122,891.40 Proceeds from sale of building Present value = $40,000  Present value factor for i = 10% and n = 10 = $40,000  .38554 (from Table 4) = $15,421.60 Total present value

= $122,891.40 + $15,421.60 = $138,313

Since the present value of future cash flows exceeds the purchase price of $90,000, it appears that Barry made a wise investment. b. Barry Smith Income Statement For the Year Ended December 31, 2011 Rental revenue ................................................................................................... $ Management expenses ......................................................................................... Depreciation expense............................................................................................ Net income ..........................................................................................................

12

65,000 (45,000) (5,000) $ 15,000


P3–7

Concluded Barry Smith Balance Sheet As of December 31, 2011

Assets

c.

Liabilities & Stockholders' Equity

Cash Building Accumulated depreciation

$ 20,000 90,000 (5,000)

Total assets

$ 105,000

Liabilities Contributed capital Retained earnings Total liabilities and stockholders' equity

$

0 90,000 15,000

$ 105,000

Present value of future cash flows on December 31, 2011: Present value of annual net rentals

Present value of sale proceeds

Total present value

= $20,000  Present value of an ordinary annuity factor for i = 10% and n = 9 = $20,000  5.75902 (from Table 5) = $115,180.40 = $40,000  Present value factor for i = 10% and n = 9 = $40,000  .42410 (from Table 4) = $16,964

= $115,180.40 + $16,964.00 = $132,144.40

Economic income = Net cash received during 2011 + (12/31/11 present value – 1/1/11 present value) = ($65,000 cash inflow – $45,000 cash outflow) + ($132,144.40 – $138,313.00 (from part [a]) = $20,000.00 – $6,168.60 = $13,831.40 Accounting income differs from economic income because economic income incorporates the time value of money. Hence, economic income reflects that the purchasing power of $1.00 received on December 31, 2011 is not equivalent to the purchasing power of $1.00 received on December 31, 2012. Accounting income, through the stable dollar assumption, ignores the time value of money. Further, economic income considers future events (i.e., discounted future cash inflows and outflows), whereas accounting income considers only past events. d. The book value of the building on December 31, 2011 equals the cost of the building less the associated accumulated depreciation. Therefore, the book value is $85,000 ($90,000 – $5,000). The present value of the building equals the present value of future cash flow, which as of December 31, 2011, is $132,144.40 (from Part [c]).

13


P3–8 a. Book value on 12/31/11

= Total book value of assets – Total value of liabilities = $124,000 – ($8,000 + $20,000) = $96,000

b. The economic value of Myers and Myers equals its future cash flows discounted to reflect the time value of money. Myers and Myers have two streams of future cash flows. The first type is annual cash flows, which is an annuity, and the second type is the cash flow from the sale of the business. The present values of these two cash flows are calculated below. Annual cash flows Present value = = =

$20,000  Present value of ordinary annuity factor for i = 10%, n = 10 $20,000  6.14457 (from Table 5) $122,891.40

Proceeds from sale of business Present value = $80,000  present value factor for i = 10%, n = 10 = $80,000  .38554 (from Table 4) = $30,843.20 Total present value of future cash flows c.

Liquidation value

= = =

= =

$122,891.40 + $30,843.20 $153,734.60

Total fair market value of assets – Total value of liabilities $124,000 – ($8,000 + $20,000) $96,000

d. Book value is based upon the original cost of individual assets. This value provides little indication of a company's current value due to price changes. The problem is magnified as the company's assets age. Liquidation value is based upon the fair market values of individual assets and liabilities. This value provides an accurate measure for a company planning to cease operations. However, such a measure provides little indication for a company that is a going concern. Under the going concern assumption, accountants are concerned with providing accounting numbers for companies that will continue operating indefinitely. Present value is based upon future cash flows; as such, it incorporates all non-quantifiable assets, such as employee loyalty and customer loyalty. This value more accurately reflects the economic value of a company, since it captures items not included on a balance sheet under GAAP. Unfortunately, it is difficult, if not impossible, to accurately predict future cash flows. Hence, in most instances, present value amounts do not satisfy the principle of objectivity. A difference between a company's book value and its economic value (i.e., present value of future cash flows) can arise for two reasons. First, this difference can be due to a difference between a company's book value and the fair market value of its individual assets and liabilities. The assets are usually carried on the books at their original cost. However, over time the actual value of the assets would be expected to diverge from their original cost. Second, the difference between a company's book value and its economic value can be due to the excess of the company's economic value over the fair market value of its net assets (i.e., total assets less total liabilities). The net assets are worth more grouped together than individually. Companies generate customer loyalty and name recognition that has value, yet is not reflected in the value of any particular asset. This value is, however, reflected in its economic value. The excess of the company's economic value over the fair market value of its net assets represents goodwill. In this case, Myers and Myers goodwill would be approximately $153,734.60 – $96,000, or $57,734.60.

14


P3–9 a. Ending retained earnings = Beginning retained earnings + Net Income – Dividends $40,000 = $16,000 + Net Income – $0 Net Income = $24,000 b. 2012 FMV

= = =

FMV of total assets – Total liabilities $148,000 – ($6,000 + $20,000) $122,000

2011 FMV

= = =

FMV of total assets – Total liabilities $124,000 – ($8,000 + $20,000) $96,000

2012 Net income

c.

= = =

2012 FMV – 2011 FMV $122,000 – $96,000 $26,000

Present value of future cash flows as of December 31, 2012: Annual cash flows Present value = $20,000  Present value of ordinary annuity factor for i = 10%, n = 9 = $20,000  5.75902 (from Table 5) = $115,180.40 Proceeds from sale of business Present value = $80,000  Present value factor for i = 10%, n = 9 = $80,000  .42410 (from Table 4) = $33,928.00 Total present value of future cash flows

= =

$115,180.40 + $33,928.00 $149,108.40

Economic income = Net cash received during 2012 + (12/31/12 present value – 12/31/11 present value) = $20,000 + ($149,108.40 – $153,734.60 = $20,000 – $4,626.20 = $15,373.80

15


P3–9

Concluded

d. All three income measures provide a performance measure of Myers and Myers. Of the three measures, economic income is the only one that incorporates the time value of money. In theory, holding everything else constant, economic income is probably more accurate than the other measures. Unfortunately, in the real world it is extremely rare that the future cash flows can be predicted with any reasonable degree of accuracy. So in the end, economic income is simply a guess based upon estimates of the timing and amount of future cash flows. Lack of objectivity can also plague income computed using fair market values. If a strong market exists for each of the company's assets, such as with marketable securities, then the company could probably obtain reasonably accurate estimates of what it could receive for selling the assets. In this case, net income would be more relevant than income computed using original costs, since more up-to-date values are being used. Unfortunately, it is not possible to find a market for all assets. For example, a manufacturing company may use highly specialized equipment in its production process. If no other company would use this equipment, does the equipment have a fair market value? Do we assign it a value of zero, assign it a scrap value, or assign it an arbitrary fair market value? The end result is that the value assigned to some assets will be arbitrary and not objective. Computing net income under GAAP circumvents the problem of arbitrary values and lack of objectivity. The values assigned to most assets are based upon their original costs. Assets are usually acquired in arm's-length transactions. Since each party would have opposing interests, the purchase price should accurately reflect the value of the asset on the purchase date. Further, anybody examining the value of the asset could verify the original cost. Although using original cost as a basis for valuing assets provides objective values, original cost amounts can be extremely outdated.

P3–10 a. ABC Inventory Method

Depreciation Method

Income

XYZ Working Capital

Income

Working Capital

B Y $28,000 $26,000 $24,000 $30,000 B X 20,000 26,000 16,000 30,000 A Y 18,000 16,000 14,000 20,000 A X 10,000 16,000 6,000 20,000 Note: Changes in the companies' inventory balances affect net income through Cost of Goods Sold. b. ABC and XYZ both have the highest net income and working capital under the combination of Method B and Y depreciation. Managers could have many reasons for selecting one accounting method over another method. Management is a party to many contracts that may rely on accounting numbers. For example, a manager may have an incentive compensation contract based upon accounting income. A company may have a debt covenant with a creditor that stipulates a minimum level of working capital (or some other relevant measure). Or a manager may have incentives to minimize the company's tax liability. To the extent that the accounting methods used for tax reporting must also be used for financial reporting, a manager may select those methods that provide a tax benefit. In selecting a particular accounting method, a manager will consider the factors that provide an incentive for selecting one method over another, and in the end the manager would be expected to select the accounting method that gives him or her the greatest benefit. In some cases, the accounting method selected by the manager may actually cause net income to decrease. The most likely reason for a manager to select an

P3–10 Concluded 16


accounting method that would cause net income to decrease would be to minimize taxable income, thereby minimizing cash outflows for taxes. c. As an investor, one must realize that different companies may face different environments. To the extent that two companies face different environments, we would expect them to select the accounting methods appropriate to their particular environments. Further, an investor must realize that managers have their own interests and will work to satisfy their interests. In some cases the interests of the managers will be congruent with the investors' interests, and in some cases they will not. Generally accepted accounting principles allow companies to use different accounting methods because it is impossible to select a method that would be appropriate across different companies and environments. Consequently, companies are allowed to select those methods that they deem appropriate for their situation. To an investor, the underlying economic reality (i.e., expected future cash flows) of the company is of interest. Consequently, if companies use different accounting methods, the effects of the different methods on the amounts reported in the financial statements must be considered in comparing different companies.

P3–11 a. Revenues Assumption 1 [$2,400,000  (2/12)] [$2,400,000  (6/12)] [$2,400,000  (3/12)] [$2,400,000  (1/12)] Assumption 2 [$2,400,000  (380/1,140)] [$2,400,000  (380/1,140)] [$2,400,000  (285/1,140)] [$2,400,000  (95/1,140)] Assumption 3 [$2,400,000  (600/2,400)] [$2,400,000  (900/2,400)] [$2,400,000  (300/2,400)] [$2,400,000  (600/2,400)]

Year 1

Year 2

Year 3

Year 4

$400,000 $1,200,000 $600,000 $200,000

800,000 800,000 600,000 200,000

600,000 900,000 300,000 600,000

17


P3–11 Concluded b. Costs Assumption 1 [$1,140,000  (2/12)] [$1,140,000  (6/12)] [$1,140,000  (3/12)] [$1,140,000  (1/12)] Assumption 2 [$1,140,000  (380/1,140)] [$1,140,000  (380/1,140)] [$1,140,000  (285/1,140)] [$1,140,000  (95/1,140)] Assumption 3 [$1,140,000  (600/2,400)] [$1,140,000  (900/2,400)] [$1,140,000  (300/2,400)] [$1,140,000  (600/2,400)]

Year 1

Year 2

Year 3

$190,000 $570,000 $285,000 $ 95,000

380,000 380,000 285,000 95,000

285,000 427,500 142,500 285,000

Net Income Assumption 1 $ 400,000 1,200,000 600,000 200,000

Year 1 – – – –

$190,000 570,000 285,000 95,000

$210,000

Assumption 2 $ 800,000 800,000 600,000 200,000

– – – –

$ 380,000 380,000 285,000 95,000

420,000

Assumption 3 $600,000 900,000 300,000 600,000

– – – –

$ 285,000 427,500 142,500 285,000

315,000

Year 2

Year 3

Year 4

$630,000 $315,000 $105,000

420,000 315,000 105,000

472,500 157,500 315,000

c. Assumption 1 Assumption 2 Assumption 3

Year 4

Total RevenueTotal Cost $2,400,000 2,400,000 2,400,000

Total Net Income $ 1,140,000 1,140,000 1,140,000

18

$1,260,000 1,260,000 1,260,000


P3–12 a. Hydra Aire would recognize the following revenue in each of the 3 years based on the number of toasters produced times the selling price per toaster. Year 1: Year 2: Year 3:

200 200 100

  

$100 $100 $100

= = =

$20,000 $20,000 $10,000

b. Hydra Aire would recognize the following revenue in each of the 3 years based on the number of toasters delivered times the selling price per toaster. Year 1: Year 2: Year 3:

150 200 150

  

c. Assumption 1 Revenues (from part [a]) Expenses Net income

$100 $100 $100

= = =

$15,000 $20,000 $15,000

Year 1

Year 2

Year 3

Total

$ 20,000 8,000 * $ 12,000

$ 20,000 8,000* $ 12,000

$10,000 4,000* $ 6,000

$ 50,000 20,000 $ 30,000

* Expenses = Number of units produced  $40 per unit.

Assumption 2 Revenues (from Part [b]) Expenses Net income

Year 1

Year 2

Year 3

Total

$ 15,000 6,000 * $ 9,000

$ 20,000 8,000* $ 12,000

$ 15,000 6,000* $ 9,000

$ 50,000 20,000 $ 30,000

* Expenses = Number of units delivered  $40 per unit. d. If Hydra Aire’s management is compensated based on the net income of the company, they would prefer to recognize revenues at the point of production. Why? Because it results in higher net income in year 1 and therefore in a higher bonus for the management.

P3–13 a. Cost of Error 1: If Joe McGuire requires disclosure of the lawsuit, and Nelson Repairs, Inc., does not lose the lawsuit, McGuire could incur some costs. If the president of Nelson Repairs, Inc., is serious about not wanting the lawsuit disclosed and McGuire requires that it be disclosed, Nelson could fire McGuire. In this case, McGuire would lose the audit fees of his biggest client. If these audit fees make up a substantial portion of McGuire's total revenues, it is even possible that the loss of Nelson Repairs, Inc., as a client could cause McGuire to cease operations. Cost of Error 2: If McGuire does not require disclosure of the lawsuit, and Nelson Repairs, Inc., loses the lawsuit, McGuire could incur some costs. If any of the stockholders or creditors relied on the financial statements and incurred a loss, these stockholders could sue McGuire for their losses. Since the lawsuit could force Nelson Repairs, Inc., out of business, the potential losses to stockholders and creditors could be quite substantial. A Type 2 error could also damage McGuire's reputation. Financial statement users might view McGuire as a "low quality" auditor and might be unwilling to accept financial statements auditied by McGuire. Loss of reputation might cause some potential clients to no longer be interested in hiring McGuire. Even if some 19


P3–13 Concluded new clients did hire McGuire, they might demand a lower audit fee to compensate for a "lower quality" service. Furthermore, some of McGuire's existing clients may no longer wish to engage him as their auditor. Consequently, it appears that the cost of a Type 2 error exceeds the cost of a Type 1 error. McGuire faces a tough decision, though. Although the cost of a Type 2 error appears higher, the probability that Nelson Repairs, Inc., will lose the lawsuit is not very likely. Which factor should McGuire focus on: the cost of the errors or the probability of an adverse outcome? Luckily, McGuire can refer to Statement of Financial Accounting Standards No. 5, "Accounting for Contingencies" for guidance on whether to disclose this lawsuit. =

Cost of an error  Probability of an error

Expected cost of a Type 1 error

= =

$10,000  80% $8,000

Expected cost of a Type 2 error

= =

$50,000  20% $10,000

b. Expected cost of an error

Based upon the expected cost of each type of error, it appears that Joe McGuire should disclose the lawsuit. c.

Conservatism means that "when in doubt, understate rather than overstate." This statement means that when a company faces some uncertainty concerning how to value or record an event, the company should understate, rather than overstate, the financial health of the company. In this case, McGuire has some doubt as to whether Nelson Repairs, Inc., will win or lose the pending lawsuit. McGuire's framework focuses on the relative costs of errors. He wants to determine the cost of a Type 1 error compared to the cost of a Type 2 error. Since the cost of overstatement (i.e., a Type 2 error) exceeds the cost of understatement (i.e., a Type 1 error), McGuire should risk making a Type 1 error. Hence, due to McGuire's doubt concerning the outcome of the lawsuit, he should understate the financial health of Nelson Repairs, Inc., by requiring management to disclose the lawsuit.

20


ISSUES FOR DISCUSSION ID3–1 a. Revenue recognition refers to the recording of revenues when they are earned. Matching refers to recognizing costs as expenses when the costs help generate a benefit (such as revenue). The criteria for recognizing revenue are: (1) The company must have completed a significant portion of the production and sales effort. (2) The amount of the revenues can be objectively measured. (3) The major portion of the costs have been incurred, and the remaining costs can be reasonably estimated. (4) The eventual collection of cash is reasonably assured. The FASB requirement that airlines defer a portion of the current revenues is consistent with these criteria in that at the time of selling a ticket the airlines have not completed a significant portion of the production and sales effort associated with the eventual free trip. In essence the airlines are charging more for the tickets now to cover the cost of "free" trips later. Accordingly, deferred revenue should be established for each ticket sold and then recognized as revenue when passengers use their "free" tickets. Related costs should be capitalized and matched against the revenue when it is recognized. b. As a result of implementing this new accounting policy, Continental Airlines would recognize less revenue. This would cause income to be lower than if Continental continued to use its previous policy for frequent flyer tickets. The new approach to recognizing revenue does a better job of matching revenues with expenses. Part of the overall revenue is recognized at the time when the passenger uses the frequent flyer ticket and the expense for the ticket is incurred by Continental Airlines.

ID3–2 a. Priceline’s method of booking revenue has the potential to mislead investors. It is not the same method that traditional companies in this industry use. It does not make sense from the standpoint that Priceline is reporting revenues for products and services that it does not provide. Priceline is not an airline or a hotel but yet is reporting the revenues that relate to those activities. Priceline provides a service of matching buyers and sellers (like stock brokers) and should only report revenues that relate to the service that it actually provides. b. If investors are going to value the stock of a company based on a multiple of revenue then management has an incentive to report the highest amount of revenue as possible. So by reporting these “gross bookings” as revenue Priceline is able to increase its stock price. This is particularly significant for a company that is losing a lot of cash in its operations. The most common way for a company that is losing cash from its operations is to raise money by selling stock. Typically companies that are losing money do not have the option of issuing bonds and so the only way the company can fund itself is to sell stock. A higher stock price allows the company to give up fewer shares for the needed amount of cash. c.

Allowing internet companies to record revenues differently than traditional companies has a couple of impacts, both of which are negative. One of the goals of GAAP accounting is to have financial statements comparable from one company to another. If different accounting methods are used then this is not possible. If investors are going to value the stock of a company based on a multiple of revenue then 21


management has an incentive to manipulate this number. It is much easier to manipulate revenues than net income.

ID3–3 a. Because Blockbuster is the franchiser, it can dictate policies that the franchises must follow. For example, Blockbuster could dictate when new franchises must purchase merchandise from Blockbuster and how much they must purchase. Blockbuster cannot, however, dictate when the franchises will actually generate revenue; that depends on the franchises' customers. By recognizing revenue when it ships merchandise to franchises, Blockbuster can manage earnings because it has some control over when and how much merchandise is shipped to the franchises. In the case of U.S. Robotics, it is possible to “manage” earnings by shipping some orders early or conversely delaying the shipping of orders to the next period. There is always some risk of capturing the true amount of sales since dealers have the ability to return product to U.S. Robotics. b. The criteria for recognizing revenue under the revenue recognition principle are (1) the company must have completed a significant portion of the production and sales effort, (2) the amount of the revenues can be objectively measured, (3) the major portion of the costs have been incurred, and the remaining costs can be reasonably estimated, and (4) the eventual collection of cash is reasonably assured. If a company meets these criteria when it ships merchandise, then recognizing revenue when merchandise is shipped does not violate GAAP. In fact, most manufacturers and distributors recognize revenue when they ship merchandise to their customers.

ID3–4 a. If Campbell Soup had not made the accounting change, it would have reported $626 for net income in 2003. The restated amounts are more consistent—and hence more comparable—because they are all based on the same accounting principles and methods, whereas the original amounts are based on different accounting principles and methods. b. Changes in accounting methods would be referenced in the audit opinion and the footnotes to the financial statements. In addition, the dollar amount associated with a change in accounting method would be reported separately in the income statement. c.

Based on the information provided in the initial section of the question, it seems Campbell Soup is following the “big bath theory,” which means clean-up the books by taking all the possible losses that you can. The intention is to be able to report higher net income numbers from next year onwards.

ID3–5 a.

Investors are interested in earnings that can be repeated, earnings that can be counted on in future fiscal periods. Therefore, investors would be interested in the 81 cents EPS, because future quarters will not contain any more charges related to the Pfizer acquisition.

b.

Since the analysts were expecting earnings of 79 cents per share and the (repeatable) earnings were 81 cents per share, the analysts should have been pleased. The analysts would take into consideration the one-time expenses related to Pfizer and would adjust the 74 cents EPS to 81 cents.

c.

Companies such as J & J are constantly buying and selling subsidiaries, so comparing results from quarter to quarter, or from year to year, is problematic because the company changes so much over time. Excluding one-time expenses (or gains, for that matter) is one technique analysts use to improve comparability over time.

22


ID3–6 To be able to compare financial results across companies, financial statement users would like those companies to use uniform accounting methods. If the companies do not use uniform accounting methods—as is the case with General Electric and IBM—the financial statement users would have to adjust the amounts reported in the financial statements as if both companies used the same accounting methods. For example, financial statement users would have to adjust IBM's financial statements so that they would reflect accounting numbers as if IBM had used the same depreciation method as General Electric or, alternatively, adjust General Electric's financial statements to reflect accounting numbers as if General Electric had used the same depreciation method as IBM.

ID3–7 a. Capitalizing an item simply means putting that dollar amount on the balance sheet. When WorldCom put $3.9 billion on the balance sheet, that same amount was not put on the income statement; in other words, expenses were understated by $3.9 billion since the expenditures were capitalized. b. The matching principle is the first violation. WorldCom inaccurately capitalized the expenditures in the attempt to spread the cost over future years when it should have “matched” the current expenditures with the (current) revenues derived from those expenditures. Another principle that was violated was the consistency principle, as this accounting treatment was a change from past practices by the company.

ID3–8 When Citi erroneously reported results in 2008, only to correct them in 2009, the company did not accurately reflect its financial results in the 2008 fiscal period. A reader of those financial statements—such as the U.S. Government after its bailout investment in the bank—would have seen an inaccurate picture of Citi. Correcting those mistakes in 2009 again put the reader of the financial statements in the position of not seeing the results in the correct period. Financial statements are designed to tie together, in the same time period, results and the efforts needed to get those results. The statements should “match” revenues and expenditures that are related to the activities of the fiscal period; by mismatching the 2008 costs and benefits, the bank did not accurately tell the reader of its financial statements the net result of its efforts and accomplishments in 2008—some of those results were not reported until 2009. Ideally, the financial statements should consistently attach costs in a period to the benefits that resulted in that same period. An analyst reading the financial statements of CitiBank, or a taxpayer analyzing the investment made by her government, would need to judge the results of 2008’s cost and benefits, not the results of 2008 tied to the costs of 2009.

ID3–9 a. Writedown: 246 million Euros – Recovery of prior Writedowns: 23 milliion Euros = Net Writedown: 223 million Euros b. If Unilever followed U.S. GAAP, the company would not have “written up” the inventory that had previously been written down. The writedown would have been 246 million Euros. c. IFRS is driven by a focus on market values, which allows for assets such as inventory to be both written up and written down to market value. U.S. GAAP, on the other hand, is driven by 23


conservatism and objectivity, which limits inventory adjustments to writedowns, if market values have been objectively determined to be below historic cost.

ID3–10 a. Smoothing earnings means that companies are making accounting assumptions to eliminate the fluctuations in the net income over a period of time. Many suspect that companies smooth earnings to meet targets set by Wall Street analysts; often, meeting earnings targets assures a healthy stock price. Financial services companies can manipulate their earnings by adjusting the annual “bad debt expense” (see Chapter 6 for more discussion) with higher or lower estimates of the number of loans that will be uncollectible. The charge for bad debts is one of the largest for financial services companies and is subject to management discretion for estimates. A financial services company could manipulate its net income by adjusting its annual bad debt expense to achieve its earnings goals for the year. b. Some analysts argue that smoothing earnings is an accurate way for a financial services company to show its net income over a long period of time, due to the extreme fluctuations in loan losses (due to macroeconomic conditions, for example). The financial statements could be seen as more “conservative” if the company takes a large charge for bad debts today, in anticipation of future losses. However, the counterargument is that future earnings, therefore, would be inaccurately shown as higher when the financial services company does not have to take the charge, as it has already been booked.

ID3-11 The economic entity assumption states that individual entities can be shown as distinct from their owners and all other entities and that financial results can be measured for an entity separate from all others. By requiring consolidation, FASB is attempting to force companies to accurately portray their financial condition by including all resources and obligations that belong together. In Enron’s case, shareholders and creditors could not accurately gauge the financial condition of the company because certain assets, and more importantly, certain liabilities that were the responsibility of Enron were not directly shown on the Enron balance sheet. If Enron had consolidated the entities that carried the debt, shareholders and creditors (as well as government regulators) could have better understood the financial condition and the related risks of the company. That increased level of understanding would have allowed the financial markets to better judge and price the risks associated with the company.

ID3-12 a.

KeyCorp valued its marketable securities and other equity investments at fair market value. In the case of equity and real estate investments that were not able to be valued at FMV, Key made estimates of value based on present value.

b.

Investments in privately held firms, by definition not subject to valuation on public markets, can be difficult to objectively value in terms of fair market value. Mainly subjective considerations are entered into the valuation analysis. Privately held firms do not actively turn over their ownership and therefore do not have external, objective valuations. Analysts looking into KeyCorp’s assets would need to review the subjective assumptions made when the valuations were placed on the balance sheet.

c.

If the estimation of fair value used present value as its driving determinant, then the assumptions regarding time and interest rates would need to be reviewed. By definition, present value requires the future cash flows to be determined. If those future cash flows are projected based on management assumptions, a responsible analyst would review the assumptions that underlie the valuation process.

24


ID3–13 a. The FASB is suggesting that companies use a valuation that is like net realizable value. This is the fair value of the asset minus the cost of disposal or the fair value of the liabilities plus the cost of repurchase. This is slightly different than fair market value because the FASB is saying that the cost of disposal or repurchase be reflected in the value reported on the balance sheet. b. Reporting these securities at fair market value has the impact of adding more volatility to reported income. Previously these assets and liabilities would not change in value and so there would have been no impact on the income statement. Now the companies have these additional items that will have to be included on the income statement. c.

Forcing companies to report equity and debt securities at fair value would improve the value of the balance sheet. It would give a truer representation as to the value of assets and liabilities. This policy would also increase the volatility of the reported net income of the company. Profits and losses unrelated to the operations of the company could be somewhat confusing to the users of the financial statements. Overall, the benefit of having the balance sheet reflect more current values probably outweighs any potential perceived negative impacts on the income statement. d. Under U.S. GAAP, the principle of objectivity ensures that fair market values are not used unless they can be objectively determined; also, the concept of conservatism dictates that fair market value is used only if it is below historical cost. IFRS, conversely, allows adjustments to the balance sheet values of assets for changes in market value, and these adjustments can be upward or downward.

ID3–14 Economic entity assumption: This assumption states that the financial statements report financial information about an identifiable and measurable entity that is separate and distinct from its owners and all other entities. The financial statements of Nike are for Nike, Inc. and subsidiaries. Thus, the identifiable and measurable entity is Nike and its subsidiaries. Fiscal period assumption: This assumption states that the operating life of an economic entity can be divided into arbitrary time periods. Nike has broken its operating life into fiscal years, where its fiscal year is defined as the 12 months ending on May 31. The fiscal years are reflected on the company's consolidated statements of earnings, cash flows, and stockholders' equity and on its consolidated balance sheets. Going concern assumption: This assumption states that a company's life extends beyond the current period. Nike reports assets and liabilities on its consolidated balance sheets. Because assets are defined in terms of expected future benefits and liabilities are defined in terms of probable obligations that will be settled in the future, it is necessary to assume that Nike will exist beyond the current period to derive the benefits from the assets or to pay its obligations. Stable dollar assumption: This assumption states that the U.S. dollar is used to measure economic events and that the purchasing power of a dollar is constant across time. The absence of any adjustments for inflation in Nike’s financial statements is an example of the application of this assumption. Principle of objectivity: This principle states that financial accounting information must be verifiable and reliable. An excellent example of Nike applying this principle is the company’s recording of property, plant and equipment at historic cost.

25


Matching principle: This principle states that the efforts of a given period should be matched against the benefits that result from them. Examples of Nike applying the matching principle include recording expenditures, such as prepaid expenses, as assets because the expenditures have not yet helped generate a benefit; recognizing depreciation and amortization as the company consumes a portion of the fixed assets; and recognizing operating expenses for the products that were sold. Revenue recognition principle: This principle provides guidelines for when it is acceptable for a company to recognize revenue. As disclosed in the first footnote, Nike recognizes revenue upon shipment or delivery to the customer. Principle of consistency: This principle states that companies should choose a set of accounting methods and procedures and use them from one period to the next. An example of Nike applying the principle of consistency is using similar average useful lives of its fixed assets when calculating annual depreciation. Materiality: Materiality states that only those transactions dealing with dollar amounts large enough to make a difference to financial statement users need to be accounted for in a manner consistent with the principles of financial accounting. It is difficult to identify immaterial events in a company's financial statements, because by their very nature, immaterial events would not be disclosed in a way that would make them very noticeable. However, one example of Nike applying the materiality exception in reporting items is lumping together various other assets. Conservatism: Conservatism states that, when in doubt about how to record or report an event, a company should understate assets, overstate liabilities, delay recognizing revenues or gains, and accelerate recognizing expenses or losses. According to the first footnote, Nike values its inventory at the lower of cost or market, an application of the conservatism principle of U.S. GAAP.

26


122

Chapter 5

CHAPTER 4 THE MECHANICS OF FINANCIAL ACCOUNTING BRIEF EXERCISES

BE4–1 Transaction

Assets

=

Liabilities

+ Stockholders’ Equity

Paid $5,197 to purchase property, plant and equip.

+ 5,197 - 5,197

Issued common stock for $1,105

+1,105

=

+1,105

Recorded depreciation of $4,360

-4,360

=

-4,360

Net effect

-3,255 =

-3,255

b. The transaction to purchase property, plant and equipment does not appear to affect the accounting equation. This is because both sides of the transaction affect the asset side of the balance sheet. Intel pays cash for p,p,&e; this reduces cash and increases fixed assets. All of the other transactions affect both sides of the balance sheet.

BE4–2 Transaction Repaid $15 of long-term debt

Assets ,

- 15

= Liabilities + Stockholders’ Equity

=

- 15

Paid cash dividends of $201 - 201

=

- 201

Repurchased common stock -379 for $379

=

-379

Net effect

-595

=

- 15

-580

b. Both transactions reduce assets and equity and can be viewed as alternate ways to return cash to shareholders, by either paying cash in the form of dividends or paying cash in return for shares.


BE4–3 Transaction

Assets

= Liabilities + Stockholders’ Equity

Recognized revenues of $6,426, in exchange for accounts receivable.

+6,426

=

+6,426

Paid $1,322 for sales and marketing.

-1,322

=

-1,322

Issued common stock for $3638

+363

=

+363

Purchased marketable securities for $2,317

-2,317 +2,317 =

Net effect

+5,467

_____ =

+5,467

b. The first and second transactions would be reflected on the income statement. Yahoo would show $6,426 of revenue on the income statement. Yahoo would also show sales and marketing expense of $1,322. The last three transactions would be reflected on the statement of cash flows. The first transaction does not involve cash and therefore would not directly show on the cash flow statement. Paying cash for marketing expenses would be shown in the operating section; the third transaction would be in the financing section; and the fourth transaction would be in the operating section.

EXERCISES E4–1 Assets = (1) + 30,000 (2) – 20,000 + 20,000 (3) + 9,000 (4) + 8,000 (5) – 5,500 (6) – 500 Total 41,000 Note:

Liabilities

+

Stockholders' Equity + 30,000

+9,000 + – – 9,000

8,000 5,500 500 32,000

Transactions (4), (5), and (6) are initially recorded in temporary accounts and are closed into the Retained Earnings account, which is part of stockholders' equity.


E4–2

Cash

Assets Accounts + Receivable +

Land

= Liabilities + Stockholders' Equity Notes Contributed Retained = Payable + Capital + Earnings

(1) (2) (3) (4) (5) (6) Total

+ 30,000 – 20,000 + 9,000

+30,000

Note:

Transactions (4), (5), and (6) are initially recorded in temporary accounts and are closed into the Retained Earnings account, which is part of stockholders' equity.

+20,000 + 9,000 +8,000

– –

5,500 500 13,000

+ – –

8,000

20,000

9,000

30,000

8,000 5,500 500 2,000

E4–3 X Company Income Statement For the Year Ended Revenues ............................................................................................................. Operating expenses ............................................................................................. Net income ..........................................................................................................

$ 8,000 5,500 $ 2,500

X Company Statement of Stockholders’ Equity For the Year Ended

Beginning balance Net income Dividends Owner contribution Ending balance

Contributed Capital $ 0

30,000 $ 30,000

Retained Earnings $ 0 2,500 (500) _______ $ 2,000

X Company Balance Sheet As of Assets

Liabilities and Stockholders' Equity

Cash ........................................ Accounts receivable.................. Land ........................................

$ 13,000 8,000 20,000

Total assets ...............................

$41,000

Notes payable ............................. $ 9,000 Contributed capital ..................... 30,000 Retained earnings ....................... 2,000 Total liabilities and stockholders' equity .............. $ 41,000


E4–3

Concluded X Company Statement of Cash Flows For the Year Ended Cash flows from operating activities: Cash payments for expenses .............................................

$ (5,500)

Cash flows from investing activities: Purchase of land ................................................................

(20,000)

Cash flows from financing activities: Cash contributions from owners ....................................... Proceeds from bank loan .................................................. Payments of cash dividend................................................ Net cash flow from financing activities ........................ Net increase in cash................................................................

$ 30,000 9,000 (500)

Beginning cash balance .......................................................... Ending cash balance ...............................................................

38,500 $ 13,000 0 $ 13,000

E4–4 Assets +10,000 + 8,000 – 3,000 +12,000 – 2,000 (5) – 400 (6) + 7,000 – 6,000 Total 25,600 (1) (2) (3) (4)

=

Liabilities

+ 3,000 + 10,000

+

Stockholders' Equity + 10,000 + 8,000 – 6,000

– + 13,000

400 1,000 12,600

Cathedral Enterprises Income Statement For the Year Ended Fees earned ............................................................................................................. Expenses .................................................................................................................. Gain on sale of land ................................................................................................. Net income ..............................................................................................................

$ 8,000 (6,000) 1,000 $ 3,000


E4–4

Concluded Cathedral Enterprises Statement of Stockholders’ Equity For the Year Ended

Beginning balance Net income Dividends Stockholder contribution Ending balance

Contributed Capital $ 0

10,000 $ 10,000

Retained Earnings $ 0 3,000 (400) ______ $ 2,600

Cathedral Enterprises Balance Sheet As of Assets

Liabilities and Stockholders' Equity

Cash ........................................ Receivables ............................... Land ........................................

$ 17,600 2,000 6,000

Total assets ...............................

$25,600

Misc. payable .............................. $ 3,000 Long-term note ........................... 10,000 Contributed capital ..................... 10,000 Retained earnings ....................... 2,600 Total liabilities and stockholders' equity .............. $25,600

Cathedral Enterprises Statement of Cash Flows For the Year Ended Cash flows from operating activities: Cash collected from customers ......................................... Cash paid for expenses ...................................................... Net cash increase from operating activities ................. Cash flows from investing activities: Proceeds from sale of land ................................................ Cash paid for land .............................................................. Net cash increase from investing activities .................. Cash flows from financing activities: Contributions from stockholders ...................................... Dividends paid to stockholders ......................................... Net cash increase from financing activities.................. Increase in cash ......................................................................

$

6,000 (3,000) $

$

3,000

7,000 (2,000) 5,000

$ 10,000 (400) 9,600 $ 17,600


Beginning cash balance .......................................................... Ending cash balance ...............................................................

0 $ 17,600

Note: Even though $12,000 worth of land was purchased only $2,000 is shown on this statement because the balance ($10,000) was paid for with a promise to pay cash in the future (loan). So only $2,000 of cash was used this year.

E4–5 (1) This financial event does not have accounting significance. Entries are made to record financial events that affect the company's current financial condition. In this case, the new contract will affect the company's future financial condition by affecting the dollar value of future events as the new contract is implemented. Simply signing the contract does not affect the company's current financial position. (2) This financial event does have accounting significance. The receipt of cash in exchange for issuing debt affects the company's current financial position by increasing both the amount of cash the company has and the obligations the company has to other entities. Thus, an entry is necessary, and the entry would be: Cash (+A) ........................................................................................... Bonds Payable (+L) ..................................................................... Issued bonds.

200,000 200,000

(3) This event does not have accounting significance. The retirement of an official does not influence the company's current financial position. (4) This financial event does have accounting significance. Receiving cash from a customer would change the company's current financial position. The entry would be: Cash (+A) ........................................................................................... Accounts Receivable (–A) ........................................................... Collected cash from customers.

10,000 10,000

(5) This financial event does have accounting significance. Payment of a liability will change a company's current financial position by decreasing both the amount of cash the company has and the company's obligations to other entities. The entry would be: Accrued Interest Payable (–L) ........................................................... Cash (–A)..................................................................................... Paid interest previously incurred.

1,000 1,000

(6) This financial event does not have accounting significance. Long-lived assets are reported at original cost less accumulated depreciation. Increases in market value above the reported amounts are not reported because market values on long-lived assets are not objective (i.e., are not reliable). (7) This financial event does have accounting significance. The purchase of an insurance policy represents a change in the company's financial position because the company has less cash and because the company has acquired the benefit of insurance coverage. However, the value of the policy has no influence on the company. The appropriate entry would be: Prepaid Insurance (+A) ...................................................................... Cash (–A).....................................................................................

1,500 1,500


Purchased insurance coverage. (8) This financial event does not have accounting significance. Simply placing an order does not affect a company's financial position. That is, the company has not experienced a change in the amount of cash it has, the amount it owes other entities, and so forth. The company's position does not change until it legally owns the goods.

E4–6 Account

Financial Statement

Flight Equipment Balance Sheet Passenger Revenue Income Statement Retained Earnings Balance Sheet Notes Payable Balance Sheet Interest Expense Income Statement Accounts Receivable Balance Sheet Prepaid Expenses Balance Sheet Accounts Payable Balance Sheet Common Stock Balance Sheet Fuel Expense Income Statement Other Revenues Income Statement Short-Term Investments Balance Sheet Depreciation Expense Income Statement Landing Fees Income Statement

Accounting Equation Assets Owners’ Equity Owners’ Equity Liabilities Owners’ Equity Assets Assets Liabilities Owners’ Equity Owners’ Equity Owners’ Equity Assets Owners’ Equity Owners’ Equity

E4–7 Bristol-Myers Squibb Income Statement For the Year Ended December 31, 2008 Net Sales ................................................................................. Expenses: Cost of goods sold ............................................................. Selling and adm. Expense .................................................. Advertising and product expense...................................... Research and dev. expense ............................................... Restructuring Expense....................................................... Other expenses ................................................................. Total expenses.............................................................. Net income .............................................................................

$ 20,597 $

6,396 4,792 1,550 3,585 218 901 $

17,442 3,155

Accounts payable ........................ $ Accrued payables ........................

1,535 2,936

Bristol-Myers Squibb Balance Sheet As of December 31, 2008 Assets Cash and equivalents ................ Marketable securities ...............

Liabilities and Stockholders' Equity $

7,976 289


Accounts receivable.................. Other current assets ................. Current Assets ..........................

3,710 Short-term borrowings ............... 2,788 Other current liabilities............... 14,763 Current Liabilities ...........................

154 2,085 6,710

Property, plant and equipment Other noncurrent assets...........

5,405Long-term liabilities ................................. 9,384 Stockholders’ equity ...................

10,601 12,241

Total assets ............................... E4–7 Concluded

$ 29,552

Total liabilities and stockholders’ equity

$ 29,552

Bristol-Myers Squibb Statement of Cash Flows For the Year Ended December 31, 2008 Cash flows from operating activities: Net income ......................................................... Adjustments: Total adjustments.......................................... Net cash increase (decrease) due to operating activities .......................

$

3,155 552 $ 3,707

Cash flows from investing activities: Net cash increase (decrease) due to investing activities ....................................

5,079

Cash flows from financing activities: Net cash increase (decrease) due to financing activities.................................... Change in cash balance ...........................................

$

Beginning cash balance ........................................... Ending cash balance ................................................

1,772 $ 7,976

(2,582) 6,204

The company appears to be in very good financial condition. The company is very profitable with a 15.3% net income margin ($3,155/$20,597). The company has an extremely strong balance sheet with very good liquidity; working capital is $8,053 ($14,763 - $6,710).

E4–8 a.

Ending cash

= = =

Beginning cash + Cash inflows – Cash outflows $9,000 + $133,500 – $99,500 $43,000

Note: Since Cash is an asset, cash inflows are recorded on the debit, or left-hand side of the T account, and cash outflows are recorded on the credit, or right-hand side of the T account.


E4-8 Concluded b. Miller Manufacturing Statement of Cash Flows For the Year Ended December 31, 2012 Cash flows from operating activities: Cash collections from customers ...................................... Payment of salaries ........................................................... Payment of miscellaneous expenses................................. Payment of rent................................................................. Payment of interest ........................................................... Net cash increase from operating activities ................. Cash flows from investing activities: Proceeds from sale of land ................................................ Purchase of long-term investments .................................. Purchase of equipment ..................................................... Net cash decrease from investing activities ................. Cash flows from financing activities: Proceeds from issuance of common stock ........................ Proceeds from borrowing ................................................. Payment of bank loan........................................................ Payment of dividends ........................................................ Net cash increase from financing activities.................. Increase (decrease) in cash balance .......................................

$ 95,000 (26,500) (13,000) (7,000) (3,000) $ 45,500 $

7,500 (10,000) (24,000) (26,500)

$ 15,000 16,000 (12,000) (4,000) 15,000 $ 34,000

Beginning cash balance .......................................................... Ending cash balance ...............................................................

9,000 $ 43,000

E4–9 a. (1) Cash (+A)..................................................................................... Common Stock (+SE) ............................................................. Issued common stock.

15,000

(2) Cash (+A)..................................................................................... Fees Earned (R, +SE) .............................................................. Sold services for cash.

4,000

(3) Wage Expense (E, –SE) ............................................................... Cash (–A) ...............................................................................

1,600

15,000

4,000

1,600


Incurred and paid wages.

(4) Investment in Land (+A) ............................................................. Cash (–A) ............................................................................... Purchased land as an investment.

9,000

(5) Dividends (–SE) ........................................................................... Cash (–A) ............................................................................... Declared and paid dividend.

2,000

E4–9

9,000

2,000

Continued

(6) Cash (+A)..................................................................................... Land (–A) ............................................................................... Gain on Sale of Land (Ga, +SE) .............................................. Sold land.

3,500

(7) Interest Expense (E, –SE) ............................................................ Note Payable (+L) ....................................................................... Cash (–A) ............................................................................... Made principal and interest payment.

600 900

(8) Miscellaneous Expenses (E, –SE) ................................................ Cash (–A) ............................................................................... Incurred and paid miscellaneous expenses.

1,800

3,000 500

1,500

1,800

b. B.B. (1) (2) (6)

E.B.

Cash 5,000 15,000 (3) 4,000 (4) 3,500 (5) (7) (8)

1,600 9,000 2,000 1,500 1,800

11,600

c. Small and Associates Statement of Cash Flows For the Month Ended January 31, 2012 Cash flows from operating activities: Collections from customers............................................... Payment of wages ............................................................. Payment of interest ........................................................... Payment of miscellaneous expenses.................................

$ 4,000 (1,600) (600) (1,800)


Net cash from operating activities ............................... Cash flows from investing activities: Proceeds from sale of land ................................................ Purchase of land ................................................................ Net cash decrease from investing activities ................. Cash flows from financing activities: Proceeds from issuance of stock ....................................... Repayment of note............................................................ Dividend payment ............................................................. Net cash increase from financing activities.................. Increase in cash balance ......................................................... Beginning cash balance .......................................................... Ending cash balance ...............................................................

$

0

$ 3,500 (9,000) (5,500) $15,000 (900) (2,000) $

12,100 6,600

5,000 $ 11,600


E4–10 a. Cash (1) (2) (3) (4) (5) (6) (7) (8) Total

+ 12,000 + 5,000 – 10,000 – 5,000 + 10,000 – 4,000 + 2,800 – 2,200 8,600

Assets Accounts + Receivable +

Land

= Liabilities + Stockholders' Equity Notes Contributed Retained = Payable + Capital + Earnings +12,000 + 5,000

+ 10,000 +4,000 – 4,000

3,000 7,000

5,000

12,000

– 5,000 + 14,000 – 4,000 – 200 – 2,200 2,600

Ed's Lawn Service Income Statement For the Year Ended December 31, 2012 Revenue .................................................................................. Rent expense .......................................................................... Miscellaneous expense........................................................... Loss on sale of land................................................................. Net income .............................................................................

$14,000 (5,000) (4,000) (200) $ 4,800

Ed's Lawn Service Statement of Stockholders’ Equity For the Year Ended December 31, 2012 Contributed Capital Beginning balance, January 1, 2012 $ 0 Net income Dividends Stockholder contribution 12,000 Ending balance, December 31, 2012 $ 12,000

Retained Earnings $ 0 4,800 (2,200) ______ $ 2,600

Ed's Lawn Service Balance Sheet As of December 31, 2012 Assets Cash ........................................ Accounts receivable.................. Land ........................................ Total assets ...............................

Liabilities and Stockholders' Equity $

8,600 4,000 7,000 $19,600

Notes payable ............................. $ 5,000 Contributed capital ..................... 12,000 Retained earnings ....................... 2,600 Total liabilities and stockholders' equity .............. $ 19,600


E4–10

Continued Ed's Lawn Service Statement of Cash Flows For the Year Ended December 31, 2012 Cash flows from operating activities: Cash collected from customers ......................................... Rent payments on lawn equipment .................................. Payment of miscellaneous expenses................................. Net cash increase from operating activities ................. Cash flows from investing activities: Proceeds from sale of land ................................................ Cash paid for land .............................................................. Net cash decrease from investing activities ................. Cash flows from financing activities: Stockholder contributions ................................................. Proceeds from bank loan .................................................. Dividend payments............................................................ Net cash increase from financing activities.................. Increase in cash ......................................................................

$ 10,000 (5,000) (4,000) $ $

1,000

2,800 (10,000) (7,200)

$ 12,000 5,000 (2,200)

Beginning cash balance .......................................................... Ending cash balance ...............................................................

$

14,800 8,600

$

0 8,600

b. (1) Cash (+A)..................................................................................... Contributed Capital (+SE) ...................................................... Collected cash from stockholders.

12,000

(2) Cash (+A)..................................................................................... Notes Payable (+L) ................................................................. Borrowed cash from bank.

5,000

(3) Land (+A)..................................................................................... Cash (–A) ............................................................................... Purchased land.

10,000

(4) Rent Expense (E, –SE) ................................................................. Cash (–A) ............................................................................... Incurred and paid rent expense.

5,000

(5) Cash (+A)..................................................................................... Accounts Receivable (+A) ........................................................... Fees Earned (R, +SE) .............................................................. Rendered services.

10,000 4,000

(6) Miscellaneous Expenses (E, –SE) ................................................ Cash (–A) ............................................................................... Incurred and paid miscellaneous expenses.

4,000

12,000

5,000

10,000

5,000

14,000

4,000


E4–10

Continued

(7) Cash (+A)..................................................................................... Loss on Sale of Land (Lo, –SE) ..................................................... Land (–A) ............................................................................... Sold land.

2,800 200

(8) Dividends (–SE) ........................................................................... Cash (–A) ............................................................................... Declared and paid cash dividend.

2,200

3,000

2,200

Cash B. B. 0 (1) 12,000 (3) 10,000 (2) 5,000 (4) 5,000 (5) 10,000 (6) 4,000 (7) 2,800 (8) 2,200 _____________________________________ E. B. 8,600

Accounts Receivable B. B. 0 (5) 4,000 _________________________________ E. B. 4,000

Land B. B. 0 (3) 10,000 (7) 3,000 _____________________________________ E. B. 7,000

Notes Payable B. B. 0 (2) 5,000 _________________________________ E. B. 5,000

Contributed Capital B. B. 0 (1) 12,000 _____________________________________ E. B. 12,000

Retained Earnings* B. B.

_________________________________ E. B. 2,600

Dividends B. B. 0 (8) 2,200 _____________________________________ E. B. 2,200

Fees Earned B. B. 0 (5) 14,000 _________________________________ E. B. 14,000

Rent Expense B. B. 0 (4) 5,000 _____________________________________ E. B. 5,000

Miscellaneous Expenses B. B. 0 (6) 4,000 _________________________________ E. B. 4,000

Loss on Sale of Land B.B. 0 (7) 200 _____________________________________ E.B. 200 *The Ending Balance in the Retained Earnings account is derived by the following formula:

0


Beginning Balance + Revenues – Expenses – Dividends. For a check, refer to the statement of retained earnings.


E4–10

Concluded Ed's Lawn Service Income Statement For the Year Ended December 31, 2012 Revenue ............................................................................................................ Rent expense .................................................................................................... Miscellaneous expense..................................................................................... Loss on sale of land........................................................................................... Net income .......................................................................................................

$ 14,000 (5,000) (4,000) (200) $ 4,800

Ed's Lawn Service Statement of Stockholders’ Equity For the Year Ended December 31, 2012 Contributed Retained Capital Earnings Beginning balance, January 1, 2012 $ 0 $ 0 Net income 4,800 Dividends (2,200) Stockholder contribution 12,000 _____ Ending balance, December 31, 2012 $ 12,000 $ 2,600 Ed's Lawn Service Balance Sheet As of December 31, 2012 Assets Cash ........................................ Accounts receivable.................. Land ........................................

Liabilities and Stockholders' Equity $ 8,600 4,000 7,000

Notes payable ............................. Contributed capital ..................... Retained earnings ....................... Total liabilities and Total assets ............................... $19,600 stockholders' equity .............. Ed's Lawn Service Statement of Cash Flows For the Year Ended December 31, 2012 Cash flows from operating activities: Cash collected from customers ......................................... Rent payments on lawn equipment .................................. Payment of miscellaneous expenses................................. Net cash increase from operating activities ................. Cash flows from investing activities: Proceeds from sale of land ................................................ Cash paid for land .............................................................. Net cash decrease from investing activities ................. Cash flows from financing activities: Stockholder contributions ................................................. Proceeds from bank loan .................................................. Dividend payments............................................................ Net cash increase from financing activities.................. Increase in cash ...................................................................... Beginning cash balance ..........................................................

$ 5,000 12,000 2,600 $19,600

$ 10,000 (5,000) (4,000) $ $

1,000

2,800 (10,000) (7,200)

$ 12,000 5,000 (2,200) $

14,800 8,600 0


Ending cash balance ...............................................................

$

8,600


E4–11 a. Ending cash balance

=

$8,000 + $109,500 – $90,000

=

$27,500

b. Holcomb Manufacturing Statement of Cash Flows For the Year Ended December 31, 2012 Cash flows from operating activities: Cash collections from customers ...................................... Payments for inventory ..................................................... Payment of wages ............................................................. Payment of administrative expenses ................................ Payment of interest ........................................................... Net cash increase due to operating activities ..............

$ 74,000 (34,000) (16,000) (12,000) (3,000)

Cash flows from investing activities: Proceeds from long-term investments ............................. Purchase of equipment ..................................................... Net cash increase due to investing activities ...............

$ 12,500 (11,000)

Cash flows from financing activities: Proceeds from issuance of common stock ........................ Proceeds from borrowing ................................................. Repayment of bank loan ................................................... Payment of dividends ........................................................ Net cash increase due to financing activities ............... Increase in cash balance .........................................................

E4–12 a. (1) (2) (3) (4) (5)

The entry is to record rent incurred but not yet paid. The entry is to record the expiration of a previously purchased insurance policy. The entry is to record the expiration of a portion of a fixed asset cost. The entry is to record interest revenue earned but not yet collected. The entry is to record the earning of a deferred revenue.

b. (1) (2) (3) (4) (5)

Accrual adjusting entry Cost expiration adjusting entry Cost expiration adjusting entry Accrual adjusting entry Accrual adjusting entry

E4–13 Accrual adjusting entry Operating cash flow Financing cash flow Cost expiration adjusting entry Cost expiration adjusting entry Operating cash flow

(7) (8) (9) (10) (11) (12)

9,000

$

1,500

$ 14,000 9,000 (10,000) (4,000)

Beginning cash balance .......................................................... Ending cash balance ...............................................................

(1) (2) (3) (4) (5) (6)

$

Investing cash flow Cost expiration adjusting entry Operating cash flow Cost expiration adjusting entry Operating cash flow Cost expiration adjusting entry

9,000 $ 19,500 8,000 $ 27,500


E4–14 a. 12/31/12

Wage Expense (E, –SE) ................................................ Wages Payable (+L) ................................................ Incurred, but did not pay, wages. _________________ * $42,000 = $70,000  (3 days in December ÷ 5 days total)

b. 1/2/13

Wage Expense (E, –SE) ................................................ Wages Payable (–L) ..................................................... Cash (–A) ................................................................ Paid wages.

42,000* 42,000

28,000 42,000 70,000

c. Wage expense Cash outflow associated with wages

2012 $42,000 0

2013 $28,000 70,000

Total $70,000 70,000

d. The purpose of the adjusting journal entry on December 31, 2012 is to recognize an economic event that has not yet been captured by an exchange transaction. The economic event is that the Hurst Corporation consumed the benefits of its employees' labor, and in doing so, has become obligated to its employees. Hurst Corporation will not fulfill its obligation to its employees until the subsequent period when it actually pays the employees their wages. Consequently, an accrual adjusting entry is required on December 31 to record this economic event in the correct accounting period.

E4–15 a. 12/31/09

12/31/10

12/31/11

Depreciation Expense (E, –SE)..................................... Accumulated Depreciation (–A) ......................... Depreciated equipment for 2009.

150,000

Depreciation Expense (E, –SE)..................................... Accumulated Depreciation (–A) ......................... Depreciated equipment for 2010.

150,000

Depreciation Expense (E, –SE)..................................... Accumulated Depreciation (–A) ......................... Depreciated equipment for 2011.

150,000

2009 Book value, 1/1/09 ............................................................................ 2009 Depreciation expense............................................................... Accumulated depreciation, 12/31/09 ............................................... Book value, 12/31/09 ........................................................................

150,000

150,000

150,000

$450,000 $150,000 150,000 $ 300,000


E4–15

Concluded

2010 Book value, 1/1/09 ........................................................................ 2009 Depreciation expense........................................................... 2010 Depreciation expense........................................................... Accumulated depreciation, 12/31/10 ........................................... Book value, 12/31/10 ....................................................................

$150,000 150,000

2011 Book value, 1/1/09 ........................................................................ 2009 Depreciation expense........................................................... 2010 Depreciation expense........................................................... 2011 Depreciation expense........................................................... Accumulated depreciation, 12/31/11 ........................................... Book value, 12/31/11 ....................................................................

$150,000 150,000 150,000

b. Depreciation expense Cash outflow associated with the purchase of the equipment c.

$450,000

300,000 $ 150,000

$ 450,000

$

450,000 0

2009 $ 150,000

2010 $150,000

2011 $150,000

Total $450,000

450,000

0

0

450,000

The purpose of the adjusting journal entry at the end of each period is to recognize the economic event of the portion of the fixed asset cost that expired during that year. Specifically, the purpose of the adjusting journal entries is to allocate the cost of the equipment to the periods that benefited from the equipment. Since the equipment has a useful life of three years, it is assumed that it will help generate revenues for three years. The cost of a fixed asset should be matched with the periods in which the fixed asset helps generate revenues. Consequently, an adjusting entry is required on December 31, 2009, 2010, and 2011 to allocate the economic event to the correct accounting periods.

E4–16 a. With cash-basis accounting, cash inflows and outflows are the critical events. A company will recognize revenue when it has cash inflows, and the company will recognize expenses when it has cash outflows. So in this case, Washington Forest Products would recognize the following expenses under cash-basis accounting. Insurance expense Supplies expense Rent expense

$29,000 27,000 8,000

With accrual-basis accounting, inflows and outflows of assets and liabilities are the critical events. That is, a company will recognize revenue when it has an inflow of assets or an outflow of liabilities associated with operating activities. Similarly, the company will recognize expenses when it has an outflow of assets or an inflow of liabilities associated with operating activities.


E4–16

Concluded

Consider the revenues being generated when the company is entitled to cash. The company could collect the cash at the exact same time it becomes entitled to the cash (which is an asset account), the company could expect to collect the cash after it has become entitled to the cash (which would give rise to a receivable, an asset account) or the company could become entitled to cash after it had already collected the cash (which would result in the company reducing its unearned revenue, a liability account). Just as with revenues, a company can consume benefits at three different points in time relative to the cash outflow. The company would consume the benefit at the same time it disburses cash (which is an asset account), the company could intend to disburse the cash after consuming the benefit (which would give rise to a payable, a liability account), or the company could consume a benefit for which it has already disbursed the cash (which would result in the company consuming a prepaid expense, an asset account). Because accrual-basis accounting is not based on the inflow and outflow of one asset (i.e., cash), accrual-basis accounting provides a much broader measure of revenues and expenses than provided by cash-basis accounting. Thus, in the case of Washington Forest Products, the difference between its expenses under cash-basis accounting and under accrual-basis accounting is due to expenses being defined differently under the two approaches. b. Insurance: Ending balance

= = =

Beginning balance + Insurance purchased – Insurance expense $ 0 + $29,000 – $20,000 $9,000

Since the company acquired more insurance than it used during 2011, the company expects to receive future benefits from the remaining insurance. Consequently, the company has an asset, and the appropriate account title is Prepaid Insurance. Supplies: Ending balance

= = =

Beginning balance + Supplies purchased – Supplies expense $0 + $27,000 – $11,000 $16,000

Since the company acquired more supplies than it used during 2011, the company expects to receive future benefits from the remaining supplies. Consequently, the company has an asset, and the appropriate account title is Supplies Inventory. Rent: Ending balance

= = =

Beginning balance + Cash disbursed for rent – Rent expense $0 + $8,000 – $14,000 $(6,000)

Since the company incurred more expense than it disbursed in cash for rent, the company expects to fulfill the remaining obligations in the future. Consequently, the company has a liability, and the appropriate account title would be Rent Payable.


E4–17 a. Lauren Retailing Income Statement For the Month Ended July 31 Sales ....................................................................................................................... Cost of goods sold ................................................................................................... Accrued expenses .................................................................................................... Net income ..............................................................................................................

$ 8,000 (3,700) (2,500) $ 1,800

Lauren Retailing Statement of Cash Flows from Operating Activities For the Month Ended July 31 Cash flows from operating activities: Cash collections from customers ....................................................................... Payment for inventory ....................................................................................... Net cash increase due to operating activities ...............................................

$ 7,000 (2,800) $ 4,200

b. Cash flows from operating activities are based on the inflow and outflow of cash. Net income is based on the accrual method of accounting. Under accrual accounting, revenue is recognized when it is earned, and expenses are recognized when incurred. This means that cash flows do not trigger the recognition of revenues and expenses. Rather, the inflow or outflow of operating assets, which includes more than just cash, triggers revenues and expenses. Consequently, cash flows from operating activities and net income will be equal only by coincidence. The difference between the cash flows from operating activities and net income can be explained as follows: Net income ............................................................................. Adjustments: Collections on accounts receivable ................................... Excess of cost of goods sold over cash disbursed for inventory ................................................ Expenses incurred but cash not disbursed ........................ Sales generated but cash not collected............................. Total adjustments......................................................... Net cash increase due to operating activities ........................

$ 1,800 $

2,000 900 2,500 (3,000) 2,400 $ 4,200


E4–18 a. (1) Cash (+A)..................................................................................... Common Stock (+SE) ............................................................. Issued common stock.

24,000

(2) Cash (+A)..................................................................................... Accounts Receivable (–A) ...................................................... Collected cash from customers on account.

3,900

(3) Wage Expense (E, –SE) ............................................................... Cash (–A) ............................................................................... Incurred and paid wages.

1,530

(4) Investment in Land (+A) ............................................................. Cash (–A) ............................................................................... Purchased land as an investment.

12,000

(5) Dividends (–SE) ........................................................................... Cash (–A) ............................................................................... Declared and paid dividend.

6,000

(6) Cash (+A)..................................................................................... Equipment (–A) ..................................................................... Gain on Sale of Equipment (Ga, +SE)..................................... Sold equipment.

7,000

(7) Interest Expense (E, –SE) ............................................................ Note Payable (–L) ....................................................................... Cash (–A) ............................................................................... Made interest and principal payment.

1,100 900

(8) Miscellaneous Expense (E, –SE).................................................. Cash (–A) ............................................................................... Incurred and paid miscellaneous expenses.

5,000

24,000

3,900

1,530

12,000

6,000

5,000 2,000

2,000

b. Cash Beginning balance Issued common stock Collected from customer Sold equipment

4,000 24,000 3,900 7,000

Paid wages 1,530 Purchased land 12,000 Paid dividends 6,000 Paid interest 1,100 Repaid loan 900 Paid misc. expenses 5,000 ______________________________________________________________________ Ending balance 12,370

5,000


E4–18

Concluded

c. Rahal and Watson Statement of Cash Flows For the Month Ended January 31, 2012 Cash flows from operating activities: Cash collections from customers ...................................... Payments for wages .......................................................... Payments for interest ........................................................ Payments for miscellaneous expenses.............................. Net cash increase (decrease) due to operating activities ................................................... Cash flows from investing activities: Proceeds from sale of equipment ..................................... Purchase of land ................................................................ Net cash increase (decrease) due to investing activities .................................................... Cash flows from financing activities: Proceeds from issuance of common stock ........................ Dividend payment ............................................................. Repayment of note payable .............................................. Net cash increase (decrease) due to financing activities .................................................... Increase in cash balance .........................................................

$

3,900 (1,530) (1,100) (5,000) $ (3,730)

$

7,000 (12,000) (5,000)

$ 24,000 (6,000) (900)

$

Beginning cash balance .......................................................... Ending cash balance ...............................................................

17,100 8,370

4,000 $ 12,370

E4–19 a. Peters Company Income Statement For the Years Ended December 31, 20X1 and 20X2

Sales ....................................................................... Cost of goods sold ................................................... Rent expense ........................................................... Wage expense ......................................................... Net income ..............................................................

Year 1

Year 2

$ 24,000 (5,000) (6,000) (10,000) $ 3,000

$30,000 (5,000) (6,000) (12,000) $ 7,000


E4–19

Concluded Peters Company Statement of Cash Flows For the Year Ended December 31, 20X1 Cash flows from operating activities: Cash collections from customers ...................................... Payment for wages ............................................................ Payment for rent ............................................................... Net cash increase due to operating activities ..............

$ 20,000 (6,000) (12,000) $

Net cash flows due to investing activities .............................. Cash flows from financing activities: Proceeds from stockholders.............................................. Net cash increase (decrease) due to financing activities .................................................... Increase in cash balance .........................................................

2,000 0

$ 24,000 24,000 $ 26,000

Beginning cash balance .......................................................... Ending cash balance ...............................................................

0 $ 26,000

Peters Company Statement of Cash Flows For the Year Ended December 31, 20X2 Cash flows from operating activities: Cash collections from customers ...................................... Payment for wages ............................................................ Payment for inventory ...................................................... Net cash increase due to operating activities ..............

$ 34,000 (16,000) (10,000) $

Net cash flows due to investing activities .............................. Cash flows from financing activities: Payment to stockholders................................................... Net cash increase (decrease) due to financing activities .................................................................... Decrease in cash balance .......................................................

0 $ (34,000)* (34,000) $ (26,000)

Beginning cash balance .......................................................... Ending cash balance ............................................................... * $34,000 =

$

$26,000 Cash at beginning of Year 2 + $8,000 of Year 2 cash from activities + $0 Year 2 cash from investing activities

b. Performance Measure Net income Net cash from operating activities

Year 1 $3,000 2,000

8,000

Year 2 $7,000 8,000

Total $10,000 10,000

26,000 0 operating


E4–20 a. Prior to the 5 transactions mentioned in the Exercise, Goodyear’s current ratio and debt/equity ratios are as follows: 2008 Current Ratio

= Current Assets ÷ Current Liabilities = $8,340 ÷ $4,779 = 1.75 2008 Debt/Equity Ratio = Total Liabilities ÷ Total Stockholders’ Equity = $14,204 ÷ $1,022 = 13.90 Now, let us see how each transaction would independently affect the above two ratios. Effect of Transaction on Accounts

Effect on Current Ratio

Effect on Debt/Equity Ratio

1. Increases Inventory and Accounts Payable

will go down

will go up

2. Increases Cash and Stockholders’ Equity

will go up

will go down

3. Short-term liability decreases and Long-term liability increases

will go up

will stay the same

4. Fixed assets will increase and Long-term liability will increase

will stay the same

will go up

5. Decreases Cash and Short-term debt

will go up

will go down

b. Prior to any of the 5 independent transactions mentioned in the problem, Goodyear’s current ratio is 1.75. As mentioned in (a) above, only transaction (1) will lead to a decrease in the current ratio. Entering into Transaction (1) will lower the current ratio to 1.62 ($9,340/$5,779), causing a violation of a covenant. c.

Goodyear could pay $454 for a long-term investment and still be in compliance with the covenant. The current ratio with the investment would be ($8,340 – $454)/$4,779 = 1.65.

E4–21 a. Wages Payable as of 12/31/12

=

Wages Payable as of 12/31/11

+

Wage Expense on 2012 Inc. St.

$17,000

=

X

+

$39,000

$35,000

X

=

$13,000

Prepaid Rent = as of 12/31/11

+

Cash Paid for Rent during 2012

Rent Expense on on 2012 Inc. St.

$15,000

=

$12,000

+

X

$21,000

X

=

$24,000

=

Accounts Rec. as of 12/31/11

+

Sales Revenue on 2012 Inc. St.

Cash Collected during 2012

=

$14,000

+

$45,000

$38,000

b. Prepaid Rent as of 12/31/12

c. Accounts Rec. as of 12/31/12 X

Cash Paid for Wages during 2012


X

=

$21,000

E4–22 a.

Hamilton Direct method Cash collections from customers Cash paid for inventory Cash paid for other expenses Net cash flow from operating activities Indirect method Net income Adjustments: Depreciation expense Net cash flow from operating activities

$

$

900,000 (400,000) (200,000) 300,000

Watson $

$

900,000 (400,000) (200,000) 300,000

$ 250,000

$ 200,000

50,000 $ 300,000

100,000 $ 300,000

b. Cash flows from operating activities measures all the cash inflows and cash outflows associated with a company's operating assets and liabilities. Alternatively, net income measures the inflows and outflows of operating assets and liabilities, not just the cash associated with operating assets and liabilities. Thus, cash flows from operating activities can differ from net income due to items affecting net income that do not affect cash and due to cash flows that do not affect net income. In this particular case, the difference is due to depreciation expense. Depreciation is the systematic allocation of the cost of fixed assets. Since depreciation is simply the allocation of the asset's cost, depreciation does not affect cash flows. The indirect method more clearly shows that net income is different from cash flows because this method explicitly reconciles the difference between the two.

c.

Disagree. Many people think that depreciation expense represents a fund established to finance future acquisitions of fixed assets. If this were true, it would follow that companies using accelerated depreciation methods would have more cash available than companies that use straight-line depreciation. However, as demonstrated in part (a), depreciation expense has absolutely no effect on the cash flows from operating activities. Both companies have the same cash flows, even though each company uses a different method to compute depreciation. One must remember that depreciation is simply the allocation of the net cost of fixed assets. Cash flows associated with fixed assets arise when fixed assets are acquired or sold, not when the cost of the fixed asset is allocated to expenses.


E4–23 a. 1.

2.

3.

4.

5.

6.

7.

Cash (+A) ........................................................................................ Contributed Capital (+SE) ....................................................... Owner contributed capital.

20,000

Cash (+A) ........................................................................................ Payable to Bank (+L) ............................................................... Borrowed money from the bank.

60,000

Property, Plant, & Equipment (+A) ................................................ Cash (–A) ................................................................................. Purchased long-lived assets.

25,000

Inventory (+A) Cash (–A) ................................................................................. Accounts Payable (+L) ............................................................. Purchased inventory.

40,000

Cash (+A) ........................................................................................ Accounts Receivable (+A) .............................................................. Sales (R, +SE) ........................................................................... Made sale.

20,000 60,000

Cost of Goods Sold (E, –SE) ............................................................ Inventory (–A) ......................................................................... Recognized cost of inventory sold.

25,000

Operating Expenses (E, –SE) .......................................................... Payable to Bank (–L) ...................................................................... Dividend (–SE)................................................................................ Cash (–A) ................................................................................. Made cash disbursements.

18,000 5,000 2,000

Operating Expenses (E, –SE) .......................................................... Operating Expenses Payable (+L) ............................................

15,000

Depreciation Expense (E, -SE)……………………………….. ............ 1,000 Accumulated Depreciation (-A)…………………………… ................ Incurred, but did not pay, expenses; recorded annual depreciation.

20,000

60,000

25,000

25,000 15,000

80,000

25,000

25,000

15,000

1,000


E4–23

Continued

b. Tony’s Business Income Statement For the Year Ended December 31, 2012 Sales ........................................................................................................... Cost of goods sold ..................................................................................... Operating expenses ................................................................................... Net income ................................................................................................

$ 80,000 (25,000) (34,000) $ 21,000

Tony's Business Statement of Shareholders’ Equity For the Year Ended December 31, 2012

Beginning balance:1/1/2012 Net Income Dividends Stock Issuance Ending balance: 12/31/2012

Retained Earnings $ 0 21,000 (2,000) $19,000

Contributed Capital $ 0

20,000 $20,000

Tony's Business Balance Sheet December 31, 2012 Assets Cash ....................................................... Accounts receivable .............................. Inventory ............................................... Property, plant & equipment, net.........

Total assets............................................ c. Beginning balance Owner's contribution Proceeds from bank loan Proceeds from sale

Ending balance

$

25,000 60,000 15,000 24,000

$ 124,000

Liabilities & Stockholders' Equity Accounts payable...................... $ 15,000 Operating expenses payable .. 15,000 Payable to bank ........................ 55,000 Contributed capital ................... 20,000 Retained earnings ..................... 19,000 Total liabilities and stockholders' equity ................ $ 124,000

Cash 0 20,000 Purchase of fixed assets 60,000 Purchase of inventory 20,000 Operating expenses Principal payment Dividend payment 25,000

25,000 25,000 18,000 5,000 2,000


E4–23

Continued Tony's Business Statement of Cash Flows For the Year Ended December 31, 2012

Cash from operating activities: Cash collections from sales ..................................................... Cash paid for inventory ........................................................... Cash paid for expenses ............................................................ Net cash increase (decrease) due to operating activities ..........................................................................

$ 20,000 (25,000) (18,000) $ (23,000)

Cash from investing activities: Purchase of fixed assets .......................................................... Cash from financing activities: Proceeds from owner's contribution ...................................... Proceeds from bank loan ........................................................ Principal repayment on debt ................................................... Payment of dividend ............................................................... Net cash increase (decrease) due to financing activities .......................................................................... Net increase (decrease) in cash.................................................... Beginning cash balance, January 1, 2012 ..................................... Ending cash balance, December 31, 2012....................................

(25,000)

$ 20,000 60,000 (5,000) (2,000) 73,000 $ 25,000 0 $ 25,000


E4–23

Concluded

d. Tony's Business Statement of Cash Flows For the Year Ended December 31, 2012 Cash from operating activities: Net income ................................................................ Depreciation Expense …………………………… .................. Adjustments: Increase in accounts receivable ........................... Increase in inventory............................................ Increase in accounts payable ............................... Increase in operating expense payable................ Total adjustments ........................................... Net cash increase (decrease) due to operating activities ...........................

$ 21,000 1,000 $ (60,000) (15,000) 15,000 15,000 (45,000) $ (23,000)

Cash from investing activities: Purchase of fixed assets ............................................ Cash from financing activities: Proceeds from owner's contribution ........................ Proceeds from bank loan .......................................... Principal repayment on debt ..................................... Payment of dividend ................................................. Net cash increase (decrease) due to financing activities .............................................. Net increase (decrease) in cash...................................... Beginning cash balance, January 1, 2012 ....................... Ending cash balance, December 31, 2012......................

(25,000)

$ 20,000 60,000 (5,000) (2,000) 73,000 $ 25,000 0 $ 25,000


PROBLEMS P4–1 a. (1) Equipment (+A)........................................................................... Cash (–A) ............................................................................... Purchased equipment.

150,000

(2) Wage Expense (E, –SE) ............................................................... Cash (–A) ............................................................................... Incurred and paid wages.

30,000

(3) Cash (+A)..................................................................................... Accounts Receivable (–A) ...................................................... Collected cash from customers.

15,000

(4) Cash (+A)..................................................................................... Accounts Receivable (+A) ........................................................... Fees Earned (R, +SE) .............................................................. Rendered services.

16,000 8,000

(5) Interest Expense (E, –SE) ............................................................ Note Payable (–L) ....................................................................... Cash (–A) ............................................................................... Made interest and principal payment.

10,000 40,000

(6) Advertising Expense (E, –SE)....................................................... Cash (–A) ............................................................................... Purchased advertising.

5,000

(7) Building (+A) ............................................................................... Cash (–A) ............................................................................... Long-Term Note Payable (+L) ................................................ Purchased building.

250,000

(8) Cash (+A)..................................................................................... Investments (–A) ................................................................... Gain on Sale of Investments (Ga, +SE) .................................. Sold investments.

35,000

150,000

30,000

15,000

24,000

50,000

5,000

130,000 120,000

20,000 15,000


P4–2 a. Sold services worth $28,000; received $7,000 cash and an account receivable for the balance of $21,000. Assets Increased by $28,000 and Owners’ Equity increased by $28,000 via increase in Retained Earnings. b. Purchased $6,000 worth of inventory on credit. Assets would increase and Liabilities would increase by $6,000 each. c.

Paid $2,000 cash to suppliers on previously purchased inventory. Assets and liabilities would both decrease by $2,000 each.

P4–2

Concluded

d. Purchased equipment worth $50,000 by paying $20,000 cash and signing a note payable for the balance of $30,000. Assets and Liabilities would go up by $30,000 each. e. Incurred and paid rent of $1,200 cash. Assets and owners’ equity would go down by $1,200 each. f.

Collected $5,000 cash from the customers on account of previously made credit sales. No effect on the accounting equation as assets would increase and decrease by the same amount.

g. Issued common stock for $25,000 cash. Assets and Owners’ Equity both would go up by $25,000.

P4–3 a. (1) Cash (+A)..................................................................................... Common Stock (+SE) ............................................................. Issued common stock for cash.

95,000

(2) Rent Expense (E, –SE) ................................................................. Cash (–A) ............................................................................... Incurred and paid rent ($2,600 per month for 12 months).

31,200

(3) Cash (+A)..................................................................................... Accounts Receivable (+A) ........................................................... Fees Earned (R, +SE) .............................................................. Rendered services.

65,000 125,000

(4) Land (+A)..................................................................................... Cash (–A) ............................................................................... Purchased land.

32,000

(5) Cash (+A)..................................................................................... Long-Term Note Payable (+L) ................................................ Borrowed cash.

75,000

(6) Salary Expense (E, –SE) ............................................................... Cash (–A) ............................................................................... Incurred and paid salaries.

80,000

95,000

31,200

190,000

32,000

75,000

80,000


(7) Other Expenses (E, –SE) .............................................................. Cash (–A) ............................................................................... Incurred and paid expenses.

40,000

(8) Cash (+A)..................................................................................... Accounts Receivable (–A) ...................................................... Collected cash from customers on account.

56,000

(9) Dividends (–SE) ........................................................................... Cash (–A) ............................................................................... Declared and paid dividends.

26,000

P4–3

40,000

56,000

26,000

Continued

b. Cash B.B. (1) (3) (5) (8)

0 95,000 65,000 75,000 56,000

(2) 31,200 (4) 32,000 (6) 80,000 (7) 40,000 (9) 26,000 _____________________________________ E.B. 81,800

Accounts Receivable B.B. 0 (3) 125,000 (8) 56,000 _________________________________ E.B. 69,000

Land B.B. 0 (4) 32,000 _____________________________________ E.B. 32,000

Notes Payable B.B. 0 (5) 75,000 _________________________________ E.B. 75,000

Common Stock B.B. 0 (1) 95,000 _____________________________________ E.B. 95,000

Dividends B.B. 0 (9) 26,000 _________________________________ E.B. 26,000

Fees Earned B.B. 0 (3) 190,000 _____________________________________ E.B. 190,000

Rent Expense B.B. 0 (2) 31,200 _________________________________ E.B. 31,200

Salary Expense B.B. 0 (6) 80,000 _____________________________________

Other Expense B.B. 0 (7) 40,000 _________________________________


E.B.

80,000

E.B.

40,000


P4–3

Continued

c. Hope, Inc. Income Statement For the Year Ended December 31, 2012 Fees earned ............................................................................ Operating expenses: Salary expense ................................................................... Rent expense ..................................................................... Other expenses ................................................................. Total operating expenses ............................................. Net income .............................................................................

$ 190,000 $ 80,000 31,200 40,000 151,200 $ 38,800

Hope, Inc. Statement of Stockholders’ Equity For the Year Ended December 31, 2012

Beginning Balance, January 1, 2012 Net income Dividends Stock Issuance Ending Balance, December 31, 2012

Common Stock $ 0

95,000 $ 95,000

Retained Earnings $ 0 38,800 (26,000) ______ $ 12,800

Hope, Inc. Balance Sheet As of December 31, 2012 Assets

Liabilities and Stockholders' Equity

Cash ........................................ $ 81,800 Accounts receivable.................. 69,000 Land ........................................ 32,000

Long-term notes payable ............ $ 75,000 Common stock ............................ 95,000 Retained earnings ....................... 12,800 Total liabilities and stockholders’ equity $ 182,800

Total assets ............................... $ 182,800


P4–3

Concluded Hope, Inc. Statement of Cash Flows For the Year Ended December 31, 2012 Cash flows from operating activities: Cash collections from customers ...................................... Cash paid for rent .............................................................. Cash paid for salaries......................................................... Cash paid for other expenses ............................................ Net cash decrease due to operating activities .............

$ 121,000 (31,200) (80,000) (40,000)

Cash flows from investing activities: Purchase of land ................................................................ Net cash decrease due to investing activities ..............

$ (32,000)

Cash flows from financing activities: Proceeds from stock issuance ........................................... Proceeds from debt issuance ............................................ Dividend payment ............................................................. Net cash increase due to financing activities ............... Increase in cash balance .........................................................

$ (30,200)

(32,000) $ 95,000 75,000 (26,000) 144,000 $ 81,800

Beginning cash balance .......................................................... Ending cash balance ...............................................................

0 $ 81,800

P4–4 a. (1) Cash (+A)..................................................................................... Fees Earned (R, +SE) .............................................................. Rendered services.

7,000

(2) Cash (+A)..................................................................................... Accounts Receivable (–A) ...................................................... Collected cash from customers on account.

3,000

(3) Liabilities (–L) .............................................................................. Cash (–A) ............................................................................... Made payment on outstanding liabilities.

3,000

(4) Long–Term Assets (+A) ............................................................... Note Payable (+L) .................................................................. Purchased long–lived assets.

6,000

(5) Miscellaneous Expenses (E, –SE) ................................................ Cash (–A) ............................................................................... Incurred and paid miscellaneous expenses.

4,000

(6) Dividends (–SE) ........................................................................... Cash (–A) ............................................................................... Declared and paid dividends.

800

7,000

3,000

3,000

6,000

4,000

800


P4–4

Continued

b.

a b c

Transaction

Current Ratio

Return on Equity

Debt/Equity Ratio

1. 2. 3. 4. 5. 6.

Increase No Effect Increasea No Effectb Decrease Decrease

Increase No Effect No Effect No Effect Decrease Increasec

Decrease No Effect Decrease Increase Increase Increase

Assuming that liabilities on balance sheet are current. Assuming the note payable is long-term liability in nature. Assuming that return on equity is computed after closing Dividend account to the Retained Earnings account.

c. Morrison Home Services Income Statement For the Month Ended January 31, 2012 Revenues ............................................................................................................. Miscellaneous expenses ...................................................................................... Net income ..........................................................................................................

$ 7,000 4,000 $ 3,000

Morrison Home Services Statement of Stockholders’ Equity For the Month Ended January 31, 2012

Beginning balance Net income Dividends declared Ending balance

Common Stock $ 10,000 ______ $ 10,000

Retained Earnings $ 8,000 3,000 (800) $ 10,200

Morrison Home Services Balance Sheet January 31, 2012 Assets

Liabilities and Stockholders' Equity

Cash ........................................ Receivables ............................... Long-term assets ......................

$ 12,200 1,000 16,000

Total assets ...............................

$ 29,200

*Represents nontrade notes payable.

Liabilities ..................................... $ 3,000* Notes payable ............................. 6,000 Common stock ............................ 10,000 Retained earnings ....................... 10,200 Total liabilities and stockholders’ equity .............. $ 29,200


P4–4

Concluded Morrison Home Services Statement of Cash Flows For the Month Ended January 31, 2012 Cash flows from operating activities: Cash collections from customers ...................................... Payment of expenses ........................................................ Net cash increase due to operating activities ..............

$ 10,000 (4,000) $

Cash flows from investing activities .......................................

6,000 0

Cash flows from financing activities: Repayment of liabilities ..................................................... Dividend payment ............................................................. Net cash decrease due to financing activities .............. Net increase in cash................................................................

$ (3,000)* (800) $

Beginning cash balance .......................................................... Ending cash balance ...............................................................

(3,800) 2,200

10,000 $ 12,200

*Represents nontrade notes payable. d. Morrison Home Services Statement of Cash Flows For the Month Ended January 31, 2012 Cash from operating activities: Net income ................................................................ Adjustments: Decrease in accounts receivable ..........................

$ $

3,000

3,000

Total adjustments ........................................... Net cash due to operating activities ...........................

3,000

(1) Wage Expense (E, -SE) ................................................................ Cash (-A) ................................................................................ Paid employee wages.

5

(2) Cash (+A)..................................................................................... Accounts Receivable (–A) ...................................................... Collected cash from customers on account.

10

(3) Equipment (+A)........................................................................... Cash (–A) ............................................................................... Purchased Equipment.

5

(4) Supplies (+A) ............................................................................... Cash (-A) ................................................................................

2

$ 6,000

P4-5 5

10

5

2


Purchased Supplies. (5) Accounts Payable (–L)................................................................. Cash (–A) ............................................................................... Paid Cash to suppliers.

3

(6) Interest Payable (–L) ................................................................... Cash (–A) ............................................................................... Paid interest accrued in previous period.

3

(7) Cash (+A)..................................................................................... Accounts Receivable (+A) ........................................................... Sales (R, +SE) ......................................................................... Rendered services.

9 9

(8) Long-term Note Payable (-L) ...................................................... Cash (–A) ............................................................................... Reduced long-term debt.

15

(9) Cash (+A)..................................................................................... Common Stock (+SE) ............................................................. Issued stock.

5

(10)Unearned Revenue (-L) ............................................................... Sales (R, +SE) ......................................................................... Rendered services.

3

(11)Interest Expenses (E, –SE) ........................................................... Interest Payable (+L).............................................................. Accrued interest expense.

1

(12)Depreciation Expense (E, –SE) .................................................... Accumulated Depreciation (–A) ............................................ Depreciated equipment.

4

(13)Supplies Expense (E, -SE) ............................................................ Supplies (-A) .......................................................................... Physical count of supplies.

5

(14)Dividends (-SE) ............................................................................ Cash (–A) ............................................................................... Declared and paid dividends.

3

(15)Retained Earnings (–SE) .............................................................. Dividends (+SE) ...................................................................... Closed dividends to Retained Earnings.

3

(16)Sales (-SE) .................................................................................... Wage Expense (+SE) .............................................................. Interest Expense (+SE) ........................................................... Depreciation Expense (+SE)................................................... Supplies Expense (+SE) ..........................................................

21

3

3

18

15

5

3

1

4

5

3

3

5 1 4 5


Income Summary (+SE) ......................................................... Closed sales and expenses to Income Summary.

6

(17)Income Summary (–SE) ............................................................... Retained Earnings (+SE)......................................................... Closed Income Summary to Retained Earnings.

6 6

Tybee Corporation Income Statement For the Month Ended January 31, 2012 (in millions) Sales ................................................................................................................. $ Supplies Expense ................................................................................................. Depreciation Expense .......................................................................................... Interest Expense .................................................................................................. Wage Expense ..................................................................................................... Net income ..........................................................................................................

21 5 4 1 5 6

$

Tybee Corporation Statement of Stockholders’ Equity For the Month Ended January 31, 2012 (in millions)

Beginning balance Stock issuance Net income Dividends Ending balance

Common Stock $ 20 5

$

Retained Earnings $ 12

__ 25

$

6 (3) 15

Tybee Corporation Balance Sheet January 31, 2012 (in millions) Assets Cash ........................................ $ Accounts Receivable ................. Supplies .................................... Prepaid Insurance ..................... Current Assets .......................... Equipment ............................ 55 Less: Acc. Dep. ...................... 16 Land ........................................

Liabilities and Stockholders' Equity 12 14 3 12 41 39 10

Accounts Payable ........................ $ 1 Interest Payable .......................... 1 Unearned Revenue ..................... 9 Other short-term payables ......... 4 Current Liabilities 15 Long-term Note Payable ............. 35 Common stock ............................ Retained earnings .......................

25 15


Total assets ...............................

$

90

Total liabilities and stockholders’ equity .............. $ 90

Tybee Corporation Statement of Cash Flows For the Month Ended January 31, 2012 (in millions) Cash flows from operating activities: Cash collections from customers ...................................... Cash paid to suppliers ....................................................... Cash paid for interest ........................................................ Cash paid for expenses ...................................................... Net cash increase due to operating activities ..............

$

19 (5) (3) (5) $

Cash flows from investing activities: Cash paid for equipment ................................................... Net cash decrease due to investing activities .............. Cash flows from financing activities: Repayment of liabilities ..................................................... Stock issuance ................................................................... Dividend payment ............................................................. Net cash decrease due to financing activities .............. Net change in cash .................................................................

6

(5) (5) $

(15) 5 (3)

Beginning cash balance .......................................................... Ending cash balance ...............................................................

$

(13) (12)

$

24 12

Tybee Corporation Statement of Cash Flows For the Month Ended January 31, 2012 (in millions) Cash from operating activities: Net income ................................................................ Depreciation Expense …………………………… .................. Adjustments: Decrease in accounts receivable .......................... Decrease in supplies............................................. Decrease in accounts payable .............................. Decrease in interest payable................................ Decrease in unearned revenue ............................. Total adjustments ........................................... Net cash increase (decrease) due to operating activities ...........................

$

$

1 3 (3) (2) (3) (4) $ 6

Cash flows from investing activities: Cash paid for equipment ................................................... Net cash decrease due to investing activities .............. Cash flows from financing activities: Repayment of liabilities .....................................................

6 4

(5) (5) $

(15)


Stock issuance ................................................................... Dividend payment ............................................................. Net cash decrease due to financing activities .............. Net change in cash .................................................................

5 (3)

Beginning cash balance .......................................................... Ending cash balance ...............................................................

$

(13) (12)

$

24 12

P4–6 Accounts Direction (1) (2) (3) (4) (5) (6) (7) (8)

Net Income

Cash

+

Contributed Capital

+

Inventory Accounts Payable Accounts Receivable Fees Earned Cash Accounts Receivable Equipment Cash

+ + + + + – + –

Wages Payable Cash Cash Fees Earned

Cash Long-Term Note Payable (9) Cash Interest Expense (10) Cash Land Gain on Sales of Land

Net Operating Cash Flow

NE

NE

NE

NE

+

NE

NE NE

+ NE

– –

NE

+ +

+

+

NE

NE

+

NE

– – – + + – +


P4–7 (1) (2) (3) (4) (5) (6) (7)

Classification

Assets

Liabilities

A C A C C C C

NE – + NE – – NE

+ NE NE – NE NE –

Stockholders' Equity – – + + – – +

Revenues

Expenses

NE NE + + NE NE +

+ + NE NE + + NE

P4–8 a. Supplies Expense (E, –SE) .................................................................. Supplies Inventory (–A) .............................................................. Adjusted for supplies used.

55,000

b. Rent Expense (E, –SE) ........................................................................ Rent Payable (+L) ........................................................................ Incurred, but did not pay, rent.

2,400

c.

Unearned Service Revenues (–L) ....................................................... Fees Earned (R, +SE) ................................................................... Rendered services for cash collected in advance.

12,000

d. Depreciation Expense (E, –SE) ........................................................... Accumulated Depreciation (–A) ................................................. Depreciated fixed assets for 2011.

50,000

e. Interest Expense (E, –SE) ................................................................... Interest Payable (+L) ................................................................... Incurred, but did not pay, interest.

600

f.

55,000

2,400

12,000

50,000

600

Advertising Expense (E, –SE) ............................................................. Advertising Payable (+L) ............................................................. Incurred, but did not pay, advertising.

28,000

g. Insurance Expense (E, –SE) ................................................................ Prepaid Insurance (–A) ............................................................... Adjusted for expiration of prepaid insurance.

175

28,000

175

P4–9 (1) Rent Expense (E, –SE) ........................................................................ Prepaid Rent (–A) ....................................................................... Adjusted for expiration of prepaid rent.

2,700

(2) Insurance Expense (E, –SE) ................................................................ Prepaid Insurance (–A) ............................................................... Adjusted for expiration of prepaid insurance.

700

(3) Depreciation Expense (E, –SE) ........................................................... Accumulated Depreciation (–A) ................................................. Depreciated fixed assets.

2,400

2,700

700

2,400


P4–9

Concluded

(4) Salary Expense (E, –SE) ...................................................................... Salaries Payable (+L) ................................................................... Incurred, but did not pay, salaries.

1,200

(5) Unearned Revenues (–L) ................................................................... Fees Earned (R, +SE) ................................................................... Rendered services for cash collected in advance.

200

1,200

200

P4–10 Beginning payable Interest expense Interest paid Ending payable

Case 1 $ 400 2,800 (3,000) $ X

Case 2 $ 800 2,800 X $ 300

Case 3 $ 400 2,800 (2,300) X

Case 4 $ X 2,800 (2,600) $ 200

Case 5 $ 200 2,800 X $ 400

Case 6 $ X 2,800 (2,500) $ 0

X equals

$

$ (3,300)

$

$

$ (2,600)

$

200

900

0

(300)

P4–11 a. (1) Cash (+A)..................................................................................... Unearned Insurance Premium (+L) ....................................... Collected insurance premium in advance.

240,000

(2) Unearned Insurance Premium (–L) ............................................ Premium Revenue (R, +SE) .................................................... Earned insurance premium.

70,000*

240,000

70,000

* $70,000 = ($240,000 premium ÷ 24 month total coverage) x 7 months expired (3) The purpose of the adjusting journal entry is to recognize an economic event not captured on a timely basis by a transaction. That is, the adjusting entry is to record the fact that Prustate fulfilled part of its obligation to Jacobs by providing insurance coverage to Jacobs, and in doing so, Prustate earned part of the $240,000 it collected from Jacobs on May 31, 2011. The journal entry results in Prustate allocating the revenue to the period in which it earned the revenue. (4) Insurance revenue Cash receipts

$

2011 70,000 240,000

2012 $120,000 0

2013 $50,000 0

b. (1) Prepaid Insurance (+A) ............................................................... Cash (–A) ............................................................................... Paid insurance in advance.

240,000

(2) Insurance Expense (E, –SE) ......................................................... Prepaid Insurance (–A) .......................................................... Adjusted for expiration of prepaid insurance.

70,000

Total $240,000 240,000

240,000

70,000


P4–11 (3)

Concluded The purpose of the adjusting journal entry is to recognize an economic event not captured on a timely basis by a transaction. That is, the adjusting entry is to record the fact that Jacobs Printing Corporation consumed a portion of one of its assets (i.e., prepaid insurance), and in doing so, Jacobs Printing incurred an expense. The journal entry results in Jacobs Printing allocating the cost of the insurance policy to those periods for which Jacobs Printing received coverage from the policy.

(4) Insurance expense Cash payment

2011 $ 70,000 240,000

2012 $120,000 0

2013 $50,000 0

Total $240,000 240,000

P4–12 Net Income NE NE NE + NE

(1) (2) (3) (4) (5)

Current Ratio + – – + – +b NE –

Working Capital + – – + –

Debt/Equity Ratio – NE – –a +

(6) NE NE – (7) NE NE NE (8) – – +c __________________ a Transaction (4) increases revenues, thereby increasing the net income amount closed into Retained

b

c

Earnings. Since increasing Retained Earnings increases stockholders' equity, Transaction (4) decreases the debt/equity ratio. Note that the inventory was sold above cost. Transaction (6) reduces cash and accounts payable by the same amount. Reducing the numerator and the denominator by the same amount has the impact of increasing the ratio ($11,872/$6,769 becomes $11,672/$6,569). Therefore this transaction has the impact of increasing the current ratio. Transaction (8) increases expenses, thereby decreasing the net income amount closed into Retained Earnings. Since reducing Retained Earnings reduces stockholders' equity, Transaction (8) increases the debt/equity ratio.

P4–13 a. Current ratio

= = =

Current assets ÷ Current liabilities ($10,000 + $40,000) ÷ $20,000 2.50

Debt/equity

= = =

Total liabilities ÷ Total stockholders’ equity ($20,000 + $20,000) ÷ ($30,000 + $50,000) .50

Book value

= = =

Net assets ÷ Number of shares outstanding $80,000 ÷ 12,000 6.67


P4–13

Concluded

b. Financial Alternative 1. Stock issuance 2. Long-term note

Current Ratio 2.50 2.50

Debt/Equity Ratio .36 .88

Book Value 7.33 6.67

3. Open account

1.00

.88

6.67

c.

Stock Issuance: Issuing stock is attractive because it improves the company’s debt/equity ratio. By decreasing this ratio, the company might become more attractive to creditors in the future, thereby allowing the company to borrow money at more favorable rates. Further, issuing stock, as opposed to issuing debt, will not hurt future cash flows. Stock does not require mandatory interest payments, and it never matures. On the negative side, issuing stock has the potential to dilute the ownership rights of existing stockholders. Long-Term Note: Issuing debt could place severe cash restrictions on the company. Each year the note is outstanding the company will incur $3,600 in interest charges. Also, the company will have to pay back the $30,000 when the note matures. If the company is unable to meet any of these obligations, it could be forced into bankruptcy. On the positive side, interest on the loan is deductible for tax purposes, so issuing the note would decrease the company’s cash outflows from income taxes. Further, with a long-term note, as opposed to a short-term note, the company has more time in which to generate cash to pay off the obligation. Open Account: Purchasing the equipment on account would allow the company to avoid interest charges. However, the company would be placed in a severe financial bind by purchasing the equipment on account. The $30,000 must be repaid within 30 days, and the company currently has only $10,000 in cash. The company would have to raise an additional $20,000 either through operations or by selling some of its assets. Either solution may not be desirable. Diverting money from operations to pay off the obligation means that the company would not be able to plow as much of its earnings back into the business. If the company reduces the amount reinvested in the business, it could suffer long-term negative consequences. Using money raised by selling assets may cause the company to hurt its base of operations, thereby hurting future operations. Of the three options, issuing stock appears to be the most attractive.

d. The balance in Retained Earnings represents the excess of cumulative net income over cumulative net losses and declared dividends. Retained Earnings does not represent cash (or any other particular asset). Retained Earnings is simply a measure of one source of capital. In other words, Retained Earnings is an accounting concept and does not indicate a company’s financial wherewithal to purchase items or declare dividends. Items are purchased and dividends are paid with assets (usually cash). Since the company has only $10,000 in cash, it would have to use other assets to purchase the equipment and pay a dividend. In doing so, the company would hurt its future base of operations.


P4–14 Return on Equity

Current Ratio

Debt/ Equity

1. Purchase inventory on account.

NE

+

+

2. Sell land for cash at a gain. 3. Provide services to customers, receiving cash in return. 4. Make a principal payment on an outstanding long-term liability.

+

+

+

+

NE

5. Issue common stock for cash.

+

Return on Sales

Current Ratio

Debt/ Equity

NE

NE

NE

NE

+

– NE

– –

+ +

NE

+

Return on Assets

Current Ratio

NE

+

+

+

+

+

P4–15

1. Purchase equipment for cash. 2. Purchase machinery in exchange for a long-term note payable. 3. Pay salaries, which have not been accrued, to employees. 4. Declare a dividend. 5. Issue common stock to satisfy a current obligation.

P4–16

1. Purchase equipment in exchange for a note payable. 2. Pay cash for marketing its services. 3. Sell equipment for an amount less than its book value. 4. Pay wages that were accrued in a previous period 5. Provide a service for which cash was collected in a previous period


P4–17 a. All T-accounts for P4−17 appear in this section. Transactions are keyed to numbers in parentheses, adjusting journal entries are keyed to lowercase letters, and closing entries are keyed to upper case letters. Cash B. B. (1a) (5) (7) (9)

170,000 350,000 850,000 37,000 120,000

E. B.

9,000

Accounts Receivable (3) (6) (8) (11) (12)

400,000 870,000 148,000 50,000 50,000

B. B. (1a)

188,000 1,350,000

E. B.

616,000

Interest Receivable B. B.

1,620

E. B.

1,620

B. B (10)

200,000 820,000

E. B.

320,000

E. B.

B. B. 44,400

B. B. (4)

72,000

E. B.

40,000 110,000

25,000

E. B.

25,000

40,000

B. B. (12)

160,000 50,000

E. B.

180,000

E. B.

29,600

B. B.

480,000

B. B. (g)

98,000 48,000

B. B.

950,000

E. B.

480,000

E. B.

146,000

E. B.

950,000

Equipment

Accum. Depr.—Equipment

Accum. Depr.—Machinery B. B.

B. B.

47,500

E. B.

277,500

Wages Payable

(g) E. B.

30,000

Accounts Payable

75,000 (6)

(g)

(7)

Machinery

Patent 230,000

110,000

Long-Term Investments

0

(c)

700,000

Supplies Inventory

0 72,000

Prepaid Advertising

74,000 (a)

(3)

(1b)

(b)

Prepaid Insurance B. B.

850,000 72,000

Notes Receivable

0

(f)

(5) (10)

Merchandise Inventory B. B. (2)

870,000

B. B. (2)

220,000 820,000

E. B.

170,000

12,500

62,500

Op. Exp. Payable

Interest Payable

B. B.

73,000

B. B.

0

B. B.

0

(e)

43,000

(d)

6,000

(h)

3,361

E. B.

43,000

E. B.

6,000

E. B.

3,361

73,000

Short-Term Notes Payable

Mortgage Payable

Bonds Payable

B. B. (4)

0 110,000

B. B.

300,000

B. B.

500,000

E. B.

110,000

E. B.

300,000

E. B.

500,000


P4–17

Continued Common Stock B. B. (9)

Retained Earnings 500,000 120,000 (C)

E. B.

620,000

B. B. (1a)

0 1,700,000

50,000

Sales

Dividends

B. B.

416,000

(B)

243,859

E. B.

609,859

E. B.

0

E .B.

B. B. (3) (e)

0

(A) E. B.

E. B.

B. B. (8)

0

E. B.

B. B. (g)

E. B.

(c) (d) (A)

25,000 36,000 87,000

(a)

(A) 0

0 47,500

48,000

0

(A) E. B.

Interest Expense B. B. (h)

44,400

Depr. Expense—Machinery B. B. (g)

(A)

0 44,400

E. B.

0 48,000

E. B.

0 12,500

0

Insurance Expense B. B.

0

110,000

0

(A)

0

370,000

Depr. Expense—Equipment

0 110,000

Amortization Expense B. B. (g)

0 148,000

E. B.

(A)

0

0

42,000

Supplies Expense B. B. (b)

E. B.

7,000

Miscellaneous Op. Expense

0 36,000 6,000 (A)

0 7,000

0 327,000 43,000

700,000

Operating Expense B. B. (d) (d)

1,620 (A)

B. B. (7)

Wage Expense

0 700,000 (A)

0

0

1,620

Cost of Goods Sold

E. B.

E. B.

50,000

Gain on Sale of Investment

B. B. (f)

(A)

0 50,000 (C)

Interest Revenue

(A) 1,700,000

B. B. (1b)

B. B. (11)

47,500

0

Income Summary

0 3,361

12,500

(A) E. B.

3,361

(B)

243,859

(A)

243,859

0

b. Entries are posted to the T-accounts in Part (a). (1a) Cash (+A)..................................................................................... Accounts Receivable (+A) ...................................................... Sales (R, +SE) .................................................................... Made sales.

350,000 1,350,000

(1b) Cost of Goods Sold (E, –SE)......................................................... Merchandise Inventory (–A)............................................. Recorded cost of inventory sold.

700,000

1,700,000

700,000


P4–17 (2)

Continued Merchandise Inventory (+A).................................................. Accounts Payable (+L) ...................................................... Purchased inventory on account.

820,000

Wage Expense (E, –SE) .......................................................... Wages Payable (–L) ............................................................... Cash (–A) .......................................................................... Paid wages.

327,000 73,000

Supplies Inventory (+A) ......................................................... Short–Term Notes Payable (+L)........................................ Purchased supplies by note.

110,000

Cash (+A) ............................................................................... Accounts Receivable (–A) ................................................. Collected cash from customers.

850,000

Accounts Payable (–L) ........................................................... Cash (–A) .......................................................................... Made payment to suppliers.

870,000

Cash (+A) ............................................................................... Long–Term Investments (–A) ........................................... Gain on Sale of Investment (Ga, +SE) ............................... Sold investment.

37,000

Miscellaneous Operating Expense (E, –SE) ........................... Cash (–A) .......................................................................... Incurred and paid expenses.

148,000

Cash (+A) ............................................................................... Common Stock (+SE) ........................................................ Issued common stock.

120,000

(10) Notes Receivable (+A) ................................................................ Accounts Receivable (–A) ................................................. Accepted a note in payment of an open account.

72,000

(11) Dividends (–SE) ........................................................................... Cash (–A) .......................................................................... Declared and paid a cash dividend.

50,000

(12) Long–Term Investments (+A) ..................................................... Cash (–A)......................................................................... Purchased investments.

50,000

(3)

(4)

(5)

(6)

(7)

(8)

(9)

820,000

400,000

110,000

850,000

870,000

30,000 7,000

148,000

120,000

72,000

50,000

50,000


122

P4–17

Chapter 5

Continued

c. Account Cash Accounts Rec. Interest Rec. Merchandise Inv. Supplies Inventory Prepaid Insurance Prepaid Advert. Notes Receivable L-T Investments Equipment Accum. Depr.—Equip. Machinery Accum. Depr.—Mach. Patent Accounts Payable Wages Payable Interest Payable Oper. Exp. Payable S-T Notes Payable Mortgage Payable Bonds Payable Common Stock Retained Earnings Dividends Sales Interest Revenue Gain on Investment Cost of Goods Sold Wage Expense Operating Exp. Miscellaneous Op.Exp. Insurance Exp. Supplies Exp. Interest Exp. Amortization Exp. Depr. Exp.—Equip. Depr. Exp.—Mach. Income Summary Total

Unadjusted Trial Balance Dr.

Adjusting Entries Dr.

Cr.

Cr.

9,000 616,000 (f)

1,620

320,000 150,000 74,000

(b) 110,000 (a) 44,400 (c)

25,000

72,000 180,000 480,000 98,000

(g)

Adjusted Trial Balance Dr. Cr.

Cr.

230,000

(g) (g)

47,500 12,500

146,000

146,000

(e) (h) (d)

43,000 3,361 6,000

277,500

277,500 62,500

170,000 43,000 3,361 6,000 110,000 300,000 500,000 620,000 416,000

110,000 300,000 500,000 620,000 416,000 50,000

950,000

62,500

170,000

(C)

50,000

50,000 1,700,000 (f)

1,700,000 1,620 7,000

1,620

7,000 700,000 327,000

(e) (d)

43,000 42,000

(a) (b) (h) (g) (g) (g)

44,400 110,000 3,361 12,500 48,000 47,500

148,000

(c&d) 61,000

377,381

377,381

700,000 370,000 42,000 87,000 44,400 110,000 3,361 12,500 48,000 47,500 4,300,481

4,300,481

(B) (C)

243,859 50,000

(A) (A) (A) (A) (A) (A) (A) (A) (A) (A) (A)

700,000 370,000 42,000 87,000 44,400 110,000 3,361 12,500 48,000 47,500 243,859 2,002,479

170,000 43,000 3,361 6,000 110,000 300,000 500,000 620,000 609,859

(A) 1,700,000 (A) 1,620 (A) 7,000

(B) 4,151,000

Cr.

9,000 616,000 1,620 320,000 40,000 29,600 25,000 72,000 180,000 480,000

950,000

75,000

4,151,000

Final Trial Balance Dr.

9,000 616,000 1,620 320,000 40,000 29,600 25,000 72,000 180,000 480,000

48,000

950,000

Closing Entries Dr.

243,859 2,002,479

2,785,720

2,785,720


Chapter 5

P4–17

53

Continued

d. Entries are posted to the T–accounts in Part (a). (a) Insurance Expense (E, –SE) ......................................................... Prepaid Insurance (–A) .......................................................... Adjusted for expiration of prepaid insurance.

44,400 44,400

(b) Supplies Expense (E, –SE) ........................................................... Supplies Inventory (–A) ......................................................... Adjusted for supplies used.

110,000

(c) Prepaid Advertising (+A) ............................................................. Miscellaneous Operating Expense (E, –SE) ........................... Recognized advertising to be used in subsequent periods.

25,000

(d) Operating Expense (E, –SE)......................................................... Operating Expense (E, –SE)......................................................... Miscellaneous Operating Expense (E, –SE) ........................... Op. Exp. Payable (+L) ............................................................. Incurred, but did not pay, and reclassify.

6,000* 36,000

110,000

25,000

36,000* 6,000

* $6,000 = ($3,500 per month  12 months) – $36,000 misc. exp. payment (e) Wage Expense (E, –SE) ............................................................... Wages Payable (+L) ............................................................... Incurred, but did not pay, wages.

43,000

(f) Interest Receivable (+A) ............................................................. Interest Revenue (R, +SE) ...................................................... Earned, but did not collect, interest.

1,620*

43,000

1,620

* $1,620 = $72,000  9%  3/12 (g) Depreciation Expense—Equipment (E, –SE)............................... Depreciation Expense—Machinery (E, –SE) ............................... Amortization Expense (E, –SE).................................................... Accumulated Depreciation—Equipment (–A) ....................... Accumulated Depreciation—Machinery (–A) ....................... Patent (–A) ............................................................................ Amortized fixed and intangible assets.

48,000 47,500 12,500

(h) Interest Expense (E, –SE) ............................................................ Interest Payable (+L).............................................................. Incurred, but did not pay, interest.

3,361*

* $3,361 = $110,000  10%  110/360

48,000 47,500 12,500

3,361


P4–17

Continued

e. Closing entries are posted to the T–accounts in Part (a).

a.

(A) Sales ........................................................................................... Interest Revenue ....................................................................... Gain on Sale of Investment ........................................................ Income Summary .................................................................. Cost of Goods Sold ............................................................... Wage Expense ...................................................................... Operating Expense ................................................................ Misc. Operating Expense ...................................................... Insurance Expense ................................................................ Supplies Expense .................................................................. Interest Expense ................................................................... Amortization Expense ........................................................... Depreciation Expense—Equipment ..................................... Depreciation Expense—Machinery ...................................... Closed revenues and expenses into Income Summary.

1,700,000 1,620 7,000

(B) Income Summary........................................................................ Retained Earnings.................................................................. Closed Income Summary into Retained Earnings.

243,859

(C) Retained Earnings ...................................................................... Dividends .............................................................................. Closed Dividends into Retained Earnings.

50,000

243,859 700,000 370,000 42,000 87,000 44,400 110,000 3,361 12,500 48,000 47,500

243,859

50,000

J.D.F. Company Income Statement For the Year Ended December 31, 2012 Revenues: Sales............................................................... Interest revenue ............................................ Gain on sale of investment ............................ Total revenues .......................................... Operating expenses: Cost of goods sold ......................................... $700,000 Wage expense ............................................... 370,000 Operating expense ........................................ 42,000 Miscellaneous operating expense ................. 87,000 Insurance expense......................................... 44,400 Supplies expense ........................................... 110,000 Amortization expense ................................... 12,500 Depreciation expense—Equipment .............. 48,000 Depreciation expense—Machinery ............... 47,500 Total operating expenses ......................... Interest expense ................................................. Total expenses .................................................... Net income .........................................................

$

1,700,000 1,620 7,000 $1,708,620

$

1,461,400 3,361 $

1,464,761 243,859


P4–17

Continued J.D.F. Company Statement of Retained Earnings For the Year Ended December 31, 2012 Retained earnings balance, January 1, 2012 ....................................................... Plus: Net income.................................................................................................. Less: Dividends declared ..................................................................................... Retained earnings balance, December 31, 2012 .................................................

$ 416,000 243,859 (50,000) $ 609,859

J.D.F. Company Balance Sheet December 31, 2012 Assets Current assets: Cash ............................................................... Accounts receivable ...................................... Interest receivable......................................... Inventory ....................................................... Supplies inventory ......................................... Prepaid insurance .......................................... Prepaid advertising expenses ........................ Notes receivable ............................................ Total current assets .................................. Long-term investments ...................................... Fixed assets: Equipment ..................................................... Less: Accumulated depreciation ................... Net book value of equipment ....................... Machinery...................................................... Less: Accumulated depreciation ................... Net book value of machinery ........................ Total fixed assets ...................................... Patent ................................................................. Total assets ......................................................... Liabilities & Stockholders' Equity Current liabilities: Accounts payable .......................................... Wages payable .............................................. Interest payable............................................. Operating expense payable ........................... Short-term notes payable ............................. Total current liabilities ............................. Long-term liabilities: Mortgage payable ......................................... Bond payable ................................................. Total long-term liabilities .........................

$

9,000 616,000 1,620 320,000 40,000 29,600 25,000 72,000 $1,113,220 180,000

$ 480,000 146,000 $ 334,000 $ 950,000 277,500 672,500 1,006,500 62,500 $ 2,362,220

$ 170,000 43,000 3,361 6,000 110,000 $

332,361

$ 300,000 500,000 800,000


Common stock .................................................... Retained earnings ............................................... Total liabilities & stockholders' equity ...............

$

620,000 609,859 2,362,220


P4–17

Continued J.D.F. Company Statement of Cash Flows (Direct Method) For the Year Ended December 31, 2012 Cash flows from operating activities: Collections from customers on cash sales ........................ Collection from customers on open accounts .................. Payments for wages .......................................................... Payments to suppliers on open account ........................... Payment for misc. oper. exp ............................................. Net cash increase (decrease) due to operating activities ................................................... Cash flows from investing activities: Proceeds from sale of investment..................................... Purchase of investment ..................................................... Net cash increase (decrease) due to investing activities .................................................... Cash flows from financing activities: Proceeds from issuance of common stock ........................ Payment of dividends ........................................................ Net cash increase (decrease) due to financing activities .................................................... Decrease in cash balance ....................................................... Beginning cash balance .......................................................... Ending cash balance ...............................................................

$ 350,000 850,000 (400,000) (870,000) (148,000) $ (218,000) $

37,000 (50,000) (13,000)

$ 120,000 (50,000) 70,000 $(161,000) 170,000 $ 9,000


P4–17 b.

Concluded J.D.F. Company Statement of Cash Flows (Indirect Method) For the Year Ended December 31, 2012 Cash flows from operating activities: Net income ........................................................... Adjustments: Depreciation expense—Equipment ................ Depreciation expense—Machinery ................. Amortization expense ..................................... Gain on sale of investments ............................ Increase in accounts receivable ...................... Increase in interest receivable ........................ Increase in merchandise inventory ................. Increase in prepaid advertising ....................... Increase in notes receivable ............................ Decrease in prepaid insurance ........................ Decrease in accounts payable ......................... Decrease in wages payable ............................. Increase in operating exp. payable ................. Increase in short-term notes payable ............. Increase in interest payable ............................ Total adjustments ...................................... Net cash increase (decrease) due to operating activities.....................................

$ 243,859 $ 48,000 47,500 12,500 (7,000) (428,000) (1,620) (120,000) (25,000) (72,000) 44,400 (50,000) (30,000) 6,000 110,000 3,361

Cash flows from investing activities: Proceeds from sale of investment........................ Purchase of investment ........................................ Net cash increase (decrease) due to investing activities ...................................... Cash flows from financing activities: Proceeds from issuance of common stock ........... Payment of dividends ........................................... Net cash increase (decrease) due to financing activities...................................... Decrease in cash balance .......................................... Beginning cash balance .......................................................... Ending cash balance ...............................................................

(461,859) $ (218,000) $ 37,000 (50,000) (13,000) $ 120,000 (50,000) 70,000 $ (161,000) $

170,000 9,000


P4–18 For 2012 the net cash flow from operations is $62,400 which is composed of: (1) cash collections from services rendered, and (2) cash payments due to operating activities. Let us compute the cash collections from services rendered for 2012: Cash Collections from Services Rendered for 2012

$54,000

=

=

Revenue from Services per Income Statement in 2012

$54,700 + $600

+

Decrease in Accounts Receivable

Increase in Accounts Receivable

+

Increase in Unearned Revenue

Decrease in Unearned Revenue

$1,300

Now, we can compute the cash payments due to operating activities: Net Cash Flow from Operations during 2012

=

Cash Collections from Services Rendered during 2012

+

Cash Payment Due to Operating Activities during 2012

$62,400

=

$54,000

+

X

X

=

$8,400

P4–19 In order to answer this question, we need to know how much total revenue is generated by Mayberry from each of its 2 sources: (1) advertising display sales; and (2) consulting services. We are given the total revenues as reported on the Income Statement, and selected activity of cash receipts and changes in Accounts Receivable account for the advertising display sales. First, based on the following relationships, we will compute the revenue generated by advertising display sales for 2012 and 2011. Cash Collections from Advertising Display Sales during the Year

=

Advertising Display Sales Revenue as Reported on the Income St.

+ –

Decrease in Acc. Rec. Increase in Acc. Rec.

For 2011 $41,500

=

X + $2,800

X

=

$38,700

$43,500

=

X + $2,700

X

=

$40,800

For 2012

Based on the above calculations we know that total revenue generated from advertising display sales during 2011 and 2012, respectively, was $38,700 and $40,800.


P4–19

Concluded

Second, based on the above information and the total revenue as reported in the 2011 and 2012 income statements, we can compute the amount of revenue generated by Mayberry Enterprises from its consulting services. For 2011 $76,000

– $38,700

=

$37,300

– $40,800

=

$48,700

For 2012 $89,500

Overall, the revenue from each segment can be broken down as follows.

2011 2012 Total

Display Sales $ 38,700 40,800 $ 79,500

Consulting Service $37,300 48,700 $ 86,000

Total $ 76,000 89,500 $ 165,500

Based on the above, it seems that revenue stream is growing for both product lines. However, the growth is higher in the consulting service product line.

P4–20 Cash Account ..................................................................................... Accumulated Depreciation ................................................................ Equipment .................................................................................. Gain on Sale of Equipment ......................................................... a b

23,400 6,600a 24,400b 5,600

$24,300 + $8,700 – $26,400 = $6,600 $84,800 + $37,000 – $97,400 = $24,400

It is true that Badger’s net income has more than tripled from 2011 to 2012. However, this increase is entirely due to selling the equipment with a book value of $17,800 ($24,400 – $6,600) for $23,400. This has resulted in a gain of $5,600. Since net income is $5,200, it appears that Badger has lost money from its regular, recurring operations. I am not sure if we should extend any loans to Badger, let alone grant the preferential interest rates.


ISSUES FOR DISCUSSION ID4–1 a. The amount at which MCI would report the SBS system on its balance sheet is the amount MCI gave up to receive it. Since MCI "paid" $376 million in stock and took on a note for $104 million (with the proceeds going to IBM), MCI would report the SBS system at $428 million (i.e., $480 million less $52 million of miscellaneous assets received). b. The exchange increased assets by $480 million, liabilities by $104 million, and stockholders' equity by $376 million. The accounting equation remained in balance, as it should for all transactions. c.

MCI's balance sheet would show new assets composed of $52 million in miscellaneous assets and $428 million ($480 million – $52 million) for SBS. Its liabilities now include a note payable for $104 million. MCI's stockholders' equity would increase by $376 million for the common stock MCI issued to IBM.

d.

Account Miscellaneous Assets (asset) Investment in SBS (asset) Notes Payable (liability) Common Stock (equity)

Effect Increase Increase Increase Increase

Dollar Value $ 52 million 428 million 104 million 376 million

e. Miscellaneous Assets (+A) ................................................................. Investment in SBS (+A) ...................................................................... Notes Payable (+L) ...................................................................... Common Stock (+SE) .................................................................. Acquired SBS and other assets from IBM.

52,000,000 428,000,000 104,000,000 376,000,000

ID4–2 a.

From Campbell Soup’s perspective, assets increased (inventory, fixed assets, intangible assets—less the decrease in cash) and liabilities increased (short-term notes payable).

b.

From Unilever’s perspective, since Campbell’s paid a premium for the acquisition, assets were increased (cash received increased more than the drop in fixed assets and inventory) and the premium received would have increased equity (through a gain).

c.

See a above

d.

Fixed Assets and Inventory (+A)

100

Trademarks (+A)

490

Goodwill (+A)

330

Cash and ST Notes Payable(-A, +L) ID4–3

920


a. Issuing stock increases stockholders' equity and assets (for the cash raised) by the value of the stock issued. If the proceeds were then used to reduce debt, liabilities would decrease by the amount of liabilities paid off and assets would decrease for the cash used to retire the debt. If the proceeds from the stock issuance were used to acquire plant and equipment, assets would increase by the value of the new plant and equipment but decrease for the cash paid out for the new plant and equipment. The effect would be similar for current assets. Thus, the net effect of issuing stock to reduce debt would be to increase stockholders' equity and decrease liabilities. There may be a positive or negative residual effect on assets if the amount of cash raised through the stock issuance is different from the amount of cash used to pay off the debt. The net effect of issuing stock to acquire plant and equipment or to increase current assets would be to increase stockholders' equity and increase assets. b. A ratio that is often used to assess the riskiness of a company is the debt/equity ratio. This ratio is calculated as a company's total liabilities divided by the company's total stockholders' equity. The higher the ratio is, the less the stockholders have at stake in the company relative to the debtholders. This implies that the higher the ratio, the more risky the company is. By issuing additional stock, the company increases its stockholders' equity and thereby decreases its debt/equity ratio. In this way, issuing stock would affect the riskiness of the company and, hence, its credit rating.

ID4–4 a. There are a number of items that could explain why cash flows from operations would be less than net income. These items fall into two general classes: (1) items that affect net income but not cash flows from operating activities and (2) items that affect cash flows from operating activities but not net income. Specific examples of the former include gains, sales on account, accrual of other receivables, and so forth. Specific examples of the latter include payments to suppliers for inventory purchased previously on account, payments of other previously accrued payables, and so forth. b. There are four places in the financial statements that would probably be particularly informative about cash flow problems. First, a big increase in the balance of Accounts Receivable, particularly if the balance in Sales has not increased, could indicate that the company is having trouble collecting on credit sales. This would imply that the company has less cash flowing in. Second, a big increase in the company's payables could signal cash flow problems. That is, if a company is having cash flow problems, it would try to delay paying its payables as long as possible so that it can use its cash for more immediate problems. Third, a big decrease in inventory could result from cash flow problems because the company cannot afford to tie up as much cash in inventory. Finally, the statement of cash flows can provide lots of information about a company's cash flows and solvency. For example, a company that repeatedly fails to generate cash from operating activities could experience some solvency problems because a company cannot borrow or issue stock indefinitely. In addition, the investing activities section of the statement of cash flows would reveal whether the company is selling off lots of assets. This could indicate that the company is liquidating its productive base in an effort to raise enough capital to stay in business.

ID4–5


a. When WorldCom capitalized an expenditure as an asset, it did not show that dollar amount as an expense and therefore underreported expenses and overstated earnings. For example, if the company spent $10 million on its telecommunications lines and should have recorded a $10 million expense, it actually put the $10 million on the balance sheet as an asset, with the expense to be recorded in future periods as the asset depreciated. Future profits, of course, would be lower under this treatment, as those time periods would be charged with a depreciation expense. WorldCom was putting off the inevitable; the expenditures would eventually be called expenses but due to earnings pressure the company opted to capitalize the expenditure and show the expense at a later date. The company clearly violated the matching principle with this approach. b. If management feels pressure from public investors and Wall Street analysts, it may be tempted to employ techniques to defer expenses so that current earnings are higher than permitted under GAAP. Many observers have commented that management of publiclytraded companies often act with short-term profit targets in mind, ignoring long-term health and strategy. c. Given the size of publicly-traded companies, auditors cannot look at each and every transaction. Employing the materiality exception, auditors will focus their time on larger transactions. However, many small transactions (not material taken singly) become material when viewed in aggregate. d. Internal controls are designed to act as a “check-and-balance” mechanism so that the decisions of one manager are subject to review by an independent party. Had the audit committees of the Boards of Directors of WorldCom, Tyco and HealthSouth been truly committed to protecting shareholders, they would have established internal procedures where auditors—reporting directly to the Board, not to senior management—would have been reviewing the accounting procedures. An internal auditor of WorldCom, for example, would have spotted the inaccurate treatment of current expenditures and would have notified the Board’s audit committee, who could have immediately addressed the situation. ID4–6 Mr. Bailey is drawing a distinction between cash flows from operating activities and net income, and stressing how cash flows are more important for the company's solvency. Net income is a measure of the net assets the company generated during the year from operating activities, whereas cash flows address only one asset (i.e., cash). Thus, the generation of net income (i.e., "my bottom line is black") does not mean that the company generated cash. One thing that Mr. Bailey specifically refers to is credit sales (which increases net income) for which the company is not collecting cash on a timely basis. Because cash is the medium of exchange in the U.S. economy, cash is necessary for companies to be able to pay their employees, to obtain merchandise to sell, to pay obligations, to pay dividends, and so forth. Thus, without cash a company cannot stay in business. For these reasons, Mr. Bailey advises to "watch cash flow."

ID4–7


A.

Net income would not be affected. BMW’s change moves items from one part of the income statement (operating) to another (non-operating), but the items are still deducted as expenses when the final net income is calculated. The change will be to a line item within the income statement called “operating income”. B. BMW, as a global manufacturing company, wants its financial statements to conform to international standards so that users of financial statements can accurately compare BMW’s financial performance to that of other global companies. Additionally, BMW will show a higher operating income if it moves expenses into the non-operating section of the income statement. Many financial analysts focus on operating income as a measure of a company’s ability to generate a profit by the daily running of its core business. A higher operating income might improve BMW’s reputation in the financial community. C. An investment bank such as Goldman Sachs has significant business relationships with companies all across the globe and in all different industries. Goldman Sachs would ideally like to compare companies, using the same set of standards and measures. If a European company uses one set of accounting rules while a U.S. company uses another, it will be difficult for Goldman to determine the relative strengths and weaknesses of the companies. D. BMW would need to follow U.S. accounting standards if it would choose to be listed on the NYSE. Countries are attempting to synchronize their accounting standards, but certain differences still exist. If BMW wanted access to U.S. capital markets, it would have to conform to U.S. standards. In the age of globalization, international standards become that much more important. A global concern like BMW might want U.S. and European capital, but the different accounting standards, and the different requirements of the national capital markets, makes that more challenging.

ID4–8 The military has many users of its financial statements; these users need to be able to rely upon the integrity of the financial information in order to make good decisions. While these decisions are of a different nature than the typical commercial enterprise, they have significant financial impact on the enterprise. There are a couple of systematic errors that could develop from having to use poor financial information to make decisions. The first is that the decision makers decide to not use financial information at all due to the lack of faith in the numbers that are being generated. The second problem would be if the producers of the financial information (the military) resorts to overriding the financial systems in place and thereby “creating” numbers as they think they should be. In both cases the quality of the decisions made will be poor. This could lead to money being spent on the wrong projects and having the military unable to perform its function properly.

ID4–9 Borrowing money to pay dividends affects the right side of the accounting equation. Liabilities increase (Notes Payable, for example) while Stockholders’ Equity (through Retained Earnings) decreases. The combined effect is to leverage up a company’s balance sheet, causing it to finance assets with relatively more debt (and relatively less equity). The risks, of course, revolve around the increased debt on the company’s books and the consequent requirement to repay that debt. A blip in the company’s operations, or a downturn in general economic activity, and the company may be unable to repay its now-increased obligations. The benefit to companies such as Intel comes from cheap financing. When rates are low and borrowing money is relatively inexpensive and easy, companies can please their shareholders with a return on capital without incurring expensive debt. The low rates from 2006 to 2010 made such a move attractive.

ID4–10


The consolidation process of Umbro’s statements with those of Nike, Inc. has two major obstacles: 1) the conversion of pounds to U.S. dollars so that the statements can be presented in a common denomination; and 2) the conversion of Umbro’s IFRS-based statements (with its principles-based approach) to GAAP statements (with its rules-based approach). The currency conversion is relatively straight-forward and is something many multinational companies handle in their global operations. The second conversion, however, is trickier as management of Nike has to take statements in essentially a foreign language and translate them into U.S. GAAP—and do so in such a manner that share owners, analysts, regulators and other readers of Nike’s statements can understand the company’s business performance and see its results presented in a consistent manner.

ID4-11 Information obtained on a real-time basis is information that is available as soon as the underlying transaction has taken place. In the past financial systems might update once a week or once a month. In that environment managers were only able to see what sales were after the system was updated at the end of the month. Today, however, managers are able to access financial systems to see what sales are so far today. Today, virtually any transaction that hits the information system on a real-time basis can be tracked on a real-time basis. The most common items that are tracked this way are sales, inventory, accounts receivable and accounts payable. For service firms it might also include hours worked on each account. The items that are least likely to be available on a real-time basis are items that have very little day to day activity or that require manual input of some kind. This might include items like depreciation, bad debts expense and prepaid expenses. Real-time accounting improves the quality of the information available to the manager to make decisions. It is not always possible to wait until the end of the month to make certain decisions and, therefore, with real-time accounting better information is at the managers’ fingertips everyday to make those decisions.

ID4–12 a. According to the first footnote Nike’s prepaid expenses include advertising and promotion expenses of its products. Nike does not expense advertising costs until the ads are first run. Nike pays various media outlets in advance and converts the payment to an expense when the ad runs; until that conversion the expenditures is capitalized as a current asset for prepaid advertising expenses. Assets and Stockholders’ Equity are reduced when the advertising runs. Nike carries $280.0 million and $266.7 million of prepaid expenses as of 5/31/2009 and 5/31/2008, respectively. b. At times, Nike will book an expense before the cash is paid, “accruing” the liability to the balance sheet (to record the obligation to pay cash in the future) and recording the expense in the current period (and thereby reducing net income and, through Retained Earnings, equity). The Matching Principle is a driving force behind the accrual process. c. The Statement of Cash Flows shows that $455.7 million was spent on Capital Expenditures (Investing section) and that $466.7 million was spent on Dividends (Financing section). When Nike spent cash on long-term assets, total assets did not change, but cash was reduced and fixed assets were increased. When the company spent cash on dividends, assets and equity were reduced.


The Statement of Cash Flows indicates that $186.6 million of cash was raised from the sale of stock for options(Financing section). When the stock was sold, assets and equity increased. d. The Balance Sheet shows $1,031.9 million of accounts payable as of 5/31/2009. If Nike purchases inventory (cloth, rubber, zippers, etc.) without paying cash at the time of the purchase, a trade accounts payable is created. e. Management represents that its internal controls are designed to provide “reasonable assurance” that assets are protected and that the accounting transactions are properly recorded. Further, management represents that it continually reviews and updates its internal controls. f. The statement of cash flows represents the in- and out-flows of cash during a period of time. The indirect presentation of the statement of cash flows begins with net income (a number based on accrual accounting that does not have anything to do with cash) and, through a series of steps, converts this number to cash from operating activities. The first step to convert net income to cash from operations is to add back depreciation expense (and any other non-cash charges). Depreciation reduced net income but it did not reduce the company’s cash balance; in order to get the net income number closer to a cash number, we add back any expenses that reduced profits but did not reduce cash. The second group of steps required to convert net income into a cash figure is to look at the changes to the operating (current) accounts, including accounts receivable. In Nike’s case, accounts receivable increased over the period of fiscal 2009, meaning that the company booked sales but did not collect all the cash from those sales—the amount not collected from the sales is still sitting in the receivables account. In order to convert the profit number (which, after all, started from sales on the income statement) into a cash number, any sales that have not yet resulted in cash (meaning those sales still represented in the receivables balance) should be subtracted away.


CHAPTER 5 USING FINANCIAL STATEMENT INFORMATION BRIEF EXERCISE BE5–1 (a) ROE = Net Income/Average Stockholders Equity ROA = (Net Income +[Interest Expense (1-Tax Rate)])/ Average Total Assets Common Equity Leverage = Net Income/(Net Income + [Interest Expense(1-Tax Rate)]) Capital Structure Leverage = Average Total Assets/ Average Stockholders’ Equity Return on Sales = Net Income + [Interest Expense (1- Tax Rate)]/Net Sales Asset Turnover = Sales/Average Total Assets

Coke 27.3%

Pepsi 34.8%

14.7%

15.2%

94.4%

95.5%

1.97

2.39

19.3% .76

12.4% 1.22

Pepsi earns considerably more relative to its equity base and slightly more relative to its assets. Coke, however, has a higher return on sales (profits relative to sales). Pepsi shows higher use of leverage (Capital Structure Leverage ratio) and is much more efficient generating sales from its asset base (Asset Turnover ratio). (b) ROA x Common Equity Leverage x Capital Structure Leverage = ROE Coke: .147 x .944 x 1.97 = .273 Pepsi: .152 x .955 x 2.39 = .347 (rounding) (c) Return on Sales x Asset Turnover = ROA Coke: .193 x .76 = .147 Pepsi: .124 x 1.22 = .151 (rounding) (d) Pepsi’s advantage in producing a return for shareholders’ equity investment is driven by more aggressive use of leverage and better returns from its asset base (ROA). Pepsi’s advantage in ROA is driven by its ability to generate sales from its assets; Pepsi’s lower return on sales is offset by higher sales volume. Given the higher returns on equity and the companies’ respective costs of capital, both companies are creating value for their shareholders.

1


BE5–2 (a) With $24.6 billion in 2008 revenues, J & J’s pharmaceutical business is the largest. From 2006 to 2008, the consumer division showed the largest percentage growth at 64.25%. (b)

2006 2007 2008 55.8% 53.1% 50.7%

(c) Based on 2008 results, the consumer division generated the highest percentage of sales outside the U.S. at 56.8% (with medical devices next at 54.4%).

EXERCISES E5–1 Profitability Ratios: Return on Equity = Net Income ÷ Average Stockholders’ Equity 2008: $6,134 ÷ 36,500 2007: $8,052 ÷ 32,916.5

= .168 = .245

Return on Sales = (Net Income + [Interest Expense (1 – Tax Rate)]) ÷ Net Sales 2008: ($6,134 + [346 x (1 - .20)]) ÷ $29,131 = .220 2007: ($8,052 + [319 x (1 - .20)]) ÷ $33,099 = .251 Solvency Ratios: Current Ratio = Current Assets ÷ Current Liabilities 2008: $ 44,177 ÷ 2007: $ 35,699 ÷

$13,655 = $13,858 =

3.24 2.58

Leverage Ratios: Capital Structure Leverage Ratio = Average Total Assets ÷ Average Total Stockholders’ Equity 2008: $63,431 ÷ $36,500 = 1.74 2007: $56,037 ÷ $32,916.5 = 1.70 Overall, by examining the above computed ratios, it appears that Cisco would be a good investment. Profitability declined but still remained strong, while solvency improved. Leverage is up only slightly.

2


E5–2 Profitability Ratios: Return on Equity = Net Income ÷ Average Stockholders’ Equity 2008: 2007:

$5,292 $6,976

÷ ÷

40,925 39,757

= =

.129 .176

Return on Sales = (Net Income + [Interest Expense (1 – Tax Rate)]) ÷ Net Sales 2008: 2007:

($5,292 + [8 x (1 - .31)]) ÷ $37,586 = .141 ($6,976 + [15 x (1 - .31)]) ÷ $38,334 = .182

Solvency Ratios: Current Ratio = Current Assets ÷ Current Liabilities 2008: 2007:

$19,871 $23,885

÷ ÷

$7,818 $8,571

= =

2.54 2.79

Leverage Ratios: Capital Structure Leverage Ratio = Average Total Assets ÷ Average Total Stockholders’ Equity

2008: 2007:

$53,183 $52,009.5

÷ ÷ $39,757

$40,925 = 1.31

=

1.30

Profitability numbers are down significantly, solvency is down slightly and leverage is steady. The trends are due to the economic environment of 2008; the company is still quite strong and would represent a good investment opportunity.

E5–3 Based on the information provided by Ginny’s Fashions, we can compute the following ratios: 1. Return on Equity = Net Income ÷ Average Stockholders’ Equity* 2011: $17,000 ÷ $31,000 = .548 2012: $18,000 ÷ $35,500 = .507 *2010 numbers needed for averages, so the above ratios use the ending number from the balance sheet 2. Return on Sales = (Net Income + [Interest Expense (1 – Tax Rate)]) ÷ Net Sales 2011: 2012:

$17,000 + [2,000 (1 - .3)] ÷ $70,000 = .263 $18,000 + [2,000 (1 - .3)] ÷ $74,000 = .262

3


E5–3

Concluded

3. Current Ratio = Current Assets ÷ Current Liabilities 2011: 2012:

$14,000 $21,000

÷ ÷

$7,000 $9,000

= =

2.0 2.33

4. Debt/Equity Ratio = Total Liabilities ÷ Total Stockholders’ Equity 2011: 2012:

$33,000 $33,000

÷ ÷

$31,000 $40,000

= =

1.065 .825

Generally, a lot more information is available to a bank loan officer to decide upon a long-term loan. However, given the limited information, I would support only a short-term loan rather than a long-term loan. The return on equity has declined while the return on sales has remained stable, indicating that the company is not making any gains in its profitability. The current ratio is encouraging, indicating the company’s short-term solvency is not in question. In terms of cash flows, the company’s cash flow from operating activities is positive but declining considerably. It seems it is financing its asset base partly from long-term loans and partly from its own operations. Overall, as a bank loan officer, my bank’s interest will be safely protected if I approve only a short-term loan and not a long-term loan.

E5–4 a. Profitability Ratios: Return on Equity

= = =

Net Income ÷ Average Stockholders' Equity $16,500 ÷ [($29,000 + $36,500) ÷ 2] .504

Return on Assets

= = =

(Net Income + [Interest Expense (1–Tax Rate)]) ÷ Average Total Assets ($16,500 + [$5,000 x (1- .34)]) ÷ [($81,000 + $99,000) ÷ 2] .22

Earnings per Share

= = =

Net Income ÷ Average Number of Common Shares Outstanding $16,500 ÷ [(2,000 shares + 2,000 shares) ÷ 2] $8.25

Return on Sales

= = =

(Net Income + [Interest Expense (1 – Tax Rate)])÷ Net Sales ($16,500 + [$5,000 x (1 - .34)]) ÷ $72,000 .275

Interest Coverage

= (Net Income Before Taxes and Interest Expense) ÷ Interest Expense = $30,000 ÷ $5,000 = 6.00

4


E5–4

Continued

Solvency Ratios: Current Ratio

= = =

Current Assets ÷ Current Liabilities ($9,000 + $12,000 + $18,000) ÷ $16,500 2.36

Quick Ratio

= = =

(Cash + Marketable Securities + Accounts Receivable) ÷ Current Liabilities ($9,000 + $12,000) ÷ $16,500 1.27

Activity Ratios: Receivables Turnover =

Inventory Turnover

Net Credit Sales ÷ Average Accounts Receivable = $72,000 ÷ [($9,000 + $12,000) ÷ 2] = 6.86 = = =

Cost of Goods Sold ÷ Average Inventory $30,000 ÷ [($15,000 + $18,000) ÷ 2] 1.82

= = =

Return on Equity – Return on Assets .504 – .22 .284

Capitalization Ratios: Financial Leverage

Debt/Equity

= = =

Total Liabilities ÷ Total Stockholders' Equity ($16,500 + $46,000) ÷ ($20,000 + $5,000 + $11,500) 1.71

Market Ratios: Price/Earnings Ratio

Dividend Yield

= = = = = =

Return on Investment = Market Price0

Market Price per Share ÷ Earnings per Share $36 ÷ $8.25 4.36

Dividends per Share ÷ Market Price per Share ($9,000 ÷ 2,000 shares) ÷ $36 .125 (Market Price1 – Market Price0 + Dividends per Share) ÷ = =

($36 – $30 + $4.50) ÷ $30 .35

5


E5–4

Concluded

b. Balance Sheet Cash Accounts receivable Inventory Long-lived assets (net) Total assets

2012

2011 9% 12% 18% 61% 100%

9% 11% 19% 61% 100%

Accounts payable Long-term liabilities Common stock Additional paid-in capital Retained earnings Total liabilities & stockholders' equity

17% 46% 20% 5% 12% 100%

15% 49% 25% 6% 5% 100%

Income Statement Sales Cost of goods sold Gross profit Operating expenses Income from operations Interest expense Income from continuing operations (before taxes) Income taxes Net income

100% 42% 58% 17% 41% 7% 34% 12% 22%

c.

The company is making a handsome return of 27.5% on sales. Its return on equity is more than 50%. Since the return on equity measures a company’s ability to use equity investor’s capital to generate net assets through operations, a return of more than 50% indicates that Ken’s Sportswear has exceptional earning power. The current ratio of Ken’s Sportswear has gone down from 2.58 for the year 2011 to 2.36 for the year 2012, but it is still very good. It is indicative of the fact that the company has more than twice the current assets to meet its short-term obligations. Just as the current ratio provides information about the short-term solvency position of the company, the debt/equity ratio provides information about the long-term solvency of the company. Ken’s Sportswear has a debt/equity of 1.79 in 2011 and 1.71 in 2012. This means that the company has more debt than equity. A ratio of 1 would indicate that 50% of the company is financed by the stockholders and the remaining 50% is financed by the creditors. Therefore, a ratio of more than 1 indicates that the company has more debt than equity, which often can be a cause for concern. In summary, the company does not have any solvency problems in the short run but could face solvency problems in the future if its return on equity, financial leverage, and other activity ratios decline.

6


E5–5 a. Current Ratio = Current Assets ÷ Current Liabilities 2006: $5,029 ÷ $2,272 = 2.21 2007: $4,086 ÷ $2,433 = 1.68 2008: $4,005 ÷ $2,158 = 1.86 b. Gross Profit as a % of Sales = Gross Profit ÷ Net Sales 2006: $5,657 ÷ $15,923 = .36 2007: $5,692 ÷ $15,763 = .36 2008: $5,447 ÷ $14,526 = .38 c.

Inventory Turnover = Cost of Goods Sold ÷ Average Inventory 2007: $10,071 ÷ [($1,575 + $1,796) ÷ 2] = 5.98 2008: $ 9,079 ÷ [($1,506 + $1,575) ÷ 2] = 5.89 Average Days Supply of Inventory = 365 ÷ Inventory Turnover 2007: 2008:

365 ÷ 5.98 = 61 days 365 ÷ 5.89 = 62 days

Accounts Payable Turnover = Cost of Goods Sold ÷ Average A/P 2007: $10,071 ÷ [($772 + $1,006) ÷ 2] = 11.33 2008: $ 9,079 ÷ [($1,006 + $975) ÷ 2] = 9.17 Average Days Accounts Payable = 365 ÷ A/P Turnover 2007: 2008:

365 ÷ 11.33 = 32 days 365 ÷ 9.17 = 40 days

d. Over the period shown, solvency has deteriorated, as best demonstrated by the drop in the current ratio and the longer time required to pay trade payables.

7


E5–6 a. 2010 2010 Ending Cash Balance

= = =

2010 Beginning Cash Balance + Change in Cash $0 + $78 $78

Change in Cash = Cash from Operating Activities + Cash from Investing Activities + Cash from Financing Activities $78 = Cash from Operating Activities – $400 + $800 Cash from Operating Activities = $(322) 2011 2011 Ending Cash Balance = $76 = 2011 Beginning Cash Balance =

2011 Beginning Cash Balance + Change in Cash Beginning Cash Balance – $2 $78

or 2011 Beginning Cash Balance

= =

2010 Ending Cash Balance $78

Change in Cash = Cash from Operating Activities + Cash from Investing Activities + Cash from Financing Activities $(2) = $(252) + Cash from Investing Activities + $400 Cash from Investing Activities = $(150) 2012 2012 Ending Cash Balance $156 Change in Cash

= 2012 Beginning Cash Balance + Change in Cash = $76 + Change in Cash = $80

Change in Cash = Cash from Operating Activities + Cash from Investing Activities + Cash from Financing Activities $80 = Cash from Operating Activities + $150 – $200 Cash from Operating Activities = $130 Beecham was using debt and/or equity during 2010 and 2011 to finance the acquisition of productive assets (i.e., investing activities) and to cover cash outflows from operating activities. During 2012, the company started generating cash from operating activities and used this cash—along with selling productive assets—to reduce its debt and/or equity and to build a cash reserve.

8


E5–6

Concluded

b. Other than at the beginning of 2010, the company always had a positive cash balance. From that standpoint the company was solvent throughout the three-year period. A more detailed analysis of Beecham's solvency, however, requires an analysis of the company's operating performance, financial flexibility, and liquidity. During 2010 and 2011, Beecham did not generate cash flows from operating activities. The company remained solvent by issuing additional debt or equity. Since the company was able to acquire additional debt or equity financing in 2010 and 2011 and was able to sell off assets during 2012, it appears that Beecham does have some financial flexibility. However, without having the associated balance sheets, it is not possible to adequately assess Beecham's financial flexibility and liquidity. Based upon the limited information provided, it appears that Beecham faced some potential solvency problems in 2010 and 2011, but was able to overcome these problems by issuing additional debt or equity.

E5–7 a. (1) Current Ratio = Current Assets ÷ Current Liabilities 2011: 2012:

$385,000 ÷ $170,000 = 2.26 $400,000 ÷ $460,000 = 0.87

(2) Quick Ratio = (Cash + Marketable Securities + Accounts Receivable) ÷ Current Liabilities 2011: 2012:

($30,000 + $10,000 + $95,000) ÷ $170,000 = 0.794 ($15,000 + $225,000+ $90,000) ÷ $460,000 = 0.717

b. Receivables Turnover = Net Credit Sales ÷ Average Accounts Receivable 2011: 2012:

$780,000 ÷ [($100,000 + $95,000) ÷ 2] = 8.00 $800,000 ÷ [($95,000 + $90,000) ÷ 2] = 8.65

Number of Days Outstanding = 365 ÷ Receivables Turnover 2011: 2012: c.

365 ÷ 8.00 = 45.625 365 ÷ 8.65 = 42.197

Solvency refers to a company's ability to meet its debts as they come due. Current liabilities represent the debts that are expected to come due first. Therefore, to be solvent, a company must have sufficient cash or near-cash assets to meet these current liabilities. Total current assets is one measure of near-cash assets. As indicated by the change in the company's current ratio, the company has insufficient current assets available to settle its current liabilities. The company's quick ratio worsened during 2012. Given that the company has insufficient current assets and insufficient cash, marketable securities, and accounts receivable to meet its debts, it can probably be concluded that the company's overall solvency position is not strong.

9


E5–8 a. Return on Equity = Net Income ÷ Average Stockholders' Equity 2009: 2010: 2011: 2012:

$510,000 ÷ [($100,000 + $100,000) ÷ 2] = 5.10 $490,000 ÷ [($100,000 + $290,000) ÷ 2] = 2.51 $515,000 ÷ [($290,000 + $315,000) ÷ 2] = 1.70 $505,000 ÷ [($315,000 + $510,000) ÷ 2] = 1.22

It appears that the additional capital provided by the owners has not been used to generate net income. The company's net income has been relatively constant from 2009 to 2012. If the company had been effective at using the additional capital, the company's net income should have increased, and return on equity should have been relatively constant or increasing over time. However, if the company has used the additional capital for long-term projects, such as a new product, these projects may not generate any net income for several years. Once these projects begin generating income, the company's return on equity may increase to more appropriate levels. Therefore, the effectiveness of the company at using the owners' capital cannot be adequately evaluated without additional information. b. It appears that the company has overinvested in inventory. The inventory turnover and the days' supply of inventory for each year are:

Inventory turnover Days' supply

2009

2010

2011

2012

12.00 30.42

5.93 61.55

4.85 75.26

4.09 89.24

These ratios indicate that the company went from having one month's supply of inventory on hand to having almost three months of inventory on hand. It appears that the company has more inventory on hand than is warranted, given demand for the inventory. The company could reduce inventory on hand and invest the proceeds in income-producing assets such as marketable securities. Such a move would make the company more profitable and provide owners a greater return on their investments. This change in investment policy would increase the company's return on equity.

E5–9 a. Current Ratio = Current Assets ÷ Current Liabilities 2009: 2010: 2011: 2012:

$20,000 ÷ $8,000 = 2.500 $24,000 ÷ $13,000 = 1.846 $31,000 ÷ $25,000 = 1.240 $35,000 ÷ $30,000 = 1.167

Debt/Equity Ratio = Total Liabilities ÷ Total Stockholders' Equity 2009: 2010: 2011: 2012:

($8,000 + $15,000) ÷ ($20,000 + $10,000) = 0.767 ($13,000 + $35,000) ÷ ($20,000 + $20,000) = 1.200 ($25,000 + $40,000) ÷ ($20,000 + $32,000) = 1.250 ($30,000 + $40,000) ÷ ($20,000 + $38,000) = 1.207

10


E5–9

Continued

Return on Assets = (Net Income + [Interest Expense (1 – Tax Rate)]) ÷ Average Total Assets 2009: 2010: 2011: 2012:

($13,000 + [$2,000 (1 - .3)]) ÷ [($53,000] = 0.272 ($14,000 + [$4,000 (1 - .3)]) ÷ [($53,000 + $88,000) ÷ 2] = 0.238 ($21,000 + [$5,000 (1 - .3)]) ÷ [($88,000 + $117,000) ÷ 2] = 0.239 ($24,000 + [$5,000 (1 - .3)]) ÷ [($117,000 + $128,000) ÷ 2] = 0.224

b.

c.

2012

2011

2010

2009

Current assets Noncurrent assets Total assets

27.34% 72.66 100.00%

26.50% 73.50 100.00%

27.27% 72.73 100.00%

37.74% 62.26 100.00%

Current liabilities Long-term liabilities Capital stock Retained earnings Total liabilities and stockholders' equity

23.44% 31.25 15.62 29.69

21.37% 34.19 17.09 27.35

14.77% 39.77 22.73 22.73

15.09% 28.30 37.74 18.87

100.00%

100.00%

100.00%

100.00%

Solvency measures a company's ability to meet its debts as they come due. The current ratio provides one measure of a company's solvency. Based upon this ratio, Lotechnic has sufficient current assets to meet its current obligations. However, the trend in its current ratio indicates that the company's excess of current assets over current liabilities is decreasing. Therefore, the company has relatively fewer current assets available to meet its current obligations. This trend indicates that Lotechnic Enterprises' solvency position may be worsening. The debt/equity ratio provides an indication of a company's capitalization, which, in turn, indicates how risky a company is. Lotechnic is relying increasingly on debt relative to stockholders' equity to finance operations. At some point in time, the company will have to repay this debt. The company will either have to repay this debt by (1) generating cash from operations, (2) selling assets, (3) borrowing additional cash, or (4) acquiring cash by issuing stock. From the statement of cash flows, the cash generated from operations has been decreasing and is now negative; therefore, it appears that the company cannot rely on operations to generate cash. The statement of cash flows also indicates that the company has been using cash for investment purposes every year. This implies that the company may have some assets that it could sell. But if these assets are used in operations, the company's operations may be adversely affected by selling them. Since total assets equal the sum of total liabilities and stockholders' equity, the proportion of total liabilities to the sum of total liabilities and stockholders' equity reported on the common-size balance sheet equals the proportion of total liabilities to total assets. This measure indicates the proportion of total assets (based upon book value) that would have to be sold to satisfy all the company's obligations. To meet its obligations, Lotechnic Enterprises would have to sell approximately 55% of its total assets, which would virtually decimate its asset base. Based upon the trend in the current ratio, the debt/equity ratio, cash flows from operations, and the proportion of total liabilities to total assets, it appears that Lotechnic Enterprises may face severe solvency problems as its long-term debt matures.

11


E5–9

Concluded

Earning power is defined as a company's ability to increase its wealth through operations and to generate cash from operations. Earning power and solvency are closely related. A company must have adequate resources to generate wealth. If a company experiences solvency problems, it will most likely have to divert its resources to paying its obligations. Therefore, due to its solvency problems, Lotechnic Enterprises may not have strong earning power. Although Lotechnic's net income has increased every year, the company's effectiveness at managing capital, as indicated by ROA, has decreased every year. This trend indicates that the company may have limited earning power. This conclusion is also supported by the trend in the company's cash flows from operations. It must be remembered, however, that this analysis is based on very limited information. To adequately analyze a company, additional information would be needed. Complete financial statements, financial information for similar companies, and general economic information should all be considered when analyzing a company's earning power and solvency position.

E5–10 Transaction

Quick Ratio

Current Ratio

Debt/Equity Ratio

(1) – – + (2) NE NE + (3) – – – (4) – – +a b (5) + + – (6) + + – __________________ a Wage Expense would be closed into Retained Earnings at the end of the accounting period as part of the closing process. Thus, recording wage expense would decrease stockholders' equity, and thereby increase the debt/equity ratio. b

This transaction would increase both Sales and Cost of Goods Sold. Both of these accounts would be closed into Retained Earnings as part of the closing process. Since the sales price exceeds the cost of the inventory, the net effect of this transaction would be to increase Retained Earnings. Thus, total stockholders' equity would increase, and thereby decrease the debt/equity ratio.

E5–11 a. Debt/Equity Ratio

= Total Liabilities ÷ Total Stockholders' Equity = ($130,000 + $150,000) ÷ $200,000 = 1.40

b. The maximum debt that Montvale can have outstanding is 1.5 times its total stockholders' equity. This means that the total debt Montvale can have outstanding is $300,000 (1.5  $200,000). Since Montvale already has $280,000 of outstanding debt, it can incur an additional $20,000 in debt without violating its debt covenant.

12


E5–11 c.

Concluded

The minimum level of stockholders' equity that Montvale can have is total debt divided by 1.5. This means that the total stockholders' equity Montvale can have is $186,667 ($280,000 ÷ 1.5). Since Montvale currently has $200,000 of stockholders' equity, and since dividends decrease stockholders' equity, the maximum dividend that Montvale can declare is $13,333.

d. If Montvale had declared, but not paid, a $20,000 dividend prior to obtaining the loan, then the $20,000 is already included in the current liabilities reported on the balance sheet. Paying the dividend would decrease both current assets and current liabilities by $20,000. Thus, the debt/equity ratio after paying the $20,000 would be 1.3 ([($130,000 – $20,000) + $150,000] ÷ $200,000). Since this ratio is less than the maximum debt/equity ratio allowed under the debt covenant, Montvale could pay the $20,000 dividend without violating its debt covenant.

E5–12 a.

2006: 2007: 2008:

$1,217 ÷ $1,766 ÷ $1,823 ÷

b. Price Earnings Ratio

2007: 2008:

$3,544 = 34.3% $2,395 = 73.7% $4,313 = 42.3% = Market Price per Share ÷ Earnings per Share = Market Price per Share ÷ (Net Income ÷ Average Number of Common Shares Outstanding)

$58.91 ÷ {$2,395 ÷ [(1,188 + 1,234) ÷ 2]} = 29.79 $62.19 ÷ {$4,313 ÷ [(1,127 + 1,188) ÷ 2]} = 16.67

Dividend Yield = Dividends per Share ÷ Market Price per Share 2006: 2007: 2008:

($1,217 ÷ 1,234 shares) ÷ $44.33 = .0223 ($1,766 ÷ 1,188 shares) ÷ $58.91 = .0252 ($1,823 ÷ 1,127 shares) ÷ $62.19 = .0260

Stock Price Return= Market Price0 2007: 2008:

(Market Price1 – Market Price0 + Dividends per Share) ÷

($58.91 – $44.33 + $1.49) ÷ $44.33 = 36.3% ($62.19 – $58.91 + $1.62) ÷ $58.91 = 8.32%

13


E5–12 c.

Concluded

An investment in McDonald’s stock from 2006 to 2008 would have provided a very good return for investors in 2007, but a much lower return in 2008 (which matches overall stock market performance for that time period). The dividend yield showed consistent improvement over this time period but the capital appreciation from the stock price lagged in 2008.

E5–13 a. (1)

Earnings per Share = Outstanding

Net Income ÷ Average Number of Common Shares = =

$7,808 ÷ [(2,169 + 2,133.3) ÷ 2] $3.63

(2)

Price/Earnings

= = =

Market Price per Share ÷ Earnings per Share $30.40 per Share ÷ $3.63 per Share 8.37

(3)

Dividend Yield

= = =

Dividends per Share ÷ Market Price per Share ($3,278.5 ÷ 2,151.15 Shares) ÷ $30.40 per Share .050

(4)

Stock Price Return = (Market Price1 – Market Price0 + Dividends per Share) ÷ Market Price0 = ($30.40 – $57.37 + $1.52) ÷ $57.37 = (44.4%)

b. Return on equity equals net income divided by average stockholders' equity. Thus, only those items that affect net income or stockholders' equity would affect a company's return on equity. Declaring and paying dividends would decrease stockholders' equity but would not affect net income. Thus, the return on equity ratio would increase for item (1). A change in the market price of a company's stock would not affect return on equity. Repurchasing common stock would decrease stockholders' equity, but it would not affect net income. Thus, the return on equity ratio would increase for item (3).

E5–14

a. The Medical devices unit generated operating profits as a percentage of sales at 31.2% in 2008, compared to 31.0% for the Pharmaceutical unit. In the previous year, however, Pharmaceuticals outperformed Medical devices at 26.3% versus 22.3%. (In 2006 Medical devices was ahead, 30.2% to 29.6%.) b. In terms of growth in operating profits from 2006 to 2008, the Consumer division was tops at 94.6%.

14


E5–15 The formulas that are used by the ROEmodel are as follows: ROE

=

ROA

*

Common Equity Leverage

=

Profit Margin

*

Capital Structure Leverage

and ROA

*

Asset Turnover

The first item that stands out is the steady increase in ROE from 2010 through 2012. This is being driven completely by the increase in Capital Structure Leverage. Both ROA and Common Equity Leverage have been decreasing over the three years. ROA has dropped because of the drop in Asset Turnover. From 2010 through 2012 both ROA and common equity leverage have decreased slightly. But the significant increase in capital structure leverage has more than offset the declines in the other components so that ROE has risen in both 2011 and 2012. These ratios show that the improvement in ROE is due primarily to increased debt leverage used by the company. ROA has decreased because of the decrease in asset turnover. ROA can be computed by multiplying profit margin x asset turnover. In this example, while the profit margin has increased from 2010 through 2012 asset turnover has deteriorated significantly. This reduction in ROA could be due to slowing sales or a large increase in the asset base of the company. With the increase in the capital structure leverage the most likely reason is an increase in the assets of the company.

E5–16 The formulas that are used by the ROE model are as follows: ROE

=

ROA

*

Common Equity Leverage

=

Profit Margin

*

Capital Structure Leverage

and ROA

*

Asset Turnover

From 2010 through 2012 both the common equity leverage and the capital structure leverage have remained fairly stable. ROA has dropped significantly which has caused the decline in ROE. In this example asset turnover has remained stable from 2010 through 2012. During this same time the company’s profit margin has declined precipitously. Therefore the drop in LBS Products’ profit margin has caused its ROA to fall which in turn caused its ROE to also fall.

E5–17 a. ROE = ROA * Common Equity Leverage therefore, ROE = .095 * 0.685 * 2.50 = 16.3% b. ROE = ROA * Common Equity Leverage ROE will equal .095 * 0.685 * 2.75 = 17.9%

*

*

Capital

Structure

Capital Structure Leverage, therefore 2013

c. ROA = Profit Margin * Asset Turnover and ROE = ROA * Common Equity Leverage * Capital Structure Leverage; therefore, 0.625 * .16 = .1; ROE = .1 * .685 * 2.50 = 17.1% 15

Leverage;


E5–18

Sales Net income Total assets Total shareholders’ equity

2008 $63,747 12,949 84,912 42,511

Historic Relationship NA 20.3% of Sales 1.33 times Sales 50.1% of Assets

If Sales are projected to grow at 8% and if the historic relationships maintain, the projected 2009 financial statements can be summarized below:

Sales Net income

2009 $68,847 13,976

Total assets Total shareholders’ equity

91,567 45,875

16


PROBLEMS P5–1 (1) Current Ratio

(2) Quick Ratio Liabilities

=

Current Assets ÷ Current Liabilities = $976 ÷ $504 = 1.94 =

(Cash + Marketable Securities + Accounts Receivable) ÷ Current

= =

($97 + $378) ÷ $504 0.94

(3) Receivables Turnover

= Net Credit Sales ÷ Average Accounts Receivables = $2,155 ÷ ((378 + 507) ÷ 2) = 4.87

(4) Interest Coverage =

(Net Income Before Taxes + Interest Expense) ÷ Interest Expense = ($-35 + $2) ÷ $2 = (16.5)

(5) Return on Assets =

(Net Income + [Interest Expense (1 – Tax Rate)]) ÷ Avg. Total Assets = NA (need Total Assets)

(6) Inventory Turnover

= = =

Cost of Goods Sold ÷ Avg. Inventory $1,805 ÷ $364.5 4.95

(7) Return on Equity

= = =

Net Income ÷ Avg. Stockholders' Equity $-33 ÷ $999.5 (.033)

Receivables and Inventory turnover ratios are at respectable rates, but the loss on the income statements hurts the other ratios. The current ratio remains solid.

P5–2 a. Return on Equity = Net Income ÷ Average Stockholders' Equity 2011: $14,000 ÷ [($34,000 + $38,000) ÷ 2] = .389 2012: $25,000 ÷ [($38,000 + $51,000) ÷ 2] = .562 Return on Assets = (Net Income + [Interest Expense(1 – Tax Rate)]) ÷ Average Total Assets 2011: ($14,000 + [$1,000 (1 - .30)]) ÷ [($49,000 + $55,000) ÷ 2] = .283 2012: ($25,000 + [$5,000 (1 - .342)]) ÷ [($55,000 + $113,000) ÷ 2] = .337 Common Equity Leverage = Net Income ÷ [Net Income + (Interest Expense (1 – Tax Rate)] 2011: $14,000 ÷ [$14,000 + ($1,000 (1 - .30)] = .952 2012: $25,000 ÷ [$25,000 + ($5,000 (1 - .342)] = .883

17


P5–2

Concluded

Capital Structure Leverage = Average Total Assets ÷ Average Shareholders’ Equity 2011: [($49,000 + $55,000) ÷ 2] ÷ [($34,000 + $38,000) ÷ 2] = 1.444 2012: [($55,000 + $113,000) ÷ 2] ÷ [($38,000 + $51,000) ÷ 2] = 1.888 Profit Margin = (Net Income + [Interest Expense(1 – Tax Rate)]) ÷ Net Sales 2011: ($14,000 + [$1,000 (1 - .30)]) ÷ $45,000 = .327 2012: ($25,000 + [$5,000 (1 - .342)]) ÷ $70,000 = .404 Asset Turnover = Sales ÷ Average Total Assets 2011: $45,000 ÷ [($49,000 + $55,000) ÷ 2] = .865 2012: $70,000 ÷ [($55,000 + $113,000) ÷ 2] = .833 b. Return on Equity

= =

Return on Assets =

($25,000 + $4,000 interest saved) ÷ {[($38,000 + $40,000) + ($51,000 + $40,000 + $4,000 interest saved)] ÷ 2} .335

[($25,000 + $4,000 interest saved) + $1,000] ÷ {[$55,000 + ($113,000 + $4,000 cash saved on interest)] ÷ 2} = .349

Common Equity Leverage = ($25,000 + $4,000 interest saved) ÷ [($25,000 + $4,000 interest saved) + ($1,000 (1 – .34)] = .978 Capital Structure Leverage = [($55,000 + ($113,000 + $4,000 interest saved)) ÷ 2] ÷ [$38,000 + ($51,000 + $4,000 interest saved) ÷ 2] = 1.849 Profit Margin = [($25,000 + $4,000 interest saved) + ($1,000 (1 – .34)] ÷ $70,000 = .424 Asset Turnover = $70,000 ÷ [($55,000 + ($113,000 + $4,000 interest saved)) ÷ 2] = .814 c.

The company appears stronger by issuing equity rather than debt if one examines return on assets, common equity leverage and profit margin. However, based on return on equity, capital structure leverage and asset turnover, the company appears stronger by issuing debt rather than equity. The company can effectively manage additional debt capital to the benefit of its stockholders, but it does not effectively manage additional equity capital to the benefit of its stockholders. In fact, the company manages equity capital to the detriment of its stockholders. Therefore, based upon this limited ratio analysis, it appears that the company made the correct decision to issue debt rather than equity. Remember, though, that the company may want or need to consider other factors in deciding whether to issue debt or equity. It may want to consider the effect of the alternative financing arrangements on existing debt covenants, incentive compensation schemes, taxes, and so forth.

18


P5–3 a.

Assets Current assets: Cash Short-term marketable securities Accounts receivable Inventory Other current assets Total current assets Property, plant, & equipment Other assets Total assets Liabilities and Stockholders' Equity Current liabilities: Short-term borrowings Accounts payable Wages payable Dividend payable Income taxes payable Other current liabilities Total current liabilities Long-term debt Contributed capital Retained earnings Total liabilities and stockholders' equity

Dollar Change

Percentage Change

$

2,276

70.7%

$

(1,182) 105 (31) (1,031) 137 51 2,150 2,338

(73.4%) 3.9% (1.2%) (45.1%) 1.1% 0.6% 35.9% 8.7%

5,432 (38) (53) 23 (9) 2,318 7,673 1,434 (2,599) (4,152)

1,312.1% (4.1%) (6.4%) 4.5% (3.8%) 91.9% 141.1% 18.1% (153.2%) (35.2%)

2,338

8.7%

$

$

$

19


P5–3

Concluded

b.

2008

2007

Change

Assets Current assets: Cash Short-term marketable securities Accounts receivable Inventory Other current assets Total current assets Property, plant, and equipment Other assets Total assets

18.82% 1.47% 9.51% 8.53% 4.30% 42.63% 29.53% 27.84% 100.00%

11.99% 5.99% 9.95% 9.39% 8.50%* 45.82% 31.91% 22.27% 100.00%

56.96% (75.46%) (4.42%) (9.16%) (49.41%) (6.96%) (7.46%) 25.01%

Liabilities and Stockholders' Equity Current liabilities: Short-term borrowings Accounts payable Wages payable Dividend payable Income taxes payable Other current liabilities Total current liabilities Long-term debt Contributed capital Retained earnings Total liabilities and stockholders' equity

20.01% 3.03% 2.64% 1.84% 0.78% 16.52%* 44.82% 32.07% (3.09%) 26.20% 100.00%

1.54% 3.44% 3.07% 1.91% 0.89% 9.40%* 20.24% 29.52% 6.31% 43.93% 100.00%

1,199.35% (11.92%) (14.01%) (3.66%) (12.36%) 75.74% 121.44% 8.64% (148.97%) (40.36%)

* Other Current Assets and Other Current Liabilities are rounded to result in a value of 100% in column addition. c.

Common-size financial statements provide relative comparisons of account balances rather than absolute comparisons of account balances. Absolute comparisons only provide information about whether an account balance has increased or decreased. Alternatively, relative comparisons provide information about whether an account balance has increased or decreased relative to a benchmark measure. This relative comparison allows financial statement users to determine more easily if a company is altering the composition of its assets, liabilities, or stockholders' equity. Relative comparisons of account balances may also provide users with insights into why account balances are changing.

P5–4 (1)

Return on equity measures a company's effectiveness at managing equity investments. Return on equity is calculated as net income divided by average stockholders' equity. 2011: 2012:

$515,000 ÷ [($450,000 + $755,000) ÷ 2] = .855 $510,000 ÷ [($755,000 + $795,000) ÷ 2] = .658

The company generated returns on its owners' investments in excess of 65%, which appears to be rather substantial. However, without being able to compare Gidley Electronics' performance to 20


industry averages, it is difficult to conclude whether the company is really effective in managing the owners' capital.

21


P5–4 (2)

Continued Return on equity measures a company's effectiveness at managing owners' investments, while return on assets measures a company's effectiveness at managing all investments, both debt and equity. The excess of return on equity over return on assets indicates a company's effectiveness at using debt to generate returns for the owners. This measure is called financial leverage. Since financial leverage is calculated using return on assets, the first step is to calculate return on assets. Return on assets is calculated as the sum of net income and [interest expense x (1- tax rate)] divided by average total assets. Gidley's return on assets for 2011 and 2012 is: 2011: 2012:

($515,000 + [$165,000 (1 - .40)]) ÷ [($1,450,000 + $1,470,000) ÷ 2] = .421 ($510,000 + [$150,000 (1 - .40)]) ÷ [($1,470,000 + $1,465,000) ÷ 2] = .409

Gidley's financial leverage for 2011 and 2012 is, therefore: 2011: 2012:

.855 – .421 = .434 .658 – .409 = .249

The company is using debt to the benefit of its equity owners. The positive leverage indicates that proceeds from debt are generating sufficient profits to provide a return for the equity owners. In other words, the return from using debt exceeds its cost. (3)

The current ratio measures whether a company has sufficient current assets to meet its current liabilities. The current ratio equals current assets divided by current liabilities. Gidley's current ratio for 2011 and 2012 is: 2011: 2012:

$1,010,000 ÷ $275,000 = 3.673 $980,000 ÷ $290,000 = 3.379

Gidley Electronics' current assets are over three times greater than its current liabilities. The company therefore appears to have no solvency problems. However, the company may be unable to convert some of its current assets to cash quickly enough to meet some of its current liabilities. Another measure of solvency that compares near-cash assets to current liabilities is the quick ratio. The quick ratio equals the sum of cash, marketable securities, and accounts receivable divided by current liabilities. Gidley's quick ratio for 2011 and 2012 is: 2011: 2012:

($115,000 + $220,000 + $400,000) ÷ $275,000 = 2.673 ($110,000 + $175,000 + $350,000) ÷ $290,000 = 2.190

Gidley Electronics appears to have sufficient cash and near-cash assets available to meet its current obligations. Therefore, the company should have no significant short-term solvency problems.

22


P5–4 (4)

Concluded The price/earnings ratio measures the sensitivity of stock prices to changes in earnings. This ratio is calculated by dividing the market price per share by earnings per share. Since this ratio uses earnings per share in the calculations, the first step is to calculate earnings per share. Earnings per share is calculated by dividing net income by the average number of common shares outstanding during the year. Gidley's earnings per share for 2011 and 2012 are: 2011: 2012:

$515,000 ÷ [(17,000 + 17,000) ÷ 2] = $30.29 $510,000 ÷ [(17,000 + 22,000) ÷ 2] = $26.15

Gidley's price/earnings ratio for 2011 and 2012 is: 2011: 2012:

$69.00 ÷ $30.29 = 2.278 $54.00 ÷ $26.15 = 2.065

It appears that the price of Gidley Electronics' stock is rather sensitive to changes in earnings. A change in earnings per share should cause the market price to change by approximately twice the change. To obtain a better idea of how sensitive the company's stock is to changes in earnings, the company's price/earnings ratio should be compared to industry averages. (5)

The average number of days accounts receivable are outstanding is calculated as 365 days divided by accounts receivable turnover. The accounts receivable turnover is, in turn, calculated by dividing net credit sales by average accounts receivable. Gidley's accounts receivable turnover for 2011 and 2012 is: 2011: 2012:

$3,010,000 ÷ [($400,000 + $400,000) ÷ 2] = 7.525 $2,450,000 ÷ [($400,000 + $350,000) ÷ 2] = 6.533

The number of days outstanding for receivables during 2011 and 2012 is: 2011: 2012:

365 ÷ 7.525 = 48.505 days 365 ÷ 6.533 = 55.870 days

The average number of days accounts receivable are outstanding increased slightly. Therefore, customers are not paying their open receivables as quickly as before. If this problem persists, Gidley may have to consider more stringent credit and/or collection policies.

P5–5 a. Return on equity provides a measure of a company's effectiveness at managing the owners' capital. The formula for calculating return on equity is net income divided by average stockholders' equity. The 2012 return on equity for Hathaway Toy Company and Yakima Manufacturing would be: Hathaway: Yakima:

$875,000 ÷ [($1,585,000 + $2,460,000) ÷ 2] = .433 $755,000 ÷ [($70,000 + $825,000) ÷ 2] = 1.687

Note: Beginning Stockholders' Equity = Ending Stockholders' Equity – Net Income Based on return on equity, Yakima Manufacturing has been almost four times more efficient than Hathaway Toy Company at managing owners' capital.

P5–5

Concluded 23


b. Return on assets provides a measure of a company's effectiveness at managing all investors' capital. The formula for calculating return on assets is the sum of net income and tax-adjusted interest expense divided by average total assets. The 2012 return on assets for Hathaway Toy Company and Yakima Manufacturing would be: Hathaway: Yakima:

($875,000 + $0) ÷ [($1,825,000 + $2,700,000) ÷ 2] = .387 ($755,000 + $195,000[1 – 0.00]) ÷ [($1,945,000 + $2,700,000) ÷ 2] = .409

Note: Beginning Total Assets

= =

Ending Total Assets – Net Income Total Liabilities and Stockholders' Equity – Net Income

Based on return on assets, Yakima Manufacturing and Hathaway Toy Company are essentially equal in their abilities to manage all investors' capital.

c.

Earnings per Share = Net Income ÷ Average Number of Common Shares Outstanding Hathaway: Yakima: Note:

$875,000 ÷ [(80,000 + 80,000) ÷ 2] = $10.94 $755,000 ÷ [(35,000 + 35,000) ÷ 2] = $21.57

The number of shares outstanding equals the balance in Common Stock divided by $10 par value per share.

d. Yes, stockholders are realizing a return on their capital of 168.7% (from Part [a]), while debtholders are realizing only a return on their capital of approximately 10.5% ($195,000 of interest expense ÷ $1,850,000 balance in mortgage payable). This difference in returns is due to the company using debt rather than equity to finance operations. Since the debtholders are only entitled to interest, any earnings from operations in excess of interest accrues to the stockholders. Thus, Yakima Manufacturing has efficiently used debt to benefit its stockholders.

P5–6 In order to consider an investment in Goodyear, let us first compute the following ratios: 1. Return on Equity

=

Net Income ÷ Average Stockholders’ Equity

2007: $ 602 ÷ [($-758 + $2,850) ÷ 2] = 57.6% 2008: $(77) ÷ [($2,850 + $1,022) ÷ 2] = (4.0%) 2. Return on Sales 2007: 2008:

=

(Net Income + [Interest Expense (1 – Tax Rate)]) ÷ Net Sales

($602 + [$450 (1 – 0.30)]) ÷ $19,644 = 4.7% ($(330) + [$320 (1 – 0.30)]) ÷ $19,488 = (0.5%)

24


P5–6

Concluded

3. Current Ratio =Current Assets ÷ Current Liabilities 2007: 2008:

$10,172 $ 8,340

÷ ÷

$4,664 $4,779

= =

2.18 1.75

4. Debt/Equity Ratio = Total Liabilities ÷ Total Stockholders’ Equity 2007: $14,341 2008: $14,204

÷ ÷

$2,850 $1,022

= =

5.03 13.90

Generally, an equity investor would have much more information available to make a decision than what is provided by Goodyear. However, based on the information provided, an equity investment in Goodyear would be unwise. The company is showing a net loss on its income statement, has high leverage on its balance sheet and is generating negative cash from operations. Until Goodyear can turn around its operations, it is not an attractive opportunity.

P5–7 a. Return on Equity = Net Income ÷ Average Stockholders' Equity Robotronics: Technology:

$610,000 ÷ [($1,005,000 + $1,005,000) ÷ 2] = .607 $1,675,000 ÷ [($1,440,000 + $1,440,000) ÷ 2] = 1.163

Based on return on equity, Technology is almost twice as efficient as Robotronics at managing the stockholders' capital. If unusual items were not considered, return on equity for each company would be: Robotronics: Technology:

$610,000 ÷ [($1,005,000 + $1,005,000) ÷ 2] = .607 ($1,675,000 – $1,300,000) ÷ [($1,440,000 + $1,440,000) ÷ 2] = .260

Technology now appears to be considerably worse than Robotronics at managing the stockholders' capital. Including unusual items in calculating return on equity does provide a more complete measure of how efficiently a company managed its stockholders' equity in the current year. However, since unusual items are, by definition, items that occur infrequently, these items do not indicate a company's continued ability to efficiently manage the stockholders' capital. Thus, unusual items probably should not be used to calculate return on equity.

25


P5–7

Concluded

b. Financial leverage indicates how effectively a company uses debt for the benefit of stockholders. Financial leverage equals return on equity less return on assets. Thus, return on assets must be calculated before calculating financial leverage. Return on Assets = (Net Income + Interest Expense (net of tax)) ÷ Average Total Assets Robotronics: Technology:

($610,000 + $100,000) ÷ [($3,360,000 + $3,360,000) ÷ 2] = .211 ($1,675,000 + $175,000) ÷ [($1,870,000 + $1,870,000) ÷ 2] = .989

Financial Leverage = Return on Equity – Return on Assets Robotronics: Technology:

.607 – .211 = .396 1.163 – .989 = .174

From this analysis, Robotronics is approximately twice as effective as Technology at using debt to generate returns for its stockholders. If unusual items are not considered, the return on assets for each company would be: Robotronics: Technology:

($610,000 + $100,000) ÷ [($3,360,000 + $3,360,000) ÷ 2] = .211 [($1,675,000 – $1,300,000) + $175,000] ÷ [($1,870,000 + $1,870,000) ÷ 2] = .294

Therefore, the financial leverage of the two companies would be: Robotronics: Technology:

.607 – .211 = .396 .260 – .294 = –.034

If unusual items are not considered, Technology has negative financial leverage. That means that Technology is not generating a large enough return on its debt to even cover the interest expense. Thus, Technology is using debt to the detriment of its stockholders. It appears, therefore, that unusual items can affect the conclusions one draws when analyzing a company.

26


P5–8 Return on Sales .08 Net Sales

= = =

Cost of Goods Sold

Net Income ÷ Net Sales $25,000 ÷ Net Sales $312,500 =

Net sales x (1 – Gross Margin Percentage) = $312,500 x (1 – 40%) = $187,500

Net Income $25,000 Expenses =

= Net Sales – Cost of Goods Sold – Expenses = $312,500 – $187,500 – Expenses $100,000

Inventory Turnover 5 Ending Inventory

= = =

Cost of Goods Sold ÷ Average Inventory $187,500 ÷ [($0 + Ending Inventory) ÷ 2] $75,000

Receivables Turnover

= =

Net Credit Sales ÷ Average Accounts Receivable $312,500 ÷ [($0 + Ending Accounts Receivable)÷2) Ending $78,125

Quick Ratio

=

.5 Cash

= =

(Cash + Accounts Receivable + Marketable Securities) ÷ Current Liabilities (Cash + $78,125 + $0) ÷ $200,000 $21,875

8 Accounts Receivable

=

Tumwater Canyon Campsites Income Statement For the Year Ended December 31, 2012 Sales Cost of goods sold Gross profit Expenses Net income

$312,500 187,500 $125,000 100,000 $ 25,000

Tumwater Canyon Campsites Statement of Current Assets and Liabilities December 31, 2012 Current assets Cash Accounts receivable Inventory Total current assets

$ 21,875 78,125 75,000 $175,000

Current liabilities Accounts payable

$200,000

Total current liabilities

________ $200,000

27


P5–9 a. Mountain-Pacific Railroad Common-Size Balance Sheet December 31, 2012 and 2011 2012 Assets Current assets: Cash Short-term marketable securities Accounts receivable Inventory Prepaid expenses Total current assets Long-term investments Property, plant, and equipment Accumulated depreciation Total assets Liabilities and Stockholders' Equity Current liabilities: Accounts payable Wages payable Dividends payable Income taxes payable Current portion of long-term debt Total current liabilities Mortgage payable Common stock ($10 par value) Additional paid-in capital Retained earnings Total liabilities and stockholders' equity

2011

Dollar

%

10,000 125,000 500,000 200,000 50,000 $ 885,000 225,000 430,000 (65,000) $ 1,475,000

0.68% 8.47% 33.90% 13.56% 3.39% 60.00% 15.25% 29.15% (4.40%) 100.00%

$

$

10,000 5,000 125,000 50,000 100,000 290,000 350,000 200,000 135,000 500,000 1,475,000

$

$

$

29

Dollar

%

312,000 120,000 150,000 210,000 75,000 $ 867,000 225,000 540,000 (100,000) $ 1,532,000

20.36% 7.83% 9.79% 13.71% 4.90% 56.59% 14.69% 35.25% (6.53%) 100.00%

0.68% 0.34% 8.47% 3.39% 6.78% 19.66% 23.73% 13.56% 9.15% 33.90%

$

50,000 2,000 5,000 35,000 175,000 267,000 450,000 110,000 95,000 610,000

3.26% 0.13% 0.33% 2.29% 11.42% 17.43% 29.37% 7.18% 6.20% 39.82%

100.00%

$ 1,532,000

100.00%

$


P5–9

Continued Mountain-Pacific Railroad Common-Size Income Statement For the Years Ended December 31, 2012 and 2011 2012

Revenue: Net cash sales Net credit sales Total revenue Cost of goods sold: Beginning inventory Net purchases Cost of goods available for sale Less ending inventory Cost of goods sold Gross profit Selling & administrative expenses: Depreciation expense General selling expenses General administrative expenses Total selling & administrative exp. Income from operations Interest expense Income from continuing operations (before taxes) Income taxes Income before unusual items Unusual loss (net of tax benefit of $60,000) Net income

2011

Dollar

%

Dollar

%

1,955,000 4,150,000 6,105,000

32.02% 67.98% 100.00%

$

2,775,000 1,410,000 4,185,000

66.31% 33.69% 100.00%

210,000 4,005,000 4,215,000 200,000 4,015,000 2,090,000

3.44% 65.60% 69.04% 3.28% 65.76% 34.24%

$

300,000 2,475,000 2,775,000 210,000 2,565,000 1,620,000

7.17% 59.14% 66.31% 5.02% 61.29% 38.71%

$

75,000 575,000 480,000

1.23% 9.42% 7.86%

$

90,000 600,000 420,000

2.15% 14.34% 10.04%

$ $

1,130,000 960,000 50,000

18.51% 15.73% 0.82%

$ $

1,110,000 510,000 65,000

26.53% 12.18% 1.55%

$

14.91% 5.08% 9.83%

$

$

910,000 310,000 600,000

$

445,000 151,000 294,000

10.63% 3.61% 7.02%

$

115,000 485,000

1.88% 7.95%

$

0 294,000

0.00% 7.02%

$ $ $ $ $ $

$

$ $ $

By looking at the common-size balance sheets and income statements, we can observe the following: 1. The proportion of current assets to total assets has increased slightly from 57% to 60%. The composition of current assets has changed dramatically. Cash balance has declined by about 19% and accounts receivables have gone up by about 24%. 2. Since current liabilities are up by 2% and long-term debt is reduced by only 6%, it is not clear, without the statement of cash flows, where the cash went. 3. Such a dramatic increase of 24% in the accounts receivable can be a cause for concern. Is the company relaxing its credit policies or is it having a poor collection year? Depending upon the cause, one may anticipate that net cash flow problems could occur in 2013.

30


P5–9

Concluded

4. Since retained earnings are down by approximately 6%, and the net income is slightly up, one can only conclude, in the absence of other information, that a hefty dividend was probably paid. This could also explain the dramatic decrease in the cash balance. 5. It seems that the relative composition of cash versus credit sale is switching from 2011 to 2012. This corroborates the dramatic increase in accounts receivable. 6. It is also clear that the company’s increased cost of goods sold has cut into its gross profit, which is down by almost the same amount (4%) as the increment in the cost of goods sold. 7. The company has been quite successful in running tight operations, as is evidenced by a reduction in the total selling and administrative expenses. Reductions such as this would add value to its stock in the long run. b. The proportion of credit sales and cash sales to total sales changed dramatically from 2011 to 2012. The company made approximately twice as many credit sales during 2012 as it made during 2011. This shift flowed through to the balance sheet. Fewer cash sales caused (1) the Cash balance to decrease and (2) the Accounts Receivable balance to increase during 2012. c.

Common-size financial statements allow people to make comparisons across time and across companies by providing a benchmark against which to make the comparisons. Standard financial statements allow only absolute comparisons. By providing a benchmark, common-size financial statements allow relative comparisons. Such comparisons allow financial statement users to focus on the relative importance of an account rather than on whether an account simply increased or decreased in absolute terms. Further, common-size financial statements can provide financial statement users with insights as to why an account balance changed or why a certain trend has developed. For example, in the case of Mountain-Pacific Railroad, the shift in the relative importance of cash and accounts receivable can be explained by examining the shift in the relative importance of cash and credit sales.

P5–10 a. Return on Equity = Net Income ÷ Average Stockholders' Equity 2011: $294,000 ÷ $815,000 = .361 2012: $485,000 ÷ [($815,000 + $835,000) ÷ 2] = .588 Current Ratio = Current Assets ÷ Current Liabilities 2011: 2012:

$867,000 ÷ $267,000 = 3.247 $885,000 ÷ $290,000 = 3.052

Quick Ratio = (Cash + Marketable Securities + Accounts Receivable) ÷ Current Liabilities 2011: 2012:

($312,000 + $120,000 + $150,000) ÷ $267,000 = 2.180 ($10,000 + $125,000 + $500,000) ÷ $290,000 = 2.190

31


P5–10

Continued

Return on Assets = (Net Income + [Interest Expense (1 – Tax Rate)]) ÷ Average Total Assets 2011: ($294,000 + [$65,000 (1 - .34)]) ÷ $1,532,000 = .220 2012: ($485,000 + [$50,000 (1 - .34)]) ÷ [($1,532,000 + $1,475,000) ÷ 2] = .345 Receivables Turnover = Net Credit Sales ÷ Average Accounts Receivable 2011: 2012:

$1,410,000 ÷ $150,000 = 9.400 $4,150,000 ÷ [($150,000 + $500,000) ÷ 2] = 12.769

Earnings per Share = Net Income ÷ Average Number of Common Shares Outstanding 2011: 2012:

$294,000 ÷ 11,000 = $26.73 $485,000 ÷ [(11,000 + 20,000) ÷ 2] = $31.29

Price/Earnings Ratio = Market Price per Share ÷ Earnings per Share 2011: 2012:

$45.00 ÷ $26.73 = 1.684 $70.00 ÷ $31.29 = 2.237

Debt/Equity Ratio = Total Liabilities ÷ Total Stockholders' Equity 2011: 2012:

($267,000 + $450,000) ÷ $815,000 = .880 ($290,000 + $350,000) ÷ $835,000 = .766

Return on Sales = (Net Income + [Interest Expense (1 – Tax Rate)]) ÷ Net Sales 2011: 2012:

($294,000 + [$65,000 (1 - .34)]) ÷ $4,185,000 = .081 ($485,000 + [$50,000 (1 - .34)]) ÷ $6,105,000 = .085

Financial Leverage = Return on Equity – Return on Assets 2011: 2012:

.361 – .220 = .141 .588 – .345 = .243

Dividend Yield = Dividend per Share ÷ Market Price per Share 2011: 2012:

($10,000 ÷ 11,000 shares) ÷ $45 = .020 {$595,000 ÷ [(11,000 + 20,000) ÷ 2]} ÷ $70 = .548

Return on Investment = Market Price0 2011: 2012:

(Market Price1 – Market Price0 + Dividends per Share) ÷

($45 – $50 + $0.91) ÷ $50 = –.082 ($70 – $45 + $38.39) ÷ $45 = 1.409

32


P5–10

Continued

Interest Coverage Ratio = (Net Income Before Taxes and Interest Expense) ÷ Interest Expense 2011: 2012:

($445,000 + $65,000) ÷ $65,000 = 7.846 ($910,000 + $50,000) ÷ $50,000 = 19.200

Inventory Turnover = Cost of Goods Sold ÷ Average Inventory 2012: $4,015,000 ÷ [($210,000 + $200,000) ÷ 2] = 19.585 It appears that during 2012 the company became more efficient at using capital provided by all investors and by equity owners. Both return on assets and return on equity increased by over 50% during 2012. Further, as evidenced by the increase in financial leverage, the company was also more efficient at using debt to benefit the equity owners. The dramatic increase in inventory turnover is probably the primary reason the company became more efficient at using capital. The higher number of inventory turns allowed the company to generate more profits, thereby increasing return on equity, return on assets, financial leverage, and earnings per share. The company also became slightly more efficient at managing its costs, as evidenced by the increase in return on sales. The company has more than sufficient current assets to meet its current liabilities, as evidenced by its current ratio. The company's receivable turnover increased dramatically during 2012, which indicates that it is doing a better job of collecting from its customers. Closer inspection of the receivable turnover, however, reveals that Mountain-Pacific may actually be doing a worse job of collecting from its credit customers. If the 2012 receivable turnover is calculated using just the December 31, 2012 receivables balance instead of using the average receivables balance, the receivable turnover falls to 8.3, which is less than the turnover rate in 2011. Further, the increase in the inventory turnover may indicate future solvency problems. As the inventory turnover increases, the company will have to acquire inventory more often. Therefore, increasing the number of inventory turnovers places added pressure on the company to have sufficient cash to meet its debts as they come due. If the company is unable to generate cash from its receivables on a timely basis, and if it continues to suffer a decline in cash sales, it could very well experience severe solvency problems.

33


P5–10

Concluded

b. Based on the average of the company's 2011 and 2012 ratios, Mountain-Pacific's return on equity, current ratio, and return on assets are almost identical to the industry averages. While the absolute levels of these ratios are similar, the trend of Mountain-Pacific's ratios provides additional information on the company's performance. Based on return on equity, the company has become more efficient during 2012 at managing the equity owners' capital, and based on return on assets, the company has also become more efficient at managing the capital provided by both debt and equity investors. These trends imply that Mountain-Pacific has also become more effective at using debt to benefit the equity owners. Further, the company is now more efficient than the average company in the industry. Several other ratios, such as receivables turnover, return on investment, and times interest earned, indicate that Mountain-Pacific is also performing better than the industry average. When Mountain-Pacific’s quick ratio is compared to the industry average, the company's solvency position appears to be better. Without having additional information about Mountain-Pacific's solvency position, it is difficult to conclude how the company is performing relative to the rest of the industry.

P5–11 As a loan officer, I would be concerned with whether a potential borrower has the ability to meet its debts as they come due. Since both companies are requesting only nine-month loans, I would be interested in the potential borrowers' short-term solvency. Therefore, I would examine their current ratios and quick ratios. Further, I would consider the effect of the potential loan on these ratios. The current ratio is calculated as current assets divided by current liabilities. Selig Equipment: Mountain Bike:

$715,000 ÷ ($285,000 + $125,000) = 1.74 $835,000 ÷ ($325,000 + $125,000) = 1.86

It appears that both companies have more than sufficient current assets to meet their current obligations, including the new loan. However, some current assets, such as prepaid expenses and inventory, are not near-cash assets. Thus, a better measure of a potential borrower's ability to meet its current obligations is the quick ratio. This ratio is calculated as the sum of cash, marketable securities, and accounts receivable divided by current liabilities. Again, the effect of the new loan should be considered. Selig Equipment: Mountain Bike:

($15,000 + $215,000) ÷ ($285,000 + $125,000) = .561 ($160,000 + $470,000) ÷ ($325,000 + $125,000) = 1.400

34


P5–11

Concluded

Based on the quick ratio, Mountain Bike, Inc. appears to be a much better risk than Selig Equipment. Mountain Bike has approximately 2.5 times more near-cash assets available than Selig Equipment to meet its current obligations. Therefore, Mountain Bike does not have to rely as heavily on converting other assets to cash as Selig does to meet its obligations. The company that can most readily convert its inventory and receivables to cash might be the better risk. Two possible measures of a company's ability to generate cash from its receivables and inventory are the turnover and number-of-days ratios. Receivables turnover is calculated as net credit sales divided by average accounts receivable, and the number of days for receivables is calculated as 365 divided by the receivables turnover. Receivables turnover: Selig Equipment: $1,005,000 ÷ $215,000 = 4.67 Mountain Bike: $1,625,000 ÷ $470,000 = 3.46 Number of days: Selig Equipment: Mountain Bike:

365 ÷ 4.67 = 78.16 365 ÷ 3.46 = 105.49

These ratios indicate that Selig Equipment, on average, collects its receivables 27 days quicker than Mountain Bike. Therefore, Selig Equipment can more easily convert its receivables to cash than Mountain Bike can. Inventory turnover is calculated as cost of goods sold divided by average inventory, and the number of days is calculated as 365 divided by inventory turnover. Inventory turnover: Selig Equipment: Mountain Bike:

$755,000 ÷ $305,000 = 2.48 $960,000 ÷ $195,000 = 4.92

Number of days: Selig Equipment: Mountain Bike:

365 ÷ 2.48 = 147.18 365 ÷ 4.92 = 74.19

These ratios bode well for Mountain Bike. Mountain Bike sells its inventory, on average, 73 days sooner than Selig Equipment sells its inventory. This difference implies that Mountain Bike generates more sales which, in turn, implies that it generates more accounts receivable. Although Mountain Bike does not turn over its receivables as often as Selig Equipment, it has a larger amount of receivables to turn over. Thus, Mountain Bike potentially has more assets that can easily be converted into cash than Selig Equipment. Based upon Mountain Bike's superior quick ratio and potential ability to generate cash from its larger receivables base, I would recommend that the bank grant the loan to Mountain Bike, Inc.

35


P5–12 a. Watson Metal Products' 2013 income statements under the different financing alternatives would be as follows.

Income from operations* Interest expense Net income before taxes Income taxes Net income * $16,500,000

Alternative 1

Alternative 2

Alternative 3

$ 16,500,000 4,000,000 $ 12,500,000 5,000,000 $ 7,500,000

$ 16,500,000 4,750,000 $ 11,750,000 4,700,000 $ 7,050,000

$ 16,500,000 4,375,000 $ 12,125,000 4,850,000 $ 7,275,000

= $15,000,000 2013 income from operations from non-French operations per the 2012 income statement + $1,500,000 2013 income from operations from French operations.

The formulas for the requested ratios are: Earnings per Share Return on Equity Return on Assets Financial Leverage Debt/Equity Ratio Note:

= = = = =

Net Income ÷ Average Number of Common Shares Outstanding Net Income ÷ Average Stockholders' Equity (Net Income + [Interest Expense (1– Tax Rate)]÷ Average Total Assets Return on Equity – Return on Assets Total Liabilities ÷ Total Stockholders' Equity

Although several of the ratios use averages, ending balances were used as specified in the problem.

Alternative 1 EPS: $7,500,000 ÷ (2,000,000 shares* + 200,000 shares) = $3.41 * 2,000,000 shares = $6,600,000 2009 net income ÷ $3.30 2012 earnings per share ROE: $7,500,000 ÷ ($45,000,000 + $5,000,000a + $7,500,000b) = .1304 a $5,000,000 = 200,000 shares  $25 per share b $7,500,000 = 2013 net income

ROA: ($7,500,000 + [$4,000,000 (1 – .4)]) ÷ ($35,000,000 + $45,000,000 + $5,000,000 $7,500,000) = .1070 Leverage: .1304 – .1070 = .0234 Debt/Equity:

$35,000,000 ÷ ($45,000,000 + $5,000,000 + $7,500,000)

36

= .609

+


P5–12

Continued

Alternative 2 EPS: $7,050,000 ÷ 2,000,000 shares = $3.53 ROE: $7,050,000 ÷ ($45,000,000 + $7,050,000)

= .1354

ROA: ($7,050,000 + [$4,750,000 (1 - .40]) ÷ ($35,000,000 + $45,000,000 + $5,000,000 $7,050,000) = .1076

+

Leverage: .1354 – .1076 = .0278 Debt/Equity:

($35,000,000 + $5,000,000) ÷ ($45,000,000 + $7,050,000) = .768

Alternative 3 EPS: $7,275,000 ÷ (2,000,000 shares + 100,000 shares) = $3.46 ROE: $7,275,000 ÷ ($45,000,000 + $2,500,000* + $7,275,000) = .1328 * $2,500,000 = 100,000 shares  $25 per share ROA: ($7,275,000 + [$4,375,000 (1 - .4)]) ÷ ($35,000,000 + $45,000,000 + $5,000,000 $7,275,000) = .1073

+

Leverage: .1328 – .1073 = .0255 Debt/Equity: ($35,000,000 + $2,500,000) ÷ ($45,000,000 + $2,500,000 + $7,275,000) = .685 b. Alternative 2 prevents a dilution of the stockholders' position. Since this alternative did not require any additional shares of stock to be issued, it provides the largest earnings per share. Alternative 2 allows the company to more effectively manage its stockholders' investment, as evidenced by return on equity, and all investments, as evidenced by return on assets. The only potentially serious drawback of this alternative is that it makes the company more risky, as evidenced by it having the largest debt/equity ratio. Further, Alternative 2 allows the company to use debt to benefit stockholders more effectively than is allowed with either of the other two alternatives. Under Alternative 3, stockholders earn a slightly smaller return on their equity, but incur fewer risks, since the company has issued less debt. Alternative 1 provides a marginally lower return to stockholders, but imposes even less risk on them. Stockholders must trade off the risk from issuing debt against the benefits of issuing debt. If the company is close to violating debt covenants or projects weak future cash flows, then Alternatives 1 or 3 would probably be preferable. Otherwise, Alternatives 2 or 3 would probably be preferable.

37


P5–12 c.

Concluded

Alternative 1 $3.30 = ($6,600,000 + Net income from expansion project) ÷ (2,000,000 shares + 200,000 shares) Net income from expansion project = $660,000 Alternative 2 $3.30 = ($6,600,000 + Net income from expansion project) ÷ 2,000,000 shares Net income from expansion project = $0 Alternative 3 $3.30 = ($6,600,000 + Net income from expansion project) ÷ (2,000,000 shares + 100,000 shares) Net income from expansion project = $330,000

P5–13 Note:

Although some ratios use average balances, year-end balances were used in the ratios as directed in the problem.

a.

Return on Equity = .75 = Total Stockholders' Equity =

b.

Debt/Equity Ratio = .4 = Total Liabilities =

c.

d.

e.

f.

Total Assets

= = =

Return on Assets .65 Interest Expense

Total Liabilities ÷ Total Stockholders' Equity Total Liabilities ÷ $600,000 $240,000 Total Liabilities + Total Stockholders' Equity $240,000 + $600,000 $840,000

= = =

(Net Income + Interest Expense) ÷ Total Assets ($450,000 + Interest Expense) ÷ $840,000 $96,000

Net Income After Taxes = $450,000 = Net Income Before Taxes = Return on Sales = .2 = Net Sales =

Net Income ÷ Total Stockholders' Equity $450,000 ÷ Total Stockholders' Equity $600,000

Income Before Taxes  (1 – Tax Rate) Net Income Before Taxes  (1 – 34%) $681,818

Net Income ÷ Net Sales $450,000 ÷ Net Sales $2,250,000

g. Credit Sales = Net Sales  80% = $2,250,000  80% = $1,800,000 38


P5–13

Concluded

h. Receivables Turnover

= 25 = Accounts Receivable =

i.

Net Credit Sales ÷ Accounts Receivable $1,800,000 ÷ Accounts Receivable $72,000

Net Sales  55% = $2,250,000  55% = $1,237,500

Cost of Goods Sold=

j.

Average Days' Supply of Inventory = 12.167 = Inventory Turnover =

k.

Inventory Turnover 30 Inventory

l.

Current Liabilities =

m.

n.

Current Ratio 3.00 Current Assets

Cost of Goods Sold ÷ Inventory $1,237,500 ÷ Inventory $41,250

= =

Total Liabilities  35% $240,000  35% $84,000

= = =

Current Assets ÷ Current Liabilities Current Assets ÷ $84,000 $252,000

Quick Ratio Liabilities 2.0 Marketable Securities

o. Noncurrent Assets=

p.

= = =

365 ÷ Inventory Turnover 365 ÷ Inventory Turnover 30

=

(Cash + Accounts Receivable + Mkt. Securities) ÷ Current

= =

($68,000 + $72,000 + Marketable Securities) ÷ $84,000 $28,000

Total Assets – Current Assets = $840,000 – $252,000 = $588,000

Earnings per Share $16.00 Common Shares Outstanding =

= Net Income ÷ Number of Common Shares Outstanding = $450,000 ÷ Number of Common Shares Outstanding 28,125 shares

39


P5–14 a. Total Revenues equal $43,269 with the percentage breakdown as follows: Frito Lay North America Quaker Foods North America Latin America Foods Americas Beverages U.K & Europe Middle East, Africa, Asia

28.9% 4.4% 13.6% 25.3% 14.9% 12.9%

b. Ranking the segments on profit margin (Profits/Revenues) yields: 1. Quaker Foods North America 30.6% 2. Frito Lay North America 23.7% 3. Americas Beverages 18.5% 4. Latin America Foods 15.2% 5. U.K. & Europe 12.6% 6. Middle East, Africa, Asia 12.0% c.

Ranking the segments on Return on Assets (Profits/Assets) yields: 1. Quaker Foods North America 56.2% 2. Frito Lay North America 47.1% 3. Latin America Foods 29.7% 4. Americas Beverages 26.4% 5. Middle East, Africa, Asia 16.8% 6. U.K. & Europe 9.4%

d. Quaker Foods is the smallest of the segments, but it is also the most profitable. Pepsico’s management should focus efforts on growing the Quaker Oats business in terms of revenue (while maintaining the profit margins). Frito Lay is the largest segment (slightly ahead of the beverage operation for North, Central and South America) and its profit margins are the second highest; this division clearly distinguishes Pepsico from its rival Coca-Cola. One final conclusion can be seen in the ROA numbers for Europe and the Middle East; the company has dedicated significant investment in these markets (seeing them as future growth areas), but the profits have not yet matched the investment in assets.

40


P5–15 a. ROE Net Income/Average Stockholders’ Equity 5,285/11,167.5 = 47.3% ROA [Net Income + (1 – Tax Rate)x (Interest Expense)]/Average Total Assets [5,285 + (1 - .26)(506)]/36,722 = 15.4%

Return On Sales [Net Income + (1 – Tax Rate)x (Interest Expense)]/Net Sales [5,285 + (1 - .26)(506)]/40,523 = 14.0%

Common Equity Leverage Net Income/[Net Income + (1 – Tax Rate)x (Interest Expense)] 5,285/[5,285 + (1 - .26)(506)] = 0.93

Capital Structure Leverage Average Total Assets/Average Stockholders’ Equity [(36,142 + 37,302)/2]/ [(9,948 + 12,387)/2] = 36,722/11,167.5 = 3.29 Debt/Equity Ratio Average Total Liabilities/Average Stockholders’ Equity [(25,770 + 24,483)/2]/ [(9,948 + 12,387)/2] = 25,126.5/11,167.5 = 2.25 Long-term Debt Ratio Long-term Liabilities/Total Assets 11,970/(24,967 + 11,175) = 0.33 Current Ratio Current Assets/Current Liabilities 11,175/13,800 = 0.81 Quick Ratio (Cash + Marketable Securities + Accounts Receivables)/Current Liabilities (2,561 + 3,823)/13,800 = 0.46 Interest Coverage (Net Income + Tax Expense + Interest Expense)/Interest Expense (5,285 + 1,844 + 506)/506 = 15.1 Accounts Payable Turnover Cost of Goods Sold*/Average Accounts Payable 21,342/[(7,824 + 8,017)/2] = 2.69 365 days/2.69 times = 135.7 days Receivables Turnover Sales/Average Accounts Receivable 40,523/[(3,823+ 4,194)/2] = 10.11 365 days/10.11 times = 36.1 days Inventory Turnover Cost of Goods Sold*/Average Accounts Payable 21,342/[(3,889 + 3,894)/2] = 5.48 365 days/5.48 times = 66.6 days 41


Fixed Asset Turnover Sales/Average Fixed Assets 40,523/[(5,957 + 6,284)/2] = 6.62 Total Asset Turnover Sales/Average Total Assets 40,523/[(36,142 + 37,302)/2] = 1.10 P5–15 Concluded *Cost of Goods Sold found in footnotes Based on the Current and Quick Ratios, the company will have to look to the conversion of long-term assets to Cash (from either operating those assets or liquidating them) to satisfy current liabilities. The company pays its trade suppliers extremely slowly, but it can cover its interest payments very easily. From an earnings power perspective, the high returns are matched by strong and consistent cash flow generated from operations. b. As demonstrated in the ROE Model, Return on Equity is driven by Return on Assets, Capital Structure Leverage and Common Equity Leverage. Although the 2006 numbers are not provided (for 2007 averages), it is obvious that Equity levels are down in 2008 (9.9, down from 12.4) and that Liabilities levels are up— meaning that Unilever has leveraged up its position in 2008. At the same time, profits for 2008 are significantly higher than those in 2007. Those two factors (higher profits, lower equity) indicate that the 2008 ROE figure is substantially higher than that of 2007. Given that asset levels dropped only 3% (37.3 down to 36.1), the 27.8% increase in profit (5,285, up from 4,136) indicates that Return on Assets were also higher. The growth in ROE was driven by both a growth in profitability relative to the asset base AND a higher use of leverage.

42


ISSUES FOR DISCUSSION ID5–1 (1) Bank of America (2) Bed, Bath & Beyond (3) HP

(4) Kelly Services

Bank of America is Company #1. As a commercial bank, B of A generates service revenue (see Income Statement) and carries receivables (loans) that are funded by deposits (current liabilities on the balance sheet). Banks also are highly leveraged; Company #1 has the lowest equity of the four companies shown. Bed, Bath & Beyond is Company #2. As a retailer, BB & B generates sales revenue from providing inventory to its customers. Company #2 is the only company of the four shown with 100% sales revenue. Further, the company has a significant investment in inventories, as do all retailers. Hewlett Packard is Company #3. As a technology and consulting company, HP will generate both sales revenue (from the sale of products such as printers) and service revenue (from its consulting business). Of the four companies, only #3 has a mix of sales and service revenue. The company also has investment in inventories, receivables and long-term assets, common for a manufacturing concern. Kelly Services is Company #4. As a temp agency, Kelly will generate service revenue from placing workers and will also carry accounts receivable from its large clients. The company does not carry inventory and does not need a significant investment in long-term assets to operate.

ID5–2 (1) General Electric

(2) EchoStar

(3) Walgreen’s

(4) Campbell’s Soup

General Electric is Company #1. GE operates as both a manufacturer and a diversified financial services firm, generating both sales and service revenue. The financial services business, similar to Bank of America in ID5-1, will carry a significant amount of receivables. EchoStar is Company #2. The company sells customers satellite dishes and the television services that are received by the dishes. Further, satellite television services are a relatively new business; start-up operations often show losses for a number of years and Company #2 is showing a loss on the income statement. Walgreen’s is Company #3. As a retailer, the company will generate sales revenue and carry heavy investment in inventories. (The inventory line is greater in #3 than it is in #4.) Campbell’s is Company #4. The description indicates that Campbell’s has been growing by acquisition, which usually generates goodwill, an intangible asset carried under “Other Assets”. Further, manufacturers typically carry balances in receivables, inventory and long-term assets such as property, plant and equipment. 43


ID5–3 Book value (stockholders’ equity) is a GAAP number that is derived from accounting values that may not reflect current market prices. For example, the book value of a company might include fixed assets that were purchased many, many years ago and were recorded on the books at historic costs; if market values for those fixed assets have increased, there will be a significant difference between the book value of the company (based on historic costs) and the market value (based on current market prices of the assets). A price-to-book ratio would be high if a company had many assets on its books at historic costs that are well below current market prices; if inflation and market appreciation have caused the market value of fixed assets to grow well above the original cost, and if many years have passed since the purchase of the assets, then the price-to-book ratio of a company will be high. Businesses with assets that have been held for a long time, or businesses with assets that have grown significantly in value (real estate in Manhattan, for example) will have high PB ratios. On the other hand, if a business owns assets that have lost significant market value due to a recession (like banks owning mortgage securities in 2008-2009), then the book values will be above market prices, creating a low PB ratio.

ID5–4 (a) A consensus earnings forecast is the most common or average earnings forecast by Wall Street analysts. Companies wish to consistently beat the estimates because, in general, the stock of their company tends to go higher as they beat the consensus estimate. If a company misses a forecast then analysts could downgrade the stock and the stock price could fall. (b) Companies don’t want to beat the estimates by too much because it may result in future expectations beyond what the company can deliver. So while the stock price might go up significantly now it could lead to a future earnings disappointment which could cause the stock to fall significantly. (c) Management can do two things. The first is to manage the expectations of research analysts. Management can “guide” analysts in terms of sales and expenses. The second is that management may try to “manage earnings” by increasing accruals or taking write-offs in good quarters and reducing accruals or taking gains in weak quarters. In both cases the idea is to keep earnings on a smooth path from quarter to quarter. Analysts have concern about companies who manage earnings because they feel they are not seeing the true picture of a company’s performance if the bottom line is smoothed by management assumptions regarding accruals and estimates.

ID5–5 (a) It is likely that analysts saw Kodak’s move coming. Analysts closely monitor industry conditions and technological changes and therefore understood that digital photography was the future. Pinpointing the exact dollar amount of the charge, however, would be more difficult for the analysts. Management, with its access to all the data and numbers of the company, is in a much better position to estimate dollar amounts for restructuring. The stock market probably applauded the move, as it knew that Kodak’s traditional business had been shrinking in recent years. Financial markets often reward companies with a boost to the stock price when restructuring announcements are made. The fact that management is proactively addressing a problem is often seen as a positive. 44


(b) Management of Kodak was taking all of the bad news at once. By booking the entire restructuring expense at the date of the announcement, and not as the payments are made, is a move companies take to show all the negatives today, holding the positives for the future. The thought is that the negative news is going to hit anyway, so the company might as well take all of the bad publicity at once. Future periods, therefore, will not be burdened with the expenses and will show better results. Earnings management of this type is very common among public companies. If the company overestimates the charges that it takes today it allows the company to “improve” its performance in future periods as it takes the overestimated expenses back into income. (c) It would be most prudent to take the restructuring expense out of the financial statements and calculate your ratios excluding the charge. If the charge is a one-time adjustment, to get a true picture of the company’s trends, the expense should not be included in the analysis. (d) Several cases of companies facing technological obsolescence exist. For example, network television companies such as ABC, NBC and CBS are vulnerable to the advances of cable and satellite television providers. Companies specializing in the delivery of documents such as Federal Express and UPS (as well as the US Postal Service) are vulnerable to new methods of communication like email. Music companies (and musicians) that historically made their profits by selling album and compact discs to consumers are vulnerable to the technological advances of transmitting and sharing music over the internet. Blockbuster’s brick-and-mortar movie rental business is vulnerable to online movie streaming.

ID5–6 Staples might want to separate the losses from its dot-com operations in order to help maintain a high stock price. By excluding the losses from the dot-com operation Staples will be reporting a higher net income. This will tend to have a positive effect on its stock price. This is especially true if its competitors include similar types of losses in their reported income. Management may justify not including the results from the dot-com operation in its reported income because Staples’ reported income in past years did not include this operation and so by including these numbers would reduce the consistency and in some cases the comparability of the numbers. Management may also may the claim that including the dot-com operations would be confusing to investors and not allow investors to evaluate the primary Staples business. The policy of not including the dot-com operations in reported income doesn’t seem to fit with the overall goals of full disclosure and would seem to be shielding investors from valuable information. The dot-com operation is not any different than a company entering into a new market (product or geography) and having losses during the early years. This seems to be a very questionable accounting policy and not an appropriate one.

ID5–7 A stock price that has already fallen dramatically has less risk than one that is trading at its all time high. A number of investors have already sold the stock and as the stock price falls there may be other investors who become interested in the stock. A falling stock price could also be reflecting that short sellers of the stock have been knocking the price down. Short sellers eventually have to buy back the shares that they have shorted.

45


The fact that the company reported positive free cash flow means that the company is able to fund itself through its operations and is not forced to go to the capital markets (sell stock) when the stock price is low. Using conservative accounting methods implies that the company is not inflating its earnings by using questionable accounting methods. Conservative accounting methods allow investors to have more confidence in the reported numbers. A build-up in raw inventory materials and partially-completed inventory would imply that management is preparing for higher sales levels by increasing its inventory in the pipeline. Since finished inventory has not been increasing this would seem to say that sales are in good shape. An increase in discretionary expenditures like research and development would be a strong statement by management that future business seems to be in very good shape. If management did not have a positive view on future sales then it probably would not increase spending in areas that were not critical. Many companies have undervalued assets on the balance sheet because fixed assets are carried at their original cost. Years later many of those assets have appreciated in value (buildings) but yet this increase in value has not been shown on the income statement. Having little or no debt while still having a high return on assets is a very positive sign for the company. This means that the company could borrow money for high return projects or to buyback the company’s stock. Consistency between what the president’s letter says and what actually happens again provides a lot more confidence for investors that the company is going to be able to execute on its strategy. Many companies say they will do a lot of different things and then never actually get to them. Confidence in what management says will draw more investors to the company’s stock.

ID5–8 a. An efficient market implies that all information about a company is reflected in the price of the company's stock as soon as the information becomes available to the public. Since the information in financial reports is available to the public, market efficiency implies that as soon as the financial report is made public, all the information in the report is reflected in the company's stock price. Thus, investors could not use financial reports to identify undervalued securities because the investors would not have any information that is not already reflected in the stock price. b. Although accounting information may not be useful in identifying undervalued or overvalued stocks, it can be useful in assessing a company's solvency and earning power, which are important components of an investment decision. For example, banks use financial statements to determine the creditworthiness of loan applicants.

ID5–9 Intangibles refer to the many assets that do not have physical characteristics. The majority of these are not recorded on the balance sheet of a company. A very valuable intangible asset that is not shown on the balance sheet is the brand name of a company. Examples of these would be the Coca-cola name, the Nike swoosh, and the Mercedes mark. While these assets are not included on the balance sheet they indirectly impact the income statement. The whole idea of a valuable brand name is that consumers want to buy your product and this is reflected in higher sales. In talking about the balance sheet it appears that his comments are exactly correct. Compare the book value of a company to its market value. This gives a good indication of the value of the company that is not 46


reflected on the balance sheet. The income statement reflects that value of a brand name in the reported sales.

ID5–10 Buffett is making the case that the reaction of a weak management to a difficult operating environment is to try and slide through by using questionable accounting measures to make up for poor performance. A strong management would be able to work through the difficult times and not have to fudge the accounting numbers. Evaluating how management handles adversity is probably a good indicator of which companies will perform the best during good times. If a company’s customers are suffering in a difficult economic environment a strong management team would work on making sure that sales were made to viable customers while a weak management team would sell to any customer to show higher sales and then would probably understate the potential for bad debts in the financial statements. Eventually some of these sales will not be collected and those disappointments will lead to lower stock performance. Financial statement users should look at the accounting standards employed by companies and determine what this is saying about the quality of the management team and their ability to operate the company. Over time a good management team will figure out a way to make a company successful while a weak management team will almost always underperform its competitors..

ID5–11 a.

Receivables turnover and inventory turnover are the two ratios that probably best capture the suggested indicators.

b.

Inventory turnover is calculated as cost of goods sold divided by average inventory, and receivables turnover is calculated as net credit sales divided by average accounts receivable. A decreasing inventory turnover means that the company is taking longer to sell its inventory. Similarly, a decreasing receivables turnover means that the company is taking longer to collect its receivables. Both ratios have implications for solvency and earning power. If a company is turning over its inventory more slowly, the company will be generating fewer cash and credit sales. That is, the company will be generating fewer assets (such as cash or accounts receivable) through operations, thereby having lower earning power. Lower sales implies lower cash collections from cash or credit sales, which, in turn, implies that it will be more difficult for the company to pay its obligations as they come due (i.e., the company has lower solvency). Less sales also implies that the company will have fewer assets to reinvest in productive assets, which, in turn, implies that the company will be less profitable in the future (i.e., the company has lower earning power). If the company is collecting its receivables more slowly, it will have less cash available to pay its obligations—which implies lower solvency—and to invest in productive assets—which implies lower earning power. Because solvency and earning power are important to investors, any indication that a company is having solvency and/or earning power problems would be expected to result in a drop in the price of the company's stock. The ratios are more likely than reported earnings to pick up this information because management has much more discretion (at least in the short run) over reported earnings than over the items used to calculate the ratios.

47


ID5–12 Theoretically, the price of a company's stock equals the present value of the stock market's expectations of the cash flows an investor would realize from investing in that company's stock. Thus, the price of a company's stock is a function of the market's expectations about future cash flows. If the market receives information that causes it to revise its expectations, the price of the company's stock should change. Earnings information is one source of information about future cash flows. It is possible that an earnings announcement of lowered results can also give the market information about the company’s future that indicates an increase in future cash flows. Likewise, an announcement of higher earnings can contain information that is negative about the company’s prospects. The stock price is determined by the market’s interpretation of future results.

ID5–13 a. Many of the Internet companies that became publicly-traded in the 1990s ended in failure. The researchers were suggesting that a similar problem might happen again in the future with the 2007 companies going public before establishing themselves as profitable businesses. b. The worry is that the investors buying the IPO shares are buying the hype and promise of the company without having any proof of either the company’s viability or the soundness of its business model. In effect, the investors are paying now for profits that will supposedly be earned in the future. c.

Companies (and their investment bankers) are incented to sell shares when the selling price is high. When the overall market is at record levels, those selling stand to receive more cash from the proceeds of the IPO. The temptation to “go public” before profitability is strong when market prices are exceedingly high.

d. Investors should be examining a company’s Income Statement to see revenues versus expenses, but investors should also analyze the Statement of Cash Flow to understand the inflows and outflows of cash due to the company’s operations, investments and financings. Ideally, this analysis should be done on both historic data, as well as on projections of future periods.

ID5–14 The case asks you to assess Eli Lilly’s financial statements using the ROE model. Relevant ratios for Lilly are calculated below: (tax rate equals 24%; interest expense taken from the footnotes equals $228.3 million in 2008 and $228.3 million in 2007 ) 2008 Return on Equity = 2007 Return on Equity =

(20.5%) 24.1%

2008 Return on Assets = 2007 Return on Assets =

(6.8%) 12.8%

2008 Common Equity Leverage 2007 Common Equity Leverage

= =

.1.09 .945

2008 Capital Structure Leverage 2007 Capital Structure Leverage

= =

2.77 1.99

ROE = ROA X C.E.L. x C.S.L. 48


2008 2007

(.205) =(.068) x 1.09 x 2.77 .241 =.128 x .945 x 1.99

2008 Return on Sales = 2007 Return on Sales =

(9.32%) 16.78%

2008 Asset Turnover 2007 Asset Turnover ID5–14 Concluded

= =

.723 .763

ROA = P.M. x A.T. 2008 2007

(.068) = (.0932) x .723 (rounding) .128 = .1678 x .763

From 2007 to 2008, Eli Lilly experienced a sharp drop in Return on Equity from positive 24 percent to a negative 20 percent. In other words, investors used to receive 24 cents in return for every dollar invested in Lilly’s equity; those same investors lost 20 cents for each dollar invested in 2008. The ROE model can be used to pinpoint the variable(s) that is(are) driving the change in shareholder return. Three variables directly feed into ROE. As shown above, Eli Lilly experienced a strong decrease in Return on Assets but experienced higher leverage (from both Capital Structure Leverage and Common Equity Leverage). The leverage increases would normally drive up ROE, so the decrease in ROE is driven by the decrease in ROA. Two variables directly affect ROA: return on sales and asset turnover. Return on sales determines how well the company converts sales into profits, while asset turnover measures the company’s ability to generate sales from assets. In both cases, Lilly did a better job in 2007 than it did in 2008, driving the decrease in ROA. The company generated less sales from its assets and then less profits from its sales.

ID5–15 Profile #1 – EchoStar

Profile #2 – US Airways

Profile #3 – Wal-Mart

EchoStar is a relatively new company with a relatively new product. The company has been growing, as more consumers switch from traditional and cable television services to satellite television. The growth of the company has generated positive cash from operations. However, the company has had to take that cash and invest it back into its business; the negative cash from investing in all three years shows that the company is growing its business by investing in long-term assets. The growth is not sufficiently funded by operations, so the company is raising money from debt and equity sources (financing activities) to fund its business. At the present time, the company is not returning cash to its shareholders or repaying debt. It is borrowing additional funds and raising additional equity to fund its growth. US Airways operates in a difficult industry. The company has been unable to generate a positive cash flow from its operations, a definite red flag for any business. The cash outflow from operations has been paired with negative cash from investing activities, as the company must reinvest in long term assets (aircraft) to remain competitive in its industry. The only source of cash to fund the negative operations and the investments is financing (issuing debt and selling equity). The long term prospects for the company do not look healthy. No business can sustain itself through endless financing activities to fund cash; if the company is going to survive, it will eventually have to generate cash from its operations so that it can eventually repay debt and return some cash to shareholders. Wal-Mart is an excellent example of a successful company that has reached the stage in its maturity that its operations throw off enough cash to fund the company’s still-impressive growth (investing activities) AND

49


to return cash to shareholders and/or repay debt (financing activities). The company can rely on its daily operations to generate cash, which is then used to add more stores and fund dividends and debt reduction.

ID5–16 a) Financial ratios for Nike are outlined below: (tax rate = 24%)

Return on Equity

2008 18.0%

Return on Assets

11.8%

Return on Sales

7.9%

Common Equity Leverage

0.98

Capital Structure Leverage

1.56

Debt/Equity

0.52

Long-term Debt Ratio

0.097

Current Ratio

2.97

Quick Ratio Interest Coverage

1.93 49.67 ([1,486.7 +469.8 +40.2*]/40.2*)

Accounts Payable Turnover/Days on Hand

9.12 / 40.0 days

Accounts Receivable Turnover/Days on Hand

6.75 / 54.1 days

Inventory Turnover/Days on Hand Fixed Asset Turnover Asset Turnover

4.41 / 82.8 days 9.96 1.49

*see Footnote #1 and comparison of Interest Income and Interest Income, net Nike has generated impressive profitability numbers in the year ending 2008. Returns on equity equaled eighteen percent, while the return on sales is also a strong at nearly eight percent. Debt levels are very low; the company is very solvent, carrying almost three dollars in current assets for every dollar in current obligations; the quick ratio demonstrates that the company can easily cover its current obligations, even if its inventory is worthless. Inventory sits for 83 days before sale, while receivables are collected in 54 days. Finally, the company’s efficiency at converting assets into sales is shown by the last two turnover ratios.

50


ID5–16 Continued b) The United States operating segment generated 34% ($6,542.9/$19,176.1) of Nike’s sales. The Asia segment had the highest pre-tax profit margin at 25.68%, while the Americas segment had the highest asset turnover at approximately 68 times ($1,284.7/$18.8). c) ROE = ROA x Common Equity Leverage x Capital Structure Leverage ROA = Return on Sales x Asset Turnover Nike 2008 .180 = .118 x .98 x 1.56 .118 = .079 x 1.49 Nike’s strong return on equity is driven primarily by its ROA. The company’s equity base is relatively large, but the company is still able to generate a significant return on this equity by throwing off a strong return from its assets. The company appears to be debt-averse (low leverage), which means that it carries relatively more equity. To generate a satisfactory return for its shareholders, the company must do an excellent job converting its assets into acceptable returns. Driving the ROA are the two factors: converting sales from assets and converting those sales into profits. The company appears to be doing an excellent job generating revenue and then converting that revenue into bottom-line earnings. A true analysis would 1) take the company’s financial ratios over the last five years to see the changes and trends in the various performance metrics, and 2) compare Nike to its competitors to determine its relative strengths and weaknesses.

51


ID5–16 Concluded d) Based on the financial statements, some reasonable assumptions regarding historic ratios can be used to help predict future financial statements. Sales Growth was 14% in 2008 and only 3% in 2009; an average of the two years yields an assumption of 8.5% that could be used for predicting 2010 levels. Expense/Sales Ratio was 92.25%, 90% and 91% for the last three years, so a reasonable assumption would be 92% (yielding net income at 8% of sales). Sales/Total Assets was 1.45 and 1.49 in the last two years, so the more conservative 1.45 is a reasonable assumption. Finally, Liabilities/Total Assets was 34% and 37% for the last two years, so a 35% figure appears to be sound for future projections. Calculations for the projections are as follows: Sales: $19,176.1 x 1.085 = $20,806.1 Expenses: $20,806.1 x .92 = 19,141.6 (Net Income = $1,664.5) Assets: $20,806.1/Assets = 1.45; Assets = $14,349 Liabilities: $14,349 x .35 = $5,022.2 (Equity = $9,326.8) Projected Financial Statements: Nike, 2010 Assets

Net Sales Net Income

$14,349.0

Liabilities Equity

$ 5,022.2 9,326.8

$20,806.1 1,664.5

52


CHAPTER 6 THE CURRENT ASSET CLASSIFICATION, CASH, AND ACCOUNTS RECEIVABLE BRIEF EXERCISES BE6–1 a. Total Accounts Receivable

= Net Receivables + Allowance for Uncollectibles

2009 Total Accounts Receivable =

$3,623

2009 Total Accounts Receivable =

$3,716

+

$93

2009 Uncollectibles as a Percentage of Total Accounts Receivable = $93/$3,716 = 2.50% 2008 Total Accounts Receivable =

$4,618

2008 Total Accounts Receivable =

$4,708

+

$90

2008 Uncollectibles as a Percentage of Total Accounts Receivable = $90/$4,708 = 1.91% Therefore, the percentage increased. b. Since Emerson Electric is using the percentage of accounts receivable method (balance sheet approach), bad debt expense for 2009 would be the amount needed to adjust the allowance for doubtful accounts to $93. This number (bad debt expense) is impacted by the balance in the uncollectible account at the beginning of the year and the write-offs taken during the year by Emerson Electric.

BE6–2 a.

2007: Ending Allowance Balance Write-Offs + Recoveries $4,238 Bad Debt Expense for 2007 2008: Ending Allowance Balance Write-Offs + Recoveries $5,325 Bad Debt Expense for 2008

=

Beginning Allowance Balance + Bad Debt Charge –

= =

$3,945 + 4,431 – $5,966 + 1,828 $4,431

=

Beginning Allowance Balance + Bad Debt Charge –

= =

$4,238 + 7,518 – $8,162 + 1,731 $7,518

As the economy went into recession in 2008, companies that extend credit (such as GE’s capital division) had a more difficult time collecting receivables.

1


b.

2007: $5,966 write-offs; $4,138 write-offs, net of recoveries 2008: $8,162 write-offs; $6,431 write-offs, net of recoveries

c.

The allowance account grew by 25.7% (($5,325- $4,238)/$4,238) from 2007 to 2008. The allowance account grew at such a large rate due to the deterioration of the economy and GE’s belief that a greater amount future receivables will prove to be uncollectible as a result. By increasing the allowance balance the company is taking into consideration that receivables will not be as collectible as when the economy was healthier. Increasing the allowance lowers the “net realizable value” of the receivables on the balance sheet, which is prudent behavior given the economic climate.

BE6–3 a.

GE bad debts as a percentage of total revenues = $7.5/$182 = 4.1%; as a percentage of GECS revenues, the calculation is $7.5/$71 = 10.6%. GE overall revenues should be used since the bad debt provision is for GE and not for GECS.

b. On a balance sheet for GECS accounts receivable would be expected to be the largest account. Its primary role is as a financing company, and the receivables from the buyers of appliances would be a large asset of GECS. c.

GE is very large and has many subsidiaries that make it difficult to classify it as just a manufacturing, retail or service company. The overall GE business is best known as a manufacturing company. It does not have its own retail stores, so it is not a retailer. While there are many services offered with its products, its primary focus is as a manufacturer. Services, primarily financial services, are becoming more important for the consolidated operation.

EXERCISES E6–1 a. Cash. Money held in checking accounts is defined as cash, and there are no restrictions on the account. b. Cash. Checks are considered cash unless the checks cannot be cashed until a later date (i.e., postdated). In this case, the check date has passed, so the checks are considered cash. c. Investment. Certificates of deposit contain penalties for early withdrawal. Since the certificates mature outside the time frame of current assets, this source of cash is not readily available and should not be classified as Cash. d. Cash. Because banks have the right to demand notice prior to a withdrawal from a savings account, the cash in savings accounts is technically not readily available. However, since banks rarely exercise this right, savings accounts are considered cash. e. Cash. Petty cash is always considered cash. f. Restricted cash. Because the company does not have ready access to these funds, the $50,000 should not be reported as cash. The portion of the $50,000 corresponding to short-term loans (i.e., $15,000) should be classified in current assets as restricted cash, and the remaining $35,000 should be classified as a long-term investment or as an other asset. g. Cash. See (a).


E6–2 a. unrestricted cash b. investment c. restricted cash

d. e. f.

investment investment restricted cash

g.

unrestricted cash

E6–3 a. 12/12

1/5

Accounts Receivable (+A)........................................................... Sales (R, +SE) ....................................................................... Made sale on account.

40,000

Cash (+A) .................................................................................... Accounts Receivable (–A) .................................................... Collected cash from customer.

40,000

Cash (+A) .................................................................................... Cash Discount (–R, –SE) ............................................................. Accounts Receivable (–A) ....................................................

39,200 800

40,000

40,000

b. 12/20

40,000

The timing of the cash receipts would affect the income statement due to the cash discount. Cash discount is a contra sales account and is deducted from the sales revenue. Therefore, the net income is reduced by the amount of the cash discount during year 1 in case (b). Under option (a) the statement of Cash Flows will show no inflow during year 1, but during year 2 it will show an inflow from operating activities of $40,000. Under option (b) no inflow from operating activities will be shown in year 2, but during year 1 there will be an increase of $39,200 from operating activities.

E6–4 5/1/12

Accounts Receivable (+A)........................................................... Sales (R, +SE) ....................................................................... Sold lobster on account.

30,000

Accounts Receivable (+A)........................................................... Sales (R, +SE) ....................................................................... Sold cod on account.

20,000

Cash (+A) .................................................................................... Cash Discount (-R, -SE) ............................................................... Accounts Receivable (-A)..................................................... Collected cash from customer.

29,100 900

5/31/12 Cash (+A) .................................................................................... Accounts Receivable (-A)..................................................... Collected cash from customer.

20,000

5/5/12

5/6/12

30,000

20,000

30,000

20,000


E6–5 Allowance for Uncollectibles 2011 Write-Offs

2012 Write-Offs

10,000

22,000

12/31/11 Bad Debt Expense

30,0001

12/31/12 Bad Debt Expense

28,0002

Ending Balance

26,000

___________ 1 $1,500,000  .02 = $30,000 2 $1,400,000  .02 = $28,000 Overall, the bad debts estimates are sufficient to cover the write-offs.

E6–6 a.

Ending Allowance Balance $200,000

= Beginning Allowance Balance + Bad Debt Charge – Write-Offs + Recoveries = Beginning Allowance Balance + $162,500 – $195,000 + $45,000 = $187,500

Beginning Allowance Balance _____________ * $162,500 = Sales of $3,250,000  Estimated uncollectible percentage of 5%

b. [Assume the $4,200,000 cash collections includes the $45,000 recovery] Ending Accounts Receivable = Beginning Accounts Receivable + Credit Sales during the year – Cash Collected on account receivables during the year – Accounts Receivables written off + Written-Off Accounts Receivables Recovered. $7,500,000

= X =

X + $3,250,000 – $4,200,000 – $195,000 + $45,000 $8,600,000

E6–7 a. Bad Debt Charge (E, –SE)........................................................................... Allowance for Doubtful Accounts (–A) ............................................... Recognized bad debt charge. _____________

b. Ending Allowance Balance

9

= Beginning Allowance Balance + Bad Debt Charge – Write-Offs + Recoveries = $12 + $9 – $11 + $0 = $10

9


E6–8 a. Ending Allowance Balance $1,300 Write-Offs

= Beginning Allowance Balance + Bad Debt Charge – Write-Offs + Recoveries = $1,000 + (Sales of $75,300 x Estimated uncollectible percentage of 2%) – Write-Offs + $55 = $1,261

b. Accounts Receivable (+A) .......................................................................... Sales (R, +SE)....................................................................................... Made sales on account.

75,300

Accounts Receivable (+A) .......................................................................... Allowance for Doubtful Accounts (-A) ................................................ Recovered accounts previously written off.

55

Allowance for Doubtful Accounts (+A) ...................................................... Accounts Receivable (–A) ................................................................... Wrote off accounts deemed uncollectible.

1,261

Bad Debt Expense (E, –SE)......................................................................... Allowance for Doubtful Accounts (–A) ............................................... Recognized bad debt expense.

1,506

75,300

55

1,261

1,506

Cash (+A) ............................................................................................. 73,894* Accounts Receivable (–A) ................................................................... 73,894 Collected cash from customers. _____________ * $73,894 was calculated as follows: Ending balance in Accounts Receivable of $9,400 = Beginning balance in Accounts Receivable of $9,200 + 2012 Credit sales of $75,300 – 2012 Cash collections + 2012 Recoveries of $55 – 2012 Write-offs of $1,261

E6–9 a. Extending credit to customers (and therefore creating accounts receivable on the balance sheet) is a normal part of business operations. Therefore, the transactions related to receivables, such as uncollectibles and increases/decreases in gross receivables, are treated as operating activities. b. The indirect format of the statement of cash flow is structured around converting net income (a number based in accrual accounting) into a pure cash number. The provision is a non-cash charge, similar to depreciation expense, that reduces net income but does not (by itself) reduce the company’s cash balance. Therefore, the non-cash expense is added back to net income in the calculation of cash from operating activities. The fact that receivables decreased indicates that the company collected more cash than just the amount in its current year sales—effectively, the company collected this year’s sales and some of last year’s (that were sitting as accounts receivable on the balance sheet). If receivables had increased, the company would have deducted that amount in the statement of cash flow (because the company would


c.

not have collected as much cash as it had booked in sales)..The decrease in receivables is called a “source” of cash (while an increase would be called a “use” of cash). Net cash from operating activities can be higher than net income because of non-cash expenses (as discussed above) and because of decreases in current assets (“sources” of cash as discussed above) or increases in current liabilities (also “sources” of cash). If “uses” of cash (such as increases in current assets and decreases in current liabilities) had been larger, net cash from operating activities may have been lower than net income.

E6–10 Total receivables equals the sum of the receivable balances for each age classification. Therefore, total receivables equals $290,000 + $110,000 + $68,000 + $40,000, or $508,000. Expected bad debts equals the sum of the estimated uncollectible amounts for each age classification. As shown in the following table, the total expected bad debts associated with the $508,000 currently in accounts receivable is $22,740. Age Current 1–45 days 46–90 days >90 days Total

Account Balance $290,000 110,000 68,000 40,000 $508,000

Noncollection Probability 2% 5% 8% 15%

Uncollectible Amount $ 5,800 5,500 5,440 6,000 $ 22,740

E6–11 1/1/12

Notes Receivable (+A) ................................................................ 10,450* Sales (R, +SE) ....................................................................... Made sale in exchange for a note. _________________ * $10,450 = Sales in Canadian dollars of 11,000 x Exchange rate of $0.95/Canadian dollar Notes Receivable (+A) ................................................................ 440* Exchange Gain (Ga, +SE)...................................................... Recorded foreign currency exchange gain on receivable. _________________ * $440 = Adjusted value of note – Carrying value of note = (Note for 11,000 Canadian dollars x Current exchange rate of $0.99/Canadian – $10,450

10,450

12/31/12

12/31/13

Exchange Loss (Lo, –SE).............................................................. Notes Receivable (–A) ......................................................... Incurred foreign currency exchange loss on receivable. _________________ * $990 = Adjusted value of note – Carrying value of note

440

dollar)

990* 990


=

(Note for 11,000 Canadian dollars x Current exchange rate of $0.90/Canadian

dollar)

– $10,890

E6–12 January 1, 2012 Notes Receivable (+A) ...................................................................................... Sales (R, +SE) ............................................................................................. Made sale in exchange for a note.

10,450* 10,450

Cash (+A) .......................................................................................................... 10,450* Notes Payable (+L)..................................................................................... Recorded borrowing (in Canadian dollars). _________________ * $10,450 = Note balance of 11,000 Canadian dollars x Exchange rate of $0.95/Canadian dollar December 31, 2012 Notes Receivable (+A) ...................................................................................... Exchange Gain (Ga, +SE) ............................................................................ Earned a foreign currency exchange gain on receivable.

440* 440

Exchange Loss (Lo, –SE) .................................................................................... 440* Notes Payable (+L)..................................................................................... Incurred a foreign currency exchange loss on payable. _________________ * $440 = Adjusted value of note – Carrying value of note = (Note for 11,000 Canadian dollars x Current exchange rate of $0.99/Canadian dollar) – $10,450 December 31, 2013 Exchange Loss (Lo, –SE) .................................................................................... Notes Receivable (–A) ............................................................................... Incurred a foreign currency exchange loss on receivable.

10,450

440

990*

Notes Payable (–L) ........................................................................................... 990* Exchange Gain (Ga, +SE) ............................................................................ Earned a foreign currency exchange gain on payable. _________________ * $990 = Adjusted value of note – Carrying value of note = (Note for 11,000 Canadian dollars x Current exchange rate of $0.90/Canadian – $10,890

990

990

dollar)

Exchange gains and losses on debt and receivables for the same amount and expressed in the same foreign currency always offset each other. When the exchange rate increases, both the receivable and the payable increase by the same amount. The increase in the receivable gives rise to a gain (i.e., the company will receive more U.S. dollars), while the increase in the liability gives rise to a loss (i.e., the company must pay out more U.S. dollars). When the exchange rate decreases, both the receivable and the payable decrease by the same amount. The decrease in the receivable gives rise to a loss, while the decrease in the liability gives


rise to a gain. Therefore, through hedging Outreach Inc. is able to avoid wide fluctuations in its net income from fluctuations in exchange rates.


PROBLEMS P6–1 a. No. The $285,000 is comprised of the $225,000 in savings and checking accounts and the $60,000 compensating balance. The $225,000 can properly be classified as a current asset, since no restrictions on the access or on the use of these funds exist. However, Print-O-Matic is restricted from using the $60,000 compensating balance. Whether restricted cash should be classified as a current or as a noncurrent asset depends on whether the restriction will end within the time frame of current assets or outside this time frame. In this case the $60,000 is restricted until the loan matures on October 1, 2016. Consequently, the $60,000 will not become available within the time frame of current assets, thereby not qualifying the $60,000 for classification as a current asset. The $60,000 should be classified as a noncurrent asset. b. The concept of interest expense is viewed differently by economists and accountants. Economists would define interest expense as the total cost of borrowing. Such costs would include the actual interest charged on the borrowing plus an opportunity cost incurred from borrowing. In this case, the actual interest charged on the borrowing for 2011 was $6,250. Print-O-Matic also incurred an opportunity cost. By borrowing the money, the company lost the opportunity to invest the $60,000 compensating balance. Assuming that Print-O-Matic would have invested this money in its savings and checking accounts at an annual rate of 6%, the company incurred an opportunity cost of $900 (i.e., $60,000 x 6% x 1/4). The opportunity cost should be considered when making business decisions. Accountants would define interest expense as the outflow of assets or the inflow of liabilities associated with borrowing. Since opportunity costs represent lost opportunities and do not represent outflows of assets or inflows of liabilities, opportunity costs are not properly classified as expenses. Consequently, accountants would classify only the $6,250 as interest expense for 2011.

P6–2 a. 3/3

3/8

3/11

3/28

Accounts Receivable (+A)........................................................... Sales (R, +SE) ....................................................................... Made sale on account.

1,400

Accounts Receivable (+A)........................................................... Sales (R, +SE) ....................................................................... Made sale on account.

800

Cash (+A) .................................................................................... Cash Discount (–R, –SE) ............................................................. Accounts Receivable (–A) .................................................... Collected cash from customer.

1,372 28

Cash (+A) .................................................................................... Accounts Receivable (–A) .................................................... Collected cash from customer.

800

1,400

800

1,400

800


P6–2 3/29

b.

Concluded Accounts Receivable (+A)........................................................... Sales (R, +SE) ....................................................................... Made sale on account.

1,800 1,800

The annual interest rate of forfeiting a cash discount is calculated as follows: Annual rate = Cash discount rate x (365 days ÷ Number of days receipt collected after the end of the discount period) = 2% x (365 ÷ 10 days) = 73% Since BBB can borrow money at an annual interest rate of 9%, BBB should have borrowed money at the 9% rate and paid its obligation to QNI Corporation. BBB would have saved itself some interest costs by borrowing the money rather than forfeiting the cash discount.

P6–3 a. 2010 Allowance for Doubtful Accounts (+A) ...................................................... Accounts Receivable (–A) ................................................................... Wrote off accounts deemed uncollectible. Bad Debt Charge (E, –SE)........................................................................... Allowance for Doubtful Accounts (–A) ............................................... Recognized bad debt charge. _____________ * $5,400 = Credit sales of $180,000 x Estimated uncollectible percentage of 3% 2011 Allowance for Doubtful Accounts (+A) ...................................................... Accounts Receivable (–A) ................................................................... Wrote off accounts deemed uncollectible. Bad Debt Charge (E, –SE)........................................................................... Allowance for Doubtful Accounts (–A) ............................................... Recognized bad debt charge. _____________ * $6,000 = Credit sales of $200,000 x Estimated uncollectible percentage of 3% 2012 Allowance for Doubtful Accounts (+A) ...................................................... Accounts Receivable (–A) ................................................................... Wrote off accounts deemed uncollectible. Bad Debt Expense (E, –SE)......................................................................... Allowance for Doubtful Accounts (–A) ............................................... Recognized bad debt charge. _____________ * $6,150 = Credit sales of $205,000 x Estimated uncollectible percentage of 3%

6,000 6,000

5,400* 5,400

10,000 10,000

6,000* 6,000

11,000 11,000

6,150* 6,150


P6–3

Concluded

b. January 1, 2010 balance .................................................... Write-offs during 2010 ...................................................... 2010 bad debt charge .......................................................

$10,000 (6,000) 5,400

December 31, 2010 balance.............................................. Write-offs during 2011 ...................................................... 2011 bad debt charge .......................................................

c.

$

9,400 (10,000) 6,000

December 31, 2011 balance.............................................. Write-offs during 2012 ...................................................... 2012 bad debt charge .......................................................

$

5,400 (11,000) 6,150

December 31, 2012 balance..............................................

$

550

CNG should consider increasing the percentage of credit sales that is considered uncollectible. From 2010 through 2012, write-offs exceeded bad debt expense, with the difference increasing over time. Write-offs as a percentage of credit sales increased from 3.33% ($6,000 ÷ $180,000) in 2010 to 5.37% ($11,000 ÷ $205,000) in 2012. This trend indicates that the December 31, 2012 balance in Allowance for Doubtful Accounts is probably understated, thereby causing an overstatement of the company's assets. A more appropriate bad debt percentage would be 4.5% to 5.0% of credit sales.

P6–4 a. Bad Debt Charge (E, –SE)........................................................................... Allowance for Doubtful Accounts (–A) ............................................... Recognized bad debt expense. _____________ * $49,500 = Net sales x 3% = ($1,800,000 – $130,000 – $20,000) x 3% b. Ending Allowance balance

c.

49,500

= Beginning Allowance balance + Bad Debt Charge + Recoveries – Write-Offs = $65,000 + $49,500 (from Part [a]) + $0 - $70,000 = $44,500

Bad Debt Expense (E, –SE)......................................................................... Allowance for Doubtful Accounts (–A) ............................................... Recognized bad debt expense. _____________ * $40,500 = Net sales x 3% = ($1,500,000 – $100,000 – $50,000) x 3%

d. Ending Allowance balance

49,500*

40,500*

= Beginning Allowance balance + Bad Debt Expense + Recoveries – Write-Offs = $44,500 (from Part [b]) + $40,500 (from Part [c]) +

$0 – $85,000 =

$0

40,500


P6–5 a. Ending Allowance balance

2010 Ending Allowance Balance

= Beginning Allowance balance + Bad Debt Charge + Recoveries – Write-Offs

= =

$0 + $4,200* + $0 - $3,000 $1,200

= =

$1,200 + $7,600** + $0 - $6,000 $2,800

= =

$2,800 + $9,600*** + $0 - $8,400 $4,000

=

Beginning A/R balance + Credit Sales Cash Collections – Write-Offs

=

A/R balance – Allowance balance

= = =

$0 + $105,000 - $92,000 - $3,000 $10,000 $10,000 - $1,200 = $8,800

Balance

= = =

$10,000 + $190,000 - $161,000 - $6,000 $33,000 $33,000 - $2,800 = $30,200

Balance

= = =

$33,000 + $240,000 - $214,000 - $8,400 $50,600 $50,600 - $4,000 = $46,600

*2010 credit sales x .04 2011 Ending Allowance Balance **2011 credit sales x .04 2012 Ending Allowance Balance ***2012 credit sales x .04 b. Ending A/R balance

A/R (net) 2010 Ending A/R Balance 2010 A/R (net) 2011 Ending A/R

2011 A/R (net) 2012 Ending A/R

2012 A/R (net)

c. In each of the three years shown, Albertson’s Locksmith Corporation has estimated bad debts slightly higher than the annual write-offs. The estimates are adequate and provide some cushion for future years in case of a larger-than-anticpated write-off.


P6–6 2008 2007 2006 January 1, balance ............................................................. $9,234 $7,279 $7,090 Provision for losses............................................................ 23,765 6,549 3,231 Recoveries ..................................................................... 929 829 842 Less: December 31, balance .............................................. ($23,164) ($9,234) ($7,279) Actual Write-offs .............................................................. $10,764 $5,423 $3,884 The 2008 provision (an expense based on an estimate of uncollectible receivables) jumped significantly due to the start of a recession. During poor economic times, banks will have more difficulty collecting loans; the higher provision and the higher balance in the allowance account reflect the economic realities of 20082009.

P6–7 a.

Hadley Company Income Statement For the Year Ended December 31, 2011 Sales ...................................................................................... $ 200,000 Cost of goods sold .................................................................. 102,000 Expenses ................................................................................. 115,000* Net income (loss) .................................................................... $ (17,000) _________________ * $115,000 = $65,000 of previously reported expenses + $50,000 of bad debt expense associated with the Litzenberger account Hadley Company Balance Sheet December 31, 2011 Assets Cash .......................................... Accounts receivable, net ............ Other assets ................................ Total assets .................................

$

5,000 35,000 40,000

$ 80,000

Liabilities & Stockholders' Equity Current liabilities .................. $ 13,000 Long-term notes payable ..... 80,000 Stockholders' equity............. (13,000) Total liabilities and stockholders' equity ........... $ 80,000

After considering the adjustment for potential bad debts, Hadley generated a net loss for 2011. Therefore, it appears that Hadley Company did not have a very successful first year of operations. b.

Auditors have their own interests. They must consider factors affecting their own well-being. One item that could adversely affect auditors' well-being is being the defendant in a lawsuit. If the auditors did not require an adjustment for the Litzenberger account, and Litzenberger was subsequently unable to pay its debt, users of Hadley's financial statements could sue the auditors for any losses incurred. Conservatism, which states "when in doubt, understate rather than overstate," applies to this situation. Since the auditors are uncertain as to whether Hadley Company will ever collect the money


from Litzenberger, the auditors would prefer to understate Hadley Company's financial health rather than overstate it.

P6–7 c.

Concluded

While it is true that Litzenberger is still operating, Hadley's CFO is ignoring the revenue recognition principle and the matching principle. Under the revenue recognition principle, revenue should not be recognized if post-sales costs can not be adequately estimated (subject to materiality). In this case, the actual bad debt cost associated with Litzenberger will not occur until a subsequent period. However, if this cost cannot be adequately estimated, Hadley Company should not even recognize the revenue from the sale to Litzenberger. Assume that Hadley Company can adequately estimate the bad debt cost. In this case Hadley Company is allowed to recognize the revenue. But under the matching principle, all costs associated with generating revenue should be matched against that revenue. Hence, any costs associated with making a sale, whether incurred in the current period or in subsequent periods, should be recorded in the period of the sale. Since the bad debt cost is associated with generating revenue, Hadley Company should record the bad debt cost in the current period as an expense.

P6–8 a. The effect of the auditors’ findings on 2011 Fees Earned, Accounts Receivable, Allowance for Doubtful Accounts, current ratio, working capital, and net income can be determined as follows. Fees Earned:

Fees Earned would decrease from $240,000 to $230,000.

Accounts Receivable:

Accounts Receivable would decrease from $68,000 to $58,000.

Allowance for Doubtful Accounts: This account should have a balance equal to 10% of the new Accounts Receivable balance. The correct balance would be $5,800, or an increase of $2,400. Current Ratio: The current ratio before the auditors’ findings was 1.62 ($105,000 ÷ $65,000). Current assets after adjusting for the auditors’ findings would be $92,600 ($105,000 – $10,000 decrease in Accounts Receivable – $2,400 increase in Allowance for Doubtful Accounts). Current Liabilities would be unaffected by the auditors’ findings. Thus, the new current ratio would be 1.42 ($92,600 ÷ $65,000). Working Capital: Working capital would decrease from $40,000 ($105,000 – $65,000) to $27,600 ($92,600 – $65,000). Net Income: Net income would decrease by the reduction in Fees Earned of $10,000 and by the increase in Bad Debt Charge of $2,400. The new net income would be $2,600. b. Prior to the auditors’ findings, Finley, Ltd. was in compliance with its debt covenants. However, after adjusting the books for the auditors’ findings, Finley, Ltd. has violated both requirements of its debt covenants. The company’s current ratio has fallen to 1.42, which is below 1.5 as specified in the loan agreement. Further, the company declared dividends equal to 192.3% of the adjusted net income. It appears that one possible explanation for Finley’s decision to record the sale and not record an adequate amount for bad debts was to avoid violating its debt covenants.


P6–9 a. Current Method Bad debt charge Accounts rec. value Allowance Method Bad debt charge Accounts rec. value Note:

2008

2009

$ 10,000

$ 50,000

950,000

900,000

$ 80,000

$ 78,000

880,000

802,000

2010 $

20,000

2011 $

1,200,000

$

82,000 1,040,000

80,000

2012 $

1,175,000

$

82,560 1,012,440

90,000 1,095,000

$

79,200 943,240

Under the allowance method, the net value of Accounts Receivable equals the balance in Accounts Receivable less the balance in Allowance for Doubtful Accounts.

b. Total bad debt charge

Current Method $250,000

Allowance Method $401,760

The allowance method provides a measure of net income consistent with both the revenue recognition and the matching principles. Over the five-year period Fine Linen Service wrote off a total of $250,000 in Accounts Receivable from sales made in prior years. Under the current method, the $250,000 did not reduce revenue in the year of the sale. Hence, net income in each individual year was misstated. Under the allowance method, bad debt expense is calculated using the year’s sales. Consequently, the allowance method, while an estimate, provides better matching of expenses with the associated revenues.

P6–10 a. Bad Debt Charge (E, –SE)........................................................................... 15,000* Allowance for Doubtful Accounts (–A) ............................................... Estimated bad debt charge. _____________ * $15,000 = $750,000 December credit sales x 2% Estimated uncollectible percentage b. Ending allowance balance

= Beginning allowance balance + Annual bad debt charge + Recoveries – Write-offs = $70,000 + ($11,940,000 x 2%) + $0 – $239,000 = $69,800

c. Ending accounts receivable balance = Beginning accounts receivable balance + Credit sales + Recoveries – Cash collections – Write-Offs $3,250,000 (from the aging) = Beginning accounts receivable balance + $11,940,000 + $0 – $12,126,000 – $239,000 Beginning accounts receivable balance = $3,675,000

15,000


P6–10 Concluded d. Account Age Current 1–30 days 31–75 days > 75 days Total

Account Balance $700,000 1,200,000 550,000 800,000

Noncollection Probability 2.0% 5.5% 10.0% 25.0%

Uncollectible Amount $ 14,000 66,000 55,000 200,000 $335,000

Bad Debt Charge (E, –SE)........................................................................... 280,200* Allowance for Doubtful Accounts (–A) ............................................... Estimated bad debt charge. _____________ * $280,200 = $335,000 – Unadjusted Allowance balance = $335,000 – ($70,000 + $223,800 of monthly bad debt charge adjusting entries for January through November – $239,000 of accounts written off)

280,200

e. Some companies use the percent-of-sales method to estimate bad debts but also periodically use the aging method. Companies would use the two methods for several reasons. The percent-of-sales method is very easy and comparatively inexpensive to use. Further, the percent-of-sales method emphasizes revenues and expenses, since estimated bad debts are a function of revenues. For these reasons a company is more apt to use the percent-of-sales method than the aging method. Additionally, the aging method can be very costly and time-consuming. For all but the smallest companies, preparing an aging of Accounts Receivable requires a computerized accounting system. However, in return for this increased cost and time, the aging method provides several types of useful information. First, the aging method identifies slow-moving accounts, which may require additional collection efforts. Further, the dollar amount of any particular aging classification dictates the maximum amount that the company should expend in trying to collect the accounts. A company would generally not want to spend more to collect an account than it expects to ultimately collect. Second, an aging can help a company define its credit policies. For example, if an aging identifies a particular customer as a consistently slow payer, the company may decide to no longer extend credit to this customer. If an aging indicates a large percentage of old accounts, this information could be used to decide whether to offer cash discounts and the terms of such discounts. Finally, an aging can indicate the accuracy of the percent-of-sales method. A detailed analysis of Accounts Receivable will usually provide an accurate estimate for the required balance in Allowance for Doubtful Accounts. This estimated balance can be compared to the balance obtained using the percent-of-sales method. Significant differences would indicate a need to revise the percent of sales considered uncollectible. To obtain this information, companies will periodically prepare an aging of Accounts Receivable.


P6–11 a. Looking at the net income for the years 2010, 2011, and 2012, it is true that Ticheley has reported profit increases consistently over the last three years. However, what remains to be seen is whether the increase in the profits is due to regular recurring operations or whether it is due to some income manipulation strategy adopted by Ticheley. b. A company would establish such a system of rewarding the management only on the belief that an increase in return on Stockholders’ equity (Net Income as a % of total Stockholders’ Equity) would lead to an increase in the market price of its common stock. This belief is based on the fact that an increase in the market price of the company’s common stock (1) is what the investors are looking for and (2) is the true measure of the success of a company during any one year. c.

It appears that Ticheley is using an “income-smoothing” reporting strategy. Even though we have the data only for 3 years, the fluctuating bad debt change as a percentage of accounts receivables points to that conclusion. 2010: 2011: 2012:

2,100 2,900 1,700

÷ ÷ ÷

23,100 23,200 27,400

= = =

9.09% 12.50% 6.20%

d. Ticheley is probably using the “income-smoothing” reporting strategy to demonstrate to its stockholders that it is steadily growing. The company is setting aside more than normal (i.e., 12.5%) charge for bad debt expenses during a good year (i.e., 2011) and by dipping into the charge to cover the bad debt losses during the not so good year (i.e., 2012).

P6–12 a.

Value of Transaction In Foreign Currency 320,000 pounds 350,000 yen 500 euros 200,000 Canadian dollars

(1) (2) (3) (4) b. (1)

(2)

(3)

Exchange Rate 1 dollar/.50 pound 1 dollar/125 yen 1 dollar/.75 euro 1 dollar/1.10 Canadian dollar

Value of Transaction in U.S. Dollars $640,000.00 2,800.00 666.67 181,818.18

Accounts Receivable (+A) ................................................................ Sales (R, +SE) ............................................................................ Made a sale on account expressed in British pounds.

640,000

Notes Receivable (+A) ..................................................................... Sales (R, +SE) ............................................................................ Sold a note expressed in yen.

2,800

Inventory (+A) ................................................................................. Notes Payable (+L)....................................................................

666.67

640,000

2,800

666.67


Purchased inventory for a note expressed in euros.

P6–12 (4)

c.

Concluded Inventory (+A) ................................................................................. Accounts Payable (+L) .............................................................. Purchased inventory on account expressed in Canadian dollars.

181,818.18 181,818.18

Adjusted Value Carrying Value Exchange Gain (Loss) (1) $533,333.33a $ 640,000.00 ($106,666.67) b (2) 3,043.48 2,800.00 243.48 (3) 588.24c 666.67 (78.43) (4) 190,476.19d 181,818.18 8,658.01 ________________ a $533,333.33 = 320,000 pounds x (1 dollar/.6 pound) b $3,043.48 = 350,000 yen x (1 dollar/115 yen) c $588.24 = 500 euro x (1 dollar/.85 euro) d $190,476.19 = 200,000 Canadian x (1 dollar/1.05 Canadian)

d. Receivables and payables are stated in a particular currency, for example, in British pounds. Assume that money from the receivable/payable is to be converted into U.S. dollars. At a given point in time, one British pound can be converted into a certain number of dollars. These dollars can, in turn, be used to purchase items. At a different point in time, one British pound can be converted into a different number of dollars. Holding everything else constant, these dollars can now be used to purchase either more or less goods than before. This change in purchasing power affects a company's wealth. Changes in wealth are captured by gains and losses. Since these gains or losses arise due to fluctuations in exchange rates, they are aptly called exchange gains or exchange losses.

P6–13 a. Carrying value of receivable

= = =

Receivable in British pounds x Exchange rate 40,000 British pounds x ($1.70/1 British pound) $68,000

b. The current ratio is calculated as current assets divided by current liabilities. If International Services is to maintain a current ratio of at least 1.5, its current assets after considering the effect of exchangerate fluctuations must be 1.5 times its current liabilities. The company's current assets not affected by exchange-rate fluctuations are $12,000 ($80,000 – $68,000 from Part [a]). Therefore, the minimum acceptable exchange rate would be calculated as follows: (Current assets not affected by exchange rates + Current assets affected by exchange rates) ÷ Current liabilities = 1.5 = = =

($12,000 + Current assets affected by exchange rates) ÷ $50,000 = 1.5 [$12,000 + (Receivable in British pounds x Exchange rate)] ÷ $50,000 = 1.5 [$12,000 + (40,000 pounds x Exchange rate)] ÷ $50,000 = 1.5


Exchange rate = 1.575

P6–13 Concluded c.

For International Services to maintain a current ratio of at least 1.5, its current assets after considering the effect of exchange-rate fluctuations must be 1.5 times its current liabilities adjusted for the effects of exchange-rate fluctuations. The company's current liabilities not affected by exchange-rate fluctuations are $48,400 ($50,000 - $1,600 payable to British bank). Therefore, the minimum acceptable exchange rate would now be calculated as follows: (Current assets not affected by exchange rates + Current assets affected by exchange rates) ÷ (Current liabilities not affected by exchange rates + Current liabilities affected by exchange rates) = 1.5 = ($12,000 + Current assets affected by exchange rates) ÷ ($48,400 + Current liabilities affected by exchange rates) = 1.5 = [$12,000 + (Receivable in British pounds x Exchange rate)] ÷ [$48,400 + (Payable in British pounds x Exchange rate)] = 1.5 = [$12,000 + (40,000 pounds x Exchange rate)] ÷ [$48,400 + (1,000 pounds x Exchange rate)] = 1.5 Exchange rate = 1.574 (rounded to 3 decimals)

d. Increases in the exchange rate cause exchange gains when holding receivables and exchange losses when holding payables. Alternatively, decreases in the exchange rate cause exchange gains when holding payables and exchange losses when holding receivables. Therefore, changes in the exchange rate have opposite effects on receivables and payables. By holding a payable in the same amount as a receivable, exchange gains will perfectly offset exchange losses. In this way, a company can hedge against exchange-rate fluctuations. A company never enjoys an exchange gain but also never incurs an exchange loss.


ISSUES FOR DISCUSSION ID6–1 a. A potential investor is interested in the solvency of a company. Since solvency is associated with the availability of cash to pay off debts, an investor would want to know about any existing restrictions on a reported cash balance. b. Because the cash will remain restricted for two years, it should be disclosed as noncurrent. If the cash will not be available to cover current obligations (which it won’t according to the loan documents), then it would be inaccurate to designate the balance as current. c. Excluding the cash balance will reduce working capital, the current ratio and the quick ratio. A user of the financial statements should rightly calculate that the solvency of the company is lowered due to the inability to use the restricted cash to cover current obligations.

ID6–2 a. For a long time, IBM rightfully enjoyed the reputation as the “epitome of financial conservatism”. This reputation was built by IBM by using conservative financial reporting practices. Such a reporting strategy underestimates the revenues and overestimates the expenses. The maneuver by IBM to immediately record all the revenue from a long-term lease contract without realizing all the cash associated with the transaction is an example of liberal accounting policy (i.e., policy that tends to record the revenue as soon as possible). Such maneuvers occurring too often will surely affect IBM’s reputation as the “epitome of financial conservatism.” b. The use of accelerated revenue recognition methods has its own costs and benefits to IBM and its management. Accelerated revenue recognition methods not only inflate the sales and net income on the income statement but also tend to inflate receivables on the balance sheet. Therefore, a number of ratios, such as return on stockholders’ equity, working capital, current ratio, and return on total assets are also inflated. As a result of these inflated ratios, IBM may tend to gain in terms of its ability to comply with a number of provisions in its debt covenants. The cost of these benefits to IBM would be its image in the capital markets as the company that engages in the “window-dressing” of its financial statements. Such an impression about a company within the financial markets could lead to a higher level of scrutiny of its financial statements by financial analysts on one hand, and possibly a higher cost of capital to the firm on the other hand. The major benefits to the management of adopting such an aggressive revenue recognition strategy would be (1) higher bonus, if based on net income or any other measure that involves net income or sales in the calculation; and (2) less pressure to worry about the credit ratings of the debt. The major cost would be increased scrutiny by the auditors to ensure that management is not fraudulent. c.

IBM’s policy of requiring all employees to swear that they have read the company’s “Business Conduct Guidelines” is in direct contrast to its own behavior. On one hand it discourages its employees from organizing financial information in a potentially deceptive way, but on the other hand the management


of the company engages in aggressive revenue recognition tactics to boost the bottom-line. This is a classic example of management not “walking the talk.” Such behavior is non exemplary and sends a very subtle signal to the employees that, during hard times, ethics have no place within the company.

ID6–3 a. Working Capital 1992: 1993: 1994:

=

Current Assets

– Current Liabilities

$1,256.20 – $1,087.5 = $168.70 $1,067.60 – $1,105.1 = $ (37.50) $1,253.60 – $1,259.1 = $ (5.50)

According to the covenant, Quaker Oats Co. must maintain a Working Capital of more than $150 million. At the end of 1992, the company came very close to violating this restriction, as its Working Capital was only $168.70 million. Since then, the Working Capital has been negative, leading to the violation of the debt covenant during 1993 and 1994. b. The 1994 annual report changes the restrictions from the Working Capital to the minimum Shareholders’ Equity. The new restrictions require the company to maintain a minimum shareholders’ equity of $300 million. It seems creditors want to protect their debt to the company, and by imposing the requirement of minimum share capital they are, in essence, securitizing their debt.


ID6–4 a.

Allowance for Credit Losses 2008 2007 2006

Allowance for loan losses at beginning of year Provision for credit losses—consumer Provision for credit losses—corporate Consumer credit losses Corporate credit losses Consumer recoveries Corporate recoveries Allowance for loan losses at end of year

$16,177 $8,940 $9,782 28,282 15,599 6,224 5,392 1,233 96 20,002 10,645 8,629 1,922 948 312 1,600 1,661 1,547 149 277 232 29,616 16,177 8,940

Across the three years, Citi is estimating more and more of its loans to be uncollectible. This trend is caused by the deteriorating economy that moved into a recession in 2008. Another trend of interest is the fact that in 2006 Citi underestimated its credit losses in both consumer and corporate but then in 2007 and again in 2008 the bank overestimated its losses (compare the provision lines to the losses lines). It appears that CIti recognized that its previous habit of underestimating had left its Allowance for Credit Losses too small to handle all the bad loans of a full-blown recession. By estimating so high in 2007 and 2008, the bank is setting itself to absorb the losses that may come in 2009 and beyond.

b. Consumer loans Corporate loans

2008 Loans $519,673 174,543

Net Write-Offs % $18,402 3.54% 1,773 1.02%

The above information indicates that the Consumer loan portfolio is the riskier of the two.

ID6–5 The Reserve for Loan Losses for a bank (or a mortgage company making home loans) is the same as the Allowance for Uncollectibles account for a company carrying accounts receivable from customers. The account is a contra asset account which lowers the carrying value of the loan portfolio to the amount that the company estimates it can actually collect. The account is increased when the lender takes an expense (often called a Provision for Loan Loss) to reflect the fact that addition loans have been granted and that collectability is not estimated to be 100%. A company such as New Century might be reluctant to increase the Reserve because, as noted above, increasing the account is done through an expense to earnings. Increasing the account will have the immediate effect of reducing the company’s profits. In addition, in the specific case of New Century, an increased Reserve is an acknowledgement that the company extended credit to borrowers who are either unable or unwilling to repay the obligation. A sharp increase in the reserve is an admission of a flaw in the company’s business model. Investors in New Century, however, need to know the amount of loans that will not convert into cash. Factors such as personal disposable income, housing prices, overall consumer debt levels and inflation will factor into loan defaults, especially in vulnerable markets such as sub-prime mortgage lending.

ID6–6 The Provision for Loan Loss is the expense that feeds into a bank’s Allowance for Loan Losses, the contra asset account that adjusts downward the carrying value of the bank’s loan portfolio. When


banks determine that their loans are less collectible, they bring the portfolio down in value by increasing the Allowance with a higher Provision on the Income Statement. A weakened housing market directly affects the collateral that supports the bank’s first mortgage home loan. When a homeowner purchases a home by borrowing money from a bank, the bank takes the house as collateral. If the homeowner is unable to repay the loan, the bank has the right to seize the house (in a foreclosure) and sell the house to recover its money. If housing prices have dropped, the bank will receive less money from the sale of the foreclosed home and will therefore be less likely to recoup all of its cash. To deal with this potential loss, prudent accounting calls for an increase in the Allowance (through the Provision on the Income Statement) to properly reflect the value of the loan.

In addition to the effect on the bank’s collateral, a weakened housing market may also affect the bank’s customers and their ability to repay their loans. If the housing market suffers, all the individuals who work in industries supporting that market (construction, real estate sales and development, mortgage lending, insurance, retail, etc.) will suffer and collectively will have less available cash flow for debt service. As discussed above, if bank customers have employment problems that weaken their monthly cash flow, the bank’s loans are less collectible and the bank will need to lower the value of the loans by increasing the Allowance through a Provision on the Income Statement.

Analysts will question a bank’s estimates of Uncollectibility because they understand that bank management has the incentive to keep earnings (and therefore the stock price) high. If a bank acknowledges problem loans, the bank will need to increase its Provision on the Income Statement— meaning the bank will report lower earnings. Analysts fear that bankers will understate problem loans in order to keep earnings high to support stock prices.

ID6–7 a. The Bad Debt Provision is an expense that represents management’s estimate of future uncollectible receivables. Like other expenses, the provision can be found on the Income Statement. b. The statement of cash flow reconciles net income (an accrual accounting number) with cash generated by the various areas of business activities. The operating section deals with those activities involved in the daily running of the business (like granting credit to clients and ultimately collecting cash), so activities related to receivables are included in the operating section. The provision is a non-cash expense which lowered net income for Target and Toyota but did not lower the cash levels for those companies. To convert net income into a cash number, the companies have added back those expenses (like the provision) that reduced earnings bu did not reduce cash balances. The change to accounts receivable is included because the company collected a different dollar amount of cash than it booked as sales. In the case of Target and Toyota, the companies had an increase in receivables, meaning that they have not yet collected all the cash due from sales; the amount of the receivable increase is therefore subtracted away to help convert earnings to cash from operations. c. Both companies have large non-cash expenses (such as depreciation and the provision) that will cause cash to be larger than earnings. In addiiton, current assets other than receivables (for these companies, mainly inventory) may have decreased in amount, effectively providing cash for the business. Finally, some current liabilities (such as trade payables) may have increased, effectively providing cash for the companies’ operations. d. GAAP and IFRS treat receivables and bad debts in a very similar manner.

ID6–8


Selling on account to customers who will not ultimately pay cash for the purchase will boost sales and profits in the short-term but cause huge problems in the long-term. Many companies chase sales growth by granting credit (selling today with the promise of receiving cash tomorrow) to customers that have not demonstrated the ability to honor obligations. Sales will grow, but in future periods the benefit will be reduced when a large expense is booked to increase the Allowance for Uncollectibles account to cover the write-off of the bad receivable. At the time of the sale, Assets (in the form of Accounts Receivable) and Equity (from Sales, which is closed into Net Income as part of Retained Earnings) increase. Also, if the company is correctly following the Matching Principle, a portion of the Receivable is booked as an expense and as an increase to the Allowance account. However, if the entire amount of the receivable is ultimately deemed to be uncollectible (because the company was granting credit to unworthy customers), then the Allowance will need to be significantly increased with another Bad Debt Expense (which will decrease Assets and Equity). Accounts Receivable Days can be calculated by dividing the Accounts Receivable Turnover Ratio (discussed in Chapter 5) into 365 days. For example, an Accounts Receivable Turnover Ratio of 6.0 translates into Accounts Receivable Days of 60.83. The “days” represents the average time period it takes the company to collect the cash from the receivable. Low days indicates the company quickly converts the receivable into cash, while higher days implies the company is slow at collecting its cash. Analysts can track a company’s collection period by calculating A/R Days to determine the “quality” of the company’s earnings. Profits that are boosted by receivables that will ultimately be uncollectible are of low quality. Tracking the collection period is an excellent tool to determine if the company’s stated profits will hold up over time.

ID6–9 a. Sales concentration (having a large percentage of credit sales and accounts receivable with one customer) is a risk simply due to the fact that if the customer fails to pay its account then a large bad debt charge will significantly reduce earnings and operating cash flow. Having a large portion of business with one account puts a company in a vulnerable position if that customer fails. A better position would be to have receivables spread across many customers (in many different industries), lessening the impact if one customer fails to pay. b. The failure of one large account would cause a company to increase its Allowance for Uncollectibles by taking a large Bad Debt Expense. Earnings, assets and equity would all decline.

ID6–10 a. The company could reduce its bad debt reserve from $6.5 million to $5.39 million because its actual writeoffs were lower than expected. If there was an increase in the credit quality of its loans it could support a reduction in the reserve for bad debts. It could also be reducing its reserve for bad debts because its reported earnings were going to be lower than expected, and management made a decision to lower its reserve in order to pump up earnings in the reported period. In a year where sales have increased by 30% it is difficult to believe that the credit quality of its portfolio of loans has improved so much that there should be a reduction in the reserve account. If sales were flat in the reporting period this might be reasonable, but often when there is a significant


increase in sales this increase may have been the result of lowering the credit standards so that more buyers could qualify to purchase time shares. b. “that’s tomorrow’s problem?” means that in future years Fairfield may have to take very large charges for bad debts. By reducing its provision this year (when sales were up 30%), Fairfield may be setting themselves up to have to take a very large charge in order to increase reserves to the appropriate level relative to sales.

ID6–11 One of the issues in the recent real estate crisis was excess capacity, a situation caused by real estate developers building too many projects (often due to the easy access those developers had to capital). With too many projects built and not enough tenants to rent and occupy spaces, the real estate firms saw lower cash flow (both from vacant properties and from depressed rents in the properties that were occupied). With less cash flow the developers were less solvent, less able to meet their obligations. Firms such as Hill International that had extended credit to their developer clients were often left with a receivable that was uncollectible. As with banks and other companies that extend credit, when a receivable is uncollectible the company takes an expense (a provision for bad debts) that reduces the carrying amount of the receivable and reduces equity (through lowered retained earnings from the expense charged to income). An analyst following the industry could “link the chain” of cash flow problems, starting from tenants who pay lower rents to developers who default on payments (due to lower rent receipts) to support firms (such as Hill International) who have bad receivables and to banks who have extended credit to both developers and those firms that support the developers.

ID6–12 a. If a company holds receivables or payables stated in a foreign currency (as would be the case for companies that operate internationally), the receivables and payables must be converted to U.S. dollars when preparing financial statements. As the value of the U.S. dollar fluctuates relative to other currencies, the value of the receivables and payables in U.S. dollars also fluctuates. This gives rise to foreign currency gains and losses, which can result in substantial variations in income and other reported values from one reporting period to the next. In addition, foreign currency gains and losses have economic consequences in that they can affect a company's stock prices, credit ratings, debt covenants, and so forth. By centralizing the treasury function, a company can better monitor its overall exchange gains and losses. Without a centralized treasury function, a company may not realize it is incurring exchange gains and losses because nobody within the company realizes that it is their responsibility to monitor such items. b. The main strategy many U.S. companies use to reduce the risks of holding receivables or payables stated in a foreign currency is hedging. c.

Hedging involves taking a position in a foreign currency in an equal and opposite amount to a particular receivable or payable stated in that currency. For example, if IBM had a receivable for 100,000 British pounds, it could hedge its position by creating a payable for 100,000 British pounds. The reason hedging protects a company is that any loss realized from holding a receivable when currency rates fluctuate will be perfectly offset from the gain realized from holding the payable, and vice versa. Hedging is valuable to companies because without it companies would experience large fluctuations in income and reported receivables and payables. Unexpected fluctuations could cause a company to violate an existing debt covenant. Through hedging, companies are able to manage away such unexpected fluctuations and thereby decrease the chance that they will violate a debt covenant.


ID6–13 a. Nike’s current ratio in 2009 was 2.97 ($9,734/$3,277), up from the 2008 level of 2.66. The company’s working capital in 2009 was $6,457 which changed from 2008’s working capital of $5,517.8. The change that had the biggest impact on these numbers was the increase in Short Term Investments. b. Cash and cash equivalents are included. Cash equivalents refer to any short term, highly liquid investments that have an original maturity of less than three months. c.

Nike’s net receivables comprise 29.6% (31.6% in 2008) of Current Assets and 21.48 (22.5% in 2008) of Total Assets as of 5/31/2009. Receivables represent a significant investment by the company and are crucial to strong cash flow; the management of those receivables, therefore, is an important part of the company’s focus. The allowance for uncollectible receivables was $110.8 and $78.4 in 2009 and 2008, respectively. The balance in the Allowance account (Footnote #14) is $64 million as of the end of 2006 ($75 million in 2005). This amount represents 1.7% of total accounts receivable in 2006 (2.2% in 2005). Write-offs have been in excess of $20 million in each of the last three years, underscoring the importance of sound receivable management.

d. Footnote #19 indicates that 66% of Net Revenue comes from international sources. Footnote #18 discloses that Nike enters into forward hedging contracts to protect itself against fluctuations in foreign currencies. e. The increase in accounts receivable is an operating activity (dealing with the daily extension of credit to and collection of cash from customers) that acted as a “use” of cash, meaning the company has yet to collect some of the receivables from the credit sales on the income statement.


CHAPTER 7 MERCHANDISE INVENTORY BRIEF EXERCISES BE7–1 The inventory purchases made by Hewlett-Packard during 2008 can be calculated as follows: Beginning inventory $ 8.0 billion + Purchases X – Cost of Goods Sold 69.3 =Ending Inventory $ 7.9 billion Purchases = $69.2 billion

BE7–2 a. From the footnote it is apparent that Johnson & Johnson is a manufacturer. A retailer or a service company would not have accounts called Raw materials and supplies or a Goods in process within the detail of their inventory. These accounts are only used by manufacturing companies. b. From this disclosure it appears that Johnson & Johnson uses the FIFO inventory cost flow assumption. If a company uses LIFO it must disclose the amount of the LIFO reserve imbedded in the valuation of the inventory.

BE7–3 If General Electric used the FIFO inventory cost flow assumption instead of LIFO, its inventory balance for 2008 would be ($13.7 + 0.706) = $14.406 billion. This disclosure is useful to financial statement users because it can make it easier to compare GE’s results with a company that uses a FIFO assumption. It also tells the reader the financial statement and tax liability impact on GE if it were to switch to a FIFO assumption.

EXERCISES E7–1 (1) Since the goods were shipped FOB shipping point, legal title to the goods passes to the buyer when the goods are shipped on December 30, 2011. Since Dallas is the buyer, Dallas has legal title to the inventory as of December 31, 2011. Further, Dallas rightfully included the items in its inventory. There will be no misstatement on any of the financial statements. (2) The goods were shipped FOB shipping point, so legal title passes to the buyer when the goods are shipped on December 31, 2011. Since Dallas is the seller, not the buyer, legal title passed from Dallas on December 31, 2011. Dallas, wrongfully included the items in its ending inventory. This would result in an overstatement of inventory on the balance sheet. Assuming

1


E7–1

Concluded

that Dallas has properly recorded the sale but did not yet record the COGS, there will be an understatement of COGS on the income statement and an overstatement of net income and retained earnings. (3) Since the goods were shipped FOB destination, legal title to the goods passes to the buyer when the goods reach their destination on January 2, 2012. Since Dallas is the seller, not the buyer, Dallas has legal title to the inventory as of December 31, 2011. Dallas has rightfully included the items in its inventory. Assuming no other entries regarding the sale have been made, there will not be any misstatement on any of the financial statements. (4) The goods were shipped FOB destination, so legal title to the goods passes to the buyer when the goods reach their destination on December 31, 2011. Since Dallas is the buyer, Dallas has legal title to the inventory as of December 31, 2011. Dallas has rightfully included the items in its inventory, and assuming that the goods were correctly included in purchases as of December 31, 2011, there will not be any misstatement on any of the financial statements. (5) The goods were shipped FOB destination, so legal title to the goods passes to the buyer when the goods reach their destination on January 3, 2012. Since Dallas is the buyer, Dallas does not have legal title to the inventory as of December 31, 2011. Dallas has wrongfully included the items in its ending inventory. This would result in an overstatement of inventory on the balance sheet. Assuming that Dallas has also improperly recorded the purchases, there will be no effect on the COGS or the net income.

E7–2 10/10

Inventory (+A) ................................................................................... Accounts Payable (+L) ................................................................ Purchased inventory on account.

76,000

Inventory (+A) ................................................................................... Accounts Payable (+L) ................................................................ Purchased inventory on account.

36,000

Accounts Payable (–L) ....................................................................... Cash (–A) .................................................................................... Inventory (–A) ............................................................................ Paid supplier. _________________ * $1,520 = $76,000  2% discount

76,000

10/30

36,000

10/11

10/20

Accounts Payable (–L) ....................................................................... Cash (–A) .................................................................................... Paid supplier.

76,000

36,000

74,480 1,520*

36,000


E7–3 3/3

Inventory (+A) ................................................................................... Accounts Payable (+L) ................................................................ Purchased inventory on account.

50,000

Inventory (+A) ................................................................................... Accounts Payable (+L) ................................................................ Purchased inventory on account.

140,000

Accounts Payable (–L) ....................................................................... Cash (–A) .................................................................................... Inventory (–A) ............................................................................ Paid supplier. _________________ * $4,200 = $140,000  3% discount

140,000

4/25

50,000

3/10

3/20

50,000

140,000

Accounts Payable (–L) ....................................................................... Cash (–A) .................................................................................... Paid supplier

E7–4 12/31/06: Ending inventory: Cost of Goods Sold $11,713 Ending Inventory

= = =

Goods available for sale – Ending Inventory $14,314 – Ending Inventory $2,601

12/31/07: Goods available for sale: Goods available for sale = Cost of Goods Sold + Ending Inventory Goods available for sale = $12,735 + $2,852 Goods available for Sale = $15,587 Purchases: Purchases Purchases Purchases

= Goods available for Sale – Beginning Inventory* = $15,587 - $2,601 = $12,986

* Beginning inventory for 2007 is the Ending Inventory for 2006 12/31/08: Goods available for sale: Goods available for sale = Goods available for sale = Goods available for sale =

Beginning Inventory** + Purchases $2,852 + $13,540 $16,392

135,800 4,200*

50,000


**Beginning inventory for 2008 is the Ending inventory for 2007 Ending inventory: Ending Inventory Ending Inventory Ending Inventory

= = =

Goods available for sale – Cost of goods sold $16,392 - $13,379 $3,013

E7–5 With the perpetual method, the balance in the Cost of Goods Sold account is perpetually updated for sales of inventory, as is the balance in the Inventory account for sales and acquisitions of inventory. This implies that the balance in Cost of Goods Sold should correspond to a balance in the Inventory account of $52,000, and that no entry is necessary at the end of the year to record Cost of Goods Sold. Ending Inventory $52,000 Cost of Goods Sold

= = =

Beginning Inventory + Net Purchases – Cost of Goods Sold $32,000 + ($85,000 + $4,300) – Cost of Goods Sold $69,300

However, since the physical count indicates that Telly's has $2,000 less inventory than is recorded in its Inventory account, the following adjusting entry is necessary at the end of the year. Inventory Shrinkage (E, –SE) ............................................................. Inventory (–A) ............................................................................ Incurred inventory shrinkage.

2,000 2,000

E7–6 a. Error in Ending Inventory in 2007: The $50 understated error in the Ending inventory means that the Ending Inventory should have been $268 + $50 = $318. This would change the Cost of goods sold to $1,174 - $318 = $856 which would then increase the Gross profit to $421 ($1,277 - $856).

b. Error in Ending Inventory in 2008: = The 2007 error in the Ending Inventory changes the Beginning Inventory in 2008 and the Goods Available for sale to $318 + $857 = $1,175. To calculate the Cost of Goods Sold the Ending Inventory for 2008 is deducted from the revised Goods Available for Sale: $1,175 – ($239 - $50) = $986. The gross profit would then be $1,262 - $986 = $276.

c. Original Cost of Goods Sold Corrected Cost of Goods Sold

2007 $906 $856

2008 $886 $986

E7–7 a.

Net cash from operating activities is the sum of all cash inflows and outflows that are related to the daily running of the company’s business operations. Net income is the difference between revenues (which do not have to be collected in cash) and expenses (which do not have to be paid out in cash). It is very possible for a company to show a positive cash from operations even though its net income is negative; it is possible that many of the expenses (which were subtracted from revenue to get net income) did not require a cash outlay,


just as it is possible that cash inflows exceeded sales for the period (because, for example, cash was also collected from sales from the prior period). b. The basic form for the journal entry is: Inventory Write Down Expense (E, -SE) Inventory (-A) As shown, this expense will reduce net income, retained earnings and stockholders’ equity but will NOT reduce cash. To reconcile net income (revenue minus expenses) with cash from operations (all cash flows related to operations), all non-cash expenses should be added back in the statement of cash flow. c.

If Sony’s change to inventory (160,432) is being added back to net income in the statement of cash flow, then the change must represent a “source” of cash, which would mean that the inventory decreased from 2007 to 2008. When assets decrease, they free up cash, while asset increases “use” cash.

E7–8 a. If Marian wants to maximize profits and ending inventory, she should sell the customer the lowest priced coat (i.e., Coat 4). If she sells Coat 4, Marian would report the following gross profit and ending inventory.

Revenues COGS of Coat 4 Gross profit

Gross Profit $ 12,000 6,800 $ 5,200

Ending Inventory Coat 1 Coat 2 Coat 3 Total

$

8,400 7,100 7,600 $ 23,100

Marian may have several reasons to maximize profits and ending inventory. If Marian's Furs has borrowed money and entered into debt covenants, the debt covenants may contain clauses stipulating a certain current ratio, debt/equity ratio, and so forth. By maximizing profits and inventory, Marian can also minimize the probability that she will violate one of these ratios, thereby decreasing the chance that she will violate her debt covenants. Further, if Marian has a bonus linked to accounting earnings, she could maximize her bonus by maximizing profits. b. If Marian wants to minimize profits and ending inventory, she should sell the customer the highest priced coat (i.e., Coat 1). If she sells Coat 1, Marian would report the following gross profit and ending inventory. Gross Profit Revenues COGS of Coat 1 Gross profit

$ 12,000 8,400 $ 3,600

Ending Inventory Coat 2 $ 7,100 Coat 3 7,600 Coat 4 6,800 Total $ 21,500

The most likely reason Marian would want to minimize profits and ending inventory is to minimize taxes. Minimizing profits would minimize current tax payments, thereby minimizing the present value


of tax payments. Further, some states charge taxes on a company's assets, thereby providing an incentive to minimize assets.

E7–9 a. FIFO cost flow assumption: Cost of Goods Sold

Gross Profit

= = =

= = =

(75 units  $450) + (50 units  $500) + (5 units  $600) $33,750 + $25,000 + $3,000 $61,750

Sales – Cost of Goods Sold (130 units  $1,000) – $61,750 $68,250

Ending Inventory

= =

(60 units  $600) $36,000

Averaging cost flow assumption: Cost per Unit

= [(75 units  $450) + (50 units  $500) + (65 units  $600)] ÷ (75 units + 50 units + 65 units) ($33,750 + $25,000 + $39,000) ÷ 190 units = $514.47 per unit (rounded) = (130 units  $514.47) = $66,881.10

Cost of Goods Sold

Gross Profit

= = =

Ending Inventory

Sales – Cost of Goods Sold (130 units  $1,000) – $66,881.10 $63,118.90 = =

60 units  $514.47 $30,868.20

LIFO cost flow assumption: Cost of Goods Sold

Gross Profit

= = =

Ending Inventory

= = =

(65 units  $600) + (50 units  $500) + (15 units  $450) $39,000 + $25,000 + $6,750 $70,750

Sales – Cost of Goods Sold (130 units  $1,000) - $70,750 $59,250 = =

(60 units  $450) $27,000


E7–9

Concluded

b. If the monitors are identical, customers would be indifferent between any two monitors. Hence, Vinnie could simply give a customer the monitor that allows him to either minimize or maximize cost of goods sold, thereby maximizing or minimizing gross profit. If Vinnie wants to maximize net income, he would first sell to customers the lowest-priced monitors, followed by the second lowest-priced monitors, and so forth. Since the cost of the monitors is increasing, this strategy is identical to the FIFO cost flow assumption. Therefore, the highest gross profit Vinnie could report is $68,250 (from part [a]). Vinnie may want to maximize net income for several reasons. First, if Vinnie receives any incentive compensation, such as a bonus that is tied to net income, then he can maximize his compensation by maximizing net income. Second, if Vinnie has any existing debt covenants, they may specify a maximum debt/equity ratio. By increasing net income, Vinnie would increase equity, thereby decreasing his debt/equity ratio. In this manner, Vinnie decreases the probability that he will violate the debt covenant. Finally, if Vinnie is in the process of trying to obtain debt, potential creditors may use net income as a factor in determining whether or not to loan money to Vinnie or what interest rate to charge. If Vinnie wants to minimize net income, he would first sell to customers the highest-priced monitors, followed by the second highest-priced monitors, and so forth. Since the cost of the monitors is increasing, this strategy is identical to the LIFO cost flow assumption. Therefore, the lowest gross profit Vinnie could report is $59,250 (from part [a]). The most likely reason Vinnie would want to minimize net income is for tax purposes. If he uses the same set of books for tax and financial reporting purposes, then by minimizing book income, Vinnie minimizes taxable income. Minimizing taxable income, in turn, minimizes the present value of cash outflows for taxes.

E7–10 2010 Cost of goods sold Gross profit (Sales – COGS) Ending inventory

FIFO

Weighted Average

160 290

170 280 170

2011

FIFO

Weighted Average

Cost of goods sold Gross profit (Sales – COGS) Ending inventory

245

262.5 437.5 262.5

290

180 270

180

455

LIFO

160 LIFO 290 410 225

If the business is growing (inventory levels rising) and the cost of inventory is increasing, then if LIFO is chosen, the company will lower its net income which will reduce its tax liability. This increases the cash flow of the company. Using FIFO will increase its reported net income and tax liability but will also increase its current assets. This choice impacts the company’s operating and liquidity ratios.


E7–11 a. LIFO cost flow assumption: Year 2008 2009 2010 2011 2012

Calculation 5,000 units  $12 (12,000 units  $16) + (4,000 units  $12) 2,000 units  $18 10,000 units  $21 (2,000 units  $23) + (3,000 units  $18) + (1,000 units  $12)

Total

Amount $ 60,000 240,000 36,000 210,000 112,000 $ 658,000

FIFO cost flow assumption: Year 2008 2009 2010 2011 2012 Total

Calculation 5,000 units  $12 (5,000 units  $12) + (11,000 units  $16) (1,000 units  $16) + (1,000 units  $18) (4,000 units  $18) + (6,000 units  $21) (4,000 units  $21) + (2,000 units  $23)

Amount $ 60,000 236,000 34,000 198,000 130,000 $ 658,000


E7–11

Concluded

Averaging cost flow assumption: Year 2008

Cost/unit

2009

COGS Cost/unit

2010

COGS Cost/unit

2011

COGS Cost/unit

2012

COGS Cost/unit COGS

Total

= = = = = = = = = = = = = = =

Calculation $120,000 ÷ 10,000 units $12 per unit 5,000 units  $12 [(5,000  $12) + (12,000  $16)] ÷ 17,000 units $14.82 per unit 16,000 units  $14.82 [(1,000  $14.82) + (5,000  $18)] ÷ 6,000 units $17.47 per unit 2,000 units  $17.47 [(4,000  $17.47) + (10,000  $21)] ÷ 14,000 units $19.99 per unit 10,000 units  $19.99 [(4,000  $19.99) + (2,000  $23)] ÷ 6,000 units $20.99 per unit 6,000 units  $20.99

Amount

$

60,000

237,120

34,940

199,900

125,940 $ 657,900 (rounded)

b. Over the life of a company, Cost of Goods Sold would be the same regardless of the cost flow assumption employed. Over the life of a business, all the units of inventory will be sold. Consequently, all costs associated with inventory will be expensed. The choice of a cost flow assumption affects only the allocation of inventory costs to particular accounting periods; it does not affect total inventory costs. c.

Assume that accounting earnings equals tax earnings. Over the life of a business, a company's total earnings are the same regardless of the cost flow assumption employed. Therefore, a company's total tax liability over the company's life is the same, regardless of the cost flow assumption employed, as long as tax rates are unchanged. The choice of a cost flow assumption does, however, affect the allocation of inventory costs to particular years. These different cost allocations give rise to different earnings in particular years. The different earnings amounts under different cost flow assumptions then give rise to different tax liabilities (i.e., cash outflows) in particular years. Due to the time value of money, the timing of cash flows affects the present value of the total tax payments. In periods of inventory build-up, the LIFO cost-flow assumption will result in lower earnings while FIFO will result in higher earnings. The opposite is true in times of inventory liquidation. Consequently, LIFO results in lower tax payments when a company builds up its inventories and FIFO results in higher tax payments. The timing of the tax payments means that the present value of tax payments under LIFO is less than the present value of tax payments under FIFO. In times of deflation, the opposite situation arises. The present value of tax payments under FIFO is less than the present value of tax payments under LIFO.


E7–12 a. Inventories on LIFO basis ............................................................. Add: Adjustment to LIFO basis ..................................................... Inventories on FIFO basis .............................................................

$8,781 3,183 $ 11,964

b. Accumulated tax savings can be computed by multiplying the tax rate by the total decrease in net income due to LIFO adoption. Accumulated Tax Savings

c.

= = =

Tax Rate  (2008 LIFO Reserve) .21  ($3,183) $668

The 2008 reported net income under the FIFO cost flow assumption would be $6,740 ($3,557 + $3,183) even if Caterpillar had chosen to change from LIFO to FIFO years earlier. d. The information generated in parts (a), (b), and (c) could be useful to the users from several perspectives. First, users could use the information to compare Caterpillar with other companies within the industry that use FIFO cost flow assumption. Second, the users can readily see the tax savings that the company has generated as a result of its choice of LIFO cost flow assumption. Thirdly, along with other information, users can use this information to assess the quality of earnings of Caterpillar. e. Under IFRS the last-in, first-out (LIFO) inventory cost flow assumption is prohibited. The cost of inventory generally is determined using the first-in, first-out (FIFO) or averaging assumption. Caterpillar would have to abandon its LIFO method and the related benefits.

E7–13 a. Loss on Inventory Write-down (Lo, –SE) ................................................... Inventory (–A) ..................................................................................... Wrote inventory down to market value. Cash (+A) ............................................................................................. Sales (R, +SE)....................................................................................... Sold Item #1.

12 12

50 50

Cost of Goods Sold (E, –SE) ....................................................................... Inventory (–A) ..................................................................................... Recorded cost of goods sold for Item #1. Cash (+A)

.............................................................................................

28 28 50


Sales (R, +SE)....................................................................................... Sold Item #2. Cost of Goods Sold (E, –SE) ....................................................................... Inventory (–A) ..................................................................................... Recorded cost of goods sold for Item #2.

E7–13

40 40

Concluded

b. Item #1 Item #2 c.

50

2011 $12 loss 0

2012 $22 profit 10 profit

Total $10 profit 10 profit

As demonstrated in Part (b) for Item #1, a company can trade off a loss in one period for increased profits in a later period. This implies that if a company is having a good year, it can hide some of those profits by writing down its inventory and then recognize increased profits in future periods when the company's profits may be lower.

E7–14 a. Unilever is a manufacturer. Manufacturing companies carrying raw materials inventory in addition to finished goods inventory, while retailers only carry finished goods. b. Unilever uses the First-In, First-Out (FIFO) method; under IFRS, the Last-In, First-Out (LIFO) method is prohibited. c. An inventory writedown is an adjustment to the carrying value of inventory when the market value has decreased below the cost of the inventory. An inventory recovery is an adjustment to the carrying value of inventory when the market value of inventory previously written down has increased; the recovery restores some of the amount previously lost due to the writedown. The appropriate journal entries for the writedown and recovery are: Inventory Writedown Expense (E, -SE) Inventory (-A)

246

Inventory (+A) Inventory Recovery (R, +SE)

23

246

23

d. If Unilever used U.S. GAAP instead of IFRS, the company would have the option to use LIFO as an inventory method. Secondly, the company would use a different market valuation for its inventory when determining the amount of any writedown expense. (Under GAAP, the market value is normally the replacement cost; under IFRS, the market value is normally the realizable value, the amount at which the inventory could be sold.) Finally, the company would not be allowed to book the recovery of inventory value; under GAAP, inventory may be written down but is never allowed to be written up in value.


PROBLEMS P7–1 11/15

11/26

12/2

12/2

Inventory (+A) ................................................................................... Accounts Payable (+L) ................................................................ Purchased inventory on account.

8,000

Inventory (+A) ................................................................................... Accounts Payable (+L) ................................................................ Purchased inventory on account.

12,000

Accounts Payable (–L) ....................................................................... Cash (–A) .................................................................................... Paid supplier.

8,000

Accounts Payable (–L) ....................................................................... Cash (–A) .................................................................................... Inventory (–A) ............................................................................ Paid supplier.

12,000

Inventory (+A) ............................................................................ Accounts Payable (+L) ......................................................... Purchased inventory on account.

30,000

Inventory (+A) ............................................................................ Accounts Payable (+L) ......................................................... Purchased inventory on account.

60,000

Accounts Payable (–L) ................................................................ Cash (–A) ............................................................................. Inventory (–A) ..................................................................... Paid supplier.

30,000

8,000

12,000

8,000

11,760 240

P7–2 a.

3/5

3/10

3/13

30,000

60,000

29,400 600


7/18

P7–2 b.

3/10

Accounts Payable (–L) ................................................................ Cash (–A) ............................................................................. Paid supplier.

60,000 60,000

Concluded Inventory (+A) ............................................................................ Accounts Payable (+L) ......................................................... Purchased inventory on account.

60,000

Accounts Payable (–L) ................................................................ Cash (–A) ............................................................................. Inventory (–A) ..................................................................... Paid supplier. _____________ * $800 = ($60,000  2/3)  2% discount

40,000

8/7

20,000

3/19

Accounts Payable (–L) ................................................................ Cash (–A) ............................................................................. Paid supplier.

60,000

39,200 800*

20,000

P7–3 The correct amount that should be reported for Cost of Goods Sold is calculated using the following formula. Error in Ending Inventory = Error in Beginning Inventory + Error in Purchases – Error COGS 2006: $500 = $0 + $0 – Error in COGS Error in COGS = ($500). Therefore, COGS as reported is understated $500. Correct COGS = $3,547 + $500 = $4,047 2007: ($150) Error in COGS Correct COGS

= $500 + $0 – Error in COGS = $650: COGS as reported is overstated $650. = $4,249 – $650 = $3,599

2008: $320 Error in COGS

= ($150) + $0 – Error in COGS = ($470). Therefore, COGS as reported is understated $470.

in


Correct COGS

= $4,383 + $470 = $4,853

The restated income statements follow. 2008 $ 20,378 4,853 $ 15,525 18,067 $ (2,542)

Sales Cost of goods sold Gross profit Expenses Net income

2007 $ 18,634 3,599 $ 15,035 11,432 $ 3,603

2006 $ 15,691 4,047 $ 11,644 9,481 $ 2,163

P7–4 a. Cost of Goods Available for Sale

Number of Units Available for Sale

FIFO: Ending Inventory

Cost of Goods Sold

LIFO: Ending Inventory

Cost of Goods Sold

Averaging: Cost per Unit

= =

= Cost of Goods in Beginning Inventory + Cost of Goods Purchased = (15,000 units x $1) + (6,000 units  $1.30) + (9,000 units  $1.50) + (7,000 units  $1.60) = $47,500 = Number of Units in Beginning Inventory + Number of Units Purchased = 15,000 + 22,000 = 37,000 units

(7,000 units  $1.60) + (4,000 units  $1.50) $17,200 = Cost of Goods Available for Sale – Ending Inventory = $47,500 – $17,200 = $30,300

= (11,000 Units  $1.00) = $11,000 = = =

Cost of Goods Available for Sale – Ending Inventory $47,500 – $11,000 $36,500

= Cost of Goods Available for Sale ÷ Number of Units Available for Sale = $47,500 ÷ 37,000 Units = $1.284 per Unit

Ending Inventory

Cost of Goods Sold

= Number of Units in Ending Inventory  Cost per Unit = 11,000 units  $1.284 per unit = $14,124 =

Cost of Goods Available for Sale – Ending Inventory


= =

P7–4

$47,500 – $14,124 $33,376

Continued Lumbermans and Associates Income Statements For the Year Ended December 31, 20XX

Sales Cost of goods sold Gross profit Other expenses Income before taxes Income taxes Net income

FIFO $ 55,000 30,300 $ 24,700 15,000 $ 9,700 2,910 $ 6,790

Averaging $ 55,000 33,376 $ 21,624 15,000 $ 6,624 1,987 $ 4,637

LIFO $ 55,000 36,500 $ 18,500 15,000 $ 3,500 1,050 $ 2,450

b. By using LIFO rather than FIFO, Lumbermans and Associates would save $1,860 ($2,910 – $1,050) in taxes. c.

Ending inventory at market value = 11,000 units  $1.35 per unit = $14,850 Lower-of-cost-or-market value: Cost Market value Excess of cost over market value (cannot be negative)

FIFO $ 17,200 14,850

Averaging $ 14,124 14,850

LIFO $11,000 14,850

2,350

0

0

FIFO method: Loss on Inventory Write-down (Lo, –SE) ........................................................ Inventory (–A) .......................................................................................... Adjusted inventory to LCM. Averaging method: No entry is necessary.

2,350 2,350


LIFO method: No entry is necessary. d. Cost of Goods Available for Sale

=

Cost of Goods in Beginning Inventory + Cost of Goods Purchased = (15,000 units  $1.60) + (6,000 units  $1.40) + (9,000 units  $1.30) + (7,000 units  $1.20) = $52,500

Number of Units Available for Sale

P7–4

= Number of Units in Beginning Inventory + Number of Units Purchased = 15,000 + 22,000 = 37,000 units

Concluded

FIFO: Ending Inventory

Cost of Goods Sold

= (7,000 units  $1.20) + (4,000 units  $1.30) = $13,600 = = =

Cost of Goods Available for Sale – Ending Inventory $52,500 – $13,600 $38,900

LIFO: Ending Inventory

Cost of Goods Sold

= 11,000 units  $1.60 = $17,600 = = =

Cost of Goods Available for Sale – Ending Inventory $52,500 – $17,600 $34,900

Averaging: Cost per Unit

= Cost of Goods Available for Sale ÷ Number of Units Available for Sale = $52,500 ÷ 37,000 units = $1.419 per unit

Ending Inventory

Cost of Goods Sold

= Number of Units in Ending Inventory  Cost per Unit = 11,000 units  $1.419 per unit = $15,609 = Cost of Goods Available for Sale – Ending Inventory = $52,500 – $15,609 = $36,891 Lumbermans and Associates


Income Statements For the Year Ended December 31, 20XX FIFO $ 55,000 38,900 $ 16,100 15,000 $ 1,100 330 $ 770

Sales Cost of goods sold Gross profit Other expenses Income before taxes Income taxes Net income

Averaging $ 55,000 36,891 $ 18,109 15,000 $ 3,109 933 $ 2,176

LIFO $ 55,000 34,900 $ 20,100 15,000 $ 5,100 1,530 $ 3,570

LIFO gives rise to the highest net income in this case. Under FIFO, the oldest costs flow into COGS before the most recent costs. Under LIFO, the most recent costs flow into COGS before the older costs. Under the averaging method, all the costs are averaged to determine COGS. In this case, the cost of the inventory is decreasing, so the LIFO cost flow assumption uses lower, newer costs in computing COGS than the other two methods. Since these lower costs flow into COGS under LIFO, the older, higher costs flow into ending inventory.

P7–5 a. Cost of Goods Available for Sale

= Cost of Goods in Beginning Inventory + Cost of Goods Purchased = (500 units x $70) + (1,000 units  $75) + (3,000 units  $80) + (4,000 units  $82) = $678,000

Number of Units Available for Sale

= Number of Units in Beginning Inventory + Number of Units Purchased = 500 + 8,000 = 8,500 units

Units Sold Units remaining in Inventory

= =

FIFO: Ending Inventory

Cost of Goods Sold

LIFO: Ending Inventory

Cost of Goods Sold

= =

6,000 units 2,500 units

2,500 units  $82 $205,000 = Cost of Goods Available for Sale – Ending Inventory = $678,000 – $205,000 = $473,000

= (500 units  $70) + (1,000 units x $75) + (1,000 units x $80) = $190,000 = = =

Cost of Goods Available for Sale – Ending Inventory $678,000 – $190,000 $488,000


Averaging: Cost per Unit

= Cost of Goods Available for Sale ÷ Number of Units Available for Sale = $678,000 ÷ 8,500 Units = $79.76 per Unit

Ending Inventory

Cost of Goods Sold

P7–5

= Number of Units in Ending Inventory  Cost per Unit = 2,500 units  $79.76 per unit = $199,400 = = =

Cost of Goods Available for Sale – Ending Inventory $678,000 – $199,400 $478,600

Continued Laundryman’s Corporation Income Statements For the Year Ended December 31, 20XX FIFO $ 900,000 473,000 $ 427,000 125,000 $ 302,000 90,600 $ 211,400

Sales Cost of goods sold Gross profit Other expenses Income before taxes Income taxes Net income

Averaging $ 900,000 478,600 $ 421,400 125,000 $ 296,400 88,920 $ 207,480

LIFO $ 900,000 488,000 $ 412,000 125,000 $ 287,000 86,100 $ 200,900

b. By using LIFO rather than FIFO, Laundryman’s would save $4,500 ($90,600 – $86,100) in taxes. c.

Ending inventory Cost

Market /unit Writedown

FIFO 2,500 units @ $82

$78

Averaging LIFO 2,500 units @ $79.76 500 units @ $70 1,000 units @ $75 1,000 units @ $80 $78 $78

2,500 x ($82 - $78) 2,500 x ($79.76 - $78)

1,000 x ($80 - $78)


FIFO method: Loss on Inventory Write-down (Lo, –SE) ........................................................ Inventory (–A) .......................................................................................... Adjusted inventory to LCM.

10,000

Averaging method: Loss on Inventory Write-down (Lo, –SE) ........................................................ Inventory (–A) .......................................................................................... Adjusted inventory to LCM. LIFO method: Loss on Inventory Write-down (Lo, –SE) ........................................................ Inventory (–A) .......................................................................................... Adjusted inventory to LCM.

10,000

4,400 4,400

2,000

P7–5 Concluded d. Cost of Goods Available for Sale

=

Cost of Goods in Beginning Inventory + Cost of Goods Purchased = (500 units x $80) + (1,000 units  $78) + (3,000 units  $77) + (4,000 units  $75) = $649,000

Number of Units Available for Sale

= Number of Units in Beginning Inventory + Number of Units Purchased = 500 + 8,000 = 8,500 units

Units Sold Units remaining in Inventory FIFO:

= =

Ending Inventory

Cost of Goods Sold

6,000 units 2,500 units

= (2,500 units  $75) = $187,500 = = =

Cost of Goods Available for Sale – Ending Inventory $649,000 – $187,500 $461,500

LIFO: Ending Inventory

Cost of Goods Sold

= (500 units  $80) + (1,000 units x $78) + (1,000 units x $77) = $195,000 = =

Cost of Goods Available for Sale – Ending Inventory $649,000 – $195,000

2,000


=

$454,000

Averaging: Cost per Unit

= Cost of Goods Available for Sale ÷ Number of Units Available for Sale = $649,000 ÷ 8,500 units = $76.35 per unit

Ending Inventory

Cost of Goods Sold

= Number of Units in Ending Inventory  Cost per Unit = 2,500 units  $76.35 per unit = $190,875 = Cost of Goods Available for Sale – Ending Inventory = $649,000 – $190,875 = $458,125

Laundryman’s Corporation Income Statements For the Year Ended December 31, 20XX

Sales Cost of goods sold Gross profit Other expenses Income before taxes Income taxes Net Income

FIFO $ 900,000 461,500 $ 438,500 125,000 $ 313,500 94,050 $219,450

Averaging $ 900,000 458,125 $ 441,875 125,000 $ 316,875 95,063 $221,812

LIFO $ 900,000 454,000 $ 446,000 125,000 $ 321,000 96,300 $224,700

Because Cost of Goods Sold is the lowest under LIFO due to deflation, LIFO yields the highest net income in this case. Under FIFO, the oldest costs flow into COGS before the most recent costs. Under LIFO, the most recent costs flow into COGS before the older costs. Under the averaging method, all the costs are averaged to determine COGS. In this case, the cost of the inventory is decreasing, so the LIFO cost flow assumption uses lower, newer costs in computing COGS than the other two methods. Since these lower costs flow into COGS under LIFO, the older, higher costs flow into ending inventory.

P7–6 a. LIFO cost flow assumption:


(1)

Purchases (+A) ...................................................................... Accounts Payable (+L) .................................................... Purchased inventory on account.

140,000

Cash (+A) ............................................................................... Sales (R, +SE) .................................................................. Made cash sales.

100,000

Accounts Receivable (+A)...................................................... Sales (R, +SE) .................................................................. Made sales on account.

200,000

Accounts Payable (–L) ........................................................... Cash (–A) ........................................................................ Purchase Discount (–A) .................................................. Made payment to supplier. _____________ * $2,800 = $140,000  2% discount

140,000

(5)

248,500

(2)

(3)

(4)

(6)

1/3

1/3

1/9

1/10

1/15

1/19

Purchases (E, –SE) ................................................................. Cash (–A) ........................................................................ Accounts Payable (+L) .................................................... Purchased inventory. Purchases (+A) ...................................................................... Accounts Payable (+L) .................................................... Purchased inventory.

(7) 1/23

140,000

100,000

200,000

137,200 2,800*

73,500 175,000

182,000 182,000

Accounts Payable (–L) ........................................................... Cash (–A) ........................................................................ Purchase Discount (–A) .................................................. Made payment to supplier. _____________ * $1,750 = ($175,000 x ½)  2% discount

87,500

(8)

Purchases (+A) ...................................................................... Cash (–A) ........................................................................ Purchased inventory on account.

112,000

Accounts Payable (–L) ........................................................... Cash (–A) ........................................................................ Made payment to supplier.

87,500

(9)

1/27

1/28

85,750 1,750*

112,000

87,500


P7–6

Continued

(10) 1/28

Accounts Payable (–L) ........................................................... Cash (–A) ........................................................................ Purchase Discount (–A) .................................................. Made payment to supplier. _____________ * $3,640 = $182,000  2% discount

182,000

(11) 1/29

Cash (+A) ............................................................................... Sales (R, +SE) .................................................................. Made cash sales.

360,000

Accounts Receivable (+A)...................................................... Sales (R, +SE) .................................................................. Made sales on account.

300,000

Purchases (+A) ...................................................................... Cash (–A) ........................................................................ Purchased inventory.

60,000

Freight-In (+A) ....................................................................... Accounts Payable (+L) .................................................... Incurred freight costs on inventory.

30,000

(12) 1/30

(13) 1/31

(14) 1/31

a. LIFO cost flow assumption

178,360 3,640*

360,000

300,000

60,000

30,000

....................

Adjusting entry 1/31 Inventory (ending) ................................................................ 393,250* Cost of Goods Sold ................................................................ 466,060 Purchase Discount ................................................................ 8,190 Purchases ....................................................................... Freight-In ........................................................................ Inventory (beginning) ..................................................... Recorded COGS and ending inventory. _____________ * Units in Ending Inventory = Units in Beginning Inventory + Units Purchased – Units Sold 18,000 = 5,000 + 30,000 – 17,000: Cost of units in inventory using LIFO: $393,250 = (5,000 units  $19.00) + (7,000 units  $20.60) + (6,000 units  $25.675) The unit costs used to calculate the $393,250 were taken from the following table. Date Number of Units Beg. Inv. 5,000 1/3 7,000 1/15 10,000 1/19 7,000 1/27 4,000 1/31 2,000 __________________

Unit Cost $19.00 20.00 24.85b 26.00 28.00 30.00

Unit Freighta $0.00 1.00 1.00 1.00 1.00 1.00

Unit Discount $ 0.00 0.40 0.175c 0.52 0.00 0.00

742,500 30,000 95,000

Total Unit Cost $19.00 20.60 25.675 26.48 29.00 31.00


P7–6

Concluded a $1.00 = $30,000 freight bill ÷ 30,000 units purchased b $24.85 unit cost = [(3,000  $24.50) + (7,000  $25.00)] ÷ 10,000 units c

$0.175 unit discount = Total discount of $1,750 ÷ 10,000 units

b. FIFO cost flow assumption: All entries throughout January would be identical under the FIFO and LIFO cost flow assumptions using the periodic method. The only difference would be in the adjusting entry to record COGS and ending inventory. Adjusting entry Inventory (ending)............................................................... 491,735* Cost of Goods Sold .............................................................. 367,575 Purchase Discount ............................................................... 8,190 Purchases ....................................................................... Freight-In ........................................................................ Inventory (beginning) ..................................................... Recorded COGS and ending inventory. _________ * The computations for ending inventory are based upon the table used in part (a) $491,735 = (2,000 units  $31.00) + (4,000 units  $29.00) + (7,000 units  $26.48) (5,000 units  $25.675) 1/31

742,500 30,000 95,000

+

P7–7 a.

Current Assets $15,820a 14,768b

÷ ÷ ÷

Current Liabilities $20,852 20,852

FIFO LIFO Decrease _____________ a $15,820 = $10,768 in cash + $5,052 in inventory b $14,768 = $10,768 in cash + $4,000 in inventory b.

= = =

Current Ratio .76 .71 .05

FIFO Sales Cost of goods sold: Beginning inventory Purchases Cost of goods available Ending inventory Cost of goods sold Gross profit Expenses Income before taxes Income tax Net income

LIFO $ 63,747

$

5,110 18,453 $ 23,563 5,052

$ 63,747 $

5,110 18,453 $ 23,563 4,000 18,511 $ 45,236 28,307 $ 16,929 3,980 $ 12,949

19,563 $ 44,184 28,307 $ 15,877 3,810 $ 12,067


P7–7

Concluded

Change in gross profit = $45,236 – $44,184 = $1,052 Change in net income = $12,949 – $12,067 = $882 c.

Tax dollars saved = $3,980 – $3,810 = $170

d. Using LIFO can have several disadvantages. First, LIFO requires a company to maintain records for older inventory acquisitions. This practice usually results in higher bookkeeping costs. Second, to avoid "eating into" a LIFO layer, which would result in older, lower inventory costs flowing into COGS and raising the company's net income and associated tax liability, managers may purchase inventory at a time or at a cost that is not advantageous to the company. Third, LIFO can adversely affect a company's and/or manager's contracts. A company's debt covenants may stipulate a minimum current ratio, or level of working capital. These both would be lower under LIFO than under FIFO (assuming inflation). Also, using LIFO reduces net income during inflationary periods. If a manager has an incentive contract linked to net income, the manager's compensation would decrease. Finally, the lower net income achieved under LIFO may mislead current and potential investors into believing that the company is performing poorly (although some current research indicates that this last point is not likely).

P7–8 a. Ending Inventory, 12/31/2011: LIFO layers: 1998 4,000 units x $5 per unit

=

$

20,000

b. Ruhe Auto Supplies Income Statement For the Year Ended December 31, 2011 Revenue ....................................................................................... Cost of goods sold: Beginning inventory ................................................................ $ 112,500 Purchases................................................................................. 902,500a Cost of goods available for sale .............................................. $ 1,015,000 Ending inventory ..................................................................... 20,000b Cost of goods sold ................................................................... Gross profit .................................................................................. Operating expenses ..................................................................... Income before income taxes ....................................................... Income taxes ................................................................................ Net income................................................................................... ___________ a $902,500 = 9,500 units purchased during 2011  $95 per unit b $ 20,000 = 4,000 units from 1998  $5 per unit The company's income tax liability is $361,500, and its net income is $843,500.

$ 3,000,000

995,000 $ 2,005,000 800,000 $ 1,205,000 361,500 $ 843,500


P7–8

Concluded

c. Revenue ....................................................................................... $ 3,000,000 Cost of goods sold: Beginning inventory ................................................................ $ 112,500 Purchases................................................................................. 1,900,000a Cost of goods available for sale ............................................... $2,012,500 Ending inventory ..................................................................... 112,500b Cost of goods sold ................................................................... 1,900,000 Gross profit .................................................................................. $ 1,100,000 Operating expenses ..................................................................... 800,000 Net income before taxes.............................................................. $ 300,000 Income taxes ................................................................................ 90,000 Net income................................................................................... $ 210,000 ___________ a $1,900,000 = (9,500 units + 10,500 units)  $95 per unit b $ 112,500 = (14,000 units  $5) + (500 units  $85) Purchasing an additional 10,500 units of inventory at $95 per unit on December 31, 2011 would cost Ruhe Auto Supplies $997,500. By incurring these costs, the company would save only $271,500 in taxes (i.e., $361,500 from part [b]) – $90,000). So on the face of it, it appears that it would not be a wise decision to acquire these additional units of inventory. However, if Ruhe Auto Supplies was planning to acquire additional inventory early in 2012 anyway, then it might not be a bad decision to acquire the inventory at the end of 2011 to lower the company's taxes.

P7–9 a.

Brady’s 2011 reported income under LIFO .................................................. LIFO Layer Liquidation during 2011 (net of income taxes) .......................... FIFO Based Net Income After Taxes .............................................................

$ 42,700 – 5,200a $ 37,500

a = $8,000 x (1- .35) = $5,200, after tax impact of no LIFO liquidation during 2011.

Brady has gone from reporting higher net income to having lower net income. b. Restatement of Brady’s 2011 reported income, if it had always been a FIFO user, can be computed as follows: Brady’s 2011 reported income under LIFO ................................................ $ 42,700 Decrease in LIFO Reserve (net of income taxes) ..................................... – 845a LIFO Layer Liquidation during 2011 (net of income taxes) ...................... – 5,200b FIFO Based Net Income After Taxes ........................................................ $ 36,655 a = ($4,800 – $3,500)  1 – .35) = $845 b = $8,000  (1– .35) = $5,200

According to the analysis given above, Brady’s restated reported income is $36,655 which is lower than Danner’s reported net income. The reason Brady’s income under FIFO is lower than under LIFO is due to the decline in the LIFO reserve and LIFO layer liquidation.


P7–9 c.

Concluded

As of the end of 2011 Brady had a LIFO reserve of $3,500. A LIFO reserve shows the accumulated benefit derived from the LIFO method. Due to the adoption of LIFO Brady reduced its cumulative pretax income by $3,500. In other words, Brady saved taxes worth $3,500  .35 = $1,225 due to its choice of LIFO. As of the end of 2010, due to LIFO adoption, Brady’s cumulative net income decreased by $4,800 on a pre-tax basis. The related tax savings were $4,800  .35 = $1,680. The impact of a LIFO liquidation shows that adoption of LIFO does not necessarily save taxes in all years. LIFO has adverse effects when the layer liquidation occurs.

d. From an income tax point it is not advisable for Brady to change its cost flow assumption. If it did so, it would have to pay taxes on the $3,500 of additional income that would be created by eliminating the LIFO reserve. However, if the company wishes to report higher income, the change may be desirable.

P7–10 a. and b. IBT Income Statements For the Year Ended December 31,2011 Sales ................................................................ Cost of sales .................................................. Gross profit ................................................... Other expenses ............................................. Income (loss) before taxes ............................ Income taxes ................................................. Net income (loss) ..........................................

Part (a) $ 67,500 17,700 a 27,000 b $ 49,800 20,000 $ 29,800 8,940 $ 20,860

$

Part (b) 67,500 $ 40,500 20,000 $ 20,500 6,150 $ 14,350

____________ a $17,700 = (350 units  $30) + (200 units  $15) + (350 units  $12) b $27,000 = (900 units  $30) c.

The primary advantage of purchasing the additional 550 units on December 20 is the effect on income taxes. Under part (a), IBT would have to pay $8,940 in income taxes. However, under part (b), IBT would have to pay only $6,150 in income taxes. So the net difference between the income statements of parts (a) and (b) is $2,790 in taxes saved. Since income taxes represent a cash flow, the strategy of acquiring the additional 550 units would save IBT $2,790 in cash from income taxes. This tax savings is not without a cost however. To obtain the savings, IBT had to purchase 550 additional units for $16,500. If IBT was planning on acquiring at least 550 units some time in the near future, then the cost of the tax savings is not $16,500, but is rather the return lost on an alternative use of the $16,500. If IBT was not planning on acquiring additional inventory, then the cost of obtaining the tax savings would be the entire $16,500 plus the opportunity cost of not investing the $16,500.


P7–11 a.

Ending Inventory = 400 x $1.50a = $600 Net Income: Sales Cost of Goods Soldb Gross Profit Inventory Writedown Expensec Net Income a

$30,500 15,000 15,500 400 $15,100

market value of $1.50 per item is less than cost (FIFO) of $2.50 per item

bCOGS (FIFO) = (500 x $2) + (5,600 x $2.50) c($2.50 - $1.50) x 400 remaining items

b. Inventory (+A) 520a Inventory Recovery(R, +SE) 520 aRecovery = ($2.80 - $1.50) x 400 items

If Helio Brothers used U.S. GAAP instead of IFRS, no 2012 entry would be made. Under IFRS previous inventory writedowns may be recovered, but under GAAP inventory recoveries are prohibited.


ISSUES FOR DISCUSSION ID7–1 If investors are solely interested in net income, then the partner is probably correct, and companies should select FIFO if they want to raise capital. However, this view is probably not valid. One must remember that net income is simply a measurement; one must not lose sight of what accountants are measuring. Net income is only valuable if it truly represents an increase in the company's net assets. FIFO will result in higher reported income, but the higher income is an illusion. That is, the increased income under FIFO is due to the difference between the inventory's current market value and the older, "understated" inventory costs matched against it. This is why FIFO results in "paper profits." Alternatively, LIFO matches the most recent, higher inventory costs against revenue, which provides a higher quality measure of the company's underlying economic condition. In addition, the reduced income under LIFO implies lower taxes. The lower taxes, in turn, provide cash that the company can plow back into the business to improve operations. Thus, although LIFO results in lower reported income, LIFO provides a higher quality measure of income and results in lower taxes.

ID7–2 a. The choice of LIFO or FIFO will affect the amounts a company reports both in its balance sheet for inventory and in its income statement for cost of goods sold (and consequently net income). Thus, in order to evaluate a company's financial position and performance, particularly in comparison with other companies' performances, investors and creditors need to know which cost-flow assumption the company is using. In addition, the choice of LIFO or FIFO can have a large effect on the company's cash flows. If inventory costs are rising, a company will have lower taxable income—and hence lower cash outflows for taxes—if it uses LIFO than if it uses FIFO. For some companies the difference can be several million dollars a year in tax savings. b. Under LIFO, the cost of the inventory sold is assumed to be the cost of the inventory purchased most recently. This implies that the cost of the inventory still on hand is assumed to be the cost of inventory purchased long ago. If inventory costs are rising, one would expect the costs assigned to the inventory still on hand to be very low relative to the most recent inventory costs. If a company sells more inventory than it acquires during the year, the company will have to dip into those older inventory costs (i.e., liquidate LIFO layers) when calculating the cost of inventory sold during the year. Because those older costs are less—in some cases much less—than the most recent inventory costs, a LIFO liquidation will result in Cost of Goods Sold being less than it would have been in the absence of the LIFO liquidation. This means that the company's income will be much greater which, in turn, implies higher tax payments. Thus, investors would be interested in LIFO liquidations because they have implications for the amount of cash the company will have to pay out in taxes. c.

According to the footnote, Deere’s 2009 ending inventory under FIFO would be $1,367 million more than under LIFO. Therefore, COGS under FIFO would be lower by the same amount and net income before tax higher by the same amount. Based on a 34% tax rate, therefore, Deere would have to pay an additional income tax of $464.8 million ($1,367  .34).

d. Under IFRS the use of LIFO is prohibited. If IFRS were to be adopted, therefore, Deere would switch to FIFO as its inventory method and would incur the additional tax expenses discussed above.


ID7–3 In times of rising inventory costs, LIFO allows companies to "hide" the value of their inventory. That is, the inventory value reported on the balance sheet is assumed to consist of "old" inventory costs; the most recent costs of inventory are allocated to cost of goods sold. However, the inventory is really worth its current market value. Thus, the difference between the "old" inventory costs and the current market value represents a "hidden reserve" of profits. By manipulating its inventory acquisition, a company can dip into this reserve and increase its reported income.

ID7–4 a.

b.

Loss on Inventory Write–down (Lo, –SE) .......................................... Inventory (–A) ............................................................................ Wrote down inventory to market value. Period 1 Loss on Inventory Write–down (Lo, –SE) .......................................... Inventory (–A) ............................................................................ Wrote down inventory to market value. Period 2 Accounts Receivable (or Cash) (+A) .................................................. Sales Revenue (R, +SE) ............................................................... Sold inventory. Cost of Goods Sold (E, –SE) ............................................................... Inventory (–A) ............................................................................ Recorded cost of goods sold.

c.

12,000,000 12,000,000

12,000,000 12,000,000

48,000,000 48,000,000

40,000,000 40,000,000

Because the lower-of-cost-or-market rule gives differential treatment to price decreases and price increases, and because it forces the recognition of losses before they are realized, it may provide inconsistent measures of net income. However, such conservative accounting treatments are employed in response to the liability faced by those who provide and audit financial statements. The costs associated with understating inventories and profits are typically less than the potential costs of overstating them.


ID7–5 a. Valero is using the lower of cost or market exception to the historical cost principle that is applied to inventory. If the market value of inventory is lower than the cost of that inventory, it must be written down to the lower value. b. The write-down will lower reported income, current assets and the equity of Valero. c.

Valero’s current ratio will decrease because inventory will be carried at a lower value, which lowers current assets, while there is no change to current liabilities. Valero’s inventory turnover ratio will increase because average inventory for the year will be lower.

d. By reducing the carrying value of inventory Valero is reducing earnings in the current quarter. As this inventory is sold in future periods Valero will report higher earnings than it would have with no writedown. Valero’s reporting strategy could be to either lower this quarter’s earnings because it produced greater earnings than it anticipated, or Valero could be trying to take a large loss this quarter in order to be able to report better earnings in future quarters. e. Under U.S. GAAP, inventory is written down, if appropriate, but never written back up. Therefore, Valero would simply leave the inventory at the written-down carrying cost, even if market prices rebound. Under IFRS, on the other hand, the inventory writedowns can be recovered if market prices move back up. In that case, if Valero used IFRS, the company would book an Inventory Recovery (which would increase profits and equity) to balance out the increase in the carrying value of the asset.

ID7–6 a. If Sherwin Williams reported inventories at the end of 2008 based on a FIFO system, the ending inventory balance would have been $1,185,480 ($864,200 + $321,280). b. The following were the tax effects to Sherwin Williams as a result of using LIFO. 2006 2007 2008 (Decrease) in net income due to LIFO (24,033) (7,844) (49,184) Pretax effect on net income (effect /(1-tax rate)) (35,870) (11,707) (73,409) (Decrease) in tax liability ($11,837) ($3,863) ($24,225)

c.

A LIFO inventory system operates on the premise that inventory that is sold is the inventory that was most recently purchased and therefore reflects the most current prices for the inventory. By taking this approach, this gives the best indication as to the future earnings potential of the company. This is particularly true during periods of inflation where the cost of inventory could increase dramatically in short periods of time. A LIFO inventory method most closely approximates the earnings power of buying a new unit of inventory today and selling it in the marketplace today. In times of deflation, LIFO still does a better job of matching current costs with current revenues.


ID7–7 a. A company “thins” its inventory when it reduces the amount of inventory it owns at any given time. Inventory is an asset that has a cost; lower levels of inventory mean lower costs to carry that iventory. Companies that are trimming their operations may opt to reduce their investment in inventory, just like they might decide to own fewer buildings or a smaller number of company cars. b. The balance sheet will show the lower investment in inventory from one time period to the next. Another area in the financial statements to determine if a company is reducing its inventory would be the Statemnt of Cash Flow (Indirect Method). If inventory is being reduced, the change in inventory will show up as a source of Cash in the Operating Activities section. c.

FedEx acts as a transportation company and does not take ownership of the goods being shipped. Although FedEx has physical possession of the item being shipped, it remains on the balance sheet of FedEx’s customer.

ID7–8 The entries on the statement of cash flows are intended to show the impact on cash of the changes in the various balance sheet accounts. A change in an asset account impacts the statement of cash flows because if the asset account increased it reduced the amount of cash, and if the asset account decreased then it increased cash. To increase the inventory account the company has to go out and buy inventory, which reduces cash. If the company sold more inventory than it purchased then this would increase the amount of cash in the company. This data reveals that at the end of 2006 J.C. Penney had $190 more inventory than it did at the end of 2005. At the end of 2007 J.C. Penney had $241 more inventory than it had at the end of 2006 and at the end of 2008 J.C. Penney had $382 less inventory on hand than at the end of 2007. Since inventory is an asset account, the increase in the investment in inventory effectively decreases the amount of cash on hand, while a decrease in inventory levels effectively increases cash balances.

ID7–9 a. Supervalu has a much larger difference between LIFO and FIFO inventories than does J.C. Penney. Supervalu’s LIFO Reserve of $258 is over 9% of the value of the inventory of $2,709, while Penney’s LIFO Reserve is under 1% of the total ($21 ÷ $3,259).

b. Supervalu: LIFO Inventory $2,709 + $258 = $2,967 FIFO Inventory J.C. Penney: LIFO Inventory $3,259 + $21 = $3,280 FIFO Inventory c.

Assuming a tax rate of 30%, the benefit would be: Supervalu: .30 x $258 = $77.4 J.C. Penney: .30 x $21 = $ 6.3


d. A grocery store such as Supervalu might adopt FIFO for “highly perishable” items such as milk and produce due to the nature of the inventory. Most grocery stores will sell the gallons of milk that are closest to the expiration date before they put the more recently purchased milk on the shelf. The adoption of FIFO, in this case, more closely matches the actual physical flow of the inventory.

ID7–10 a. When an asset such as inventory decreases, it frees up a company’s cash. A drop in an asset is a “source” of cash for a company. Similarly, when a liability such as accounts payable decreases (meaning the company no longer owes its supplier because it has paid out its cash), it “uses” up cash balances. An decrease in a liability is a “use” of cash. (An increase in an asset is also a use of cash, while an increase in a liability is a source of cash.) Since both inventory and accounts payable are used in the running of a company’s daily business operations, changes to those accounts affect the operating section of the statement of cash flows. b. From the changes to inventory and accounts payable, it appears that Target was managing its business to downsize inventory levels during a recession (2008). Inventory balances decreased as did the payables related to those inventories (implying the company was not re-ordering inventory as rapidly once that inventory was sold). Those management behaviors are consistent with a cautious approach for a recession marked by a large drop in consumer spending.

ID7–11 a. Inventory is the second largest asset on the balance sheet of Nike. As of 5/31/2009 inventory was 17.8% of the total assets (only trailing accounts receivable in size). As of 5/31/2008, inventory was 19.6% of total assets, so the change from ’08 to ’09 was a slight drop (in both dollars and as a percentage of total assets). b. Cost of sales (representing the cost to Nike of the footwear and apparel inventory that was sold to customers) was 55.1%, 55.0%, and 56.1% of sales in fiscal years 2009, 2008 and 2007, respectively. c.

We are not provided balance sheet information for 2007, so we cannot figure out average accounts payable for 2008. However, using the accounts payable balances (without averaging them) indicates that Nike is paying faster (35.6 days) in 2009 versus the prior year (45.9 days).

d. In all three years shown Nike has seen an increase in accounts receivable, draining cash balances; the amount of the use of cash in 2009 was $238 million. Inventory increased in both 2007 and 2008, using company cash, but the trend changed in 2009 with a drop in inventory, freeing up $32 million. Accounts payable and other accruals increased in both 2007 and 2008, but (because there are limits to how far a company can stretch its creditors and suppliers) those balances decreased in 2009, using $220 million.

e. Nike uses the FIFO method for its wholesale (manufacturing) inventory and the retail method for the inventory in its company-owned retail stores. The company must run a “just in time” inventory system for its manufacturing operations, resulting in little to no investment in raw materials inventory. As soon as the raw materials are delivered, the company begins to manufacture its products (shoes, apparel, etc.) The manufacturing process is relatively short, limiting the dollar amount of inventory tied up in “work in process” inventory.


CHAPTER 8 INVESTMENTS IN EQUITY SECURITIES BRIEF EXERCISES BE8–1 a.

Comprehensive income includes all non-owner changes in shareholder equity that do not already appear on the income statement. For example, the change in value of assets that have been sold and in certain assets that have not been sold appear as comprehensive income, but net income only includes the changes in assets that have been sold.

b.

The investments are considered Available-for-Sale Securities, because the Unrealized Gains/Losses are not posted to the Income Statement; instead, as shown in the disclosure the Unrealized Gains/Losses are carried directly to Stockholders’ Equity through Comprehensive Income. In 2006 the Available-for-Sale Securities increased $26 million in market value. In 2007 the Available-for-Sale Securities increased $58 million in market value. In 2008 the Available-for-Sale Securities dropped $80.5 million in market value. In each of the three years, Merck did not sell the securities at the yearend value. (An Unrealized Gain/Loss is a holding gain/loss, meaning the securities were not sold but simply marked to the current market value.)

BE8–2 Bristol-Myers Squibb designated its Marketable Securities as Available-for-Sale Securities and as such reflected holding gains/losses (unrealized) in Comprehensive Income, not on the Income Statement. The $37 million reflected a change in the market value of the Securities which were still owned by the company. The asset section of the balance sheet will also show the change, with the carrying value of the Securities adjusted by the $37 million.

BE8–3 a. Investment in Equity Securities (+A) 374 Equity in net income of affiliates (R, +SE) 374

b.

The equity in net income of affiliates does not represent a receipt of cash from the affiliates, but it does represent an increase in net income. To reconcile the net income with the receipt of cash (on the Statement of Cash Flow, Indirect Method), it is necessary to 1) back out the net income from affiliates and 2) add in the cash dividends received from the affiliates.

c.

In Pepsi’s case, in 2008 the company only received cash of $172 million ($374 - $202); in 2007 and 2006, the company received $119 and $111, respectively.

d. Company management would evaluate the affiliate investments in two key areas: the effect on Pepsi’s profitiability and the effect on Pepsi’s cash flow. As the numbers above attest, the investments are contributing increasing amounts to cash flow, but the contribution to 2008 profits was lower than prior years. 1


BE8–4 Goodwill is the amount paid above fair value when assets are acquired. P & G, in its business growth strategy, has acquired other companies and has paid, collectively, $56.5 billion above the fair value of the assets acquired.

BE8–5 Assets (+A) Goodwill (+A) Liabilities(+L) Cash(-A)

1,323 1,800 323 2,800

Goodwill is an intangible asset that represents the excess above fair value paid for assets acquired. J & J paid $2.8 billion in cash and assumed liabilities. Balancing the journal entry, the company acquired assets with a fair value of $1.323 billion. Assets and liabilities increased by $323 million with no change to equity.


EXERCISES E8–1 a. (1)

(2)

(3)

(4)

(5)

(6)

(7)

(8)

Trading Securities (+A) .................................................................... Cash (–A) .................................................................................. Invested in IBM.

50,000

Trading Securities (+A) .................................................................... Cash (–A) .................................................................................. Invested in GM.

40,000

Cash (+A) ......................................................................................... Trading Securities (–A) ............................................................. Realized Gain on Sale of Trading Securities (Ga, +SE) ............................................................... Sold IBM stock.

45,000

Cash (+A) ......................................................................................... Dividend Receivable ................................................................ Received dividend.

750

Trading Securities (+A) .................................................................... Cash (–A) .................................................................................. Invested in Xerox.

8,000

Cash (+A) ......................................................................................... Realized Loss on Sale of Trading Securities (Lo, –SE) ................................................................... Trading Securities (–A) ............................................................. Sold IBM stock.

7,500

Cash (+A) ......................................................................................... Trading Securities (–A) ............................................................. Realized Gain on Sale of Marketable Securities (Ga, +SE) ....... Sold Xerox stock.

11,600

Cash (+A) ......................................................................................... Realized Loss on Sale of Trading Securities (Lo, –SE) ...................... Trading Securities (–A) ............................................................. Sold GM stock.

30,000 10,000

50,000

40,000

37,500 7,500

750

8,000

5,000 12,500

8,000 3,600

40,000


E8–1

Concluded

b. The transactions that affected the income statement for Monroe Auto Supplies are the gains and losses Monroe Auto Supplies realized from selling marketable securities for amounts different from their purchase prices. Monroe Auto Supplies realized gains of $7,500 and $3,600, respectively, on the first sales of IBM and Xerox stock and realized losses of $5,000 and $10,000, respectively, on the second sale of IBM stock and on GM stock. Thus, the net effect of the dividends, gains, and losses on net income was to decrease net income by $3,900. The dividends received from the General Motors stock were recognized as income in the previous period.

E8–2 a. Change in the wealth level of each of the four companies can be computed by comparing the beginning and year-end balance in the short-term equity investment account. Therefore, the amounts for each company are as follows. Wearever Fabrics Frames Corp. Pacific Transport Video Magic

$12,000 2,000 (24,000) 13,000

b. The amount that should be reported as holding gains and losses on the income statement is always due to changes in the market value of trading securities. Therefore, the holding gains and losses for each company are as follows. Wearever Fabrics Frames Corp. Pacific Transport Video Magic

$20,000 (10,000) (20,000) 10,000

c.

The reason that the answers to (a) and (b) are not the same is due to the fact that in part (a) the wealth charges are computed for the trading securities and available-for-sale securities together. In part (b) the holding gains and losses that are being reported on the income statement pertain to only trading securities. For the available-for-sale securities, the holding gains and losses are taken directly to the stockholders’ equity on the balance sheet via comprehensive income.

d.

If the company were to choose the fair market value option, the change in market values for the available-for-sale securities would also be included. Therefore, the income change would match the wealth change outlined in part a above.

E8–3 a. (1)

(2)

(3)

1/28

2/18

3/15

Short-Term Investments (+A) ......................................... Cash (–A) ................................................................. Invested in Able Co.

140

Short-Term Investments (+A) ......................................... Cash (–A) ................................................................. Invested in Baker Co.

520

Cash (+A) ........................................................................ Dividend Revenue (R, +SE) ...................................... Received dividends.

10

140

520

10


E8–3 (4)

(5)

(6)

Continued 4/29

5/18

6/1

Cash (+A) ........................................................................ Short-Term Investments (–A) .................................. Realized Gain on Sale of Short-Term Investments (Ga, +SE) .......................................... Sold Able Co. stock.

75

Cash (+A) ........................................................................ Dividend Revenue (R, +SE) ...................................... Received dividends.

40

Cash (+A) ........................................................................ Realized Loss on Sale of Short-Term Investments (Lo, –SE) ................................................. Marketable Securities (–A) ...................................... Sold Baker Co. stock.

110

b. Able (5 shares) Baker (15 shares) Total

Cost

Market Value

70 390 460

85 300 385

70 5

40

20 130

June 30 Adjusting Entry (1) If Able and Baker are both considered trading securities: Unrealized Price Decrease on trading securities (Lo, –SE) .............. Trading Securities (–A) ............................................................ Revalued trading securities to market.

75 75

(2) If Able is a trading security and Baker is an available-for-sale security: Able Trading Securities (+A) ................................................................... Unrealized Price Increase on Trading Securities (Ga, +SE) ................................................................................ Baker Unrealized Price Decrease (–SE) .................................................... Available-for-Sale Securities (–A) .............................................

15 15 90 90

(3) If Able is considered an available-for-sale security and Baker is considered a trading security: Able Available-for-Sale Securities (+A) ................................................... Unrealized Price Increase on Available-for-Sale Securities (+SE)...................................................................... Baker Unrealized Price Decrease on Trading Securities (Lo,–SE) ....................................................................................... Trading Securities (–A) .............................................................

15 15

90 90


E8–3

Concluded

(4) If Able and Baker are both considered available-for-sale securities: Unrealized Price Decrease on Available-for-Sale Securities (–SE) ............................................................................ Available-for-Sale Securities (–A) ............................................. c.

75 75

Since management usually want to keep losses off the income statement, the second combination in (b) will depict the management as the most successful in the current period. All the losses ($90) are shown as unrealized losses in stockholders equity while the gain of $15 goes to the income statement.

E8–4 a.

Trading 2011 income 12/31/11 balance sheet investment value 2012 income 2012 balance sheet investment value 2013 income Total income

$ 401 2002 103 1205 206 $ 70

Available-For-Sale 01 2002 104 1205 607 $ 70 $

1. 20 shares were purchased at $8, and on 12/31/11 their market value has increased to $10 a share. An unrealized price increase of $40 for Trading Securities will go to the income statement, but the same unrealized gain for available-for-sale securities will go directly into the stockholders’ equity. Therefore there is no impact on the 2011 income statement. 2. 20 shares @ $10 a share as of 12/31/11. 3. 2012 Cash (+A)................................................................................. Realized Loss –RE) .................................................................. Trading Securities ..............................................................

90 10

12/31/12 Trading Securities (+A)............................................................ Unrealized Price Increase (+Ga, +RE) ................................

20

AFS Securities (+A) .................................................................. Unrealized Price Increase (+Ga, +SE) .................................

20

100

20

20

4. For available-for-sale securities: Cash (+A)................................................................................. Unrealized Price Increase (–Ga, –SE)....................................... AFS Securities .................................................................... Realized Gain (+Ga, +SE) ....................................................

90 20 100 10


5. Purchased 20 shares @ $8 12/31/11 Mark to Market @ $10 Sold 10 shares @ cost basis of $10 12/31/12 Mark to Market @ $12 12/31/12 balance sheet value

= = = = =

$160 40 (100) _ 20 $120


E8–4

Concluded

6. Cash (+A) ................................................................................................... Trading Securities (–A)........................................................................ Realized Gain (+Ga, +RE) ....................................................................

140

7. Cash (+A) ...................................................................................................

140 40

Unrealized Price Increase (–Ga, –SE)......................................................... AFS Securities (–A) .............................................................................. Realized Gain (+Ga, +SE) .....................................................................

120 20

120 60

b. Whether a security is classified as a trading security or available-for-sale security, the balance sheet investment value remains the same due to the mark-to-market rule. However, the classification does impact the income statement as can be seen from this problem. Overall, whether the security is a trading security or an available-for-sale security, the total income would remain the same.

E8–5 a. Fair market value of Biomet’s available-for-sale securities:

Cost Gains Losses Fair value

2007 $42.9 0.1 (3.9) $39.1

2008 $825.3 0.0 (0.6) $824.7

b. The effect on the company’s comprehensive income associated with its available-for-sale securities is: 2007 2008 Gains 0.1 0.0 Losses (3.9) (0.6) Net losses $ (3.8) $(0.6)

c. The income that would be realized would be ($0.6) [$3.8 - $4.4]. Cash (+A) ................................................................................................... Realized Loss (Loss, –SE)............................................................................ Available-for-sale Securities (–A)........................................................ Unrealized Price Decrease (-Loss, +SE) ...............................................

824.7 4.4 825.3 3.8


E8–6 a. Tom Miller Balance Sheet December 31, 2011 Assets Cash ....................................... Marketable securities ............ Inventory ...............................

$ 2,880 3,840* 1,500

Total assets ............................

$ 8,220

Liabilities & Stockholders' Equity Accounts payable................. $ 1,500 Contributed capital .............. 6,000 Securities fair value adjustments (availablefor-sale) ............................ 720** Total liabilities & stockholders' equity ......... $ 8,220

* $3,840 = 120 shares  $32 ** 720 = 3,840 – 3,120 Larry Rogers Balance Sheet December 31, 2011 Assets Cash ....................................... Marketable securities ............ Inventory ...............................

$ 2,880b 3,840a 1,500

Total assets ............................

$ 8,220

Liabilities & Stockholders' Equity Accounts payable................. $ 1,500 Contributed capital .............. 6,000 Retained earnings ................ 720c Total liabilities & stockholders' equity ......... $ 8,220

a $3,840 = 120 shares x $32 b $2,880 = $6,000 contributed by Larry – $3,120 for initial investment in Diskette + $3,840 in proceeds c

from sale of Diskette – $3,840 for reinvestment in Diskette. $720 = $3,840 proceeds from sale of Diskette – $3,120 cost of Diskette.

b. Net income Working capital Current ratio c.

Tom Miller

Larry Rogers

$0.00 6,720.00 5.48

$720.00 6,720.00 5.48

Truly speaking no one. Both have the same working capital and current ratio. However, Larry would report a net income of $720 while Tom would not.

d. Tom Miller is actually in better financial position than Larry Rogers. Both of them own the exact same marketable securities, and both own $1,500 in inventory. Tom would have more cash because he chose to simply hold on to his investment and not incur the transaction costs of buying and selling. On the other hand, Larry sold and repurchased his securities, and each transaction cost him brokerage fees.


E8–7 a. (1)

This investment should be classified as a long-term investment and accounted for using the equity method. Hartney Consulting owns 40% of the investee's common stock, which indicates that it can exert some control over the company, and Hartney Consulting intends to hold the investment for longer than the time frame of current assets.

(2)

This investment should be classified as a long-term investment and accounted for using the consolidated method. Hartney owns greater than 50% of the investee, and it appears that Hartney Consulting has no intention of disposing of the investment within the time frame of current assets.

(3)

This investment should be classified as a long-term investment and accounted for using the cost method. To be classified as a short-term investment, a ready market for the securities must exist so that the investor can dispose of the investment at any time. In this case, because the company is closely held, Hartney Consulting could not dispose of the investment when it desires.

(4)

This investment should be classified as a long-term investment and accounted for using the equity method. Hartney Consulting owns 45% of the investee's common stock, which indicates that it desires to exert some control over the company, and Hartney Consulting intends to hold the investment for five years, which is longer than the time frame of current assets.

(5)

This investment should be classified as a long-term investment and accounted for using the mark-tomarket method.

(6)

This investment should be classified as a long-term investment and accounted for using the equity method. Although Hartney Consulting owned less than 20% of the investee for a portion of the year, it owned greater than 20% of the investee as of December 31, 2012. Furthermore, the company intends to hold the investment for four years.

b. To be classified as a short-term marketable equity investment, the equity investment must meet two criteria. First, the investor must intend to dispose of the investment within the time frame of current assets. Second, a ready market for the securities must exist. A ready market for the investment enables the investor to dispose of the investment at any time. In the case of nonmarketable equity securities, the investor may not be able to dispose of the investment when it wishes. Since too much uncertainty exists as to whether the investor could actually dispose of nonmarketable investments within the time frame of current assets, the investment should be classified as a long-term investment. Classifying the investment as long-term is more conservative than classifying the investment as current because classifying the investment as long-term reduces the company's measures of solvency (e.g., working capital, current ratio, and quick ratio). To account for equity investments using the mark-to-market method, the market value of the equity securities must be known. With nonmarketable equity securities, no market value exists. Consequently, nonmarketable equity securities are accounted for using the cost method rather than the mark-to-market method.


E8–8 (1)

(2)

(3)

(4)

(5)

Investment in Equity Securities (+A) .................................................... Cash (–A) ........................................................................................ Invested in Thayers International.

260,000

Investment in Equity Securities (+A) .................................................... Cash (–A) ........................................................................................ Invested in Bayhe Enterprises.

875,000

Dividend Receivable (+A)...................................................................... Dividend Revenue (R, +SE)............................................................. Earned, but did not receive, dividend.

20,000

Cash (+A)............................................................................................... Loss on Sale of Equity Securities (Lo, –SE) ............................................ Investment in Equity Securities (–A).............................................. Sold Bayhe Enterprises stock.

135,000 22,500

Cash (+A)............................................................................................... Investment in Equity Securities (–A).............................................. Gain on Sale of Equity Securities (Ga, +SE) .................................... Sold Thayers International stock.

256,000

Available-For-Sale Securities (+A) ................................................ Cash (–A) ............................................................................... Invested in Thayers International.

260,000

Available-For-Sale Securities (+A) ................................................ Cash (–A) ............................................................................... Invested in Bayhe Enterprises.

875,000

Dividend Receivable (+A) ............................................................. Dividend Revenue (R, +SE) .................................................... Earned, but did not receive, dividend.

20,000

Cash (+A) ...................................................................................... Realized Loss on Available-For-Sale Securities (Lo, –SE) ................................................................... Available-For-Sale Securities (–A) ......................................... Sold Bayhe Enterprises stock.

135,000

Cash (+A) ...................................................................................... Available-For-Sale Securities (–A) ......................................... Realized Gain on Sale of Available-For-Sale Securities (Ga, +SE) ............................................................ Sold Thayers International stock.

256,000

260,000

875,000

20,000

157,500

208,000 48,000

E8–9 a. (1)

(2)

(3)

(4)

(5)

260,000

875,000

20,000

22,500 157,500

208,000 48,000


E8–9

Concluded

b. Securities Bayhe Thayers Total

Shares Held 20,500 2,000

Cost $ 717,500 52,000 $ 769,500

Market Value $ $

656,000 50,000 706,000

Unrealized Loss on Available-For-Sale Securities (–SE) ................................................................................... Available-For-Sale Securities (–A)....................................................... Adjusted investments to market.

63,500 63,500

E8–10 a. 2011 Investment in Equity Securities (+A) ......................................................... Cash (–A)............................................................................................. Invested in Reilly Manufacturing.

190,000 190,000

Cash (+A) ................................................................................................... Investment in Equity Securities (–A) .................................................. Received dividends (equity method).

15,000

Investment in Equity Securities (+A) ......................................................... Income from Equity Investments (R, +SE) .......................................... Earned investment revenue (equity method).

18,750*

15,000

18,750

* $18,750 = $75,000 investee net income  25% 2012 Cash (+A) ................................................................................................... Investment in Equity Securities (–A) .................................................. Received dividends (equity method). Loss on Equity Investments (Lo, –SE) ........................................................ Investment in Equity Securities (–A) .................................................. Incurred investment loss (equity method).

15,000 15,000

1,500* 1,500

* $1,500 = $6,000 investee net loss  25% b. Cost of investment Plus: Portion of investee's 2011 net income Less: Portion of investee's 2011 declared dividends Book value of investment as of December 31, 2011 Less: Portion of investee's 2012 net loss Less: Portion of investee's 2012 declared dividends

$ 190,000 18,750 (15,000) $ 193,750 (1,500) (15,000)


Book value of investment as of December 31, 2012

$ 177,250

E8–11 a. During 2008 Duke Energy reported equity losses of $102 million on its income statement. Since it owns 40% of its affiliates, the affiliates reported net losses of $102 million ÷ 40% or $255 million. b. While the income (losses) claimed from the affiliates increases (decrease) the value of the long-term equity investment, the dividends received from the affiliates decrease the value of the investment. [Beginning Investment + Equity in Affiliate Income – Dividends Received from Affiliate = Ending Investment] Duke received $121million in dividends in 2008.

E8–12 a. Since Mainmont Industries owns 30% of Tumbleweed Construction, 30% of Tumbleweed Construction's net income "flows through" to Mainmont Industries' income statement. Consequently, Tumbleweed Construction's total net income for 2011 would have been $40,000 ($12,000 ÷ 30%). b. Long-term investment in equity securities: 12/31/10 30% of Tumbleweed Construction's 2011 net income Less: 30% of Tumbleweed Construction's 2011 declared dividends Long-term investment in equity securities: 12/31/11

$25,000 12,000 X $29,000

Mainmont Industries' portion of the dividends declared by Tumbleweed Construction was $8,000. Since this amount represents 30% of the total dividends declared by Tumbleweed, the total dividends declared by Tumbleweed were $26,667 ($8,000 ÷ 30%). c.

Cash (+A) ................................................................................................... Investment in Equity Securities (–A) .................................................. Received dividends (equity method).

8,000

Investment in Equity Securities (+A) ......................................................... Income from Equity Investments (R, +SE) .......................................... Earned investment revenue (equity method).

12,000

8,000

12,000

d. On its statement of cash flows (indirect), Mainmont would deduct from net income the Income from Equity Investments net of the cash received in dividends ($4,000 net, for 2011).

E8–13 a. Cash (+A) ........................................................................................ 90,000 Accounts Receivable (+A) ............................................................... 60,000 Inventory (+A)............................................................................... 160,000 Plant & Equipment (+A)................................................................ 560,000 Goodwill (+A) ................................................................................ 330,000 Payables (+L) ....................................................................................... Cash (-A) ............................................................................................. Purchased Lipley Company.

300,000 900,000


b. The net book values of assets and liabilities represent the amounts at which the assets and liabilities are carried on the balance sheet. Some assets are carried at original cost, others at net realizable value or lower-of-cost-or-market value. If the fair market value exceeds these other valuation bases, then the net fair market value of Lipley's assets and liabilities could exceed their book values. Multiplex is purchasing all of Lipley Company. Consequently, Multiplex is not only purchasing the net assets of Lipley, but it is also purchasing the nonquantifiable items that make Lipley a viable company. These nonquantifiable items include customer loyalty, name recognition, and employee skills and loyalty. These items make Lipley more valuable than the sum of its quantifiable assets and liabilities. Therefore, Multiplex is willing to pay more for Lipley because it is more valuable than the sum of its individual assets and liabilities due to synergy.

E8–14 Purchase Price = Net Book Value + Net Market Value in Excess of Book Value + Goodwill Transactions: (1) Purchase Price = $7,000 + $1,000 + $1,000 Purchase Price = $9,000 (2) $6,000 = $6,000 + Net Market Value in Excess of Book Value + $0 Net Market Value in Excess of Book Value = $0 (3) $12,000 = Net Book Value + $4,000 + $3,000 Net Book Value = $5,000 (4) $15,000 = $10,000 + $3,000 + Goodwill Goodwill = $2,000 (5) Purchase Price = $2,000 + $1,000 + $3,000 Purchase Price = $6,000 (6) $12,000 = $4,000 + $8,000 + Goodwill Goodwill = $0

E8–15 a. Book Value per Share

Book Value ÷ Number of Common Shares Outstanding $36,000 ÷ Number of Common Shares Outstanding = $12 Number of Common Shares Outstanding = 3,000 Shares

b. Market Value/Share

c.

= =

= = =

Market Value ÷ Number of Common Shares Outstanding $51,000 ÷ 3,000 Shares $17

Conglomerate would be willing to pay more than the market value per share due to goodwill. Camden has generated goodwill, such as name recognition and employee and customer loyalty, that makes its net assets more valuable taken as a whole than taken as individual net assets. In other words, due to synergy Camden is worth more than the sum of its individual parts. Additionally, combining Camden's assets with Conglomerate's assets may provide economies of scale, thereby making Camden's assets more valuable to Conglomerate than their individual fair market values.

d. Investment in Subsidiary (+A) ...................................................................

66,000*


Cash (–A)............................................................................................. Acquired subsidiary. *66,000 = $22 per Share Purchase Price  3,000 Common Shares Outstanding

Cash (+A) ................................................................................................... Receivables (+A) ........................................................................................ Inventories (+A) ......................................................................................... Fixed Assets (+A) ....................................................................................... Goodwill (+A) ............................................................................................. Liabilities ............................................................................................. Investment in Subsidiary .................................................................... Consolidated financial statements.

66,000

15,000 24,000 25,000 47,000 15,000 60,000 66,000

E8–16 a. The consolidating entries are shown below: [Note: assets and liabilities purchased are added at Fair Market Value] Investment in Subsidiary (+A) .......................................................................... 144,000* Cash (–A)............................................................................................. Acquired subsidiary. *144,000 = $18 per Share Purchase Price  8,000 Shares (80% of Outstanding) Current Assets (+A).................................................................................... Non-Current Assets (+A)............................................................................ Goodwill (+A) ............................................................................................. Liabilities ............................................................................................. Minority Interest ................................................................................ Investment in Subsidiary ....................................................................

144,000

150,000 80,000 20,000 70,000 36,000 144,000

b. If Maxwell uses IFRS and assumes that minority shareholders have no interest in goodwill, the entry would be: Current Assets (+A).................................................................................... Non-Current Assets (+A)............................................................................ Goodwill (+A) ............................................................................................. Liabilities ............................................................................................. Minority Interest ................................................................................ Investment in Subsidiary ....................................................................

150,000 80,000 16,000 70,000 32,000* 144,000

*minority interest equals 20% of net asset value (.20 x [230,000-70,000]) c. The minority interest would be disclosed in the shareholders’ equity section of the balance sheet (under both U.S. GAAP and IFRS).


E8–17

Accounts

Glover

Adjustments and Eliminations Ward Debit Credit

Cash Accts. Receivable Inventory Investment in Sub. Fixed Assets Goodwill Total Assets

73,000 10,000 110,000 40,000 220,000 60,000 100,000 0 615,000 120,000 __30,000 _____0 1,148,000 230,000

Accounts Payable Long-Term Notes Common Stock Retained Earnings Total Liabilities & Stockholders' Equity

80,000 450,000 500,000 _118,000

70,000 80,000 70,000 _10,000

70,000 10,000 _____0

1,148,000

230,000

80,000

Consolidated Balance Sheet

10,000 100,000 5,000 _5,000 20,000

______ 100,000

83,000 150,000 290,000 0 740,000 __35,000 1,298,000 150,000 530,000 500,000 _118,000

0

1,298,000

The Adjustments and Eliminations columns adjust assets to market value, eliminate the investment account, eliminate the stockholders' equity section of Ward, and recognize goodwill. The goodwill of $5,000 equals the excess of the $100,000 purchase price over the $95,000 fair market value of Ward's individual net assets (i.e., $10,000 in cash + $40,000 in accounts receivable + $70,000 in inventory + $125,000 in fixed assets – $70,000 in accounts payable – $80,000 in notes payable).


PROBLEMS P8–1 a.

The total gains and losses reported on the income statement are as follows: Realized Gains and Losses on Trading as well as Available-for-Sale Securities +

Unrealized Price Changes for Trading Dividend Income Securities +

Security

# of Shares Sold

Realized G/L per share

Houser Co. Miller, Inc. Letter Books Nordic Equip.

60 90 5 145

$25 – $22 = $3 30 – 40 = ($10) 55 – 48 = $7 95 – 70 = $25

Security

# of Shares

Cost per Share

12/31/11 Market per Share

Houser Miller

30 90

$22 $40

$25 $35

Total unrealized price decrease on trading securities

Total Realized Gain/Loss $

Total Cost

Total Market

$

$

660 3,600 $ 4,260 =

180 (900) 35 3,625 $2,940

750 3,150 $ 3,900

$4,260 – $3,900 = $360

Effect on 2011 reported income from all investment transactions and price changes: Net Realized Gain on Sale of Trading and Available-For-Sale Securities During the Year Net Unrealized Price Decrease on Trading Securities Total Dividends Received during 2011 Total Effect on 2011 Income b.

=

$2, 940

= =

(360) 205 $ 2,785

If O’Leary used the fair market value option, then the unrealized gains and losses on the availablefor-securities (Letter and Nordic) would affect net income as well.

# of

Cost

12/31/11 Market

Total

Total


Security

Shares

per Share

per Share

Cost

Market

Letter Nordic

70 25

$48 $70

$46 $90

$ 3,360 1,750 $ 5,110

$ 3,220 2,250 $ 5,470

Total unrealized price increase on available-for-sale securities = $5,470 – $5,110 = Effect on 2011 reported income: $2,785 (from part a) + $360 = $3,145

$360

P8–2 a. (1)

(2)

(3)

(4)

(5)

(6)

(7)

(8)

(9)

3/10

3/31

5/26

7/10

9/11

9/27

10/19

11/6

12/8

Trading Securities (+A) .......................................................... Cash (–A) ........................................................................ Invested in Arctic Oil & Gas.

28,000

Trading Securities (+A) .......................................................... Cash (–A) ........................................................................ Invested in Humphries Manufacturing.

8,000

Cash (+A) ............................................................................... Dividend Revenue (R, +SE) ............................................. Earned and received dividends.

1,250

Trading Securities (+A) .......................................................... Cash (–A) ........................................................................ Invested in Kingsman Game Company.

18,000

Cash (+A) ............................................................................... Trading Securities (–A) ................................................... Realized Gain on Sale of Trading Securities (Ga, +SE)........................................................ Sold Arctic Oil & Gas stock.

28,000

Cash (+A) ............................................................................... Realized Loss on Sale of Trading Securities (Lo, –SE) ... 1,000 Trading Securities (–A) ................................................... Sold Humphries Manufacturing stock.

4,000

Trading Securities (+A) .......................................................... Cash (–A) ........................................................................ Invested in Quimby, Inc.

25,000

Cash (+A) ............................................................................... Dividend Revenue (R, +SE) ............................................. Earned and received dividend.

250

Cash (+A) ............................................................................... Trading Securities (–A) ...................................................

10,500*

28,000

8,000

1,250

18,000

22,400 5,600

5,000

25,000

250

8,600


Realized Gain on Sale of Trading Securities (R, +SE) .......................................................... Sold Arctic Oil & Gas and Humphries Manufacturing stock.

1,900

* $10,500 = (200 shares of Arctic Oil & Gas  $30) + (300 shares of Humphries Manufacturing  $15)

P8–2 Concluded Computation for adjusting journal entry on 12/31:

Security

Number of Shares

Cost/Share

Market Value/Share

0 0 1,000 1,000

$28 10 18 25

$32 14 15 26

Arctic Oil & Gas Humphries Manufacturing Kingsman Game Company Quimby, Inc. Total 12/31

$

$

Total Cost

Total Market Value

0 0 18,000 25,000 43,000

$

Unrealized Price Decrease on Trading Securities (Lo, –SE) ......................................................... Trading Securities (–A)................................................ Adjusted marketable securities to LCM.

b. Marketable securities

$

2,000

$41,000

c. Transaction (1) (2) (3) (4) (5) (6) (7) (8) (9)

3/10 3/31 5/26 7/10 9/11 9/27 10/19 11/6 12/8 12/31 Net increase

Income Statement Effect 0 0 1,250 0 5,600 (1,000) 0 250 1,900 (2,000) $ 6,000

0 0 15,000 26,000 41,000

Explanation

$

Dividend revenue Gain on sale of trading securities Loss on sale of trading securities Dividend revenue Gain on sale of trading securities Holding loss on investment portfolio

2,000


P8–3 a. 10/18/11

12/31/11

12/31/12

b. 10/18/11

12/31/11

12/31/12

c.

Available-For -Sale Securities (+A) ........................................ Cash (–A) ........................................................................

3,200

Available-For -Sale Securities (+A) ........................................ Unrealized Price Increase (+SE)......................................

800

Trading Securities (+A) .......................................................... Unrealized Price Increase (-SE) ............................................. Realized Price Increase from Reclassification (Ga, +SE)...................................................................... Available-For-Sale Securities ..........................................

5,000 800

Trading Securities (+A) .......................................................... Cash ................................................................................

3,200

Trading Securities (+A) .......................................................... Unrealized Gain (Ga, +SE) ..............................................

800

Available-For-Sale Securities (+A) ......................................... Realized Price Increase on Reclassification (Ga, +SE)...................................................................... Trading Securities (–A) ...................................................

5,000

3,200

800

1,800 4,000

3,200

800

1,000 4,000

Income Effect Transactions 10/18/11 12/31/11 12/31/12

Trading Securities

Purchase Mark-to-Market Reclassification Total

$

0 800 1,000 $1,800

Availablefor-Sale Securities $

0 0 1,800 $1,800

P8–4 a.

___Guyer Books___ Balance Sheet Income Value Effect

_____Levy Co._____ Balance Sheet Income Value Effect

Investment Classified as: Trading Securities Available-for-Sale Securities

$9,000 $9,000

$6,600 0

$9,000 $9,000

$6,600 $6,600

b. Guyer and Levy’s income statements would differ only if the security is classified as available-for-sale. The reason: price increases would flow through directly to the stockholders’ equity section, via comprehensive income. In the case of available-for-sale securities, the gain is a realized one, which would flow through the income statement.


P8–5 a. 6/15/11

12/31/11

1/16/12

10/20/12

b. 6/15/11

12/31/11

1/16/12

10/20/12

c.

Trading Securities (+A) .......................................................... Cash (–A) ........................................................................

7,500

Trading Securities (+A) .......................................................... Unrealized Gain (Ga, +RE) ..............................................

1,500

Cash (+A) ............................................................................... Trading Securities (–A) ................................................... Realized Gain (Ga, +RE) ..................................................

5,700

Cash (+A) ............................................................................... Realized Loss (Lo, –RE) .......................................................... Trading Securities (–A) ...................................................

2,600 400

AFS Securities (+A) ................................................................ Cash (–A) ........................................................................

7,500

AFS Securities (+A) ................................................................ Unrealized Price Increase (Ga, +SE) ...............................

1,500

Cash (+A) ............................................................................... Unrealized Price Increase (–Ga, +SE) .................................... AFS Securities (–A) ......................................................... Realized Gain (Ga, +SE) ..................................................

5,700 900

Cash (+A) ............................................................................... Realized Loss (Lo, –SE) .......................................................... Unrealized Price Increase (–Ga, –SE) .................................... AFS Securities (–A) .........................................................

2,600 400 600

7,500

1,500

5,400 300

3,600

7,500

1,500

5,400 1,200

3,600

Cash Effect Transactions

Trading Securities

Available-for-Sale Securities

6/15/11 Purchase 12/31/11 Mark-to-Market 1/16/12 Sale 10/20/12 Sale Net Cash Effect

$ (7,500) 0 5,700 2,600 $ 800

$ (7,500) 0 5,700 2,600 $ 800


P8–5

Concluded

d. & e. Income Effect Transactions 6/15/11 12/31/11 1/16/12

Purchase Mark-to-Market Sale—Realized Price Increase —Unrealized Price Increase reversal 10/20/12 Sale—Realized Price Decrease —Unrealized Price Increase Reversal Net Income Statement Effect f.

Trading Securities $

$

0 1,500

Available-for-Sale Securities $

0 0

1,200

1,200

(900)

0

(400)

(400)

(600) 800

$

0 800

Overall income statement effect for both the years (i.e., 2011 and 2012) would be the same whether the securities are classified as trading securities or as available-for-sale securities. However, the individual years’ income would differ due to the chosen classification. For trading securities all the unrealized price changes would flow through the income statement, while those for the available-for-sale securities would flow directly into the stockholders’ equity account.


P8–6 a. and b.

We assume that Orlean’s first investment in these securities occurred on 12/31/10. Cost of the Atwater Investment on a Per-Share Basis

Realized Price Increase During 2011

Per Share Market Value as of 12/31/11

Total Unrealized Price Increase on 1,600 Shares

=

$27,000 ÷ 1,800 Shares

=

$15/Share

=

200 Shares  ($15.50 – $15)

=

$100

=

$25,440 ÷ 1,600 Shares

=

$15.90

=

1,600 Shares  .90 =

$1,440

If the securities were classified as trading securities, Orleans would report a total gain of $1,540, of which $100 would be realized and $1,440 would be an unrealized price increase. However, if the securities were classified as available-for-sale, only $100 would be reported in the income statement. The unrealized price increase of $1,440 would go directly to the stockholders’ equity section of the balance sheet.

P8–7 a. The net change in the investment in trading securities account from 2007 to 2008 was $347.4 billion minus $414.3 billion, or a decrease of $66.9 billion. Taking into account the unrealized gain of $9.9 billion, the total decrease is 76.8 billion. b. The net change in the available-for-sale securities from 2007 to 2008 was $205.9 billion minus $85.4 billion, or an increase of $120.5 billion. The net increase per the statement of cash flows was $107.4 billion, which means that the unrealized gains on the available-for-sale securities were $13.1 billion. This amount would be shown under comprehensive income, which would affect stockholders’ equity on the balance sheet.

P8–8 a. Under the mark-to-market method, the investment is carried on the books at the market value as of the balance sheet date. Adjustments of the investment portfolio to market value "flow through" to the income statement. Dividends declared by the investee are recognized as income to the investor in the period the dividends are declared.


(1) Masonite Tires Balance Sheet January 1, 2011 Noninvestment assets ..................... Investment in .................................. Securities ....................................... Total ................................................. (2)

$ 120,000 Liabilities ...................... Stockholders’ equity ............. 40,000 $ 160,000 Total.............................

$ 70,000 90,000 $ 160,000

During 2011, Masonite Tires would make the following original and adjusting journal entries under the mark-to-market method: Cash (+A) ......................................................................................... Revenues (R, +SE) ..................................................................... Earned noninvestment revenues.

85,000

Expenses (E, –SE)............................................................................. Cash (–A) .................................................................................. Incurred expenses.

50,000

Cash (20%  $10,000) (+A) .............................................................. Dividend Revenue (R, +SE) ....................................................... Earned and received dividends.

2,000

Unrealized Loss on Securities (–SE) .............................................................................. Investment in Securities (–A) ................................................... Adjusted securities to market.

85,000

50,000

2,000

4,000 4,000

Masonite Tires Income Statement For the Year Ended December 31, 2011 Revenues .............................................................................................................. Dividend revenue ................................................................................................. Unrealized Loss………………………………………………………………. 4,000 Expenses............................................................................................................... Net income ...........................................................................................................

P8–8

Continued Masonite Tires Balance Sheet

$ 85,000 2,000 50,000 $ 33,000


December 31, 2011 Noninvestment assets ...................... $157,000 Liabilities………………. $ 70,000 Investment in Securities ................. 36,000 Stockholders' equity .......... 123,000* Total……………………………… $ 193,000 Total …………………… $193,000 * $123,000 = $90,000 Beginning balance + Net Income of $33,000 (3)

During 2012, Masonite Tires would make the following original and adjusting journal entries: Cash (+A) ......................................................................................... Revenues (R, +SE) ..................................................................... Earned noninvestment revenues.

75,000

Expenses (E, –SE)............................................................................. Cash (–A) .................................................................................. Incurred expenses.

70,000

75,000

70,000

Cash (20%  $15,000) (+A) .............................................................. Dividend Revenue (R, +SE) ....................................................... Earned and received dividends.

3,000

Investment in Securities (–A) .......................................................... Unrealized Gain on Securities (+SE) .................................................................. Adjusted securities to market.

6,000

3,000

6,000

Masonite Tires Income Statement For the Year Ended December 31, 2012 Revenues .............................................................................................................. Dividend revenue ................................................................................................. Unrealized Gain……………………………………………………………….. 6,000 Expenses............................................................................................................... Net income ...........................................................................................................

P8–8

Continued

$75,000 3,000 70,00 0 $14,000


Masonite Tires Balance Sheet December 31, 2012 Noninvestment assets ...................... $ 165,000 Investment in ................................... Securities ....................................... 42,000 Total ................................................. $ 207,000

Liabilities ........... …………… $ 70,000 Stockholders’ equity. ...... 137,000* Total ................................ $207,000

* $137,000 = $123,000 Beginning balance + Net Income of $14,000 b. Under the equity method, an investment is initially recorded at its cost and subsequently adjusted for changes in the investee's net assets. Therefore, any net income or loss generated by the investee and any dividends declared by the investee cause the investment account to be adjusted. (1)

Masonite Tires Balance Sheet January 1, 2011 Noninvestment assets ...................... $ 120,000 Liabilities.......... ………………. $ 70,000 Investment in equity securities ...... 40,000 Stockholders' equity .......... 90,000 Total ................................................. $ 160,000 Total .................................. $160,000

(2)

During 2011, Masonite Tires would make the following original and adjusting entries: Cash (+A) ......................................................................................... Revenues (R, +SE) ..................................................................... Earned noninvestment revenues.

85,000

Expenses (E, –SE)............................................................................. Cash (–A) .................................................................................. Incurred expenses.

50,000

Cash (+A) ......................................................................................... Investment in Equity Securities (–A) ........................................ Received dividends (equity method).

2,000

Investment in Equity Securities (+A) ............................................... Income from Equity Investments (R, +SE) ................................ Earned investment revenue (equity method).

3,000*

* $3,000 = Investee's $15,000 income  20%

85,000

50,000

2,000

3,000


P8–8

Continued Masonite Tires Income Statement For the Year Ended December 31, 2011 Revenues: Revenue ........................................................................................................ Income from equity investments .................................................................. Expenses............................................................................................................. Net income .........................................................................................................

$ 85,000 3,000 50,000 $ 38,000

Masonite Tires Balance Sheet December 31, 2011 Noninvestment assets ...................... $ 157,000 Investment in equity securities ...... 41,000 Total ................................................. $ 198,000

Liabilities…………… $ 70,000 Stockholders’ equity. 128,000* Total………………… $198,000

* 1/1/2011 Retained Earnings of $90,000 (see part A) plus Net Income ($38,000). (3)

During 2012, Masonite Tires would make the following original and adjusting entries: Cash (+A) ......................................................................................... Revenues (R, +SE) ..................................................................... Earned noninvestment revenues.

75,000

Expenses (E, –SE)............................................................................. Cash (–A) .................................................................................. Incurred expenses.

70,000

Cash (+A) ......................................................................................... Investment in Equity Securities (–A) ........................................ Received dividends (equity method).

3,000

Investment in Equity Securities (+A) ............................................... Income from Equity Investments (R, +SE) ................................ Earned investment revenue (equity method).

4,000*

75,000

70,000

3,000

4,000

* $4,000 = Investee's $20,000 net income  20% Masonite Tires Income Statement For the Year Ended December 31, 2012 Revenues: Revenue ........................................................................................................ Income from equity investments .................................................................. Expenses............................................................................................................. Net income .........................................................................................................

$75,000 4,000 70,000 $ 9,000


P8–8

Concluded Masonite Tires Balance Sheet December 31, 2012 Noninvestment assets ...................... Investment in equity securities ...... Total .................................................

$ 165,000 Liabilities…………….. $ 70,000 42,000 Stockholders’ equity. ....... 137,000* $ 207,000 Total………………… .......... $207,000

* 12/31/11 Retained Earnings ($128,000) plus Net Income ($9,000). c.

Using the equity method versus the mark-to-market method may be desirable due to debt covenants and incentive compensation, as long as the investee company is earning Net Income. Under the equity method, a portion of the investee's net income flows through to the investor's income statement and through the closing process into stockholders' equity. This increase in stockholders' equity will, holding everything else constant, decrease the investor's debt/equity ratio. In this problem, Masonite's debt/equity ratio as of December 31, 2012 is the same under the equity method and the mark-tomarket method. Further, increased net income would be desirable to management if they receive incentive compensation based on net income. In some cases owning 20% of a company does not constitute substantial influence over the investee. To be able to substantially influence a company means that Masonite must be able to influence general corporate policy, dividend policy, and so forth. If another investor owns a larger percentage of the company than Masonite, that investor could exert more influence than Masonite. In the extreme, some other investor may own 80% of the investee while Masonite owns the remaining 20%. In such a case, Masonite could not exert a substantial influence on the investee and should not account for the investment using the equity method.

P8–9 a. Investment in Subsidiary (+A) ................................................................... Cash (–A)............................................................................................. Purchased Martin Monthly.

62,000 62,000

Alsop Ltd. Consolidated Balance Sheet December 31, 2011 Tangible assets ............................... $ 204,000a Goodwill ........................................ 40,000b Total................................................ $ 244,000 a $204,000 b $40,000

Liabilities............................. Stockholders’ equity ........... Total....................................

$154,000 90,000 $244,000

= $180,000 of Alsop's assets – $62,000 of cash disbursed for investment + $86,000 fair market value of assets acquired. = $62,000 purchase price – $22,000 fair market value of individual net assets acquired (i.e., $86,000 – $64,000).

b. Investment in Equity Securities (+A) ......................................................... Cash (–A)............................................................................................. Invested in Martin Monthly.

62,000 62,000


P8–9

Concluded

Alsop Ltd. Consolidated Balance Sheet December 31, 2011 Noninvestment assets ....................... $ 118,000 Liabilities..................................... Investment in equity securities ....... 62,000 Stockholders’ equity ................... Total................................................... $ 180,000 Total............................................ c.

$ 90,000 90,000 $180,000

Debt/equity ratio = Liabilities ÷ Stockholders' equity Purchase method: Equity method:

$154,000 ÷ $90,000 ÷

$90,000 = $90,000 =

1.71 1.00

Alsop's debt/equity ratio is considerably lower if the investment is accounted for using the equity method than if it is accounted for using the purchase method. If Alsop has any existing debt covenants that specify a maximum debt/equity ratio, the method used to account for this investment might affect whether Alsop violates its debt covenant. The probability that Alsop will violate any existing debt covenant is greater if the investment is accounted for using the purchase method than if it is accounted for using the equity method.

P8–10 a.

Short-Term Investments at 12/31/11

Trading Securities at 12/31/11

=

Trading Securities at 12/31/11

=

Trading Securities at 1/1/11

= =

$130 $280

+

Availablefor-Sale Securities at 12/31/11

+

Purchases During the Year

Sales During the Year

+

$280

[$240 – $80]

Total short-term investment as of 12/31/11 is $290, of which $280 is in trading securities. Therefore, the remaining $10 is in Available-for-sale Securities. b. Balance Sheet Carrying Value of Trading Securities Sold During 2011

c.

=

Cash Received from the Sale of Trading Securities During 2011

= =

$240 $130

Earnings per share dollar amount reported by the affiliate:

Realized and Unrealized Gain on Short-Term Investments During 2011

[80 + 30]


Total Income of the Affiliate

÷

Total # of Shares Outstanding

Total Income of the Affiliate

= =

$40,000 ÷ .40 $100

P8–10

Concluded

Total Number of Shares Outstanding

Earnings per Share of the Affiliate =

= =

50,000 ÷ .40 125,000 shares

$100,000 ÷ 125,000 = $.80/share

d. Per share dividend declared by the affiliate: Income Equity Investment – Equity Income in Excess of Cash Received Number of Shares Purchased from Affiliate =

$40,000 − $30,000 = $.20 per share 50,000

P8–11 a. Investment in Subsidiary (+A) ................................................................... Cash (–A)............................................................................................. Invested in subsidiary.

180,000 180,000

After posting this entry, Rice's Cash account would decrease from $196,000 to $16,000, and the Investment in Subsidiary account would increase from $0 to $180,000. b. Adjustments and Eliminations Rachel Debit Credit

Accounts

Rice

Cash Accounts Receivable Inventory Investment in Sub. Fixed Assets Goodwill Total Assets

16,000 150,000 300,000 180,000 400,000 0 1,046,000

10,000 40,000 40,000 0 130,000 0 220,000

Accounts Payable L-T Liabilities Common Stock APIC, Common Stock Retained Earnings Total Liabilities & Stkhlders' Equity

80,000 300,000 400,000 140,000 126,000

20,000 50,000 90,000 10,000 50,000

90,000 10,000 50,000

1,046,000

220,000

150,000

30,000 180,000 10,000 10,000 40,000

190,000

Consolidated Balance Sheet 26,000 190,000 370,000 0 520,000 10,000 1,116,000 100,000 350,000 400,000 140,000 126,000

0

1,116,000


The adjustments and eliminations columns (1) adjust assets to market value, (2) eliminate the Investment account, (3) eliminate the stockholders' equity section of Rachel, and (4) recognize goodwill. The goodwill of $10,000 equals the excess of the $180,000 purchase price over the $170,000 fair market value of Rachel's individual net assets (i.e., $10,000 in cash + $40,000 in accounts receivable + $70,000 in inventory + $120,000 in fixed assets – $20,000 in accounts payable – $50,000 in long-term liabilities).

P8–12 a. Investment in Subsidiary (+A) ................................................................... Cash (–A)............................................................................................. Invested in subsidiary.

136,000 136,000

After posting this entry, Rice's Cash account would decrease from $196,000 to $60,000, and the Investment in Subsidiary account would increase from $0 to $136,000. b. Adjustments and __Eliminations___ Rachel Debit Credit

Accounts

Rice

Cash Accounts Receivable Inventory Investment in Sub. Fixed Assets Total Assets

60,000 150,000 300,000 136,000 400,000 1,046,000

10,000 40,000 40,000 0 130,000 220,000

Accounts Payable L-T Liabilities Minority Interest Common Stock APIC, Common Stock Retained Earnings Total Liabilities & Stkhlders' Equity

80,000 300,000 0 400,000 140,000 126,000

20,000 50,000 0 90,000 10,000 50,000

90,000 10,000 50,000

1,046,000

220,000

150,000

30,000

30,000

136,000 10,000 146,000

34,000

34,000

Consolidated Balance Sheet 70,000 190,000 370,000 0 520,000 1,150,000 100,000 350,000 34,000 400,000 140,000 126,000 1,150,000

The adjustments and eliminations columns (1) adjust assets to market value, (2) eliminate the Investment account, (3) eliminate the stockholders' equity section of Rachel, and (4) recognize minority interest. The minority interest of $34,000 equals 20% (the amount not purchased by Rice) of the 100% value of Rachel as determined by the price paid for 80% [($136,000/.80 x .20]


P8–13 a. Investment in Subsidiary (+A) ................................................................... Cash (–A)............................................................................................. Invested in subsidiary.

140,000 140,000

After posting this entry, Rice's Cash account would decrease from $196,000 to $56,000, and the Investment in Subsidiary account would increase from $0 to $140,000.

b. Adjustments and Eliminations Rachel Debit Credit

Accounts

Rice

Cash Accounts Receivable Inventory Investment in Sub. Fixed Assets Goodwill Total Assets

56,000 150,000 300,000 140,000 400,000 0 1,046,000

10,000 40,000 40,000 0 130,000 0 220,000

Accounts Payable L-T Liabilities Minority Interest Common Stock APIC, Common Stock Retained Earnings Total Liabilities & Stkhlders' Equity

80,000 300,000 0 400,000 140,000 126,000

20,000 50,000 0 90,000 10,000 50,000

90,000 10,000 50,000

1,046,000

220,000

150,000

30,000 140,000 10,000 5,000 35,000

150,000

35,000

35,000

Consolidated Balance Sheet 66,000 190,000 370,000 0 520,000 5,000 1,151,000 100,000 350,000 35,000 400,000 140,000 126,000 1,151,000

The adjustments and eliminations columns (1) adjust assets to market value, (2) eliminate the Investment account, (3) eliminate the stockholders' equity section of Rachel, and (4) recognize both goodwill and minority interest. The goodwill of $5,000 equals the excess of the valuation of 100% of Rachel based on the price paid for 80% ($140,000/.80 = $175,000) over the fair market value of the individual net assets controlled by Rice, ($10,000 + $40,000 + $70,000 + $120,000 – $20,000 – $50,000). The minority interest of $35,000 equals 20% (the amount not purchased by Rice) of the 100% value of Rachel as determined by the price paid for 80% [($140,000/.80 x .20] b. If Rice uses IFRS and assumes the minority shareholders have no equity in goodwill, then the noncontrolling interest would be $34,000 (20% of the FMV of the net assets [$240,000 - $70,000]) and goodwill would be $4,000.

P8–14


a. Net income would be affected by the values reported for inventory (through cost of goods sold), and fixed assets (through depreciation expense). Thus, any attempt to maximize net income in the following year must consider the effect of inventory, fixed assets, and goodwill. Since inventory is a current asset, the assumption is that the costs in inventory will flow through to the income statement in the following year. Thus, Rice would want to value the inventory at $65,000. Fixed assets are long-term assets and are thus depreciated over a period of time (most likely over a period of time not exceeding 10 or 12 years). Rice would be able to report the lowest amount possible for depreciation expense the following year if it capitalizes the fixed assets at $115,000 instead of at $125,000. The tradeoff of allocating $65,000 to inventory and $115,000 to fixed assets is that Rice will have to report a higher amount for goodwill. That is, if Rice makes the allocations given above, Rice will

P8–14 Concluded have to allocate $20,000 to goodwill (i.e., purchase price of $180,000 less sum of the fair market values of Rachel's individual net assets as follows: $10,000 in cash + $40,000 in accounts receivable + $65,000 in inventory + $115,000 in fixed assets – $20,000 in accounts payable – $50,000 in longterm liabilities). b. To minimize next year's reported income for tax purposes, Rice would like to maximize the value of assets that it will consume next year. Thus, Rice would like to maximize the value allocated to inventory because the inventory would probably be sold next year, and thus be allocated to cost of goods sold next year. Similarly, Rice would like to maximize the amount allocated to fixed assets relative to the amount allocated to goodwill. Thus, Rice would like to allocate $125,000 to fixed assets. These two allocations would result in the sum of the fair market values of Rachel's individual net assets being $180,000 ($10,000 in cash + $40,000 in accounts receivable + $75,000 in inventory + $125,000 in fixed assets – $20,000 in accounts payable – $50,000 in long-term liabilities). Since the purchase price was $180,000, Rice would not have to allocate anything to goodwill.

P8–15 Company X Investment in Subsidiary (+A) .......................................................................... Cash (–A) ................................................................................................... Cash (+A) .......................................................................................................... Accounts Receivable (+A) ................................................................................. Inventory (+A) .................................................................................................. Fixed Assets (+A) .............................................................................................. Goodwill ........................................................................................................... Current Liabilities (+L) ............................................................................... Long-Term Liabilities (+L) .......................................................................... Minority Interest (+L) ................................................................................ Investment in Subsidiary (–A) ................................................................... * $20,000

** $21,200

84,800 84,800 6,000 12,000 30,000 70,000 20,000* 7,000 25,000 21,200** 84,800

= Purchase Price of 100% – FMV of Net Assets Purchased = [$84,800/.80] – (FMV of Total Assets – FMV of Total Liabilities) = $106,000 – ($6,000 + $12,000 + $30,000 + $70,000 – $32,000) = Purchase Price of 100%  Percentage Not Owned by Parent = [($84,800)/.80] x 20%

Company Y Investment in Subsidiary (+A) .......................................................................... Cash (–A) ...................................................................................................

24,000 24,000


Cash (+A) .......................................................................................................... Accounts Receivable (+A) ................................................................................. Inventory (+A) .................................................................................................. Fixed Assets (+A) .............................................................................................. Goodwill (+A).................................................................................................... Accounts Payable (+L) ............................................................................... Long-Term Liabilities (+L) .......................................................................... Minority Interest (+L) ................................................................................ Investment in Subsidiary (-A) .................................................................... Potential consolidating entry. __________________ * $17,000 = Purchase Price of 100% – FMV of Net Assets Purchased

P8–15

Concluded

** $16,000

= [$24,000/.60] – (FMV of Total Assets – FMV of Total Liabilities) = $40,000 – ($4,000 + $9,000 + $12,000 + $30,000 – $32,000) = Purchase Price of 100%  Percentage Not Owned by Parent =

Cash (+A) .......................................................................................................... Accounts Receivable (+A) ................................................................................. Inventory (+A) .................................................................................................. Fixed Assets (+A) .............................................................................................. Goodwill (+A).................................................................................................... Accounts Payable (+L) ............................................................................... Long-Term Liabilities (+L) .......................................................................... Minority Interest (+L) ................................................................................ Investment in Subsidiary (–A) ...................................................................

** $5,500

12,000 20,000 16,000** 24,000

[($24,000)/.60] x 40%

Company Z Investment in Subsidiary (+A) .......................................................................... Cash (–A) ...................................................................................................

* $3,000

4,000 9,000 12,000 30,000 17,000*

16,500 16,500 2,000 7,000 18,000 15,000 3,000* 5,000 18,000 5,500** 16,500

= Purchase Price of 100% – FMV of Net Assets Purchased = [$16,500/.75] – (FMV of Total Assets – FMV of Total Liabilities) = $22,000 – ($2,000 + $7,000 + $18,000 + $15,000 – $23,000) = Purchase Price of 100%  Percentage Not Owned by Parent =

[($16,500)/.75] x 25%

P8–16 a. Debt/Equity Ratio = Total Liabilities ÷ Total Stockholders' Equity Prior to the acquisition of Atom, Inc., Mammoth's total liabilities were $230,000 and its total stockholders' equity was $270,000. Thus, Mammoth's debt/equity ratio was .85. By acquiring Atom, Inc., Mammoth would need to add Atom's liabilities to its own. If you assume that the book value of Atom's liabilities equals their fair market value, you can infer that the value of Atom's liabilities is $90,000. That is, the $180,000 fair market value of Atom's assets less $20,000 excess of market value over book value of assets less the $70,000 book value of Atom's net assets (i.e., total assets less total liabilities) equals $90,000. Thus, Mammoth's post-acquisition liabilities would be $320,000. Minority interest would equal $60,000 (representing the 50% not owned by Mammoth and valuing the entire company at Mammoth’s purchase price of half the company, $60,000/.5 x 50%). Total liabilities including minority interest would therefore equal $380,000. The acquisition of Atom would not affect Mammoth's stockholders' equity. Mammoth's post-acquisition debt/equity ratio would therefore be $380,000/$270,000 = 1.41.


b. If Mammoth accounted for this investment using the equity method, it would not have to combine Atom, Inc.'s assets and liabilities with its own. Instead, Mammoth would simply report an asset (i.e., Long-Term Investment in Equity Securities) on its balance sheet valued at $60,000. Thus, Mammoth's liabilities and stockholders' equity would not be affected, which means that its debt/equity ratio would remain at .85. As shown in part (a), however, Mammoth's debt/equity ratio would increase if it accounted for the investment using the purchase method. Therefore, by using the equity method, Mammoth presents a more favorable debt/equity ratio, which allows Mammoth to avoid moving closer to violating its debt covenant. Such a move gives the company more accounting flexibility and decreases the probability that it will eventually violate its debt covenant. For these reasons, Mammoth would probably prefer accounting for the investment in Atom, Inc., using the equity method rather than the purchase method.

ISSUES FOR DISCUSSION ID8–1 a. The market value of H&R Block’s marketable securities decreased in 2007 and 2009 but increased in 2008. b. H&R Block could manage its earnings by selling marketable securities that have either a gain or a loss in order to move its net income up or down. The gains or losses on securities that are not sold are recorded in the stockholders’ equity section of the balance sheet but are not recorded on the income statement until an actual sale has taken place. c.

These gains or losses would be reflected in the statement of comprehensive income. Since1998 companies have been required to prepare a statement of comprehensive income.

ID8–2 a. Trading securities are bought and held principally for the purpose of selling them in the near future with the objective of generating profit on short-term price changes. Investments not classified as trading securities are considered available-for-sale securities. Price increases, or decreases, would flow through directly to the stockholders’ equity section, via comprehensive income, in the case of availablefor-sale securities; a gain or loss for a trading security would flow through the income statement. b. Total comprehensive income includes all realized and unrealized gains and losses. The footnote describes the realized gains/losses; the unrealized gains/losses would also factor into the calculation of comprehensive income. c.

2009 Proceeds from sale of securities Minus: Gains on securities sold Plus: Losses on securities sold Cost of securities sold

$8.3 0.7 1.3 $ 8.9

2008 $ 13.9 0.4

2007 . $3.5 0.3

0.1 $ 13.6

0.1 $3.3

ID8–3 a. Beginning LT Equity Investment + New Investments + Income from Affiliates – Dividends Received = Ending LT Equity Investment; $2,270 + X + 819 – 165 = $2,332; x = ($592)


AT & T must have sold $592 million in long-term equity investments during 2008. b. The statement of cash flow will deduct the $819 million in “equity in net income of affiliates” (because this amount was not a cash inflow to A T & T), net of the $165 million in cash dividends received from affiliates. c.

Equity income is not a good measure of the cash AT & T received from its affiliates because there is no relationship between equity income and cash received from an affiliate. The equity income reported by AT & T represents its percentage of the income of the affiliate but does not represent the amount of cash that may have been paid in dividends to AT & T.

ID8–4 a. “Share of profits from associates” represents EADS’ share of the earnings of the companies in which EADS can assert significant influence. “Investment in associates” is the value that EADS assigns to companies in which it has significant influence; under the equity method approach, companies such as EADS record the initial investment at cost and adjust the investment carrying value with increases from associate income and decreases from associate dividend payments. “Results of companies accounted for by equity method” is a deduction on the statement of cash flows because those associate earnings are recorded on the income statement (see above) even though no cash proceeds are received. “Payments for investments in associates” represents cash outflows to purchase new equity positions in companies over which EADS can exert significant influence. “Proceeds from disposals of associates” represents cash inflows when equity stakes have been liquidated. b. Ending Balance Investments in Associates = Beginning Balance Investments in Associates plus Share of Profits from Associates + Payments for Investments in Associates – Dividends Received from Associates – Balance Sheet Value of Associates Disposed; 2,356 = 2,238 + 188 + 122 - 0 - X; X = 192 million euros c. The cash proceeds from disposal ($180) was less than the carrying value of the investments that were sold ($192), so the company recorded a 12 million euro loss.

ID8–5 a. The investment in associated companies on the balance sheet increased in value because Sony claimed a share of the associates’ earnings as an increase to the value of the investments and because Sony purchased new investments over the course of 2008. b. The 31,509 figure is a deduction on the statement of cash flows because those associate earnings were recorded on the income statement even though no cash proceeds were received. c. If Sony were to account for these investments under the fair market value option, any increases or decreases in value would flow through earnings on the income statement. For Sony to claim a change in value, the company would be required to disclose its methodology of calculating its fair value estimate. Market values based on quoted prices in active markets for identical securities are called Level 1 measurements; market values based on less reliable, observable, indirect inputs are called Level 2 measurements; and market values based on much less reliable, unobservable inputs are called Level 3 measurements. The level disclosed can aid the reader of the financial statements in determining the risks associated with changes in balance sheet value.


ID8–6 a. Under the equity method a substantive economic relationship exists between an investor and an investee. The investor records the original investment at cost. For each subsequent period the investor adjusts the carrying value of the investment by (1) the investor’s proportionate share of the income or loss of the investee, and (2) by the amount of the dividend received. In other words, the investor’s net assets increase due to the investee’s earnings and decrease due to the receipt of dividends. Generally, the amount of the dividends received is much less than the amount of earnings recognized by the investor. The investee company, just like any other business entity, pays only a small fraction of earnings in the form of dividends. Therefore, in the books of the investor there is a wide disparity in the amount of earnings recognized and the cash received due to dividends. Thus, investors must be careful in analyzing financial statements that use the equity method of accounting. b. Under the operating section on Chevron’s statement of cash flows, one will subtract “Equity income in excess of Cash Received” from the total net income.

ID8–7 a. With consolidated financial statements, the assets and liabilities of the investee company (subsidiary) are added to the assets and liabilities of the investing company (parent). Thus, the amount of assets and liabilities reported by the consolidated entity will be greater than the assets and liabilities reported by either the parent or subsidiary. Note, however, that the total amount of stockholders' equity reported by the consolidated entity will be the same as the parent's stockholders' equity. If the parent has any debt covenants, consolidated financial statements could cause the company to violate the covenant. For example, assume that the debt covenant specifies a maximum allowable debt/equity ratio. With consolidated financial statements, as mentioned above, the company's liabilities would increase due to the consolidation while its stockholders' equity would remain the same. Thus, the company's debt/equity ratio would increase, thereby pushing it closer to or over the maximum allowable debt/equity ratio. Debt covenants are not as likely to be a problem for investments accounted for using the equity method. With the equity method, the parent reports its initial investment in the subsidiary in the account Investment in Equity Securities, and this account is reported on one line on the balance sheet. Over time, the parent increases the investment account when the subsidiary generates net income and reduces this investment account when the subsidiary generates net losses and when the subsidiary declares dividends. The balance in the investment account represents the parent's portion of both the subsidiary's assets and liabilities. This means that the parent's portion of the subsidiary's liabilities are


not added to the parent's liabilities, thereby allowing the parent to engage in off-balance-sheet financing. Thus, the parent's portion of the subsidiary's liabilities would not affect the company's "official" debt/equity ratio, which, in turn, implies that the equity method should not cause a company to have "difficulties with bond indenture agreements." b. If users of financial statements are reasonably sophisticated, they should be able to understand more complex business and reporting situations. This implies that as the users' sophistication increases, the FASB has a decreased responsibility for developing standards that encompass and account for every possible business situation that arises. Thus, the FASB would be able to promote broader standards as user sophistication increases, without a lessening of its responsibilities to users of financial statements.

ID8–8 a. Unrealized losses for trading securities are reported on the income statement. If Starbucks’ $44.8 million portfolio of trading securities dropped in value by 50%, the income statement for the month of October, 2009 would include an unrealized loss of $22.4 million, significantly reducing the company’s profitability. The unrealized loss on the available-for-sale securities would not affect the company’s income statement. b. Designating all marketable securities as available-for-sale would have eliminated the effect on the income statement from a stock market crash. On the other hand, designating all securities to be trading would have increased the negative impact on the company’s profits. c.

Protecting earnings from daily fluctuations in the stock market is a sound reason that management might choose to designate securities to be available-for-sale.

ID8–9 a. Realized gains (when shares are sold) are always reported on the income statement, so the two articles do not tell us whether the investments were designated as trading or available-for-sale securities. b. Analysts are interested in earnings that are sustainable and can be repeated in future periods. Profits from operations are repeatable; one-time gains from the sale of investments are not necessarily something a reader of financial statements can count on in future reporting periods. c.

Quality of earnings and sustainability of earnings would be measured by separating operating from other profits.

ID8–10 Unrealized gains/losses are not reported on the income statement for available-for-sale securities. Firms with large investment portfolios, such as banks, can therefore keep the fluctuations in the securities values from affecting the income statement. Only realized gains/losses will hit earnings, allowing banks to time the sale of the securities to coincide with either positive or negative earnings. For example, if a bank is having an unusually strong year, surpassing analyst expectations, the bank could sell securities that have dropped in value from the purchase date; when the realized losses hit the income statement, they are offset by the higher operating earnings. Had those securities been designated trading securities, the unrealized losses would have been booked on the income statement at the time of the market price decline (which might have happened in a period where earnings were not above expectations). Designating the


securities as available-for-sale allows management to time when they want to take the realized gain/loss on the income statement.

ID8–11 Comprehensive income includes all non-owner related changes in stockholders’ equity that do not appear on the income statement and are not reflected in the retained earnings balance. As a global company, Eli Lilly sells its pharmaceutical products in foreign countires for foreign currencies. In 2008, the value of those foregin currencies relative to the U.S. dollar cost Lilly equity, but the exchange rates moved in the opposite direction in 2007 and 2006, creating equity for the company. Those changes are reflected in the statement of comprehensive income (and are not posted to retained earnings through the income statement). Similarly, the unrealized losses of the company’s portfolio of available-for-sale securities in 2006, 2007 and 2008 were posted directly to the equity section of the balance sheet (bypassing the income statement) and were reflected as deductions to comprehensive income. Finally, SFAS 158 requires companies to reflect the funding status of pension and other plans through comprehensive income. The large negative adjustment Lilly recorded in 2008 reflects a loss of equity due to the funding of its plans, but this adjustment is posted directly to stockholders’ equity and does not affect the income statement; similarly, the positive adjustments in 2006 and 2007 affect equity through comprehensive income but do not touch annual earnings on the income statement.

ID8–12 a. Minority or noncontrolling interest is recognized on the balance sheet because the acquiring company does not purchase 100% of the target company but does purchasing a controlling stake (greater than 50%) and therefore consolidates financial statements with the target company. The noncontrolling portion must be recognized because that interest in the net assets acquired is held by outsiders (not shareholders of the acquiring company). The dollar amount of the noncontrolling interest is calculated by either multiplying the number of shares owned by the minority shareholder(s) times each share’s value or, in the case of IFRS, multiplying the noncontrolling interest by the net assets acquired (following the assumption that minority shareholders do not have an ownership interest in the goodwill associated with the acquisition). b. The argument concerning the placement of minority interest is between the liability section and the shareholders’ equity section on the balance sheet. Currently, under both U.S. GAAP and IFRS, the noncontrollng interest is listed under shareholders’ equity.

ID8–13 a. Nike carries $1.2 billion of short-term investments as of the end of fiscal 2009 (up from $642 million in 2008). The first footnote indicates that the investments are designated available-for-sale securities. b. The available-for-sale securities are carried at fair values, which were determined using Level 1 (active markets) and Level 2 (less actively traded) measurements (Note 6).


c.

In March, 2008 Nike acquired 100% of Umbro, a leading European soccer equipment and apparel company (Note 4). The journal entry for the acquisition is summarized: Current Assets Non-Current Assets 90.2 Intangible Assets Goodwill Current Liabilities Non-Current Liabilities Cash

87.2 407.1 342.8 60.3 290.6 576.4

d. Goodwill was $193.5 million and $448.8 million in 2009 and 2008, respectively. The decline in goodwill indicates that Nike management has determined that future cash flows from previously acquired companies will be less robust than initially thought. The income statement shows an expense for goodwill impairment of $199.3 million to reflect this management opinion. (Note 4 reconciles the changes to the goodwill account due to the Umbro acquisition and subsequent Umbro impairment charge.) e. The impairment charge reduced net income but was a non-cash expense and therefore did not affect company cash balances. The statement of cash flows (indirect format) reconciles net income (an accrual accounting number) with cash from operations (a pure cash number). Therefore, all expenses that are noncash are added back to net income in the statement of cash flows. f.

Nike does not have investments in companies in which it does not have a controlling interest.


CHAPTER 9 LONG-LIVED ASSETS BRIEF EXERCISES BE9–1 a. The new method, straight-line depreciation, will increase net income in the early years and reduce income in the later years versus using an accelerated method. An accelerated method of depreciation increases the depreciation charges in the early years of the life of an asset and reduces the depreciation charges in the later years. b. Allegheny may have decided that it wanted depreciation charges to be spread evenly over the life of an asset so that the impact on net income in any one reporting period was less. It may also feel that it will make its financial statements easier to compare with its competitors. During periods of high fixed asset investment Allegheny’s results may look unfavorable versus other companies that use a straightline method instead of an accelerated method. c.

In the annual report one could look through the first footnote. This footnote typically highlights all of the significant accounting policies and methods used by the company to prepare the financial statements.

BE9–2 a. The recognition of depreciation and amortization affects the basic accounting equation by reducing assets and reducing retained earnings in the stockholders’ equity section. Fixed assets such as property, plant and equipment are reduced through depreciation charges (which are collected in the contra asset account Accumulated Depreciation) which lower net income. Intangible assets are reduced by amortization charges which reduce the net income of the company. This reduction in net income reduces the retained earnings of the company. b. Boeing recognized a loss of $293 million, computed as follows: Accumulated depreciation 2007 + Depreciation charges for 2008 – Accumulated depreciation 2008 Accumulated depreciation on assets sold

$12,280 million 1,325 million 12,795 million $ 810 million

PP&E 2007 + PP&E purchases for 2008 – PP&E 2008 PP&E sold

$21,042 million 1,674 million 21,579 million $ 1,137 million

1


Cost of PP&E – Accumulated depreciation on assets sold Net book value of PP&E sold Cash received from sale Loss on sale of PP&E

$1,137 million 810 million 327 million 34 million $ 293 million

Derived Journal Entry: Cash (+A) 34 Accumulated Depreciation (+A) 810 Loss on Sale (E, -SE) 293 Property, Plant & Equipment (-A)

1,137

The loss on the sale of property, plant and equipment would be shown in the income statement, usually in an “other gains and losses” section. These transactions would affect the statement of cash flows in the “funds from investing activities section”. Any sales would be a source of funds in the amount of cash received.

BE9–3 a. Johnson and Johnson invested $130 million ($886– $756) of land during 2008. b. Accumulated depreciation increased during 2008 because of depreciation expense taken by Johnson and Johnson. Instead of reducing the asset account directly, depreciation expense is added to accumulated depreciation, which offsets the asset account to show its reduction in value. c.

If the company used an accelerated method of depreciation, the assets would be shown at a lower net value in the early years. Accelerated methods take more depreciation charges in the early years of an asset’s life and less in the later years, when compared to the straight-line method.

d. Johnson and Johnson would show $14,365 million for property, plant and equipment on its financial statement for 2008. The gross amount and the accumulated depreciation would be disclosed in the footnote.

BE9–4 a. The Depreciation & Amortization adjustment and the Impairment adjustment both were recorded by charging an expense to earnings and lowering the carrying value of the long-term assets. The gain was recorded when the long term assets were sold for a price greater than the carrying value; the debit to cash and credit to the sold assets were balanced out with a credit to the temporary account “Gain on sale”. b. The Depreciation & Amortization adjustment and the Impairment charge are both non-cash expenses that need to be added back to earnings on the statement of cash flow. (The expenses reduced earnings but did not reduce cash, so the add-back is necessary in the statements conversion of accrual earnings to actual cash flow.) The gain is a deduction because activities involving long-term assets are not included in the Operating section. c. The gain is adjusted out of the Operating section of the statement of cash flow because the cash received from the sale of long-term assets is an Investing activity and needs to be reflected in a different section of the statement of cash flow; additionally, the Investing section will reflect the entire amount of cash received in the sale, not simply the gain.


d. If the company followed U.S. GAAP, similar adjustments would appear.

EXERCISES E9–1 a. Lowery, Inc., should capitalize all costs associated with getting the equipment in a serviceable condition and location. These costs would be the actual purchase price of $920,000, the transportation cost of $62,000, and the insurance cost of $10,000. Therefore, the total cost of the equipment is $992,000. b. The depreciation base equals the dollar amount of a fixed asset's cost that the company does not expect to recover over the asset's useful life, but instead expects to consume over the asset's useful life. Since the plant equipment's total cost is $992,000 and since Lowery, Inc., expects to sell the equipment for $50,000 at the end of its useful life, Lowery, Inc., does not expect to recover $942,000 of the asset's cost. Therefore, the depreciation base equals $942,000. The depreciation base always equals the capitalized cost of a fixed asset less its estimated salvage value. c.

The amount that will be depreciated over the life of the plant equipment is its depreciation base. The depreciation base equals the amount of the equipment's future benefits that the company will consume. The outflow of future benefits are expenses, in this case depreciation expense. Therefore, the total amount that Lowery, Inc., will depreciate over the equipment's useful life is $942,000.

E9–2 Lot 1

Lot 2

Lot 3

Lot 4

Revenue $ 160,000 $ 120,000 $ 60,000 $ 60,000 Expenses 128,000* 96,000* 48,000* 48,000* Net income $ 32,000 $ 24,000 $ 12,000 $ 12,000 ________________ * Expenses were calculated as follows: 1. Calculate total market value. Total Market value = $160,000 + $120,000 + $60,000 + $60,000 = $400,000 2. Allocate costs to each lot based upon relative market values. Lot 1 = $320,000  (160,000/400,000) = $128,000 Lot 2 = $320,000  (120,000/400,000) = $ 96,000 Lot 3 = $320,000  (60,000/400,000) = $ 48,000 Lot 4 = $320,000  (60,000/400000) = $ 48,000


E9–3 a. All costs that are necessary and reasonable to get an asset ready for its intended use should be capitalized as part of the cost of that asset. In the case of property, plant, and equipment, "ready for its intended use" means that the asset is in a serviceable condition and location.

Item

Land

Tract of land Demolition of warehouse Scrap from warehouse Construction of building Driveway and parking lot Permanent landscaping Total

$90,000 10,000 (7,000)

Land Improvements

Building

$140,000 $32,000 4,000 $ 97,000

$32,000

$140,000

b. Land: Since land is assumed to have an indefinite life, it is never depreciated. Land Improvements: Depreciation Expense—Land Improvements (E, –SE) ............................... Accumulated Depreciation—Land Improvements (–A) ....................... Depreciated land improvements.

1,600 1,600

Building: Depreciation Expense—Building (E, –SE) .................................................. Accumulated Depreciation—Building (–A) .......................................... Depreciated building.

7,000 7,000

E9–4 a. b. c. d. e. f. g. h. i.

Maintenance Maintenance Maintenance Betterment Maintenance Maintenance Betterment Maintenance Betterment

Note:

The classification of these expenditures can be quite subjective. Some accountants might very well classify some of these expenditures differently. For example, one might argue that the cost of the


muffler in (h) is actually a betterment expenditure if the reduced noise allows workers to work more efficiently, thereby increasing the productive capacity of the machine.

E9–5 a. (1)

Expensed immediately: Income Statement Revenues Amortization Other expenses Net income

2014

2013

2012

$ 65,000 0 20,000 $ 45,000

$ 65,000 0 20,000 $ 45,000

$ 65,000 40,000 20,000 $ 5,000

Balance Sheet Assets Current assets Long-lived assets (including land) Total assets Liabilities and Stockholders' Equity Liabilities Stockholders' equity Total liabilities & stockholders' equity

12/31/14

12/31/13

12/31/12

$ 135,000

$ 90,000

$ 45,000

50,000 $ 185,000

50,000 $ 140,000

50,000 $ 95,000

$ 35,000 150,000

$ 35,000 105,000

$ 35,000 60,000

$ 185,000

$ 140,000

$ 95,000


E9–5 (2)

Continued Amortized over two years: Income Statement Revenues Amortization Other expenses Net income

2014

2013

2012

$ 65,000 0 20,000 $ 45,000

$ 65,000 20,000 20,000 $ 25,000

$ 65,000 20,000 20,000 $ 25,000

Balance Sheet 12/31/14 Assets Current assets Long-lived assets (including land) Total assets Liabilities and Stockholders' Equity Liabilities Stockholders' equity Total liabilities & stockholders' equity (3)

12/31/13

12/31/12

$ 135,000

$ 90,000

$ 45,000

50,000 $ 185,000

50,000 $ 140,000

70,000 $ 115,000

$ 35,000 150,000

$ 35,000 105,000

$ 35,000 80,000

$ 185,000

$ 140,000

$ 115,000

2014

2013

2012

$ 65,000 13,334 20,000 $ 31,666

$ 65,000 13,333 20,000 $ 31,667

$ 65,000 13,333 20,000 $ 31,667

Amortized over three years: Income Statement Revenues Amortization Other expenses Net income Balance Sheet Assets Current assets Long-lived assets (including land) Total assets Liabilities and Stockholders' Equity Liabilities Stockholders' equity Total liabilities & stockholders' equity

b. Method 1: Method 2: Method 3:

12/31/14

12/31/13

12/31/12

$ 135,000

$ 90,000

$ 45,000

50,000 $ 185,000

63,334 $ 153,334

76,667 $ 121,667

$ 35,000 150,000

$ 35,000 118,334

$ 35,000 86,667

$ 185,000

$ 153,334

$ 121,667

2014

2013

2012

Total

$45,000 45,000 31,666

$45,000 25,000 31,667

$ 5,000 25,000 31,667

$95,000 95,000 95,000


E9–5 c.

Concluded

The balance sheets under all three methods report identical amounts for each balance sheet account. Since the asset was fully amortized by December 31, 2014, the method used to amortize the asset does not affect the amounts reported on the balance sheet as of December 31, 2014.

E9–6 a. and b. Stork Freight Company Income Statement For the Year Ended December 31 12-Year Useful Life Revenues Expenses: Operating expenses Depreciation expense Total expenses Net income

6-Year Useful Life

$ 50,000,000

$ 50,000,000

$ 25,000,000 1,250,000 26,250,000 $ 23,750,000

$ 25,000,000 2,500,000 27,500,000 $ 22,500,000

The percentage decrease in net income would be approximately 5.26% [($22,500,000 – $23,750,000) ÷ $23,750,000]. c. Net income Dividend payout percentage Dividends

12-Year Useful Life

6-Year Useful Life

$ 23,750,000 30% $ 7,125,000

$ 22,500,000 30% $ 6,750,000

The difference in dividends due simply to using different estimated useful lives for the planes would be $375,000 ($7,125,000 – $6,750,000). However, it should be noted that in the 6-year useful life example, dividends in years 7 – 12 would be higher (due to the lack of depreciation expense).

E9–7 a. An asset's book value equals the asset's initial capitalized value less the associated accumulated depreciation. With straight-line depreciation, accumulated depreciation equals depreciation expense per year times the number of years the asset has been used. Therefore, the asset's book value would be calculated as follows: Depreciation expense per year

= (Cost – Salvage Value) ÷ Useful Life = ($60,000 – $12,000) ÷ 5 years = $9,600 per year

Book Value = Capitalized Cost – Accumulated Depreciation = $60,000 – ($9,600  3 years) = $31,200


E9–7

Concluded

b. Depreciation Expense

= [(Cost – Accumulated Depreciation) – Salvage Value] ÷ Remaining Useful Life = (Book value – Salvage value) ÷ Remaining useful life = ($31,200 – $12,000) ÷ 5 remaining years = $3,840

Depreciation Expense (E, –SE) ................................................................... Accumulated Depreciation (–A) ......................................................... Depreciated asset for 2011.

3,840 3,840

E9–8 Objective (a) (b) (c) (d) (e) (f) (g) (h) 1

2

3

StraightLine

Double-DecliningBalance

Activity Method

x1 x

x1 x x x x

x1 x x2

x x x

x

x3 x

Under certain conditions, all three methods could meet this objective. However, for the straight-line method and the double-declining-balance method, this objective will be met only by chance. The activity method will always meet this objective because depreciation is based upon the actual use of the asset. It is possible that the activity method would generate the largest net income in the last year of an asset's useful life. However, this result would be due to the company's use patterns of the asset and would not be due to the depreciation method per se. See note (2). The same rationale would hold in this case too.

E9–9 a. (1)

Straight-line depreciation: Depreciation per Year = (Cost – Salvage Value) ÷ Useful Life = ($300,000 – $60,000) ÷ 4 years = $60,000 per year for 2011, 2012, 2013, and 2014


E9–9 (2)

Concluded Double-declining-balance depreciation: Date

Depreciation Factor

Depreciation Expense

Cost

Accumulated Depreciation

Book Value

1/1/11 $300,000 $ 0 $300,000 12/31/11 50% $150,000a 300,000 150,000 150,000 12/31/12 50% 75,000 300,000 225,000 75,000 12/31/13 50% 15,000b 300,000 240,000 60,000 12/31/14 50% 0 300,000 240,000 60,000 _______________ a Depreciation Expense = Book Value at Beginning of the Period  Depreciation Factor b Book Value  Depreciation Factor = $75,000  50% = $37,500. If Benick Industries depreciated $37,500 in 2013, the asset's book value would drop below its salvage value. To prevent this from happening, depreciation expense for 2013 can be only $15,000. b. A manager should consider the costs and benefits associated with each depreciation method. The most likely benefit is the impact of depreciation methods on income taxes. An accelerated method decreases the present value of tax payments. However, since there is no requirement that a company use the same depreciation method for financial reporting purposes as it does for tax reporting, tax considerations are not an issue for financial reporting. A manager should also consider the bookkeeping costs associated with each method. However, with computers the bookkeeping costs should be relatively consistent across methods. Finally, since the choice of depreciation methods affects net income, managers might consider the impact of the different depreciation methods on contracts such as debt covenants and incentive compensation contracts. Comparability with other firms in the same industry may also be a factor.

E9–10 a. Computer System (+A) .............................................................................. Cash (–A) .................................................................................... Purchased computer system. Note:

b. (1)

335,000 335,000

Capitalizing the $10,000 of training costs could be debated. But, without incurring these costs, the computer system would not be in a serviceable condition. Hence, the training costs meet the requirement to be capitalized as part of the fixed asset. Straight-line depreciation: Depreciation per Year = (Cost – Salvage Value) ÷ Useful Life = ($335,000 – $70,000) ÷ 5 years = $53,000 per year for 2011, 2012, 2013, 2014, and 2015


E9–10 (2)

Concluded Double-declining-balance depreciation:

Date

Depreciation Factor

Depreciation Expense

Cost

Accumulated Depreciation

Book Value

1/1/11 $335,000 $ 0 $335,000 12/31/11 40% $134,000a 335,000 134,000 201,000 12/31/12 40% 80,400 335,000 214,400 120,600 12/31/13 40% 48,240 335,000 262,640 72,360 12/31/14 40% 2,360b 335,000 265,000 70,000 12/31/15 40% 0 335,000 _____________ a Depreciation expense = Book value at beginning of the period  Depreciation factor b Book value  Depreciation factor = $72,360  40% = $28,944. If Stockton Corporation depreciated $28,944 in 2014, the asset's book value would drop below its salvage value. To prevent this from happening, depreciation expense for 2014 can be only $2,360. c.

Depreciation Expense (E, –SE) ............................................................ Accumulated Depreciation (–A) ................................................. Depreciated fixed asset for 2011.

134,000 134,000

E9–11 1. Activity Method: Depreciation Expense per Mile

= =

($100,000 – $20,000) ÷ 200,000 Miles $0.4/Mile

Depreciation Expense (E, –SE) ................................................................... Accumulated Depreciation (–A) ......................................................... Depreciated asset for 2011.

19,200

Depreciation Expense (E, –SE) ................................................................... Accumulated Depreciation (–A) ......................................................... Depreciated asset for 2012.

14,000

Depreciation Expense (E, –SE) ................................................................... Accumulated Depreciation (–A) ......................................................... Depreciated asset for 2013.

16,000

Depreciation Expense (E, –SE) ................................................................... Accumulated Depreciation (–A) ......................................................... Depreciated asset for 2014.

10,000

19,200

14,000

16,000

10,000


E9–11

Concluded

Depreciation Expense (E, –SE) ................................................................... Accumulated Depreciation (–A) ......................................................... Depreciated asset for 2015.

14,000

Depreciation Expense (E, –SE) ................................................................... Accumulated Depreciation (–A) ......................................................... Depreciated asset for 2016.

4,000

Cash (+A) ................................................................................................ Accumulated Depreciation (+A) ................................................................ Loss on Sale of Truck (Lo, –SE) .................................................................. Truck (–A) ........................................................................................... Sold truck.

12,000 77,200 10,800

2. Straight-line Method: Depreciation Expense per Year

4,000

100,000

= ($100,000 – $20,000) ÷ 5 Years = $16,000/year

Depreciation Expense (E, –SE) ................................................................... Accumulated Depreciation (–A) ......................................................... Depreciated asset. Note:

14,000

16,000 16,000

This entry would be made each year for five years. No entry would be made in Year 6 since the truck's estimated useful life ended at the end of Year 5, which means that the truck would have been depreciated down to its estimated salvage value.

Cash (+A) .............................................................................................. Accumulated Depreciation (+A) ................................................................ Loss on Sale of Truck (Lo, –SE) .................................................................. Truck (–A).......................................................................................... Sold truck.

12,000 80,000 8,000 100,000

E9–12 a. Depletion (E, –SE) ...................................................................................... Oil Deposits (–A) ................................................................................. Depleted oil deposits. ____________ * $1,200,000 = ($4,000,000 ÷ 100,000 barrels)  30,000 barrels extracted

1,200,000*

b. Depletion (E, –SE) ...................................................................................... Oil Deposits (–A) ................................................................................. Depleted oil deposits. ____________ * $2,000,000 = ($4,000,000 ÷ 100,000 barrels)  50,000 barrels extracted

2,000,000*

1,200,000

2,000,000


c.

$800,000

E9–13 a. Year 2011 2012 2013 2014

Depreciation Expense Per Company's Books $120,000 0 0 0

Correct Depr. Exp. $25,000 25,000 25,000 25,000

Annual Difference $95,000 (25,000) (25,000) (25,000)

Cumulative Difference $95,000 70,000 45,000 20,000

b. After adjusting entries are prepared and posted on December 31, 2013, Accumulated Depreciation will be understated by $45,000. c.

After adjusting entries, but before closing entries have been prepared and posted on December 31, 2013, Retained Earnings will be understated by $70,000.

d. After both adjusting and closing entries have been prepared and posted on December 31, 2013, Retained Earnings will be understated by $45,000.

E9–14 a. Cash (+A) .............................................................................................. Accumulated Depreciation—Office Equipment (+A) ................................ Office Equipment (–A) ........................................................................ Gain on Sale of Fixed Assets (Ga, +SE) ................................................ Sold office equipment.

235,000 300,000

b. Cash (+A) ................................................................................................ Accumulated Depreciation—Office Equipment (+A) ................................ Loss on Sale of Fixed Assets (Lo, –SE)........................................................ Office Equipment (–A) ........................................................................ Sold office equipment.

185,000 300,000 15,000

500,000 35,000

500,000


E9–15 Assuming that Paris Company kept the equipment for its entire five-year estimated useful life, the depreciation schedule on the equipment would be as follows.

Date

Depreciation Factor

Depreciation Expense

Cost

Accumulated Depreciation

Book Value

1/1/09 $25,000 $ 0 $25,000 12/31/09 40% $10,000 25,000 10,000 15,000 12/31/10 40% 6,000 25,000 16,000 9,000 12/31/11 40% 3,600 25,000 19,600 5,400 12/31/12 40% 400* 25,000 20,000 5,000 12/31/13 40% 0 25,000 20,000 5,000 __________________ * Because the equipment's book value cannot drop below its estimated salvage value, depreciation expense for 2012 cannot exceed $400. a. Accumulated Depreciation—Equipment (+A) ........................................... Loss on Disposal of Equipment (Lo, –SE) ................................................... Equipment (–A)................................................................................... Disposed of equipment.

19,600 5,400

b. Accumulated Depreciation—Equipment (+A) ........................................... Loss on Disposal of Equipment (Lo, –SE) ................................................... Equipment (-A) ................................................................................... Disposed of equipment.

20,000 5,000

c.

Cash (+A) .............................................................................................. Accumulated Depreciation—Equipment (+A) ........................................... Equipment (–A)................................................................................... Gain on Sale of Fixed Assets (Ga, +SE) ................................................ Sold equipment.

d. Fixed Asset (new) (+A) ............................................................................... Accumulated Depreciation—Equipment (+A) ........................................... Loss on Disposal of Fixed Asset (Lo, –SE) .................................................. Cash (–A)............................................................................................. Equipment (old) (–A) .......................................................................... Exchanged fixed assets.

25,000

25,000

8,000 19,600 25,000 2,600

30,000 20,000 3,000 28,000 25,000


E9–16 a. and b.

First, let us compute the original cost of the equipment that was sold in 2011 as follows:

Equipment at the End of 2010

+

$32,700

+

Equipment Purchased during 2011

Equipment sold during 2011

$12,000 X

– =

X $7,200

=

Equipment at the End of 2011

=

$37,500

Now, let us compute the related accumulated depreciation for the equipment sold during 2011 as follows: Accumulated Depreciation at the End of 2010

+

Depreciation Exp. for 2011

$14,300

+

$7,200

Accumulated Depreciation for the Sold Equipment during 2011 X X

=

= =

Accumulated Depreciation at the End of 2011

$17,600 $ 3,900

Now, we can reconstruct the journal entry. Cash ........................................................................................................... Accumulated Depreciation ........................................................................ Equipment .......................................................................................... Gain on Sale of Equipment ................................................................. ____________ * $7,200 + $2,100 – $3,900 = $5,400

E9–17 Account a. Property, plant & equipment Less: accumulated depreciation Depreciation expense Investments in property, plant & equipment

Financial Statement Balance Sheet Balance Sheet Income Statement Statement of Cash Flows

b. Property, plant & equipment – 2007 Plus: investments in property, plant & equipment Less: property, plant & equipment – 2008 Property, plant & equipment sold in 2008

$46,052 5,197 48,088 $ 3,161

c.

$29,134 4,360 30,544 $ 2,950

Accumulated depreciation – 2007 Plus: depreciation expense – 2008 Less: accumulated depreciation – 2008 Accumulated depreciation – sold property

5,400* 3,900 7,200 2,100


E9–17 Concluded d. Compute the gain on the sale: Cost of property sold $3,161 Less: accumulated depreciation 2,950 Book value of property sold $ 211 Sales price of property Less: book value of property Loss on sale of property

$100 211 $111

This loss on sale of property would appear on the income statement.

E9–18 a. First, let us compute the related accumulated depreciation for the equipment sold during 2011 as follows: Accumulated Depreciation at the End of 2010

+

Depreciation Cap. for 2011

$9,800

+

$3,800

Accumulated Depreciation for the Sold Equipment during 2011 X X

=

= =

Accumulated Depreciation at the End of 2011

$10,500 $ 3,100

Now, we can reconstruct the journal entry. Cash ........................................................................................................... Loss on Sale of Equipment ........................................................................ Accumulated Depreciation ........................................................................ Equipment ..........................................................................................

b.

Equipment at the End of 2010

+

$23,400

+

Equipment Purchased during 2011

Equipment sold during 2011

X X

– =

$8,300 $11,800

____________ Equipment purchased during 2011 = $11,800

4,300 900 3,100 8,300

=

Equipment at the End of 2011

=

$26,900


E9–19 a. Swift Corporation should capitalize these costs. Assets are defined as items that are expected to provide future economic benefits to the entity. Organization costs are costs incurred by an entity prior to starting operations. Such costs include legal fees to incorporate and accountant's fees to set up an accounting system. Without incurring these costs, most companies could not be in business. Consequently, organization costs allow a company to be in business, thereby helping it to generate future benefits. Since these costs help in generating future benefits, they should most definitely be capitalized. b. Theoretically, organization costs should be amortized over their useful life. In the extreme, organization costs provide a benefit over the entire life of a company. Since under the going concern assumption accountants assume that entities will exist indefinitely, it would seem that organization costs should be amortized over an indefinite period. Since this position is not practical, the accounting profession has decided that organization costs should be amortized over a period not to exceed forty years. Assuming that Swift Corporation amortizes its organization costs over the maximum period of forty years, the appropriate adjusting journal entry for a single year would be as follows: Amortization Expense (E, –SE) .................................................................. Organization Costs (–A) ...................................................................... Amortized organization costs. c.

1,125 1,125

As mentioned in part (b), organization costs theoretically provide benefits over the entire life of the company. Under the going concern assumption, the company is assumed to exist indefinitely. If the company is assumed to exist indefinitely and if organization costs provide benefits over the entire life of the company, then these costs should provide an indefinite benefit. Consequently, organization costs should provide a benefit for an indefinite period of time, which implies that they should be reported as an asset (i.e., future benefit) indefinitely. But if organization costs are amortized, the asset will at some point in time have a zero balance, and the cost of the asset cannot be matched against the benefits the asset will help generate in the future. This situation contradicts the matching principle and the concept of an asset.

d. A patent gives a company the exclusive right to use or market a particular product or process, thereby providing the company with an expected future benefit. Consequently, the costs incurred to acquire a patent should be capitalized as an asset and amortized over the patent's useful life. If Swift were to immediately expense the $65,000, the company would be implying that it did not expect to receive any benefits from the patent in the future. If this were the case, one would have to question why Swift purchased the patent in the first place. e. Research and development costs may or may not provide a company with future benefits. The company will not know whether or not a particular R & D expenditure will provide a future benefit until some time in the future. Due to the uncertainty of projecting the usefulness of a given R & D expenditure, the FASB, in Statement of Financial Accounting Standards No. 2, "Accounting for Research and Development Costs," requires companies to expense R & D costs in the year in which they are incurred.


E9–19 f.

Concluded

Engaging in research and development activities can lead companies to develop new products or processes that will provide them with future benefits. In such cases, the R & D costs should, theoretically, be capitalized. The R & D costs would then be allocated to those periods in which the costs help generate a benefit. From a practical standpoint, however, this matching of costs with the associated benefits is not readily possible. For example, consider a company that spends $10,000,000 trying to develop a more efficient manufacturing process. The company's attempts end in failure, but the company acquires some new technology from its R & D activities that permit it to develop a revolutionary new product ten years later. In this case, it is clear that the $10,000,000 eventually provided a future benefit. But this information is available only with hindsight. At the time the $10,000,000 was expended, all the company knew was that the R & D project was a failure. So, while capitalizing R & D costs and then amortizing the costs over their useful lives is theoretically superior to immediately expensing the R & D costs, immediately expensing R & D costs is extremely practical and lessens a manager's ability to manipulate the financial statements.

E9–20 a. (1)

Southern Robotics should report the costs incurred in acquiring the patent as an asset. Therefore, the $50,000 of legal and filing fees should be capitalized as an asset in 2011. Since it is company policy not to amortize intangible assets in the year of acquisition, the company would report the entire $50,000 as an asset as of December 31, 2011.

(2)

Since Southern Robotics successfully defended its patent, the patent is still expected to provide a future benefit to the company. Hence, the company should continue to carry the patent on its books as an asset. The amount it should report for the patent as of December 31, 2012 should be the cost of acquiring and defending the patent less the portion of these costs that have been amortized. Therefore, Southern Robotics should report $200,000 on its balance sheet (i.e., $50,000 in legal and filing fees incurred in 2011 + $200,000 in legal fees incurred in 2012 to defend the patent – $50,000 in amortization).

(3)

Amortization Expense (E, –SE)............................................................ Patent (–A) ................................................................................... Amortized patent.

50,000 50,000

b. (1)

Since the lawsuit did not take place until 2012, the patent still had value to Southern Robotics as of December 31, 2011. Therefore, the company should still report the patent at $50,000 on its books as of December 31, 2011. However, if Southern Robotics was aware of the lawsuit as of December 31, 2011, it might want to disclose the lawsuit and the potential effect on the company's financial statements in a footnote as a contingency.

(2)

Since Southern Robotics was unsuccessful in defending its patent, the company no longer has the exclusive right to use or market its robotics arm. Therefore, the patent no longer provides the company with any future benefits. Since the patent no longer provides any future benefits, it should be written off in 2012.

(3)

Loss on Patent (Lo, –SE)...................................................................... Legal Expenses (E, –SE) .......................................................................

50,000 200,000


Patent (–A) ................................................................................... Cash (–A) ...................................................................................... Incurred legal fees for patent defense and wrote off patent.

50,000 200,000

E9–21 a. The journal entry for the acquisition can be derived from the information provided and appears below: Assets (+A) (at fair market value) ......................................................... 5.1 Goodwill (+A) 3.8 Liabilities (+L) ...................................................................................... Cash (-A) ........................................................................................ 7.8

1.1

Goodwill represents the excess of the purchase price above the fair market value of the assets purchased. b. Assets increased by a net $1.1 billion ($5.1 + 3.8 – 7.8) and liabilities increased by $1.1 billion.

E9–22 a. Under US GAAP, long-lived assets must be carried at original cost less accumulated depreciation (amortization); if the market value of the asset permanently falls below the balance sheet carrying value, an impairment charge must be recorded, and cannot be reversed in later periods if the value of the asset recovers. Under IFRS, companies can either follow the US GAAP method, or they can periodically revalue their long-lived assets to fair market value – recognizing not only impairments, but also increases and recoveries of asset values. b. EADS has accounted for this asset according to U.S. GAAP methods. c. If EADS were to carry the asset at its Fair Market Value (using the IFRS approach), the company would increase the value of the asset by 1 million euros and would record a gain on its income statement of the same amount.


PROBLEMS P9–1 a. Stonebrecker should capitalize all costs that it incurred that were necessary and reasonable to get the equipment in a serviceable condition and location. The capitalizable costs are (1) the $1,000,000 purchase price, (2) the $40,000 transportation costs actually incurred by Stonebrecker, (3) the $8,000 insurance coverage, (4) the $20,000 installation fees, (5) the $15,000 to reinforce the floor, and (6) the $10,000 of employee downtime. Some accountants may disagree with capitalizing the last two items as part of the equipment. However, theoretically, these costs are necessary to get the equipment in a usable condition. Therefore, the total dollar amount that should be capitalized for the equipment is $1,093,000. b. Equipment (+A) ......................................................................................... Cash (–A)............................................................................................. Purchased equipment. c.

1,093,000 1,093,000

The depreciation base represents the capitalized cost of a fixed asset that the company does not expect to recover over the asset's estimated useful life. Since the capitalized cost of the equipment is $1,093,000 and the company expects to sell the equipment for $100,000 after ten years, the company does not expect to recover $993,000 of the capitalized cost. Therefore, the depreciation base of the equipment is $993,000.

d. As discussed in part [c], the depreciation base represents the dollar amount of a fixed asset that the company does not expect to recover from the asset at the end of the asset's estimated useful life. This implies that the depreciation base represents the dollar amount of a fixed asset that the company expects to consume over the asset's estimated useful life. Since the consumption of an asset is an outflow of that asset and since, by definition, outflows of assets are expenses, the depreciation base represents the amount that should be expensed over a fixed asset's useful life. This is true whether the company uses the straight-line method or the double-declining-balance method. Thus, every depreciation method will result in the same total amount being depreciated over a fixed asset's useful life. Although each method gives rise to the same total amount of depreciation, the timing of depreciation charges varies across depreciation methods. The straight-line method allocates depreciation evenly across time, while the double-declining-balance method allocates the depreciation base more rapidly to the early years of the asset's useful life and more slowly to the later years of the asset's useful life. Thus, Stonebrecker will depreciate a total of $993,000 under both depreciation methods.


P9–2 a. 1/1/11 FMV

Relative FMV

300,000 150,000 75,000 75,000 600,000 $ 1,200,000

300/1,200 150/1,200 75/1,200 75/1,200 600/1,200 1,200/1,200

Asset Building Office equip. Crane 1 Crane 2 Land Total

$

Purchase Price $1,000,000 1,000,000 1,000,000 1,000,000 1,000,000

=

Cost Allocation $

250,000 125,000 62,500 62,500 500,000 $ 1,000,000

Building (+A) ............................................................................................. Office Equipment (+A) ............................................................................... Cranes (+A) ............................................................................................. Land (+A) .............................................................................................. Cash (–A)............................................................................................. Purchased basket of assets.

250,000 125,000 125,000 500,000 1,000,000

b. Depreciation Expense—Building (E, –SE) .................................................. Depreciation Expense—Office Equipment (E, –SE) ................................... Depreciation Expense—Cranes (E, –SE) .................................................... Accumulated Depreciation—Building (–A) ......................................... Accumulated Depreciation—Office Equipment (–A) ......................... Accumulated Depreciation—Cranes (–A) ........................................... Depreciated fixed assets. ____________ a $8,750 = ($250,000 – $75,000) ÷ 20 years b $30,000 = ($125,000 – $35,000) ÷ 3 years c $19,000 = [$125,000 – ($15,000 + $15,000)] ÷ 5 years

8,750a 30,000b 19,000c

c.

Property, plant, and equipment: Land ........................................................................................................... Building...................................................................................................... Office equipment ...................................................................................... Cranes........................................................................................................ Less: Accumulated depreciation ............................................................... Total property, plant, and equipment....................................................... ____________ * $201,000 = ($8,750  4 years) + ($30,000  3 years) + ($19,000  4 years)

8,750 30,000 19,000

$500,000 250,000 125,000 125,000 (201,000)* $799,000


P9–3 a. Cost = Purchase Price + Transportation + Installation = $950,000 + $100,000 + $130,000 = $1,180,000 b. (1)

Double-declining-balance method: Depreciation Expense—Equipment (E, –SE) ................................... Accumulated Depreciation—Equipment (–A) .......................... Depreciated fixed asset. _____________ *$590,000 = $1,180,000  50%

(2)

282,500* 282,500

Double-declining-balance method: Cash (+A) ......................................................................................... Accumulated Depreciation: Equipment (+A) .................................. Loss on Sale of Equipment (Lo, –SE) ............................................... Equipment (–A) ........................................................................ Sold equipment.

(2)

590,000

Straight-line method: Depreciation Expense—Equipment (E, –SE) ................................... Accumulated Depreciation—Equipment (–A) .......................... Depreciated fixed asset. _____________ *$282,500 = ($1,180,000 – $50,000) ÷ 4 years

c. (1)

590,000*

250,000 590,000 340,000 1,180,000

Straight-line method: Cash (+A) ......................................................................................... Accumulated Depreciation: Equipment (+A) .................................. Loss on Sale of Equipment (Lo, –SE) ............................................... Equipment (–A) ........................................................................ Sold equipment.

250,000 282,500 647,500 1,180,000


P9–4 a.

Truck (+A) ................................................................................................ Cash (–A)............................................................................................. Purchased a truck.

48,000 48,000

b. Depreciation Per Books 2009 $48,000 2010 0 _____________ *$12,000 = ($48,000 – $12,000) ÷ 3 years

Correct Depreciation $

0 12,000*

Difference $48,000 12,000

Therefore, in 2009 expenses were overstated by $48,000, so net income was understated by $48,000. In 2010 expenses were understated by $12,000, so net income was overstated by $12,000. c. Depreciation Per Books 2009 $48,000 2010 0 _____________ *$32,000 = $48,000  2/3

Correct Depreciation $

0 32,000*

Difference $48,000 32,000

Therefore, in 2009 expenses were overstated by $48,000, so net income was understated by $48,000. In 2010 expenses were understated by $32,000, so net income was overstated by $32,000.


P9–5 a. Dryer (+A) .................................................................................................. Cash (–A)............................................................................................. Purchased a dryer.

100,000 100,000

b. In deciding how to account for service and repair costs, one must consider the effect of the cost on (1) the useful life of the asset, (2) the quality of units produced by the asset, (3) the quantity of units produced by the asset, or (4) the cost of operating the asset. If the costs increase one of the first three items or reduce the last item, they provide a future benefit to the company. Consequently, the costs should be capitalized and amortized over the asset's useful life. If the costs do not increase one of the above items, they do not provide a future benefit, and they should be expensed immediately. In this particular case, the $160,000 overhaul increased both the dryer's efficiency and useful life. Consequently, the $160,000 should be capitalized as follows: Dryer (+A) .................................................................................................. Cash (–A)............................................................................................. Overhauled dryer.

160,000 160,000

The annual service cost of $1,000 and the major repair cost of $5,000 are incurred simply to maintain the dryer's existing service potential and, therefore, do not provide expected future benefits. Consequently, these costs should be expensed as incurred. c.

2008 through 2011: Depreciation Expense

2012 through 2015: Depreciation Expense

= =

(Cost – Salvage Value) ÷ Useful Life ($100,000 – $10,000) ÷ 5 Years = $18,000 per Year

= {[(Cost – Accumulated Depreciation) + Betterments] – Salvage Value} ÷ Remaining Useful Life. = {[($100,000 – $72,000) + $160,000] – $10,000} ÷ 4 Years = $44,500 per Year

P9–6 a. Building (+A) .............................................................................................. Cash (–A)............................................................................................. Purchased a building.

1,500,000

b. Building (+A) .............................................................................................. Cash (–A)............................................................................................. Installed a new roof on building.

200,000

c.

1,500,000

Depreciation Expense (E, –SE) ................................................................... 50,500* Accumulated Depreciation (–A) ......................................................... Depreciated building. ______________ *$50,500 = {[($1,500,000 – $540,000 Accumulated Depreciation) + $200,000] – $150,000 Salvage Value} ÷ Remaining Life of 20 Years.

200,000

50,500


P9–6

Concluded

d. Cash (+A) ................................................................................................... 1,200,000 Accumulated Depreciation (+A) ................................................................ 843,000* Building (–A) ....................................................................................... 1,700,000 Gain on Sale of Building (Ga, +SE) ...................................................... 343,000 Sold building. ______________ *$843,000 = ($54,000  10 years for 2007 through 2016) + ($50,500  6 years for 2017 through 2022) Note:

The above entry assumes that the adjusting entry to record depreciation expense for 2022 had already been recorded. If this entry had not yet been made, the appropriate entry to record the sale would have been as follows:

Cash (+A) ................................................................................................... Accumulated Depreciation (+A) ................................................................ Depreciation Expense (E, –SE) ................................................................... Building (–A) ....................................................................................... Gain on Sale of Building (Ga, +SE) ...................................................... Sold building.

1,200,000 792,500 50,500 1,700,000 343,000

P9–7 a. Depreciation Expense per Year

1/1/11 Accumulated Depreciation

1/1/11 Book Value

= ($180,000 – $30,000) ÷ 10 Years = $15,000 per Year = $15,000 Depreciation Expense per Year  5 Years = $75,000

= Cost – 1/1/11 Accumulated Depreciation = $180,000 – $75,000 = $105,000

b. Depreciation Expense (E, –SE) ................................................................... 9,375 Accumulated Depreciation (–A) ......................................................... Depreciated fixed assets. ______________ *$9,375 = (Book Value of $105,000 – Salvage Value of $30,000) ÷ Remaining Useful Life of 8 Years

9,375

P9–8 When the hand-held instruments were capitalized as a prepaid expense, they were carried as Current Assets and converted to an expense as used. After the accounting change, the asset was carried as a long-lived asset and converted to an expense through a depreciation charge that spread the cost over a five-year period, lowering the amount charged against earnings when compared to the previous method. The asset will remain on the books for a longer period of time (assets and equity are higher) and income will also be higher due to the lower expense. The current ratio is lowered, because the assets are moved to the noncurrent section of the balance sheet. Finally, the fixed asset turnover ratio,


which measures sales to fixed assets, will be lower as the dollar amount assigned to fixed assets is now greater.

P9–9 a. Every depreciation method depreciates the same amount over the useful life of a fixed asset. Depreciation methods only vary the timing of depreciation charges. Therefore, both the straightline method and the double-declining-balance method will give rise to the same total amount of depreciation over the four-year useful life of this equipment. The following table shows that the total depreciation under the two methods is the same. Method

Year 1

Year 2

Year 3

Year 4

Straight-linea $15,000 $15,000 $15,000 $15,000 b Double-declining-balance 40,000 20,000 0 0 ______________ a $15,000 = ($80,000 – $20,000) ÷ 4 years b Depreciation Depreciation Historical Accumulated Date Factor Expense Cost Depreciation 1/1/11 12/31/11 12/31/12 12/31/13 12/31/14

50% 50% 50% 50%

$40,000 20,000 0 0

$80,000 80,000 80,000 80,000 80,000

$

0 40,000 60,000 60,000 60,000

Total $60,000 60,000

Book Value $80,000 40,000 20,000 20,000 20,000

b. Since, as demonstrated in part (a), both depreciation methods give rise to the same total amount of depreciation over the fixed asset’s life, the total amount of net income over the asset’s life must also be the same. Therefore, the total amount of taxes will be the same regardless of which depreciation method a company selects. The following shows that the total amount of net income and taxes are the same under the two methods. Straight-line method: Method Revenues Depreciation exp. Other expenses Pretax income Income taxes Net income

Year 1

Year 2

Year 3

Year 4

Total

$ 100,000 15,000 60,000 $ 25,000 8,750 $ 16,250

$ 100,000 15,000 60,000 $ 25,000 8,750 $ 16,250

$ 100,000 15,000 60,000 $ 25,000 8,750 $ 16,250

$ 100,000 15,000 60,000 $ 25,000 8,750 $ 16,250

$ 400,000 60,000 240,000 $ 100,000 35,000 $ 65,000

Year 1

Year 2

Year 3

Year 4

Total

$ 100,000 40,000 60,000 $ 0 0 $ 0

$ 100,000 20,000 60,000 $ 20,000 7,000 $ 13,000

$ 100,000 0 60,000 $ 40,000 14,000 $ 26,000

$ 100,000 0 60,000 $ 40,000 14,000 $ 26,000

$ 400,000 60,000 240,000 $ 100,000 35,000 $ 65,000

Double-declining-balance: Method Revenues Depreciation exp. Other expenses Pretax income Income taxes Net income c.

The double-declining-balance method is preferred for tax purposes because this method defers tax payments. Under this depreciation method, more depreciation is taken in the early years of an


asset’s life than in later years. Increasing depreciation in an asset’s early life reduces taxable income which, in turn, reduces income taxes in the early years of the asset’s life [see part b]. The reduction in income taxes in the early years of the asset’s life is offset by higher taxes in the later years of an asset’s life. However, due to the time value of money, deferring taxes is beneficial. d. Straight-line method: Present Value = $8,750 from part (b)  Present Value of an Ordinary Annuity Factor for i = 10% and n = 4 = $8,750  3.16987 (from Table 5) = $27,736.36 Double-declining-balance method: Present Value = ($7,000  Present Value Factor for i = 10% and n = 2) + ($14,000  Present Value Factor for i = 10% and n = 3) + ($14,000  Present Value for i = 10% and n = 4) = ($7,000  0.82645) + ($14,000  0.75131) + ($14,000  0.68301) = $5,785.15 + $10,518.34 + $9,562.14 = $25,865.63

Factor

In present value terms, Kimberly Sisters would save $1,870.73 ($27,736.36 – $25,865.63) in taxes on this one asset by selecting the double-declining-balance method over the straight-line method.

P9–10 (a) S-L Depreciation (10-year life)

(b) DDB Depreciation

(c) S-L Depreciation (5-year life)

$ 250,000 (80,000)b (140,000) $ 30,000 (9,600) $ 20,400

$ 250,000 (80,000)c (140,000) $ 30,000 (9,600) $ 20,400

47,600 8% 3,808

$

20,400 8% 1,632

$

47,600 75% 35,700

$

20,400 75% 15,300

$

Tax Payments: Revenues $ 250,000 Depreciation expense (40,000)a Other expenses (140,000) Net income before taxes $ 70,000 Income taxes (22,400) Net income $ 47,600 ________________ a $40,000 = ($400,000 – 0) ÷ 10 years b $80,000 = ($400,000  20%) c $80,000 = ($400,000 – 0) ÷ 5 years Bonus Payment: Net income Bonus percentage Bonus amount Dividend Payment Net income Dividend percentage Dividend amount

$ $

$ $

$

$

$

$

20,400 8% 1,632

20,400 75% 15,300


P9–11 a. Drilling Equipment (+A) ............................................................................. Mobile Home (+A) ..................................................................................... Cash (–A)............................................................................................. Purchased assets for drilling fields. b. 2011: Depletion (E, –SE) ...................................................................................... Drilling Equipment (or Accumulated Depletion) (–A)......................... Depleted drilling equipment. ____________ *$240,000 = ($800,000 ÷ 2,000,000 barrels)  600,000 barrels 2012: Depletion (E, –SE) ...................................................................................... Drilling Equipment (or Accumulated Depletion) (–A)......................... Depleted drilling equipment. ____________ *$300,000 = ($800,000 ÷ 2,000,000 barrels)  750,000 barrels 2013: Depletion (E, –SE) ...................................................................................... Drilling Equipment (or Accumulated Depletion) (–A)......................... Depleted drilling equipment. ____________ *$260,000 = ($800,000 ÷ 2,000,000 barrels)  650,000 barrels

800,000 54,000 854,000

240,000* 240,000

300,000* 300,000

260,000* 260,000

c. 2011: Depreciation Expense (E, –SE) ................................................................... Accumulated Depreciation (–A) ......................................................... Depreciated mobile home. 2012: Depreciation Expense (E, –SE) ................................................................... Accumulated Depreciation (–A) ......................................................... Depreciated mobile home. 2013: Depreciation Expense (E, –SE) ................................................................... Accumulated Depreciation (–A) ......................................................... Depreciated mobile home. ____________ *$7,000 = ($54,000 – $5,000) ÷ 7 years

7,000* 7,000

7,000* 7,000

7,000* 7,000


P9–11

Concluded

Different methods are used to allocate the costs of the drilling equipment and the mobile home based upon the link between the asset and the oil field. The drilling equipment is site-specific. Hence, its useful life is identical to the productive life of the oil field. Under the matching principle, the activity method provides the best matching of the costs with the associated benefits. On the other hand, the mobile home is not site-specific; it has a useful life beyond this oil field. The activity method would not be appropriate for the mobile home because the productive capabilities of future oil fields on which the mobile home may be used are not yet known. Consequently, Garmen Oil Company must select either the straight-line method or an accelerated method to depreciate the mobile home. d. Depletion: 2011: Depletion (E, –SE) ...................................................................................... Drilling Equipment (or Accumulated Depletion) (–A)......................... Depleted drilling equipment.

240,000 240,000

2012: Depletion (E, –SE) ...................................................................................... 300,000 Loss on Oil Field (Lo, –SE) .......................................................................... 260,000* Drilling Equipment (or Accumulated Depletion) (–A)......................... 560,000 Depleted drilling equipment. ____________ * Since the well is dry, the drilling equipment will not provide any future benefits; hence, the remaining cost of $260,000 [($800,000 – ($240,000 + $300,000)] should be written off. 2013: No journal entries are necessary. Depreciation: Since the mobile home is not site-specific, the entries for depreciation would be the same as in part (c).


P9–12 a. Cash (+A) ................................................................................................... Accumulated Depreciation (+A) ................................................................ Machinery (–A) ................................................................................... Gain on Sale of Machinery (Ga, +SE) .................................................. Sold machinery. ____________ *$240,000 = [($500,000 – $100,000) ÷ 5 years]  3 years used

325,000 240,000*

b. Depreciation Expense (E, –SE) ................................................................... Accumulated Depreciation (–A) ......................................................... Depreciated machinery for January 1 – June 30. ____________ *$40,000 = [($500,000 – $100,000) ÷ 5 years]  1/2 year

40,000*

Cash (+A) ................................................................................................... Accumulated Depreciation (+A) ................................................................ Machinery (–A) ................................................................................... Gain on Sale of Machinery (Ga, +SE) .................................................. Sold machinery.

320,000 280,000

c.

FMV of asset received: Land (+A) ................................................................................................... Accumulated Depreciation (+A) ................................................................ Loss on Trade-in (Lo, –SE).......................................................................... Machinery (–A) ................................................................................... Cash (–A)............................................................................................. Exchanged machinery for land. FMV of assets given up: Land (+A) ................................................................................................... Accumulated Depreciation (+A) ................................................................ Loss on Trade-in (Lo, –SE).......................................................................... Machinery (–A) ................................................................................... Cash (–A)............................................................................................. Exchanged machinery for land. ____________ *$250,000 = Cash given up + Appraised value of machinery given up

500,000 65,000

40,000

500,000 100,000

210,000 240,000 225,000 500,000 175,000

250,000* 240,000 185,000 500,000 175,000


P9–13 a. Most assets are reported on the balance sheet at historical cost or at historical cost less accumulated depreciation. The historical cost of a particular asset is constant over time. However, the fair market value of that same asset fluctuates over time. Consequently, the fair market value of assets can be less than, equal to, or greater than the historical cost of the assets at any point in time. b. Diversified would pay more for Specialists due to goodwill (i.e., synergy). Specialists' assets considered as a package are worth more than the sum of their individual values. Goodwill arises because certain "assets" are not included on a company's balance sheet. Items that cannot be given a value (i.e., cannot be quantified) are omitted from a balance sheet. Examples include customer loyalty and the company's name recognition. c.

Assets (+A) ................................................................................................. Goodwill (+A) ............................................................................................. Liabilities (+L) ...................................................................................... Cash (–A)............................................................................................. Purchased Specialists, Inc.

1,350,000 700,000 250,000 1,800,000

d. Until recently under GAAP, goodwill was capitalized at the time of acquisition and then amortized over a maximum of 40 years. The school of thought holding the opposite viewpoint espouses that goodwill should be expensed at the time of acquisition. They maintain that since goodwill is a plug number on the books of the acquired company and its amortization period is totally arbitrary, it need not be put on the balance sheet. Further, goodwill should be periodically tested to see if it has been “impaired” (i.e., if the fair value of the assets acquired has dropped).

P9–14 a. Goodwill would be calculated by taking the purchase price less the fair value of the net assets. In this case $3.4 billion was paid and the fair value of the net assets was $1.3 billion ($2.3 less $1.0). Therefore, Goodwill is $2.1 billion. b. There may be a number of reasons why Zimmer paid over the fair value of Centerplus. There may be assets that are not recorded on Centerplus’s books. This usually would be the value of the brand name or a high quality workforce. Another reason could be that Zimmer foresees that there will be significant synergy between the two companies. It may be that these additional locations will have a significant competitive benefit to Zimmer. Another possible reason would be to block another competitor from buying Centerplus. Other reasons could be that Zimmer management just want to make Zimmer a bigger company (management compensation is sometimes based on the size of the company), or it could be that Zimmer overpaid.


ISSUES FOR DISCUSSION ID9–1 a. Gains and losses resulting from the disposal of fixed assets are based on the difference between the proceeds received from the disposal and the asset's book value. Thus, one would have to know the book value of each individual casino and the land to be able to determine the gain or loss from selling one of the casinos. Book value equals the original cost of the fixed asset less any accumulated depreciation associated with the fixed asset. Since MGM Grand, Inc., held the casino it sold for just a short time, the casino's book value is essentially identical to its cost. Thus, in this case, one would need to know the individual costs of the two casinos and the land. When assets are purchased in a group for one purchase price, as they were in this case, a common method to determine the purchase price for each individual asset acquired is to use the assets' relative fair market values. The fair market values of the individual assets would often be determined by an independent appraiser. b. Cash (+A) ................................................................................................... Casino (–A).......................................................................................... Land (–A)............................................................................................. Gain on Sale (Ga, +SE) ........................................................................ Sold casino and land. c.

110,000,000 75,000,000 17,000,000 18,000,000

For the purchasing company, it would be necessary to allocate the total $110 million cost to the casino and land. This could be done based on an appraised fair market value. The land without the casino would be appraised first; the difference between the $110 million and the appraisal would be the value of the casino.

d. Cost of hotel: $110,000,000 – $43,000,000 = $67,000,000 Depreciation per year: $67,000,000 ÷ 25 years = $2,680,000 per year

ID9–2 One of the underlying goals of an accounting system is to properly match revenues with expenses. There are many marketing costs that will help to produce revenue for the company over multiple periods. If the company expenses all of these marketing expenses in the first year, then net income for the first year will be understated and then overstated in future years when the revenue produced is not matched with the marketing expenses incurred to generate it. At the same time it is very difficult to determine a rational way to allocate marketing costs to the revenue that it produces. Management, separate from any desires to influence the stock price, will generally want to match its marketing expenses with the revenues that these expenses produce. Management wants to be able to evaluate the impact of its marketing efforts. Shareholders may want to see a system of charging marketing expenses that will have the best impact on stock performance. In the early years of a company this may mean capitalizing heavy marketing from the early years and defering the expense until the company has higher levels of revenue to absorb these expenses. Auditors want to make sure that marketing expenses are handled consistently and in a manner that fairly represents the true economic value of these


ID9–2

Concluded

expenditures. Auditors also tend to be conservative when there is uncertainty as to the future value of an asset. Will these marketing costs from this year truly have value in future years? Since this is a subjective estimate, auditors may want to expense all marketing expenses in the year incurred.

ID9–3 a. The main issue to be considered is whether the capital expenditure is a betterment or simply maintenance. To be considered a betterment, the expenditure must (1) increase the asset's useful life, (2) increase the quality of the asset's output, (3) increase the quantity of the asset's output, or (4) reduce the cost associated with operating the asset. If the expenditure meets one of these criteria, the expenditure should be capitalized. Otherwise, the expenditure should be expensed. b. The amount may be immaterial.

c.

Depreciation per year represents the remaining net cost of an asset allocated over the asset's estimated remaining useful life. In this particular case, the remaining net cost equals the sum of the asset's book value at the time of refurbishment and the cost of the refurbishment less the estimated salvage value of the plant. This amount would be depreciated over the estimated useful life of the "new" plant.

ID9–4 a. EADS is expensing a portion of its research and development costs, but the company is not expensing the entire amount. The portion not expensed is capitalized on the balance sheet as an asset (Capitalized Development Costs) and amortized as an expense in future periods. b. Under U.S. GAAP, the entire amount would be expensed.

c.

The R & D expense under US GAAP would have been 2,699 million euros plus 31 million euros.

d. In a comparison of earnings IFRS vs. U.S. GAAP for EADS, the IFRS net income overstates earnings by the 31 million euros that were not expensed (but were instead capitalized to the balance sheet). Had the company used U.S. GAAP, total expenses would have been 31 million euros higher.

ID9–5


One of the underlying goals of an accounting system is to properly match revenues with expenses. There are many advertising and research & development costs that will help to produce revenue for the company over multiple periods. If the company expenses all of these expenses in the first year, then net income for the first year will be understated and then overstated in future years when the revenue produced is not matched with the expenses incurred to generate it. At the same time it is very difficult to determine a rational way to allocate these costs to the revenue that it produces. When these costs are incurred it is extremely difficult to know the revenue, if any, that will be produced in future periods. The capitalization of software development costs has the opposite effect. Expenses that are incurred in the current year will not impact the income statement this year but will in future years. The predictability of the future value of software is higher than for advertising and research and development costs. Management, separate from any desires to influence the stock price, will generally want to match its expenses with the revenues that these expenses produce. Management wants to be able to evaluate the impact of both its advertising and research & development costs. Shareholders may want to see a system of charging marketing expenses that will have the best impact on stock performance. In the early years of a company this may mean capitalizing heavy marketing from the early years and defering the expense until the company has higher levels of revenue to absorb these expenses. Management may also have an incentive to report higher net income, which could cause some managers to want to change accounting policies to work to their benefit. This speaks for the benefit of consistently applying accounting policies from year to year.


ID9–6 a. The effect on profits from increased capital spending will come from increased depreciation charges. Capitalized expenditures for fixed assets will eventually hit the income statement as depreciation expense. b. The balance sheet will reflect growth in the property, plant and equipment, as well future growth in the contra asset accumulated depreciation account. The income statement will show increased depreciation expense. And finally, the statement of cash flow will show greater uses of cash in the investing activities section. c.

The justification of management for the increased capital expenditures will be to remain competitive in the marketplace. If businesses do not reinvest in the long-term assets of their operations, they will not be able to remain competitive. Spending the additional money for fixed assets will affect the financial statements today, but that spending will also keep the business viable into the future. The stockholders have to allow funds to be allocated for investing activities if they want to continue to receive a return on their investment in the company.

ID9–7 a. The most likely scenario causing a restaurant’s value to be impaired is a loss in the desirability of the location. If a McDonald’s restaurant was located at a certain intersection and traffic patterns in the city changed (due to a new interstate highway, for example), the restaurant will no longer be as attractive a location. McDonald’s therefore would have to adjust downward the carrying value for that restaurant. b. McDonald’s will record the impairment by first determining the fair value of the asset. Then, the company will record an impairment expense and reduce the asset from its current carrying value down to the (new) fair value. c. As with other expenses that are somewhat at management’s discretion, shareholders are vulnerable if management decides to take an impairment expense in a year where earnings are otherwise very healthy, eliminating the need to take the expense in future years when earnings are less robust. A management team could lower current earnings (and thus future earnings expectations) by taking impairment charges in current periods.

ID9–8 a. It seems GE is engaging in income-smoothing. Whenever GE has a one-time reporting gain due to any unusual events, it also tries to book a related expense or a charge to offset that gain. This ensures that earnings do not rise so high that they cannot be topped the following year. b. Discretionary restructuring charges are used by GE to offset the one-time gains in order to avoid an abnormal peak in the company’s earnings for the year. It seems that GE’s management strongly believes in modest but consistent earnings growth. Therefore, they engage in below-the-line activities and generate gains for themselves, which they try to offset to a somewhat lesser extent by recording a restructuring charge. GE wants to pursue this strategy because capital markets reward a net increase in earnings with a higher stock price. Therefore, GE hopes to increase the price of its stock. c.

Since the proliferation of total quality movement, restructuring is usually perceived by the investors and the market as a cost-cutting exercise by the company. If the market believes that a company is


becoming leaner and meaner in its operations, it expects the company’s profits to rise in the future. Therefore, in anticipation it rewards the company by increasing its stock prices. FASB is rightfully concerned in limiting such behavior of various companies. The reason is that several times in practice, restructuring charges have had nothing to do with downsizing or rightsizing. No costs are reduced in the future and no real benefits may accrue to the company in the coming years. Since there are no guidelines, all kinds of expenses incurred by the companies are being classified under this broad category of “restructuring charges,” which the market has come to perceive as a favorable charge.

ID9–9 a. The problem with using current costs is trying to determine what the current cost is. That is, how does one determine the current cost of a specialized piece of manufacturing equipment or the current cost of an office building in a slow-moving real estate market? This difficulty in determining current costs gives managers leeway to manipulate the amounts reported in the financial statements. If managers are given the opportunity to manipulate the financial statements through subjective current costs, then financial statement users will be wary of placing any reliance on the financial statements. Thus, current costs could potentially lead to the demise of financial statements. b. Historical costs are sunk costs in that they represent the cost of an asset at the time the asset was acquired; historical costs do not indicate the magnitude of cash or net assets that an asset will generate in the future. Since sunk costs are irrelevant for decision-making purposes, historical costs are not relevant for decision-making purposes. Alternatively, current costs provide a measure of the value of an asset today. For example, the amount reported for Cost of Goods Sold and Depreciation Expense under current cost represents the current values of the inventory sold during the period and of the fixed assets "consumed" during the accounting period, respectively. These values essentially represent what it would cost the company to replace the inventory it sold and the fixed assets it "consumed." In addition, the amounts reported on the balance sheet for inventory and fixed assets essentially represent what it would cost the company to replace its inventory and fixed assets, which is essentially the same value as what the company would realize if it sold the inventory and fixed assets. Thus, current cost information would be very relevant for decision-making purposes because it provides information about cash inflows the company could generate and about cash outflows the company is likely to make. In short, current cost information is very relevant for decision-making purposes. c.

The argument comes down to reliability versus relevancy. Current cost information is more relevant than historical cost information, but it is considerably more difficult to objectively determine current costs than it is to determine historical costs. If individual financial statement users were able to dictate the valuation basis to be used in preparing financial statements, each individual would be able to determine his or her personal decision on the tradeoff between reliability and relevancy. However, financial statements are intended for general use, which means that the same financial statements will be used by a variety of people. Historically, reliability has been given more importance than relevancy because (1) relevant information is not very useful if you are not sure you can rely on the information and (2) managers and auditors are legally liable to financial statement users. That is, they are uneasy about providing information that might be subjective because it could greatly increase their legal exposure.

ID9–10


a. Asset write-downs allow Kellogg to manage earnings by reducing depreciation expenses in future periods. If Kellogg has a good quarter and decides to write down an asset this lowers the book value of the asset and thereby reduces the amount of depreciation expense that will be incurred in future periods. b. The accounting profession in general tends to prefer conservative accounting practices. By carrying the value of assets at a lower value, the auditors reduce their risk that if something goes wrong with the company that they will be sued. So if management makes estimates that reduce the value of assets, the auditors will be less likely to object. c.

The FASB has come out against this policy of “taking a bath” by companies when they have had a really bad quarter to begin with. Some companies will then go ahead and write down assets so that in future periods the amount of depreciation will be reduced thereby improving reported net income. While the write down of assets may be conservative, this approach violates the matching principle. The appropriate costs are not being matched with the related revenues in future periods.

ID9–11 a. The write-off of an outdated technology system would reduce assets and equity; equity is reduced because of the write-off expense, which reduces Retained Earnings through lower profits. b. The most likely factor in determining that a system is overvalued is the introduction of new technology products in the market that better meet the company’s needs. The old system, still carried on the balance sheet, is no longer as valuable because of the technological updates of the new systems. c.

Management could decide to take the write-off expense in a year where earnings are otherwise very healthy, eliminating the need to take the expense in future years when earnings are less robust. A management team could lower current earnings (and thus future earnings expectations) by taking the write-off charge in the current period.

ID9–12 According to U.S. GAAP, long-lived assets are recorded at original cost less accumulated depreciation. If the market value of the asset permanently falls below the balance sheet carrying value, an impairment charge must be recorded, and cannot be reversed in later periods, even if the value of the asset recovers. Under IFRS, companies can either follow the U.S. GAAP method, or they can periodically revalue their long-lived assets to fair market value, recognizing not only impairments but also recoveries of previously impaired assets and increases in asset values. Effectively, U.S. GAAP follows the more conservative “lower-of-costor market” principle, where asset values may be marked down but may never be marked up. IFRS, on the other hand, gives companies the option to value assets according to ever-changing market values, where both market value increases and decreases are recorded. Under U.S. GAAP, development costs must be expensed, while IFRS gives companies the ability, in certain circumstances, to capitalized development costs and amortize those costs over future accounting periods. U.S. GAAP requires immediate expensing, while IFRS can allow costs to be allocated to future periods.


ID9–13 a. Property, plant and equipment make up 14.8% ($1,957.7/$13,249.6) of total assets. Other long-lived assets make up 11.8% ($1,557.9/$13,249.6) of total assets. b. According to Note 3, Machinery and Equipment is the largest category within property, plant and equipment. c.

Depreciation expense (from the Statement of Cash Flow) is 1.7% ($335.0/$19,176.1) of Net Revenue. Because depreciation is a non-cash expense, it is added back in the Statement of Cash Flow in the calculation of cash from operating activities.

d. According to Note 1, Nike depreciates its assets using the straight-line method. The company uses 2 to 40 years for buildings and leasehold improvements, 2 to 15 years for machinery and equipment, and 3 to 10 years for computer software. e. The company’s largest intangible asset is Trademarks. f.

According to Note 1, Nike evaluates assets for impairment whenever business circumstances or events indicate the carrying value of assets might not be recoverable. If the evaluation determines impairment, the asset is written down to its estimated fair value.

g. The acquisition of Umbro and the subsequent determination that the carrying value of the acquisition was greater than the fair value of the acquired company led to the large impairment charge. Impairment expenses appear on both the income statement and the statement of cash flow because they are non-cash expenses (and therefore are added back to earnings in the calculation of cash from operating activities).

h. Nike spent $455.7 milliion for capital expenditures. The same year the company received $32 million in cash from the disposal of property, plant and equipment.


CHAPTER 10 INTRODUCTION TO LIABILITIES: ECONOMIC CONSEQUENCES, CURRENT LIABILITIES, AND CONTINGENCIES BRIEF EXERCISES BE10–1 a. Dividends declared during a year and the actual cash paid for dividends during the year may be different because dividends declared includes dividends that are accrued but not paid as of year end. The cash paid for dividends will include all payments that relate to dividends during the year, regardless of when those dividends were recognized in the financial statements. b. Dividend payable is a short term liability because it represents dividends that will have to be paid within the next 12 months as of the balance sheet date. c.

Dividend payable – 12/31/07 + Dividends declared – 2008 – Dividends paid during 2008 Dividend payable – 12/31/08

$ 831.1 million 3,250.4 million (3,278.5) million $ 803.0 million

BE10–2 a. Calculation of inventory purchases during 2008: Inventory @ 12/31/07 $ 6,780 million + Purchases - 2008 x – Inventory @ 12/31/08 (6,705) million Cost of Goods Sold – 2008 $ 44,157 million x = $44,082 million ............................................................................. Inventory (+A) .................................................................................................. Accounts Payable (+L)......................................................................... Record inventory purchases made during 2008. b. Calculation of payments made to suppliers during 2008: Accounts Payable @ 12/31/2007 + 2008 Inventory Purchases – Accounts Payable @ 12/31/2008 Payments made to suppliers during 2008

44,082

$ 6,721 million 44,082 million (6,337) million $44,466 million

Accounts Payable (-L) ................................................................................ Cash (-A) ............................................................................................. Record cash paid to suppliers during 2008.

1

44,082

44,466 44,466


BE10–3 a. The accrual for litigation increased the liabilities and reduced stockholders’ equity (because of lowered profits due to the environmental expense). b. Monsanto is actually following a couple of accounting principles with this expense. By recording contingency losses, the company is following the conservatism principle and the matching principle (taking the expense in the time period in which related revenues were recorded). c.

Environmental Liabilities | $ 272 25 | 15 | __ _|____ | $262

(Note: if $25 million was spent to reduce the liability in 2009, Monsanto would have accrued an additional $15 million in environmental remediation expenses in 2009 to show a decrease in the liability to the year-end value of $262 million.)

BE10–4 a. The decrease in inventory indicates that Target sold more inventory than it purchased. Therefore, the inventory purchases were COGS – change in inventory ($44,157 – 77 = $44,080). If inventory purchases were made on account, then the $44,080 was credited to accounts payable during the year. If accounts payable decreased over the course of the year, Target must have paid more to suppliers than it purchased from them (reducing previous payables). Therefore, the cash paid to suppliers was Purchases + change in accounts payable ($44,080 + 389 = $44,469). b. The increase in Accounts Receivable and the decrease in Accounts Payable both represented “uses” of cash and therefore decreased Target’s cash flows. The decrease in Inventory was a “source” of cash and increased the company’s cash flows.

EXERCISES E10–1 a. A 12% annual discount rate is equivalent to a daily discount rate of .03288%. Present value = $40,000  [(1 + .0003288) -10] = $40,000  .99672 = $39,868.80 b. Although present value probably provides a better economic measure of a company's liabilities than is provided by the liabilities' face value, accounts payable are carried on the balance sheet at their face value. This is due to both materiality and cost/benefit considerations. Because payables are usually paid off relatively quickly, the difference between the present value of a payable and its face value will be quite small (in the problem, the difference is only $131.20). The differences are so small that financial statement users' decisions would be the same whether payables are reported at their present value or at their face value. Thus, the difference is immaterial. In addition, the cost of amortizing the discounts


(i.e., the excess of the face value of a payable over its present value) would exceed the benefits because the discount amount is so small.


E10–2 a. Current Ratio

= Current Assets ÷ Current Liabilities = ($130,000 + $50,000) ÷ $80,000 = 2.25

Current assets cannot fall below 1.5 times current liabilities. Therefore, dividing current assets by 1.5 indicates the maximum level that Darrington and Darling can allow current liabilities to grow to without violating the debt covenant. So current liabilities can grow to $120,000 ($180,000 ÷ 1.5). b. Current Ratio

= ($130,000 + $20,000) ÷ $80,000 = 1.875

Using the same logic as in part (a), the current liabilities can grow to $100,000 ($150,000 ÷ 1.5). c.

Current Ratio

= ($130,000 + $0) ÷ $80,000 = 1.625

Using the same logic as in part (a), the current liabilities can grow to $86,666.67 ($130,000 ÷ 1.5).

E10–3 Reported amounts Adjustments: Rent Wages Interest Adjusted amounts

Current Assets

Current Liabilities

Net Income

$ 24,000

$ 15,000

$ 7,500

1,008 5,000a $ 25,008

50b $ 20,050

1,008 (5,000)a (50)b $ 3,458

a $5,000 = ($7,500 ÷ 15 days per pay period)  10 days left in December b $50 = $10,000  12%  15/360

Current Ratio

= Current Assets ÷ Current Liabilities = $25,008 ÷ $20,050 = 1.25

Net Income = $3,458

E10–4 a. Cash (+A) ................................................................................................... Discount on Notes Payable (–L) ................................................................ Notes Payable (+L) .............................................................................. Borrowed money from First Bank and Trust.

19,250 750 20,000


E10–4

Concluded

b. Interest Expense (E, –SE) ........................................................................... Discount on Notes Payable (+L) .......................................................... Incurred interest expense.

250* 250

* $250 = $750 discount  30/90 days Spencer Department Store should disclose the note payable on its December 31 balance sheet as follows:

c.

Notes payable............................................................................................ Less: Discount on notes payable ............................................................... $19,500 Interest = Principal  Rate  Time $750 = $19,250  Rate  90/360 days Rate = 15.58% (rounded)

$20,000 500

d. The actual, or effective, interest rate is determined by comparing the cash payments for interest to the actual amount of cash received. The cash payment for interest equals the interest rate stated in the note times the face value of the note. Thus, the only way for the stated and effective interest rates to be the same is for the amount of cash received to be the same as the note's face value. In this case, however, the actual amount of cash received is $750 less than the note's face value of $20,000. The $750 essentially represents prepaid interest.

E10–5 a. Lacey Treetoppers has to make a total of fifteen payments of $20,000 each. As of December 31, 2011, the company has made payments for 2007, 2008, 2009, 2010, and 2011. Consequently, Lacey Treetoppers has a total of ten payments remaining. The remaining liability of $200,000 must be allocated on the balance sheet between the amounts that will mature within the time frame of current liabilities and the amounts that will not mature within that time. As of December 31, 2011, only $20,000 will become due within the next year. This $20,000 should be classified on the balance sheet under current liabilities as Current Maturities of Long-Term Debt. The remaining $180,000 should be classified on the balance sheet under long-term debt. b. Current liabilities are defined as those liabilities that will be settled through the use of current assets or through the creation of other current liabilities. If a liability is to be settled through the use of noncurrent assets or through long-term refinancing, then the liability should be classified as long-term debt. In this case, Lacey Treetoppers has basically two options in trying to avoid classifying the upcoming $20,000 installment payment as a current liability. The first option is to negotiate with the creditor to refinance the payment on a long-term basis. The second option is to intend to pay off the $20,000 using noncurrent assets. For example, the company could create a sinking fund to service the entire obligation. Since the sinking fund would be classified as a long-term investment, the corresponding liability would also be classified as long-term. Lacey Treetoppers may also be able to avoid violating its debt covenants by increasing its current assets to offset the increase in current liabilities. Assume that Lacey Treetoppers intends to pay off the upcoming installment payment by selling off investments in marketable securities classified as longterm. Since these securities would be sold off within one year, the marketable securities should be reclassified as a current asset. Consequently, current assets would be increased to offset the increase in current liabilities.


E10–6 a. Cash (+A) ................................................................................................... Deferred Revenue (+L) ....................................................................... Sold gift certificates.

88,000

b. Deferred Revenue (–L) .............................................................................. Sales (R, +SE)....................................................................................... Made sales.

52,000

Cost of Goods Sold (E, –SE) ....................................................................... Inventory (–A) ..................................................................................... Cost of inventory sold.

32,000

c.

2012 Ending balance

88,000

52,000

32,000

= 2012 Beginning balance + Gift certificates sold during 2012 – Gift certificates redeemed during 2012 = ($88,000 – $52,000) + $60,000 – $80,000 = $16,000

E10–7 a. Calculation of payments made to suppliers during 2008: Step One: Inventory – 2007 + Purchases - 2008 – COGS - 2008 = Inventory-2009

$ 1,616,981 thousand x (4,335,104) thousand $ 1,642,339 thousand

x = 4,360,462 thousand Step Two: Accounts Payable – 2007 + Purchases on account - 2008 – Cash Paid to suppliers -2008 = Accounts Payable-2008

$ 570,605 thousand 4,360,462 thousand (x) thousand $ 514,734 thousand

Cash Paid to suppliers – 2008 = $4,416,333 b. If the Statement of Cash Flows is structured in the direct format, the Cash Paid to Suppliers number would be disclosed in the Operating Section. If the Statement is structured in the indirect format, this dollar amount would be disclosed on the statement of cash flows, but it is broken into three different areas. Net income, which includes the cost of goods sold, is the first line item on the statement of cash flows. The other two items on the statement of cash flows that reflect the balance of this amount are the change in accounts payable and the change in inventory.


E10–8 a. Since Zeus Power brought the lawsuit, Zeus Power is facing a gain contingency. Gain contingencies are ordinarily not disclosed in the financial statements or in the footnotes to the financial statements due to conservatism. However, if it is probable that Zeus Power will realize the gain contingency, then it may be acceptable to disclose the contingency in the footnotes to the financial statements to avoid misleading financial statement users.

b. Since Regional Supply is the defendant in the lawsuit, Regional Supply is facing a loss contingency. The appropriate accounting treatment for this lawsuit by Regional Supply depends upon (1) whether an adverse outcome to the lawsuit (from Regional Supply's perspective) is remote, reasonably possible, or probable and (2) whether the amount of the loss, given an adverse outcome, can be reasonably estimated and (3) if it is material to Regional Supply. To record an economic event, a company must be able to quantify the dollar amount of the event. If the company cannot quantify the dollar amount of the event, it is impossible for the company to prepare a journal entry. Thus, if Regional Supply cannot reasonably estimate the amount of the loss, it cannot accrue a contingent liability. At best, Regional Supply could disclose the loss contingency in the footnotes to its financial statements. Alternatively, assume that Regional Supply can reasonably estimate how much it would lose if it lost the lawsuit. If the probability that it will lose the lawsuit is remote, Regional Supply can ignore the lawsuit for financial reporting purposes. If it is reasonably possible that Regional Supply will lose the lawsuit, it should disclose the lawsuit, and the amount of the potential loss, in the footnotes to its financial statements. Finally, if it is probable that Regional Supply will lose the lawsuit, then it should accrue a contingency liability (i.e., it should prepare a journal entry in which it recognizes a loss and related liability for the lawsuit). In this particular case, it appears that the amount of the loss can be reasonably estimated. Regional Supply must decide whether "a greater that 50% chance" of losing the lawsuit means it is reasonably possible or probable that the company will lose the lawsuit. c.

Zeus Power and Regional Supply would account for this lawsuit differently due to conservatism. Under conservatism, the basic rule is "if in doubt on how to record or report an economic event, put your worst foot forward." That is, record or report the event in the way that is least favorable to the company. Since doubt exists as to who will win the lawsuit, this event qualifies for conservatism. For Zeus Power, disclosing or recording the potential gain would put it in a better position than not disclosing or recording it. Consequently, Zeus Power should probably ignore this event for financial reporting purposes. For Regional Supply, the opposite is true. Disclosing or accruing the lawsuit presents the event in the least favorable way.

E10–9 a. The owners of a corporation (i.e., the stockholders) usually want the managers to make operating, investing, financing, and reporting decisions that will maximize the owners' wealth. However, managers have their own goals, and the stockholders of a company typically are unable to observe the day-to-day activities of the company's managers. Consequently, the owners are unsure that the managers are taking actions in the best interests of the owners. If the owners and managers have similar goals, then the managers would be expected to take actions that the owners would approve. Since maximizing net income is positively associated with maximizing stockholders' wealth, one way to help align managers' goals with the owners' goals is to make maximizing net income desirable to managers. If the managers


were to receive bonuses linked to net income, then they would be expected to try to increase net income. So a company would institute a bonus plan to try to align the goals of managers and owners. If Jordan Brothers earned net income of $300,000, then the amount allocated to the bonus pool would be computed as follows. ($300,000 – $200,000)  10% = $10,000 The appropriate journal entry would be: Bonus Expense (E, –SE).............................................................................. Bonus Liability (+L).............................................................................. Incurred, but did not pay, management bonuses.

10,000 10,000

b. The managers of the company are not eligible for a bonus unless net income exceeds $200,000. So if net income is only $180,000, nothing would be allocated to the bonus pool. Assume that Jordan Brothers will eventually lose this lawsuit and that the company will have to pay the entire $60,000. If Jordan Brothers accrues the loss now, net income will be reduced by $60,000, and no additional losses will have to be recorded when the lawsuit is actually settled. Since nothing will be allocated to the bonus pool this year anyway, accruing the loss this year does not affect this year's bonus. Since no additional losses will have to be recorded in future years, accruing the loss this year will not affect future bonuses. If, however, Jordan Brothers simply discloses the loss now, the company will have to record the $60,000 loss when the lawsuit is settled. Since nothing will be allocated to the bonus pool this year anyway, simply disclosing the loss this year does not affect this year's bonus. However, future years' bonuses could be decreased. Assume that in the year the lawsuit is settled, net income, before considering the loss, exceeds $200,000. Any item that reduces net income also reduces the allocation to the bonus pool. Consequently, simply disclosing the lawsuit could have economic consequences to the managers. By accruing the loss this year, the managers are increasing the probability that they will receive a bonus in a future year.

E10–10 a. (1)

(2)

Cash (+A) ........................................................................................ Sales (R, +SE) ........................................................................... Sold outboard engines.

50,000

Warranty Expense (E, –SE) ............................................................ Contingent Warranty Liability (+L) .......................................... Estimated warranty expense.

4,000*

50,000

4,000

* $4,000 = 200 engines  $20 estimated warranty cost per engine (3)

2011 Contingent Warranty Liability (–L) ................................................ Cash (–A) ................................................................................. Made repairs under warranty. 2012

1,400 1,400


Contingent Warranty Liability (–L) ................................................ Cash (–A) ................................................................................. Made repairs under warranty.

b. _2011 Revenue Warranty expense Net income (loss)

Contingency Basis _2012

$50,000 (4,000) $46,000

_2011_

$0 0 $0

2,600 2,600

Cash Basis _2012_ $50,000 (1,400) $48,600

$

0 (2,600) $(2,600)

E10–11 a. Southwest Airlines does not recognize revenue when the cash is received because the revenue has not been earned yet. It is not earned until the passenger takes the flight. There is always the possibility that the passenger will not take the originally scheduled flight and get a refund of their money. b. Air traffic liability is a current liability. Airlines do not sell tickets for flights that are scheduled for more than 12 months in the future. The liabilites are therefore shown as current. c.Calculation of cash receipts from passengers during 2008: Air traffic liabilities @ 2007 $ 931 million + cash receipts – 2008 x – revenue recognized – 2008 (10,5) billion Air traffic liabilities @ 2008 $ 963 million x = $10.532 billion = cash receipts – 2008

E10–12 a. Year 1 Pension Expense (E, –SE) ........................................................................... Cash (–A)............................................................................................. Made pension contribution. Year 2 Pension Expense (E, –SE) ........................................................................... Cash (–A)............................................................................................. Made pension contribution. Year 3 Pension Expense (E, –SE) ........................................................................... Cash (–A)............................................................................................. Made pension contribution.

16,000 16,000

16,000 16,000

16,000 16,000

b. The amount that Seasaw Seasons should report for its pension liability equals the difference between the amount necessary to fund the benefits and the amount already paid into the pension plan. In this case, the liability would be $10,000 ($58,000 – $48,000).


E10–13 a. Income Tax Expense (E, –SE) ..................................................................... Income Tax Liability (+L) ..................................................................... Deferred Income Taxes (+L)................................................................ Incurred, but did not pay, income taxes.

28,000a 22,750b 5,250

a $28,000 = $80,000  35% b $22,750 = $65,000  35%

b. Income Tax Expense (E, –SE) ..................................................................... Income Tax Liability (+L) ..................................................................... Deferred Income Taxes (+L)................................................................ Incurred, but did not pay, income taxes.

24,000a 19,500b 4,500

a $24,000 = $80,000  30% b $19,500 = $65,000  30%

c.

Generally accepted accounting principles differ from the Internal Revenue Code. The two sets of accounting principles/procedures usually yield different income amounts. However, over the life of a company, total net income should, for all practical purposes, be the same as total taxable income. So the total tax expense recognized over the life of the entity should equal the total tax liability incurred over the life of the entity. If in a particular year the tax expense exceeds the tax liability, then in a subsequent year the tax liability must exceed the tax expense to balance out. So the balance in Deferred Income Taxes represents the amount that the entity will have to pay in the future to balance the timing differences between its tax expense and its tax liability. The balance in Deferred Income Taxes is directly linked to the tax rate. The lower the rate, the smaller the difference between the tax expense and tax liability. Consequently, there is a smaller amount that must be balanced out over the life of the entity.

E10–14 a. Conservatism Ratio

=

Reported Income Before Taxes ÷ Taxable Income

= =

$68,000 ÷ $50,000* 1.36

Income Tax Expense .................................................................................. Deferred Income Tax ($9,700 – $8,300) ............................................. Income Tax Payable (Plug).................................................................. *

$19,000 ÷ 38% = $50,000

20,400 1,400 19,000


A conservatism ratio greater than 1.0 indicates the extent to which reported income before taxes exceeds the taxable income. It seems that Busytown Industries does use some aggressive accounting policies. However, a meaningful conclusion can be drawn only with an industry-wide comparison. b. The conservatism ratio provides a reasonably good measure of the extent to which a company uses aggressive versus conservative accounting policies. The numerator of the ratio is income reported to the stockholders, which management is motivated to inflate. The denominator is the income on which the company actually would pay income taxes, which management naturally has the motivation to deflate. Thus, a ratio of the two provides a measure of the divergence between the reported income before taxes and the taxable income. The following table lists a few accounting policies and their effect on the Conservatism Ratio.

Accounting Policy 1. Straight-line method of depreciation X 2. Double-declining or any accelerated depreciation method 3. Accounting estimates, i.e., bad debts, warranties etc.

Conservatism Ratio Increase Decrease

X X

X

E10–15 a. Conservatism Ratio

=

Reported Income Before Taxes ÷ Taxable Income

= =

$145,500 ÷ $162,059* .898

Income Tax Expense .................................................................................. Deferred Income Tax ($19,400 – $18,300)................................................ Income Tax Liability (Plug) .................................................................. *

54,000 1,100 55,100

$55,100 ÷ 34% = $162,059

Since the conservatism ratio is less than 1, it appears that the company is using income-deflating policies on its income statement. b. The conservatism ratio provides a reasonably good measure of the extent to which a company uses aggressive versus conservative accounting policies. The numerator of the ratio is income reported to the stockholders, which management is motivated to inflate. The denominator is the income on which the company actually would pay income taxes, which management naturally has the motivation to deflate. Thus, a ratio of the two provides a measure of the divergence between the reported income before taxes and the taxable income. The following table lists a few accounting policies and their effect on the Conservatism Ratio.

Accounting Policy 1. Straight-line method of depreciation X

Conservatism Ratio Increase Decrease


2. Double-declining or any accelerated depreciation method 3. LIFO inventory valuation 4. FIFO inventory valuation 5. Accounting estimates, i.e., bad debts, warranties etc.

X X X X

X

PROBLEMS P10–1 a., b., and c.

Item (1) (2) (3) (4) (5) (6) (7) (8) Total

Classification Current Long-Term X X X X X

Amount Current Long-Term $170,000

X X

75,000 8,000 25,000 15,000

X X

$ 60,000 425,000

125,000 50,000 $343,000

$610,000

P10–2 The balance sheet of Linton immediately after the bank loan and purchase of equipment would be: Assets Current assets Noncurrent assets

Total assets

Liabilities and Stockholders' Equity $

$ 260,000a Current liabilities 1,860,000b Long-term liabilities

$2,120,000

Capital stock Retained earnings Total liabilities and stockholders' equity

125,000c 775,000d 1,000,000 220,000

$2,120,000

a $260,000 = $120,000 + $140,000 in cash. b $1,860,000 = $1,500,000 + $360,000 in purchased equipment. c

$125,000 = $100,000 + $25,000 in current maturities of the new note.

d $775,000 = $300,000 + $475,000 in long-term maturities of the new note.

The current ratio after recording the bank loan and the purchase of the equipment would be: Current Ratio

= Current Assets ÷ Current Liabilities = $260,000 ÷ $125,000 = 2.08


Declaring a dividend would increase current liabilities. Current assets cannot fall below 2 x current liabilities if Linton is to avoid violating its debt covenant. Since current assets are currently $260,000, Linton could declare and pay a dividend of $5,000. This dividend would increase current liabilities to $130,000 and reduce retained earnings to $215,000.

P10–3 a. (1)

Bad Debt Expense (E, –SE) ............................................................. Allowance for Doubtful Accounts (–A) ................................... Estimated bad debts.

* $1,000

(2)

(3)

(4)

(5)

(6)

1,000* 1,000

= ($50,000 Accounts receivable balance  6% Estimated uncollectible percentage) – $2,000 Balance in the Allowance account

Warranty Expense (E, –SE) ............................................................ Warranty Liabilities (+L) .......................................................... Estimated warranty expense.

7,000

Unearned Revenues (–L) ............................................................... Revenue (R, +SE) ..................................................................... Earned revenue from advance collections.

10,000

Other Current Liabilities (–L) ......................................................... Long-Term Liabilities (+L) ........................................................ Reclassified liabilities.

5,000

Income Tax Expense (E, –SE) ......................................................... Income Tax Payable (+L) ......................................................... Incurred income tax expense.

3,000

Loss on Lawsuit (Lo, –SE) ............................................................... Short-Term Contingent Liability (+L)....................................... Incurred potential loss on a lawsuit.

10,000*

7,000

10,000

5,000

3,000

10,000

* Assumes that 60% is considered probable. If 60% was considered only reasonably possible, the $10,000 would not be accrued, but would, instead, be disclosed in a footnote. b. After considering the journal entries in part (a), except for entry (6), current assets and current liabilities would be as follows. Current Assets

= $40,000 + $50,000 – $3,000 + $52,000 = $139,000

Current Liabilities =

$30,000 + $15,000 + $12,000 + $5,000 + $3,000


= Current Ratio

$65,000

= Current Assets ÷ Current Liabilities = $139,000 ÷ $65,000 = 2.138


P10–3 c.

Continued

After considering the journal entries in part (a), including entry (6), current assets and current liabilities would be as follows. Current Assets

= $40,000 + $50,000 – $3,000 + $52,000 = $139,000

Current Liabilities = $30,000 + $15,000 + $12,000 + $5,000 + $3,000 + $10,000 = $75,000 Current Ratio

= $139,000 ÷ $75,000 = 1.853

d. The company's auditors must first consider the directives of the FASB. SFAS Statement No. 5, "Accounting for Contingencies," addresses contingent liabilities. This pronouncement states that if a contingent liability is both reasonably estimable and probable, then the liability must be accrued. But if the contingent liability is reasonably estimable and only reasonably possible, or if it is not reasonably estimable, then the liability only has to be disclosed in a footnote. Finally, if a contingency is remotely likely, then the contingent liability does not have to be disclosed. Since SFAS Statement No. 5 does not provide a definition of probable, reasonably possible, or remote, judgment must be used in converting a probability estimate to one of these categories. One factor that may greatly influence the auditor's judgment is his or her legal liability regarding each classification. Assume that the company loses the lawsuit. If the liability was considered remotely likely, and hence not disclosed, the auditor would be legally liable to financial statement users. If the liability was considered reasonably possible, and hence disclosed in the footnotes, the auditor could potentially still face legal action from the financial statement users. The users could argue that the auditor should have required the lawsuit to be accrued. Finally, if the liability was considered probable, and hence accrued, the financial statement users were provided with the correct information and the auditor would not face any legal liability. Assume that the company wins the lawsuit. If the liability was considered remotely likely, the financial statement users were provided with the correct information. The managers of the company would also be happy because the company was not made to appear worse off than it actually was. If the liability was considered reasonably possible and only disclosed in the footnotes, it is doubtful that many financial statement users incurred out-of-pocket losses due to the disclosure. Some potential investors or creditors might have foregone transacting with the company, but these people only incurred opportunity losses. People cannot sue for opportunity losses. The managers might be somewhat upset that the potential loss was disclosed in the footnotes. This disclosure might make it more difficult to attract capital. However, since the contingent liability was only disclosed in the footnotes and was not accrued, the liability would not affect any of the manager's contracts such as debt covenants or bonus contracts. Finally, if the contingent liability were accrued, the auditor would probably not face any legal liability because the auditor took the most conservative action. However, the managers would probably be upset. Since the contingent liability was accrued, liabilities would be increased, and the company would have to report a loss on the income statement. Accruing the liability could decrease the manager's bonus and could also place the company in default on its debt covenants.


P10–3

Concluded

The auditor faces two conflicting interests. The auditor must consider his or her legal liability to financial statement users. On the other hand, the manager pays the audit fees. If the auditor demands that the contingent liability be accrued, the manager may fire the auditor. The auditor must trade off these two conflicting interests in trying to decide whether or not to accrue a contingent liability. In most cases, the cost of legal liability will probably be greater than lost audit fees and damage to the auditor's reputation. In this particular case, a 60% probability of losing the lawsuit would probably be considered either reasonably possible or probable. So at the minimum, the auditor should require disclosure of the contingent liability in the footnotes.

P10–4 a. From an accounting perspective, the Floor Wax Shop must consider generally accepted accounting principles. The company would find authoritative guidance in SFAS Statement No. 5, "Accounting for Contingencies." According to this statement, the two factors that the Floor Wax Shop must consider are (1) whether the amount of the loss can be reasonably estimated and (2) the probability that the Floor Wax Shop will eventually incur the loss. From an economic perspective, the Floor Wax Shop must consider the costs and benefits of the different ways to report this lawsuit. If the lawsuit is not disclosed or accrued and the company subsequently loses the lawsuit, financial statement users could sue the company for any losses they incurred from relying on the company's financial statements. If the Floor Wax Shop discloses the lawsuit but does not accrue a loss, then it is less likely that any financial statement users would be able to successfully sue the company. Also, any contracts in place, such as debt covenants or bonuses, would probably not be affected by a footnote disclosure. However, the footnote disclosure may make it more difficult for the company to attract new capital. Some suppliers may also be wary of extending credit, thereby forcing the Floor Wax Shop to pay cash for purchases or sign interest-bearing notes. In either case, the Floor Wax Shop could encounter cash flow problems. If the Floor Wax Shop accrues the lawsuit, the company would report a loss on its income statement and an associated liability on its balance sheet. In this case it would be virtually impossible for a financial statement user to successfully sue the Floor Wax Shop for providing misleading or incorrect information. However, in addition to the problems discussed associated with disclosing the lawsuit, accruing a loss for the lawsuit could affect existing contracts. Accruing a loss might also be considered an admission of guilt to the court, which could cause accruing the loss to become a self-fulfilling prophecy.


P10–4

Concluded

b. The Floor Wax Shop should accrue the lawsuit for two reasons. First, the contingent loss meets the requirements set forth in SFAS Statement No. 5, "Accounting for Contingencies," for accruing a contingent loss. That is, it is both probable that the Floor Wax Shop will lose the lawsuit, and the amount of the loss can be reasonably estimated. Second, accruing a loss minimizes my legal exposure as the company's auditor. c.

The Floor Wax Shop would have made the following entry on December 31, 2011 to accrue the contingent loss. Contingent Loss on Lawsuit (Lo, –SE) ........................................................ Contingent Liability (+L) ...................................................................... Accrued contingent loss.

742,000 742,000

The amount recorded for the contingent loss and liability on December 31, 2011 was simply an estimate. That is, $742,000 was the company's best estimate as of that date about the amount the company would eventually lose. On August 12, 2012, when the lawsuit was settled, the Floor Wax Shop was able to revise its estimate. Since changes in estimates are accounted for prospectively, a prior period adjustment is not necessary. Instead, the Floor Wax Shop should make the following entry on August 12, 2012. Contingent Liability (–L) ............................................................................ Cash (–A)............................................................................................. Recovery of a Contingent Loss (R, +SE) .............................................. Settled lawsuit.

742,000 690,000 52,000

P10–5 a. Cash (+A) ................................................................................................... Sales (R, +SE)....................................................................................... Sold cars. b. 5/30

332,500

Contingent Warranty Liability (–L) ........................................... Cash (–A)............................................................................ Parts (–A) ........................................................................... Made repairs under warranty.

3,000

Contingent Warranty Liability (–L) ........................................... Cash (–A)............................................................................ Parts (–A) ........................................................................... Made repairs under warranty.

5,000

Contingent Warranty Liability (–L) ........................................... Cash (–A)............................................................................ Parts (–A) ........................................................................... Made repairs under warranty.

6,000

Warranty Expense (E, –SE) ........................................................................ Contingent Warranty Liability (+L) ..................................................... Estimated warranty expense.

26,600*

9/2

11/15

c.

332,500

1,200 1,800

2,000 3,000

2,400 3,600

26,600


* 26,600 = 35 cars sold  $760 estimated warranty cost per car

P10–5

Concluded

d. Ending Balance

= Beginning Balance + Warranty Expense for the Year – Cost of Under Warranty = $3,500 + $26,600 [from part (c)] – ($3,000 + $5,000 + $6,000) = $16,100

Repairs

e. Under the matching principle, all costs that are incurred in generating revenue should be matched against those revenues. The matching principle does not distinguish between costs incurred before or after the point of sale. Consequently, both presale and post-sale costs should be matched against the associated revenue. The problem with matching post-sale costs against the associated revenue is that the costs have not yet been incurred by the time the revenue is earned. Thus, the only way to be able to match post-sale costs against the associated revenue at the time the revenue is earned is to estimate the value of the post-sale costs. Since post-sale costs are an example of a contingency, those post-sale costs that are both reasonably estimable and probable are recognized at the time the revenue is recognized. In the case of warranties, a company can usually use company- or industry-specific data to estimate the average cost that will subsequently be incurred to make repairs under warranty. However, if a company expects to incur a material amount of repairs under warranty but cannot reasonably estimate the amount, it is debatable whether the company should recognize the revenue in the current period. One of the criteria for recognizing revenue under the revenue recognition principle is that post-sale costs can be reasonably estimated. If this criterion cannot be met and the amount is material, a company should probably delay recognizing the revenue.

P10–6 a.

2011 Cash (+A) ………………………………… Sales (R, +SE) ……………………….. Made sales.

40,000 ........ 56,000 40,000 ......

2012 56,000

Promotion Expense (E, –SE)……………… .......... 400a Contingent Promotion Liability (+L) …. 400 Estimated promotion cost.

560b 560

= 20,000 boxes sold during the year  Expected redemption rate of 10% ÷ 5 box tops per refund  $1.00 per refund b $560 = 28,000 boxes  10% ÷ 5 box tops per refund  $1.00 per refund 2011 2012 a Contingent Promotion Liability (–L) ................... 300 400b Cash (–A)...................................................... 300 Paid promotional refund. a $400

a $300 = 1,500 box tops redeemed ÷ 5 box tops per refund  $1.00 per refund b $400 = 2,000 box tops redeemed ÷ 5 box tops per refund  $1.00 per refund

400


P10–6

Concluded

b. Ending balance

=

Beginning balance + Promotional expense – Refund payments

2011:

= =

$0 + $400 – $300 $100

2012:

= =

$100 + $560 – $400 $260

P10–7 a. Restructuing Expense (E, –SE) ................................................................... Assets (-A) ........................................................................................... Severance Liability (+L) .......................................................................

425 364 61

b. The $364 million expense was subtracted from earnings to produce net income, but the expense did not reduce the company’s cash balances. Therefore, to convert the net income figure to the cash from operating activities number, the non-cash expense was added back. c.

Reduction in taxable earnings x tax rate = tax savings $425 million x .26 = $110.5 million

P10–8 a.

2007 Pension Expense (E, –SE) ................................. 40,000 Cash (–A).................................................... Pension Liability (+L) .................................. Funded pension.

2008 40,000 32,000 8,000

32,000 8,000

2009 Pension Expense (E, –SE) .................................. 40,000 Cash (–A)..................................................... Pension Liability (+L) ................................... Funded pension.

2010 40,000 36,000 4,000

36,000 4,000

2011 Pension Expense (E, –SE) ................................. 40,000 Cash (–A).................................................... Funded pension. b.

Pension Expense 2007 2008 2009 2010 2011

$ 40,000 40,000 40,000 40,000 40,000 $200,000

Amount Funded $ 32,000 32,000 36,000 36,000 40,000 $176,000

40,000

Pension Liability $

8,000 8,000 4,000 4,000 0 $24,000


Thus, the balance in the Pension Liability account as of December 31, 2011 is $24,000.

P10–9 a. 2007 Income Tax Expense (E, –SE) ..................................................................... Deferred Income Taxes (+L)................................................................ Income Tax Liability (+L) ..................................................................... Incurred income taxes.

30,625a 4,375 26,250b

a $30,625 = ($100,000 – $12,500 in depreciation expense)  35% b $26,250 = ($100,000 – $25,000 in depreciation expense)  35%

2008 Income Tax Expense (E, –SE) ..................................................................... Income Tax Liability (+L) ..................................................................... Incurred income taxes.

30,625* 30,625

* $30,625 = ($100,000 – $12,500 in depreciation expense)  35% 2009 Income Tax Expense (E, –SE) ..................................................................... Deferred Income Taxes (–L) ...................................................................... Income Tax Liability (+L) ..................................................................... Incurred income taxes.

30,625.00 2,187.50 32,812.50*

* $32,812.50 = ($100,000 – $6,250 in depreciation expense)  35% 2010 Income Tax Expense (E, –SE) ..................................................................... Deferred Income Taxes (–L) ...................................................................... Income Tax Liability (+L) ..................................................................... Incurred income taxes.

2007 2008 2009 2010

30,625.00 2,187.50 32,812.50

Income Tax Expense

Income Tax Liability

Change in Deferred Income Taxes

Deferred Income Tax Balance

$30,625 30,625 30,625 30,625

$26,250.00 30,625.00 32,812.50 32,812.50

$ 4,375.00 0 (2,187.50) (2,187.50)

$4,375.00 4,375.00 2,187.50 0

b. The balance in Deferred Income Taxes represents the amount a company will, theoretically, have to pay the government in the future. Amount differences, such as arise when a company uses different depreciation methods for financial reporting and tax purposes, between book and taxable income reverse themselves over time. If the government lowers the tax rate, then the tax liability in future periods when the timing differences reverse themselves will be lower than the associated tax expense recorded on the books when the older tax rate was still in effect. Thus, the amount of tax expense recorded in prior periods was "incorrect" given the new tax rates, and the company should adjust its books for the decrease in the expense. If the misstatement is considered an error, then a prior period adjustment is necessary. But if the misstatement is considered to be a change in estimate (which is the


more likely view), then the correction would be accounted for prospectively and reported as a gain in the current period. In this particular case, the gain from the change in the tax rate is $1,875.

P10–9

Continued

2007

Same as in part (a)

2008

Same as in part (a)

2009 Deferred Income Taxes (–L) ...................................................................... Gain on Deferred Income Taxes (Ga, +SE) .......................................... Adjusted for change in statutory tax rate.

1,875* 1,875

* $1,875 = Excess tax depreciation of $12,500  Decrease in tax rate of 15% Income Tax Expense (E, –SE) ..................................................................... Deferred Income Taxes (–L) ...................................................................... Income Tax Liability (+L) ..................................................................... Incurred income taxes.

17,500 1,250 18,750*

* $18,750 = ($100,000 – $6,250 in depreciation expense)  20% 2010 Income Tax Expense (E, –SE) ..................................................................... Deferred Income Taxes (–L) ...................................................................... Income Tax Liability (+L) ..................................................................... Incurred income taxes.

2007 2008 2009 2009 2010 c.

Income Tax Expense

Income Tax Liability

$30,625 30,625

$26,250 30,625

17,500 17,500

18,750 18,750

Change in Deferred Income Taxes $4,375 0 (1,875) (1,250) (1,250)

17,500 1,250 18,750

Deferred Income Tax Balance $4,375 4,375 2,500 1,250 0

2007 Same as in part (a) 2008 Income Tax Expense (E, –SE) ..................................................................... Deferred Income Taxes (+L)................................................................ Income Tax Liability (+L) ..................................................................... Incurred income taxes.

26,250a 4,375 21,875b

a $26,250 = ($100,000 – $25,000 in depreciation expense)  35% b $21,875 = ($100,000 – $37,500 in depreciation expense)  35%

2009 Income Tax Expense (E, –SE) .....................................................................

26,250.00


Deferred Income Taxes (–L) ...................................................................... Income Tax Liability (+L) ..................................................................... Incurred income taxes. * $28,437.50 = ($100,000 – $18,750 in depreciation expense)  35%

2,187.50 28,437.50*


P10–9

Concluded

2010 Income Tax Expense (E, –SE) ..................................................................... Income Tax Liability (+L) ..................................................................... Incurred income taxes.

21,875* 21,875

* $21,875 = ($100,000 – $37,500 in depreciation expense)  35% Income Tax Expense

Income Tax Liability

$30,625 26,250 26,250 21,875

$26,250.00 21,875.00 28,437.50 21,875.00

2007 2008 2009 2010

Change in Deferred Income Taxes

Deferred Income Tax Balance

$ 4,375.00 4,375.00 (2,187.50) (0)

$4,375.00 8,750.00 6,562.50 6,562.50

If a company uses different depreciation methods for calculating book and taxable income, it is possible that the Deferred Income Tax account will remain on the company's books indefinitely. This would happen if a company continues to acquire fixed assets because the deferred taxes arising from the new assets would exceed the deferred income taxes being reversed from prior accounting periods. Consequently, the balance in Deferred Income Taxes would continue to grow over time.

P10–10 Based on the information provided in this problem, we can compute the conservatism ratio of each company: The lower the ratio, the higher the earning power of the company. Conservatism Ratio

= Reported Income Before Taxes ÷ Taxable Income

Owen-Foley Company Income Tax Expense (I/S) .......................................................................... Deferred Income Tax ($18,400 – $16,600) ......................................... Income Tax Liability (Plug) .................................................................. Taxable Income

= =

Conservatism Ratio

52,000 1,800 50,200

$50,200 ÷ 36% $139,444 = =

$163,000 ÷ $139,445 1.169

Amerton Industries Income Tax Expense (I/S) .......................................................................... Deferred Income Tax ($19,800 – $18,800)................................................ Income Tax Liability (Plug) .................................................................. Taxable Income Conservatism Ratio

= =

53,500 1,000 54,500

$54,500 ÷ 36% $151,389 = $158,500 ÷ $151,389 = 1.047

Amerton Industries’ conservatism ratio is lower than Owen-Foley Company’. Therefore, it has stronger earning power than Owen-Foley.


P10–11 Based on the information provided in this problem, we can compute the conservatism ratio of each company: The lower the ratio, the more conservative the company. The higher conservatism ratio indicates that management is more aggressive with its tax policies and results in higher future tax liabilities. The lower the ratio, the less aggressive management is with tax policies. Conservatism Ratio

= Reported Income Before Taxes ÷ Taxable Income

Walgreen’s Taxable Income = $898 ÷ 36.6% = $2,454 Conservatism Ratio

= =

$3,164 ÷ $2,454 1.29

The Limited Taxable Income

Conservatism Ratio

= =

$187 ÷ 51.5% $363 = =

$453 ÷ $363 1.25

The above analysis indicates Walgreen’s is slightly more aggressive than The Limited.


ISSUES FOR DISCUSSION ID10–1 a. Under the terms of its debt covenants, FedEx's current assets must be at least as great as its current liabilities. Since Federal Express had current assets of $7,116 as of May 31, 2009, the maximum amount of liabilities Federal Express could have without violating its debt covenant would also be $7,116. Therefore, FedEx could report additional current liabilities of $2,592 ($7,116 – $4,524) as of May 31, 2009. b. There are many current liabilities where either the creation of or the amount reported for the liability is under the company's control. For example, a company's board of directors decides when to declare a dividend (which results in the current liability Dividend Payable), how much the dividend will be and when the liability will be paid. Management has discretion over the amount reported for contingent liabilities such as Contingent Warranty Liability or Contingent Promotion Liability. By altering their estimates, management can decrease the amount reported for such liabilities. Management could also control to some extent the amount reported for Unearned Revenues. That is, management could simply not take any advances from customers, or management could control when it provides the goods or services for which it collected the advances from customers. c.

Violating a debt covenant results in the borrower being in technical default on the loan. If a company is in default on a loan, the creditor could require the borrower to immediately repay the outstanding balance. Alternatively, the creditor could allow the borrower to renegotiate the loan. However, in such cases the borrower is usually forced to agree to less favorable loan terms such as a higher interest rate, providing more collateral, and so forth.

d. If FedEx purchased the aircraft for cash, its noncurrent assets would increase by $3 billion but its current assets would decrease $3 billion for the cash disbursed. With no change in its current liabilities, funding the aircraft purchase with cash would decrease the company’s current ratio from 1.57 ($7,116/$4,524) to 0.91 ([$7,116 - $3,000]/$4,524). FedEx could not fund the purchase with cash without violating its debt covenant. Alternatively, if FedEx purchased the aircraft using long-term debt, neither current assets nor current liabilities would be affected, leaving the current ratio at 1.57 (in compliance with the debt covenant).

ID10–2 a. The account Customers' Advance Payments represents cash collected from customers in advance of providing desired goods or services to the customers. When Ingersoll-Rand makes these collections, it is implicitly promising to either provide the desired goods or services or refund the customer's money. Thus, the cash collected in advance for which Ingersoll-Rand has not yet provided the desired goods or services represents a liability to Ingersoll-Rand, and it should be reported in the liability section of the balance sheet in the account Customers' Advance Payments. In addition, the cash collections made during the year should be reported in the operating activities section of the statement of cash flows.


ID10–2 Concluded b. Under the revenue recognition principle, a company should not recognize revenue until (1) the company has earned the revenue, (2) the amount of revenue earned can be objectively determined, (3) any post-sale costs can be reasonably estimated, and (4) cash collection is reasonably assured. Although the cash collection is reasonably assured when Ingersoll-Rand collects cash in advance from its customers, it is not really entitled to keep the cash at the time it makes the collections. That is, the company has to provide some goods or services before it is entitled to the cash (i.e., before it has generated some revenue). Thus, Ingersoll-Rand does not earn any revenue simply from making collections in advance from its customers. It is not until the company provides some goods or services that the company should actually recognize the revenue. Once Ingersoll-Rand provides the goods or services and recognizes the revenue, the company should then match all costs—both pre-sale and postsale costs—against the revenue in accordance with the matching principle. c.

Ingersoll-Rand should not recognize revenue for simply collecting customer advances; instead, it generates revenue when it actually ships the goods. Thus, earnings per share would increase when Ingersoll-Rand ships goods (assuming that the company sells its inventory at a profit), and it would not be affected by receiving advance payments. Collecting advance payments would increase both Ingersoll-Rand's current assets (through the cash collected) and current liabilities (through the obligation arising from the advances). The actual effect on the company's current ratio depends on what the ratio was before the company made the advance collections. If the current ratio was less than 1, collecting advance payments would increase IngersollRand's current ratio. Alternatively, if its current ratio was greater than 1, collecting advance payments would decrease Ingersoll-Rand's current ratio. Shipping the related goods would decrease IngersollRand's current liabilities, which means that the company's current ratio would increase. Since collecting advance payments would increase Ingersoll-Rand's current liabilities, its debt/equity ratio would increase. Shipping the related goods would both decrease the company's current liabilities and increase its stockholders' equity (through the increased income that would be closed into Retained Earnings). Thus, the company's debt/equity ratio would decrease when the goods are shipped.

ID10–3 a.

In 2008, 20.1% ($279/$1,387) of the reserve was classified as current; in 2007, 18.5% ($280/$1,513) of the reserve was current.

b.

The journal entries appear below:

2006: Other Operating Expense (E, -SE) Professional Liability Reserve (+L)

217 217

2007: Other Operating Expense (E, -SE) Professional Liability Reserve (+L)

163 163

2008: Other Operating Expense (E, -SE)

175


Professional Liability Reserve (+L)

175

ID10–3 CONTINUED c. Professional Liability Reserve– 12/31/2007 + Professional Liability Expense - 2008 – Cash Payments re: Liability - 2008 = Professional Liability Reserve-12/31/2008

$ 1,513 million 175 million x $ 1,387 million

x = $301 million d. Earnings could be managed in the healthcare industry with manipulation to the professional liability reserve (a liability account) by adjusting the annual charge taken for the expense. If the company were having a very successful year, beating analyst earnings estimates, management could “pad” the reserve by taking larger-than-necessary expenses so that in future years, when earnings might be below targets, management could then take lower-than-necessary expenses (and therefore show higher earnings).

ID10–4 The numerous filed and potential criminal and civil lawsuits against Philip Morris are examples of loss contingencies. SFAS Statement No. 5, "Accounting for Contingencies," provides authoritative guidance in accounting for contingencies. According to this statement, the two factors that should be considered in deciding how to account for a loss contingency are (1) whether the amount of the loss can be reasonably estimated and (2) the likelihood that the company will eventually experience the loss. If the likelihood that the company will eventually experience the loss is considered to be remote, then the company does not need to provide any information about the loss contingency in its financial statements or associated footnotes. A company should disclose a loss contingency in the footnotes to its financial statements under two conditions. The first condition is that the likelihood that the company will eventually experience the loss is considered to be reasonably possible. The second condition is that the likelihood that the company will eventually experience the loss is considered to be probable, but the amount of the loss cannot be reasonably estimated. Finally, a company should accrue the loss if it is both probable that the company will eventually experience the loss and the amount of the loss can be reasonably estimated. Accruing the loss means that the company reports a loss on its income statement and, in the case of a lawsuit, an associated liability on its balance sheet. It is probable that Philip Morris would lose a number of lawsuits associated with smoking, it was probably not possible to determine which lawsuits it would win and which ones it would lose. Philip Morris would have to estimate the probability of which lawsuits it would lose, then Philip Morris has to reasonably estimate the amount of the court settlements because of the magnitude of the damage caused by smoking. Philip Morris then estimated the overall liability and booked this as a contingent liability. By disclosing the lawsuits in its footnotes, Philip Morris has provided its financial statement users with relevant information about events that could affect the company's financial health. Thus, the financial statement users would have access to a discussion about the various lawsuits and management’s estimate as to the potential financial losses. Users can then make their own assessment as to the potential liabilities that Philip Morris may face.


ID10–5 a. There are two reasons why Lifschultz's stock could be highly valued even though the company has a negative book value. First, the stock market expects Lifschultz to win the lawsuits and collect a large settlement. The expected settlement, however, is currently not reflected in the company's financial statements. Thus, the potential settlement essentially represents an unreported asset. If Lifschultz included the amount it expected to win in the lawsuits, its net book value may be positive. In addition, generally accepted accounting principles do not allow a company to report any self-generated goodwill (such as customer loyalty or name recognition). Goodwill is often a company's largest asset. Second, book value and stock prices are based on two different measures. The price of a company's stock should, theoretically, equal the present value of the future cash flows the stock market expects an investment in that company to generate. Alternatively, a company's net book value is based largely on historical cost. A company could have a negative book value, but if the stock market thought that the company could use its assets to generate enough cash to both service its debt and disburse to its stockholders, the stock market would value that company's stock positively. b. SFAS Statement No. 5, "Accounting for Contingencies," provides authoritative guidance in accounting for contingencies. According to this statement, gain contingencies should usually be ignored for financial reporting purposes. If it is highly probable that the company will eventually realize the gain, the company could arguably disclose the gain contingency in a footnote. Alternatively, there are three potential ways to account for a loss contingency. The first way is to ignore the loss contingency. This approach would be appropriate if the likelihood of the company eventually experiencing the loss is remote. The second accounting treatment is to disclose the loss contingency in a footnote to the financial statements. This approach is appropriate (1) if the likelihood that the company will eventually experience the loss is considered to be reasonably possible or (2) if the likelihood that the company will eventually experience the loss is considered to be probable but the amount of the loss cannot be reasonably estimated. The final potential accounting treatment is to accrue a loss. That is, record a loss and associated liability. This approach is appropriate when both the likelihood of the company eventually experiencing the loss is probable and the amount of the loss can be reasonably estimated. Since Lifschultz has a gain contingency, it should not disclose this information in its financial statements or footnotes. Alternatively, the three trucking companies have loss contingencies associated with the lawsuit. The fact that the three companies only disclosed the lawsuit in their footnotes indicates that either the trucking companies thought it was only reasonably possible that they would lose the lawsuit or that they could not reasonably estimate the amount of the loss. c.

The costs of overstating and understating a company's financial position and operations is very different. Assume that a company overstates its financial position and operations in its financial statements. If the users of the financial statements use the information in the financial statements to make investment decisions and then incur a loss on their investments, the financial statement users would probably sue the company and its auditors. The financial statement users could argue that they would not have made the investment if the company had not overstated its financial position and operations. The cost of defending oneself against such lawsuits—both the monetary cost and the loss of reputation—can be very large.


Alternatively, assume that a company understates its financial position and operations in its financial statements. People may invest in the company despite its reported financial position and operations, but once the understatement becomes known, the investment should increase in value. Thus, the people who invested in the company anyway will probably not be upset and pursue legal action against the company or its auditors. Although some people might argue that they incurred a loss because they would have invested in the company if it had not under-reported its financial position and operations, such losses represent opportunity costs. Opportunity costs generally do not provide a sufficient basis for pursuing legal action against a company or its auditors. Thus, the cost to a company and its auditors of overstating the company's financial position and operations is greater than the cost of understating its financial position and operations. Reporting or accruing a gain contingency could result in an overstatement but not an understatement of a company's financial position and operations. Since reporting or accruing a loss contingency could result in an understatement but not an overstatement of the company's financial position and operations, it makes sense economically that plaintiffs and defendants would account for unsettled lawsuits differently.

ID10–6 a. A "technical default" means that the borrower has violated some term of a debt agreement, usually a debt covenant. In this particular case, Citibank reacted to Campeau Corporation's technical default by giving Campeau a few days to correct the problem. If Campeau is unable to correct the problem, Citibank appears to intend to require Campeau to immediately repay the entire outstanding loan balance. Another option Citibank could have pursued would have been to renegotiate the loan with terms more favorable to Citibank. For example, Citibank could have required more collateral from Campeau or increased the interest rate it was charging Campeau. b. Campeau's technical default and Citibank's statement that they may demand immediate full repayment of the loan indicate that Campeau may be experiencing severe cash flow problems. Suppliers would usually be unwilling to ship merchandise to a customer if there is a good chance that the customer will not be able to pay for the merchandise. If the customer is in danger of declaring bankruptcy, the suppliers' unwillingness would be even greater. This is because suppliers are usually unsecured creditors in that the goods were purchased on account, which means that the suppliers have only the customer's promise of payment; the merchandise usually does not serve as collateral. If the customer were to declare bankruptcy, the secured creditors would be paid first, and only if there was any cash left, would the unsecured creditors be paid. The suppliers' unwillingness to ship merchandise only increases Campeau's cash flow problems. Without the new spring merchandise, Campeau's stores will be unable to compete against other stores who have the new fashions, and its stores will probably run low on merchandise. This means that the company will be able to generate less cash from its operations.

ID10–7 a.

Unearned Extended Warranty Revenue |29 (2008 Beginning Balance) | |51 (2008 deferrals*) (2008 Revenue ) 51 _ |_______________________

|29 (2008 Ending Balance) *the journal entry would have been Cash or Accounts Receivable (+A) 51


Unearned Extended Warranty Revenue (+L)

51

Unearned Extended Warranty Revenue |29 (2009 Beginning Balance) | |50 (2009 deferrals*) (2009 Revenue ) 51 _ |_______________________

|28 (2009 Ending Balance) *the journal entry would have been Cash or Accounts Receivable (+A) 50

Unearned Extended Warranty Revenue (+L)

50

The Revenue is only recognized when the earning process (honoring the requests under the warranty program) is complete. In the time period when Agilent honors the warranty, it will incur expenses (to repair a system, for example) which will then be matched with the Revenue from the Extended Warranty.

b. The cash from the sale of the Extended Warranty is received on the date of the sale of the warranty; the Statement of Cash Flow will reflect an inflow of cash in the period that the cash was received. The profitability of the warranty, however, is recorded as the warranty period expires, when the Revenue from the Extended Warranty is matched with any expenses incurred to honor the warranty.

ID10–8 a.

Deferred Revenue is a liability that represents the obligation that Microsoft has to its customers for work that has been paid for but has not been performed. The liability is carried on the balance sheet until the Revenue can be recognized. When the revenue is recognized (with a credit), the liability Deferred Revenue is reduced (with a debit).

b.

From a cash flow perspective, the $1.67 billion was received in prior periods when operating system upgrades were sold to customers. The current period was simply a conversion of the liability to revenue and did not (and will not in future periods) include the receipt of cash.

c.

An analyst will be concerned with Microsoft’s profitability—and the $1.67 billion in revenue certainly factors into that figure. However, an analyst will also be concerned with Microsoft’s cash flow. When converting profits to cash flow, the analyst will deduct the portion of revenue that was not cash-related to get a better idea of cash coming into and leaving the company. In addition, since the conversion of the deferred revenue came from customers opting not to cash in coupons to upgrade software, an analyst might be concerned with the market’s interest in Microsoft’s new operating system.

ID10–9 a.

Short-term borrowings: the company issued debt securities that are due within 12 months Trade accounts payable: the company purchased inventory on account, agreeing to pay in the near future Employee compensation: the company owes its workers for labor that was performed since the last pay day; the obligation will be satisfied at the next pay day Other accrued liabilities: the company has miscellaneous obligations, such as utility bills, that have been expensed on the income statement but have yet to be paid in cash Dividend payable: the company has declared but not yet paid dividends to its stockholders Income tax payable: the company has expensed its tax obligations but has yet to settle with cash


Current portion of long-term debt: the company has long-term debt with some principal payments due in the next 12 months; that portion of the long-term debt due after the next 12 months is categorized in the noncurrent liabilities section of the balance sheet b.

If these obligations are categorized as current, then they will be satisfied from the conversion of current assets. For example, the accounts payable obligation will be paid when the company sells inventory (a current asset) and collects cash.

c.

The company might be considering converting the short-term borrowings to long-term. If long-term interest rates are low and the company recognizes that it will be a borrower for some time, it might make sense to restructure the debt to long term. Perhaps the company could address some solvency issues (working capital, the current ratio) by recasting some of its short-term debt to long-term.

d.

Return on equity is the ratio of net income to shareholders’ equity. If a company can responsibly use debt, it can improve its ROE. If the debt is used to add assets that strongly contribute to earnings, then ROE will rise. If Abbott can use its current liabilities to fund inventory that can be sold for higher margins, the increased profitability will transfer to improved returns for the shareholders. On the other hand, if companies overload on debt (in an attempt to drive ROE), interest charges can drag down earnings; in a worst case scenario, if the debt is increased to the point that an interruption to earnings and cash flow prevent principal repayment, ROE can be jeopardized as the company may become insolvent. Proper management of debt allows companies to use leverage to improve earnings available to shareholders while adequately providing cushion so that the debt can be repaid.

ID10–10 A proxy is a device whereby a security holder authorizes another person or group of persons to act for him at a meeting of security holders—in effect a power of attorney. Whenever the solicitation is made on behalf of the management of the issuer and relates to an annual meeting of security holders at which directors are to be elected, a proxy statement must be furnished to the security holders pursuant to the SEC requirements. In essence, proxy statements are disclosure instruments with all kinds of information. They are furnished to the stockholders at a time when the shareholders have to make critical decisions through a proxy to the management. The new SEC required disclosures in the proxy statements regarding the executive compensation can be very informative to the stockholders. If executive compensation is linked to the shareholder returns or stock market performance, then it is most likely that management’s actions, while maximizing their own wealth, will also maximize the wealth of the stockholders. In a way, linking executive compensation to stock market returns or shareholder returns promotes goal congruence. Further, if a company’s executives own a lot of its stock, stockholders can take comfort that executives will make decisions that would really maximize the economic wealth of the company. They would try to increase the market price of each share rather than play with accounting policies to declare more income on the income statement. Any information that indicates that there has been a change in the auditors can also provide useful signals to the stockholders. Such a change can indicate the existence of serious disagreement between the auditor and the management. The shareholders can further investigate through SEC as to what were the reasons leading to such a change before assigning their proxy to the current management.


ID10–11 a. Medical and Life Insurance Expense (E, –SE) .......................... Accrued Insurance Liability (+L) ........................................ Accrued medical and life insurance expense.

2,260,000,000 2,260,000,000

b. As part of the Chapter 11 reorganization, LTV negotiated new credit agreements. These credit agreements contained debt covenants, most likely related to the amount of debt LTV could have. If LTV had waited until after emerging from Chapter 11, the $2.26 billion insurance liability might have caused the company to violate the amount of allowable debt as specified in its debt agreements. Such a violation could have forced the company back into Chapter 11 or to renegotiate its debt agreements at less favorable terms. By recording the $2.26 billion liability prior to negotiating its new debt agreements, the $2.26 billion liability would be considered by creditors in creating the debt covenants. Thus, this liability would not place LTV into an automatic violation of its debt covenants. c.

There are probably two reasons why LTV chose to take several significant charges while it was under bankruptcy proceedings. First, the significant charges would adversely affect LTV's reported results of operations and financial position, and LTV may have been trying to extract more favorable settlement terms from its creditors by demonstrating weakened performance and financial position. Second, LTV may have been positioning itself to show improved performance once it emerged from Chapter 11. By taking the $2.26 billion charge now, not only does LTV avoid having to reduce its earnings in the future, but it also reduces the company's earnings so much that its earnings can only increase next year. This latter strategy is known as "taking a bath."

ID10–12 The conservatism ratio is an excellent measure of a company’s aggressiveness in its tax strategy. The lower the ratio, the more conservative the company. The higher conservatism ratio indicates that management is more aggressive with its tax policies and results in higher future tax liabilities. The lower the ratio the less aggressive management is with tax policies. Conservatism Ratio

= Reported Income Before Taxes ÷ Taxable Income

2007 Taxable Income = $1,028 ÷ .312 = $3,295 Conservatism Ratio

= =

$3,093 ÷ $3,295 0.94

2008 Taxable Income

= =

Conservatism Ratio 2009 Taxable Income

Conservatism Ratio

$1,085 ÷ .317 $3,423 = =

= =

$3,591 ÷ $3,423 1.05

$571 ÷ .287 $1,990 = =

$2,417 ÷ $1,990 1.21


ID10–12 CONCLUDED The trends indicate that the company is becoming more and more aggressive in its tax strategy.

ID10–13 a. What Mr. Healy really means is that GM earned $200 million in 1997 not by selling autos but by working through an accounting adjustment due to a change in the tax rates in its North American operations. b. Change in the expected tax rate can lead to a positive effect on reported earnings due to the fact that tax liability was accrued at a higher tax rate and eventually paid at a lower tax rate. c.

Not really. It seems that GM estimated its tax liability to be higher, and it had to pay much less lower taxes at the end of the year. It is akin to a change of an accounting estimate.

ID10–14 a. A patent gives the holder the exclusive rights to use an idea (or a technology) and prevents competitors from using the idea (or technology). Patents are considered intangible assets and are carried on the balance sheet. b. From Vonage’s perspective, the lawsuits represent contingent losses. Depending on the evaluation (by Vonage management and legal counsel) of the merits of the lawsuits, the company can ignore the lawsuits in the financial statements (if loss is deemed remotely likely), disclose the lawsuit in the footnotes (if loss is deemed reasonably likely) or accrue an expense and a matching contingent liability (if loss is deemed highly likely). An analysis of Vonage’s balance sheet, income statement and footnotes will determine how the company and its auditors treated the contingencies related to the patent lawsuits.

ID10–15 a. Provisions under IFRS are recorded more frequently than contingent liabilities under U.S. GAAP because under IFRS provisions are accrued when the obligation is “more likely than not,” but GAAP uses a more stringent requirement of “highly probable”. Additionally, with IFRS the long term portion of any provisions are valued at the present value of the future expected cash flows; GAAP, on the other hand, does not apply any discounting when valuing contingent liabilities. Finally, when there is a range of possible outcomes associated with the liability, U.S. GAAP uses the lowest possible value, while IFRS employs the “best estimate”. b. When VW increased its provision account, it booked an expense (reducing profitability and equity) and a liability. The increase may include items such as warranties, restructurings and environmental issues. c. When VW reduced its provision account, it settled obligations by paying cash (reducing assets) and reducing the amount of its liabilities. Severance payments, costs incurred to honor automobile warranties, and costs for environmental clean-up are examples that may be included in VW’s number.


ID10–16 a. Working capital is the excess of current assets over current liabilities. It is one measure of the solvency of a company. Nike’s working capital was $5,517.8 million in 2008 and $6,457 million in 2009, showing a strong increase. b. In 2008 working capital was 44.3% of total assets, increasing to 48.7% in 2009. c.

Nike’s current ratio was 2.66 in 2008 and increased to 2.97 in 2009. The largest factor in the improved solvency is the increase in Short-Term Investments (a Current Asset). The drop in Accounts Payable also helped improve the company’s situation.

d. The most important current liability reported by Nike is Accounts Payable ($1,031.9 million). Accounts payable represent Nike’s obligation to its suppliers for inventory purchases. Without inventory items (such as rubber, nylon and canvas) the company would not be able to produce its products. The company buys its inventory items on credit, meaning accounts payable is an important item on Nike’s balance sheet. Accrued Liabilities in total is larger than Accounts Payable, but the accrued accounts contain many different items (see footnote #5).

e. Accounts payable turnover = COGS ÷ (Average accounts payable) 2009 X = $10,571.7 ÷ (($1,031.9 + $1,287.6) ÷ 2) X = 9.115* *dividing turnover into 365 days yields 40.0 days on hand for accounts payable 2008 X = $10,239.6 ÷ (($1,287.6 + $1,040.3) ÷ 2) X = 8.797* *dividing turnover into 365 days yields 41.5 days on hand for accounts payable The company is paying its suppliers 1.5 days faster, demonstrating a slight improvement in solvency. f. Per footnote #15, the company is involved with several trademark and legal issues, but none the proceedings pose a material risk to Nike’s financial condition.

of

g. According to footnote #5, the largest accrued liabilities are for compensation, benefits, taxes, restructuring charges and dividends. An accrued liability occurs when the company records an expense (or a deduction to equity in the case of dividends) with a debit and an obligation (a liability) to settle the charges at a later date with a credit. h. Per footnote #13, the company provides pension plans for employees world-wide (as mandated by local governments). Nike discloses that its pension plan currently is underfunded in the amount of $82.8 million. For its U.S. employees the company offers a profit sharing plan; Nike contributed $27.6 million, $37.3 million, and $31.8 million in the last three years, respectively. Nike also offers 401(k)


savings plans for employees; contributions to these plans were $37.6 million, $33.9 million, and $24.9 million in the last three years, respectively.

ID10–16 Concluded i. Conservatism Ratio = Reported Income Before Taxes ÷ Taxable Income 2009

2008

2007

Reported income before tax

1,956.5

2,502.9

2,199.9

÷ Taxable income

3,182.9

3,710.1

2.093.5

= Conservatism ratio

0.61

0.67

1.05

Current year tax liability*

763.9

920.1

674.1

÷ Effective tax rate*

24.0%

24.8%

32.2%

$3,710.1

$2,093.5

= Taxable Income

$3,182.9

*footnote #9 The tax strategy of Nike was more conservative over the time period (as shown by the lower conservatism ratios).


CHAPTER 11 LONG-TERM LIABILITIES: NOTES, BONDS, AND LEASES BRIEF EXERCISES BE11–1 a. During 2009 Radio Shack paid down $43.2 million of notes payable (long-term debt) and $2.3 million of other long-term debt. Further, the reduction in the unamortized discount reflects the fact that interest expense was booked at the effective rate while cash paid for interest was calculated at the stated rate of interest. When the debt obligations were repaid, the balance sheet and the statement of cash flows were adjusted. Interest expense affects the income statement. b. During 2009 $9.375 million of interest expense was recognized on the 2.5% notes (2.5% x $375 million). c.

If Radio Shack paid $300 million to retire the notes in 2009 the company would have recorded a gain of $6.8 million on the transaction. ($306.8 – $300). This amount would be found on the income statement as a gain on the early extinguishment of debt.

BE11–2 a. The life of these bonds is 20 years, from 1997 until 2017. b. The stated interest rate is 0%. c.

The effective rate is 3.2%. (The present value is $968 million, while the future value of the single sum in 20 years is $1.8 billion.)

d. Bonds typically are issued in amounts of $1,000, therefore Hewlett-Packard issued 1.8 million bonds with a total face value of $1.8 billion.

BE11–3 a. The operating lease payments reduced the reported income for the period, reduced the assets on the balance sheet (payment of cash), and impacted the statement of cash flows by reducing the net income of SuperValu for the reporting period. By reducing net income these lease payments are a use of cash from operations. b. The interest portion of capital lease payments and the depreciation on the capitalized lease assets reduce reported net income for the period. They also impact the balance sheet by reducing both the assets and the liabilities. For a capital lease the interest payments is cash from operations, principal reductions is financing and the depreciation has no impact on the statement of cash flows. c.

The $401 million of operating leases is a form of off-balance sheet financing for SuperValu. Supervalu does not have to show these future liabilities on its balance sheet. These future contractual payments are disclosed in the footnotes. One of the advantages of this approach is that these liabilities are not used in the calculation of financial ratios. 1


EXERCISES E11–1 a. Melrose Enterprises' debt/equity ratio is currently 1.25 [($200,000 + $300,000) ÷ $400,000]. The company's loan agreement specifies that debt can be twice the stockholders' equity. Consequently, the company's debt cannot exceed $800,000. Since Melrose Enterprises already has $500,000 in debt, the company can borrow an additional $300,000. b. By definition, Melrose Enterprises will settle its December 31, 2011 current liabilities sometime during 2012. The company will probably also incur new current liabilities as of December 31, 2012. Since no information is provided as to the expected amount of current liabilities as of December 31, 2012, a reasonable assumption is that these liabilities will remain at $200,000. Consequently, Melrose Enterprises would have total debt of $500,000 and total stockholders' equity of $550,000 ($400,000 + $950,000 in revenues – $800,000 in expenses). The company could now borrow a total of $1,100,000 ($550,000  2) without violating the debt covenant. Melrose Enterprises could, therefore, borrow an additional $600,000. c.

At the end of 2012, Melrose Enterprises would have $200,000 in current liabilities and $300,000 in long-term debt for total debt of $500,000, and it would have $450,000 in stockholders' equity ($400,000 + $950,000 in revenues – $800,000 in expenses – $100,000 in declared dividends). The company could now borrow a total of $900,000 (i.e., $450,000  2) without violating its debt covenant. Consequently, Melrose Enterprises could borrow an additional $400,000. If Melrose Enterprises declares, but does not pay, the $100,000 dividend, the company's debt/equity ratio will be affected. Dividends that are declared but not paid are typically classified as current liabilities. Consequently, Melrose Enterprises would have $300,000 in current liabilities and $300,000 in long-term liabilities for total liabilities of $600,000, and it would have $450,000 in stockholders' equity, which means that the company could borrow a total of $900,000 without violating its debt covenant. Consequently, Melrose Enterprises could borrow an additional $300,000. Declaring but not paying the dividend, as opposed to declaring and paying the dividend, reduced the amount of money that the company could borrow on a dollar-for-dollar basis.


E11–2 a. 1/1/11

1/1/12

1/1/13

1/1/14

1/1/15

1/1/16

$30,000

$30,000

$30,000

$30,000

$30,000 $300,000

b. All dollar amounts on the time line below are in thousands of dollars. 1/11

c.

1/12

7/12

1/13

7/13

1/14

7/14

1/15

7/15

1/16

$15

$15

$15

$15

$15

$15

$15

$15

$15

$15 $300

Total Present Value

= Present Value of Face Value + Present Value of Periodic Interest Payments

(1)

Annual interest payments: Total Present Value = ($300,000  Present Value Factor for i = 10% and n = 5) + ($30,000  Present Value Factor of an Ordinary Annuity Factor for i = 10% and n = 5) = ($300,000  .62092 from Table 4, Appendix A) + ($30,000  3.79079 from Table 5, Appendix A) = $186,276 + $113,724 = $300,000

(2)

Semiannual interest payments: Total Present Value = ($300,000  Present Value Factor for i = 5% and n = 10) + ($15,000  Present Value Factor of an Ordinary Annuity Factor 5% and n = 10) = ($300,000  .61391 from Table 4, Appendix A) + ($15,000  7.7218 from Table 5, Appendix A) = $184,173 + $115,827 = $300,000

E11–3 1 2 3 4

7/11

Par value Discount Premium Premium

for i =


E11–4 Present Value = Present Value of Face Value + Present Value of Interest Payments Note 1 Present Value

Note 2 Present Value

Note 3 Present Value

Note 4 Present Value

Note 5 Present Value

= ($1,000  Present Value Factor for i = 8% and n = 4) + [($1,000  0%)  Present Value of an Ordinary Annuity Factor for i = 8% and n = 4] = $1,000  .7350 (from Table 4 in Appendix A) + $0 = $735.00 = ($5,000  Present Value Factor for i = 6% and n = 6) + [($5,000  0%)  Present Value of an Ordinary Annuity Factor for i = 6% and n = 6] = $5,000  .7050 (from Table 4 in Appendix A) + $0 = $3,525.00 = ($8,000  Present Value Factor for i = 12% and n = 6) + [($8,000  4%)  Present Value of an Ordinary Annuity Factor for i = 12% and n = 6] = ($8,000  .5066 (from Table 4 in Appendix A) + ($320  4.1114 (from 5 in Appendix A) = $4,052.80 + $1,315.65 = $5,368.45 = ($3,000  Present Value Factor for i = 8 % and n = 7) + [($3,000  8%)  Present Value of an Ordinary Annuity Factor for i = 8% and n = 7] = ($3,000  .5835 (from Table 4 in Appendix A) + ($240  5.2064) (from Table 5 in Appendix A) = $1,750.50 + $1,249.54 = $3,000.00 = ($10,000  Present Value Factor for i = 6 % and n = 10) + [($10,000  10%)  Present Value of an Ordinary Annuity Factor for i = 6% and n = 10] = ($10,000  .5584 (from Table 4 in Appendix A) + ($1,000  7.3601 (from Table 5 in Appendix A) = $5,584.00 + $7,360.10 = $12,944.10

Table


E11–5 a.

Present Value $11,348

Present Value Factor

= Present Value of Face Value + Present Value of Interest Payments = ($20,000  Present Value Factor for i = ? and n = 5) + [($20,000  0%)  Present value of an ordinary annuity factor for i = ? and n = 5] = ($20,000  Present Value Factor) + $0 = 0.5674

Looking in the n = 5 row of Table 4 in Appendix A reveals that a present value factor of 0.5674 corresponds to an annual effective interest rate of 12%. b. Equipment (+A) ......................................................................................... Discount on Notes Payable (–L) ................................................................ Notes Payable (+L) .............................................................................. Purchased equipment by issuing a note. c.

Interest Expense

d. Balance Sheet Value

11,348 8,652 20,000

= Book Value of Debt  Effective Interest Rate = ($20,000 – $8,652)  12% = $1,361.76 = = =

Face Value of Note – Discount on Notes Payable $20,000.00 – ($8,652.00 – $1,361.76) $12,709.76

e. Interest expense is computed as the debt's book value times the effective interest rate. For a note issued at a discount, the book value will increase over time until the book value equals the face value immediately prior to the note maturing. Since the book value is greater at the beginning of Year 2 than it was at the beginning of Year 1, and the effective interest rate is constant, the interest expense recognized by Tradewell in the second year will be greater than the interest expense recognized in the first year. Intuitively this makes sense. During Year 1, Tradewell only "borrowed" $11,348. Although Tradewell has to compensate the creditor for using the creditor's money during Year 1, Tradewell did not make any such payment to the creditor during Year 1 because the stated rate on the note is 0%. Thus, the amount that Tradewell should have compensated the creditor (i.e., interest expense) is simply added on to what Tradewell "borrowed" from the creditor. During Year 2, therefore, Tradewell has to pay interest not only on the initial $11,348 it "borrowed," but also on the interest that it incurred, but did not pay, during Year 1. As proof, Interest Expense for Year 2

= $12,709.76 [from part (d)]  12% = $1,525.17

This amount exceeds the interest expense for Year 1 computed in part (c).


E11–5 f.

Concluded

Since the note has not yet matured, the same logic used in part (e) can be applied to this question. Consequently, the interest expense recognized by Tradewell in the third year will be greater than the interest expense recognized in the second year. As proof, Interest Expense for Year 3

= Book Value at Beginning of Year 3  12% = [$20,000.00 – ($8,652.00 – $1,361.76 – $1,525.17)]  12% = $1,708.19

This amount exceeds the interest expense for Year 2 computed in part (e).

E11–6 a. Stated interest rate = 8% Cash (+A) ................................................................................................... Notes Payable (+L) .............................................................................. Issued notes payable.

8,000 8,000

Interest Expense (E, –SE) ........................................................................... Cash (–A)............................................................................................. Incurred and paid interest.

640

Interest Expense (E, –SE) ........................................................................... Cash (–A)............................................................................................. Incurred and paid interest.

640

Notes Payable (–L)..................................................................................... Cash (–A)............................................................................................. Repaid notes payable.

8,000

b. Stated interest rate = 0% Face value ...................................................................................... Present value (i = 8%, n = 2) Present value of face value $8,000  .8573 (from Table 4 in Appendix A)......................... Discount on notes payable ............................................................

640

640

8,000

$ 8,000.00

6,858.40 $ 1,141.60

Cash (+A) ................................................................................................... Discount on Notes Payable (–L) ................................................................ Notes Payable (+L) .............................................................................. Issued notes payable.

6,858.40 1,141.60

Interest Expense (E, –SE) ........................................................................... Discount on Notes Payable (+L) .......................................................... Incurred interest.

548.67

Interest Expense (E, –SE) ........................................................................... Discount on Notes Payable (+L) .......................................................... Incurred interest.

592.93

8,000.00

548.67

592.93


E11–6

Concluded

Notes Payable (–L)..................................................................................... Cash (–A)............................................................................................. Repaid notes payable. c.

Stated interest rate = 6% Face value .................................................................................................. Present value (i = 8%, n = 2) Present value of face value $8,000  .8573 (from Table 4 in Appendix A) ................................... Present value of interest payments ($8,000  6%)  1.7833 (from Table 5 in Appendix A) ...................... Total present value.................................................................................... Discount on notes payable ........................................................................

8,000.00 8,000.00

$8,000.00

$6,858.40 855.98 $

Cash (+A) ................................................................................................... Discount on Notes Payable (–L) ................................................................ Notes Payable (+L) .............................................................................. Issued notes payable.

7,714.38 285.62

Interest Expense (E, –SE) ........................................................................... Discount on Notes Payable (+L) .......................................................... Cash (–A)............................................................................................. Incurred and paid interest.

617.15a

7,714.38 285.62

8,000.00

137.15b 480.00c

a $617.15 = Book Value  Effective Interest Rate = 7,714.38  8% b $137.15 = Interest Expense – Interest Payment c

$480.00 = Face Value  Stated Interest Rate = $8,000  6%

Interest expense (E, –SE) ........................................................................... Discount on Notes Payable (+L) .......................................................... Cash (–A)............................................................................................. Incurred and paid interest.

628.47a 148.47b 480.00

a $628.47 = Book Value  Effective Interest Rate = (7,714.38 + 137.15)  8% b $148.47 = $8,000.00 – [$7,714.38 + $137.15 (from prior entry)]

Notes Payable (–L)..................................................................................... Cash (–A)............................................................................................. Repaid notes payable.

8,000 8,000


E11–7 a. Present value = Present value of face value + Present value of periodic interest payments (1) Discount rate = 8% Present value of face value (i = 8%, n = 2) ($2,500  0.85734 from Table 4 in Appendix A) Present value of periodic interest payments (i = 8%, n = 2) ($200  1.78326 from Table 5 in Appendix A) Total present value (2)

(3)

Discount rate = 10% Present value of face value (i = 10%, n = 2) ($2,500  0.82645 from Table 4 in Appendix A) Present value of periodic interest payments (i = 10%, n = 2) ($200  1.73554 from Table 5 in Appendix A) Total present value Discount rate = 12% Present value of face value (i = 12%, n = 2) ($2,500  0.79719 from Table 4 in Appendix A) Present value of periodic interest payments (i = 12%, n = 2) ($200  1.69005 from Table 5 in Appendix A) Total present value

$ 2,143.35 356.65 $ 2,500.00

$ 2,066.13 347.11 $ 2,413.24

$ 1,992.98 338.01 $ 2,330.99

b. The effective interest rate is the interest rate that equates the undiscounted future cash flows with the present value of the future cash flows. In this case, the undiscounted future cash flows are (1) the $2,500 face value due in two years and (2) the interest payments of $200 due at the end of each year for two years, while the present value of the note is the proceeds of $2,413. From part (a), a discount rate of 10% equates the future cash flows and the proceeds. Therefore, the effective interest rate is 10%. c.

If Wilmes Floral Supplies originally borrowed $2,500, the $2,500 would be the present value of the future cash flows. From part (a), a discount rate of 8% equates the future cash flows with $2,500. The effective interest rate would, therefore, be 8%. Anytime the proceeds equal the face value, the effective interest rate equals the stated interest rate.

E11–8 a. The building should be capitalized at the cash value of the transaction. In this particular case, the cash value of the transaction would be assumed to equal the building's appraised value. Therefore, the building should be recorded at $550,125.


E11–8

Concluded

b. The total present value of a note equals the sum of the present value of the note's face value and the present value of the periodic interest payments specified in the note. Since the note signed by Morrow Enterprises is non-interest-bearing, there are no periodic interest payments, and the present value of the note would equal just the present value of the note's face value.

c.

(1)

Discount Rate Present Value

= = = =

6% ($693,000  Present Value Factor for i = 6% and n = 3) $693,000  0.83962 from Table 4 in Appendix A) $581,856.66

(2)

Discount Rate Present Value

= = = =

8% ($693,000  Present value factor for i = 8% and n = 3) $693,000  0.79383 from Table 4 in Appendix A) $550,124.19

(3)

Discount Rate Present Value

= = = =

10% ($693,000  Present value factor for i = 10% and n = 3) $693,000  0.75131 from Table 4 in Appendix A) $520,657.83

The effective interest rate is the rate that equates the undiscounted future cash flows with the present value of the future cash flows. In this case, the undiscounted future cash flow is the $693,000 face value due in three years, and the present value of the note is the value of the building, or $550,125. From part (b), a discount rate of 8% equates the future cash flows and the proceeds. Therefore, the effective interest rate is 8%.

d. Since the note is non-interest-bearing, the only cash flow is the face value of $693,000. Dividing the present value of $550,125 by the face value of $693,000 yields a present value factor of .79383. Looking across the n = 3 row of the present value of $1 table (i.e., Table 4) in Appendix A reveals that the annual effective interest rate on this note is 8%.

E11–9 a.

Interest Expense $16,400 Effective Interest Rate

= Effective Rate  Book Value of Debt at Beginning of the Period = Effective Rate  ($200,000 – $14,400) = 8.8% (rounded)

b. Interest Expense (E, –SE) ........................................................................... Discount on Notes Payable (+L) .......................................................... Cash (–A)............................................................................................. Incurred and paid interest. * $2,400 = Change in the balance of Discount on Notes Payable

16,400 2,400* 14,000


E11–10 a. The ten-year notes call for annual interest of $25.025 million (stated rate of 6.5% X face value of $385 million) and the repayment of $385 million in principal. The proceeds of the notes were $380 million. If the present value of the contract is $380 million and the future values are represented in the interest (ordinary annuity) and the principal (single sum), then the effective interest rate is the rate that discounts the future values to the present value of $380 million. The effective interest rate is approximately 6.7%. (The general present value formula of 1/[(1 + r) to the nth] was used in this calculation.) b. The interest expense for 2002 will be the effective rate of 6.7% multiplied by the proceeds of $380 million, or $25.46 million. This amount is made up of $25.025 million of cash paid plus $.435 million of non-cash interest from the amortization of the bond discount.

c.

The market paid less than $385 million for these bonds because the market demands 6.7% interest for their investment dollars for the risk posed by the company at the time of issuance. The notes only pay a cash interest rate of 6.5% and so the only way that investors can make their desired return is to pay less for the notes. This allows the investors to make the market rate of 6.7%.

E11–11 a. Bond A Face value ........................................................................................... Present value (i = 3%, n = 20) PV of face value ($100,000  0.55368 from Table 4 in Appendix A)........................ PV of periodic interest payments ($3,000 x 14.87747 from Table 5 in Appendix A) .......................... Total present value (i.e., proceeds) .................................................... Discount/premium ............................................................................. Bond B Face value ........................................................................................... Present value (i = 3%, n = 20) PV of face value ($400,000  0.55368 from Table 4 in Appendix A)........................ PV of periodic interest payments ($16,000  14.87747 from Table 5 in Appendix A)........................ Total present value (i.e., proceeds) .................................................... Premium ............................................................................................. Bond C Face value ........................................................................................... Present value (i = 4%, n = 10) PV of face value ($600,000  0.67556 from Table 4 in Appendix A)........................ PV of periodic interest payments ($18,000  8.11090 from Table 5 in Appendix A)..........................

$ 100,000

$ 55,368 44,632 $

100,000 0

$ 400,000

$ 221,472 238,040 459,512 $ 59,512

$ 600,000

$ 405,336 145,996


Total present value (i.e., proceeds) .................................................... Discount .............................................................................................

551,332 $ 48,668

E11–11 Concluded b. Immediately before a bond matures, its carrying value on the balance sheet must equal its face value. Thus, discounts and premiums must be amortized over time so that the carrying value approaches the bond's face value over time. For bonds that are issued at their face value, such as Bond A, the bond is already stated at its face value and there is no discount or premium to amortize. Consequently, the carrying value of the bond will remain equal to its face value over the life of the bond. Thus, the carrying value of Bond A will remain constant over its life. For bonds issued at a discount, such as Bond C, the carrying value on the date the bond is issued is less than its face value. Consequently, over the life of the bond, its carrying value must increase as the discount is amortized. Remember that a discount on a bond is deducted from the bond's face value to determine the carrying value. Thus, any reduction in the discount balance, such as when the discount is being amortized, will decrease the amount being deducted from the face value which thereby increases the carrying value of the bond. The carrying value of Bond C will, therefore, increase over its life. Alternatively, the carrying value of Bond B will decrease over its life. For bonds issued at a premium, such as Bond B, the carrying value on the date the bond is issued is greater than its face value. Consequently, over the life of the bond, its carrying value must decrease as the premium is amortized. Remember that a premium on a bond is added to the bond's face value to determine the carrying value. Thus, any reduction in the premium balance, such as when the premium is being amortized, will decrease the amount being added to the face value which thereby decreases the carrying value of the bond. Thus, the carrying value of Bond B will decrease over its life. c.

Interest expense is computed as the bond's book value at the beginning of the accounting period times the effective interest rate per period. Since accountants use the effective interest rate on the date a bond is issued to calculate interest expense, the effective interest rate is constant over the bond's life. This implies that the only factor that could affect whether the interest expense recognized each period increases, decreases, or remains constant over the life of the bond is the book value. As discussed in part (b), the book value of Bond A will remain constant over the life of the bond issue. Consequently, the interest expense recognized in each accounting period will remain constant over the life of Bond A. Alternatively, the interest expense recognized for Bonds B and C will vary across periods. Since the book value of Bond B will decrease over the life of the bond issue [see part (b)], interest expense associated with Bond B will also decrease from one period to the next. The interest expense associated with Bond C will increase from one period to the next because the book value of Bond C will increase each period [see part (b)].


E11–12 a. 1/1/11

b. 6/30/11

Cash (+A)............................................................................ Bonds Payable (+L)...................................................... Issued bonds for cash.

30,000

Interest Expense (E, –SE) ................................................... Cash (–A) ..................................................................... Incurred and paid interest.

1,500a

30,000

1,500b

a $1,500 = Book Value  Effective Interest Rate per Period = $30,000  5% b $1,500 = Face Value  Stated Interest Rate per Period = $30,000  5%

12/31/11

c.

Interest Expense (E, –SE) ................................................... Cash (–A) ..................................................................... Incurred and paid interest.

1,500 1,500

Balance Sheet Value

= Face Value – Associated Discount + Associated Premium = $30,000 – $0 + $0 = $30,000 d. Present value (i = 5%, n = 18) PV of face value ($30,000  .4155) ................................................................................. $12,465 PV of interest payments ($1,500  11.6896) ............................................................................... 17,534 Total present value.................................................................................... $30,000 e. Balance Sheet Value as of 12/31/12 = Face Value – Associated Discount + Associated Premium = $30,000 – $0 + $0 = $30,000 Present value as of 12/31/12 (i = 5%, n = 16) PV of face value ($30,000  .4581) ................................................................................. PV of interest payments ($1,500  10.8378) ............................................................................... Total present value....................................................................................

$13,743 16,257 $30,000

Notice that the balance sheet value of $30,000 is identical to the present value just calculated. Amortizing premiums and discounts using the effective interest rate results in bonds being carried on the balance sheet at an amount equal to the present value of the future cash flows of the bonds, using the effective interest rate on the date the bonds were issued as the discount rate.


E11–13 a. Face value ........................................................................................ Present value (i = 4%, n = 10) PV of face value ($500,000  0.6756 from Table 4 in Appendix A)....................... PV of interest payments ($15,000  8.1109 from Table 5 in Appendix A)......................... Total present value.......................................................................... Discount ..........................................................................................

$ 500,000

$ 337,800 121,664 $

Cash (+A) ................................................................................................. Discount on Bonds Payable (–L) .............................................................. Bonds Payable (+L) ........................................................................... Issued bonds.

459,464 40,536

459,464 40,536 500,000

18,378.56a

b. Interest Expense (E, –SE) ......................................................................... Discount on Bonds Payable (+L) ....................................................... Interest Payable (+L) ......................................................................... Accrued interest payable.

3,378.56c 15,000.00b

a $18,378.56 = Book Value  Effective Rate per Period = $459,464  4% b $15,000.00 = Face Value  Stated Rate per Period = $500,000  3% c

c.

$3,378.56 = $18,378.56 – $15,000.00

Balance sheet value as of 12/31/12

= = =

Face value – Discount as of 12/31/12 $500,000.00 – ($40,536.00 – $3,378.56) $462,842.56

d. Present value (i = 4%, n = 9) PV of face value ($500,000  0.7026 from Table 4 in Appendix A)............................... PV of interest payments ($15,000  7.4353 from Table 5 in Appendix A)................................. Total present value..................................................................................

$ 351,300.00 111,530.00 $ 462,830.00

Notice that the $462,842.56 from part [c] is essentially identical to the $462,830.00 just calculated. Amortizing discounts and premiums using the effective interest rate results in bonds being carried on the balance sheet at an amount equal to the present value of the bonds, using the effective interest rate on the date the bonds were issued as the discount rate.

E11–14 a. Face value ........................................................................................ Present value (i = 3%, n = 20) PV of face value ($100,000  0.55368 from Table 4 in Appendix A)..................... PV of interest payments ($4,000  14.87747 from Table 5 in Appendix A).......................

$ 100,000

$

55,368 59,510


Total present value.......................................................................... Premium ..........................................................................................

E11–14

$

114,878 14,878

Concluded

Cash (+A) ................................................................................................. Premium on Bonds Payable (+L)....................................................... Bonds Payable (+L) ........................................................................... Issued bonds.

114,878 14,878 100,000

3,446.34a 553.66c

b. Interest Expense (E, –SE) ......................................................................... Premium on Bonds Payable (-L) .............................................................. Interest Payable (+L) ......................................................................... Accrued interest payable.

4,000.00b

a $3,446.34 = Book Value  Effective Rate per Period = $114,878  3% b $4,000.00 = Face Value  Stated Rate per Period = $100,000  4% c

c.

$553.66 = $4,000.00 – $3,446.34

Balance sheet value as of 12/31/12

= = =

Face value + Premium as of 12/31/12 $100,000.00 + ($14,878.00 – $553.66) $114,324.34

d. Present value (i = 3%, n = 19) PV of face value ($100,000  0.57029 from Table 4 in Appendix A)............................. PV of interest payments ($4,000  14.3238 from Table 5 in Appendix A)................................. Total present value..................................................................................

$

57,029.00

57,295.20 $ 114,324.20

Notice that the $114,324.34 from part [c] is essentially identical to the $114,324.20 just calculated. Amortizing discounts and premiums using the effective interest rate results in bonds being carried on the balance sheet at an amount equal to the present value of the bonds, using the effective interest rate on the date the bonds were issued as the discount rate.

E11–15 a. Since it is one year later, two interest periods have passed. Thus, there are only eight remaining interest periods. Present value (i = 3%, n = 8) PV of face value ($20,000  0.7894 from Table 4 in Appendix A)................................ PV of interest payments ($800  7.0197 from Table 5 in Appendix A)..................................... Total present value.................................................................................

$15,788.00 5,615.76 $21,403.76

To determine whether Treadway has experienced an economic gain or loss, we need to know both the market value and the carrying value of the bonds. The market value of the bonds on December 31, 2011 should equal the present value of the bond's future cash flows discounted using the prevailing market interest rate. Thus, the market value of Treadway's bonds on December 31, 2011 is $21,403.76.


Since the bonds were issued at face value, the effective interest rate on the date of issue equaled the stated interest rate, and there was no discount or premium associated with the bonds. When bonds are issued at face value, the bonds are carried on the books at face value until the bonds mature. Consequently, the book value of these bonds is $20,000.00. Since the market value of the bonds now exceeds $20,000.00, Treadway has experienced an economic loss. That is, if Treadway wanted to retire the bonds, it

E11–15 Concluded would cost the company $21,403.76 rather than $20,000.00. The amount of the loss is the excess of the bond's market value over the bond's book value, or $1,403.76. b. Present value (i = 5%, n = 8) PV of face value ($20,000  0.6768 from Table 4 in Appendix A)................................... PV of interest payments ($800  6.4632 from Table 5 in Appendix A)........................................ Total present value....................................................................................

$13,536.00 5,170.56 $18,706.56

The market value of the bonds is now less than their book value. If Treadway Company wanted to retire the bonds through the bond market, it would have to pay less than the value of the bonds per the company's financial records. Therefore, the effective liability of the company has decreased, which implies that the company has experienced an economic gain. The gain would be the excess of the bonds' book value over the bonds' market value, or $1,293.44. c.

Companies experience economic gains and losses when their wealth changes. In the case of bonds, their market value indicates the company's effective obligation on the bonds at that particular point in time. If the market value exceeds the bonds' book value, then the company has experienced a decrease in wealth; if the market value is less than book value, then the company has experienced an increase in wealth. Such gains and losses, however, are not usually reflected in a company's financial statements because it is assumed that when a company issues bonds, the bonds will remain outstanding until they mature. That is, the company will not retire the bonds before they mature. On the date that the company issued the bonds, the company locked into the market rate of interest on that day. This means that the effective interest rate on the date the bonds are issued is the interest rate the company expects to incur over the life of the bonds. Thus, the economic gains and losses due to fluctuations in the market interest rate will essentially "wash out" over the life of the bonds because the bonds are expected to remain outstanding until they mature. Economic gains or losses associated with changes in the market interest rate are only reflected in a company's financial statements when a company retires some bonds prior to their maturity date. If you were analyzing Treadway's financial statements, you might want to adjust the amounts reported for notes and bonds to reflect the prevailing market interest rate. In this manner, the statements would more accurately reflect the company's economic liability—and hence associated gains and losses— compared to the amounts reported on the balance sheet.


E11–16 a. Cash paid to redeem the bonds

= = =

Face value  101% $500,000  101% $505,000

Bonds Payable (–L) .................................................................................... Loss on Redemption (Lo, –SE) ................................................................... Discount on Bonds Payable (+L) ......................................................... Cash (-A) ............................................................................................. Redeemed bonds.

500,000 15,000 10,000* 505,000

* $10,000 = Face Value – Book Value = $500,000 – $490,000 b. Bonds Payable (–L) .................................................................................... Premium on Bonds Payable (–L) ............................................................... Cash (–A)............................................................................................. Gain on Redemption (Ga, +SE) ........................................................... Redeemed bonds.

500,000 7,000 505,000 2,000

E11–17 a. Lilly paid $47 million to retire this debt. This is comprised of $35 million (book value of bonds), $7.2 million (after tax loss) plus $4.8 million (tax benefit of loss). b. The $4.8 million is a benefit to the company because the loss offsets against the operating income of Lilly. As a result it reduces the amount of tax that Lilly will have to pay. Therefore the loss on retiring this debt provides a tax benefit to Lilly. c.

The loss would be shown as part of the net income that is shown on the first line item on the statement of cash flows. It would be added back to compute cash from operating activities.

d. The loss would be shown as an extraordinary item on the income statement.

E11–18


a. American Greetings paid cash of $181.2 million to retire the debt (book value of $142.2 million plus loss on debt repurchase of $39.0 million). b. The company, in effect, paid extra to make the debt go away. The debt was on the books for $142.2 million, but the company—through negotiations with the holders of the debt—agreed to pay a total of $181.2 million to retire the obligations. The additional amount paid, above the book value of the debt, is the amount of the loss recorded on the income statement. c.

American Greetings refinanced the debt at much lower interest rates. A company would be willing to incur a loss on its income statement if it can refinance debt at much lower rates. Over the life of the new debt the company figures it will more than make up for the amount booked on the income statement as a loss by lowering its interest expense in all the years that the debt is outstanding.

E11–19 a. Interest Expense (E, –SE) ........................................................................... Cash (–A)............................................................................................. Discount on Bonds (+L) ....................................................................... Incurred and paid interest.

4,822.70a 4,000.00b 822.70

a $4,822.70 = Book Value  Effective Interest Rate per Period = $96,454  5% b $4,000.00 = Face Value  Stated Interest Rate per Period = $100,000  4%

b. Bonds Payable (–L) .................................................................................... Cash (–A)............................................................................................. Discount on Bonds Payable (+L) ......................................................... Gain on Retirement of Bonds Payable (Ga, +SE) ................................ Retired bonds.

100,000.00

* $2,723.30 = $3,546 Discount balance as of 12/31/11 – $822.70 Discount from 1/1/12 to 7/1/12 [from part (a)]

91,700.00 2,723.30* 5,576.70

amortized

E11–20 a. The effective interest rate can be calculated in two ways. The first way is by solving for i in the following equation where n=2 since there are two periods until maturity (12/31/11, the balance sheet date and maturity at 12/31/13). $94,650 = [($100,000  (1 + i )-2] + {$5,000  [(1 – [(1 + i)-2 + i ]} The second way is by trial and error. Simply plug an interest rate into the equation above until the right-hand side of the equation equals the left hand side. Since the bond is issued at a discount, we start with the knowledge that the effective rate is greater than the stated rate of 5%. The annual effective interest rate for the bonds is 8%. b. To determine the effective rate an investor would be earning if the bonds were purchased on 12/31/11 at the market value of $98,167, perform the same procedure using the equation. $98,167 = [($200,000  (1 + i )-2] + {$10,000  [(1 – [(1 + i)-2 + i ]}


The annual effective interest rate for the bonds is 6%. c.

The book value of the bonds on Beasley Brothers’ books at December 31, 2011, is $94,650. The market value of the bonds as of December 31, 2011, is $98,167. The difference represents a loss of $3,517. It is a loss because if Beasley Brothers were to repurchase these bonds on the market in order to retire them, it would have to pay $3,517 more than the book value.

Net income Unrealized holding loss on Bonds Payable Adjusted net income

$27,000 (3,517) $23,483

The loss is not a decrease in the wealth of the company if it intends to keep the bonds outstanding until maturity. However, if Beasley intends to retire this debt, the loss represents a decrease in wealth because Beasley will sacrifice net assets of $3,517 to do so. d. Extraordinary Realized Loss on Retirement of Debt .............................. Bonds Payable ........................................................................................ Discount on Bonds Payable ............................................................. Cash .................................................................................................

3,517 100,000 5,350 98,167

Once the repurchase has occurred, the loss has been realized because a reduction in net assets has occurred. Before this occurs, however, the loss is unrealized and may be misleading if the company intends to keep the bonds until maturity.

E11–21 a. The effective interest rate can be calculated in two ways. The first way is by solving for i in the following equation where n=2 since there are two periods until maturity (12/31/12, the balance sheet date and maturity at 12/31/14). $193,059 = [($200,000  (1 + i )-2] + {$10,000  [(1 – [(1 + i)-2 + i ]} The second way is by trial and error. Simply plug an interest rate into the equation above until the right-hand side of the equation equals the left hand side. Since the bond is issued at a discount, we start with the knowledge that the effective rate is greater than the stated rate of 5%. The annual effective interest rate for the bonds is 6.92%. b. To determine the effective rate an investor would be earning if the bonds were purchased on 12/31/12 at the market value of $186,479, perform the same procedure using the equation. $186,479 = [($100,000  (1 + i )-2] + {$5,000  [(1 – [(1 + i)-2 + i ]} The annual effective interest rate for the bonds is 8.83%. c.

The book value of the bonds on Cohort Enterprises’ books at December 31, 2012 is $193,059. The market value of the bonds as of December 31, 2012 is $186,479. The difference represents a gain of $6,580. It is a gain because if Cohort Enterprises were to repurchase these bonds on the market in order to retire them, it would have to pay $6,580 less than the book value.


Net income Unrealized holding gain on Bonds Payable Adjusted net income

$38,500 6,580 $45,080

The gain is not an increase in the wealth of the company if it intends to keep the bonds outstanding until maturity. However, if Cohort intends to retire this debt, the gain represents an increase in wealth because Cohort will acquire additional net assets of $6,580.

d. Bonds payable .......................................................................................... Discount on Bonds Payable ................................................................ Cash .................................................................................................... Extraordinary Realized Gain on Retirement of Debt ..........................

200,000 6,941 186,479 6,580

Once the repurchase has occurred, the loss has been realized because a reduction in net assets has occurred. Before this occurs, however, the loss is unrealized and may be misleading if the company intends to keep the bonds until maturity.

E11–22 a. Lease Expense (E, –SE) .............................................................................. Cash (–A)............................................................................................. Incurred and paid lease expense for 2011.

10,000

Lease Expense (E, –SE) .............................................................................. Cash (–A)............................................................................................. Incurred and paid lease expense for 2012.

10,000

Lease Expense (E, –SE) .............................................................................. Cash (–A)............................................................................................. Incurred and paid lease expense for 2013.

10,000

Lease Expense (E, –SE) .............................................................................. Cash (–A)............................................................................................. Incurred and paid lease expense for 2014.

10,000

Lease Expense (E, –SE) .............................................................................. Cash (–A)............................................................................................. Incurred and paid lease expense for 2015.

10,000

10,000

10,000

10,000

10,000

10,000

b. The effective interest rate on the lease is 8%. The following entries would be recorded on Q-Mart’s books. Facility (A) .................................................................................................. Lease Liability (L) ................................................................................ Record the capitalized lease. ($10,000*3.9927)

39,927 39,927


c.

Depreciation Expense (E, –SE) ................................................................... Accumulated Depreciation (–A) ......................................................... Record depreciation of capitalized asset for 2011. ($39,927/5)

7,985.40

Lease Liability (-L) ...................................................................................... Interest Expense (E, -SE)............................................................................ Cash (-A) ............................................................................................. Made lease payment for 2011.

6,805.84 3,194.16

Depreciation Expense (E, –SE) ................................................................... Accumulated Depreciation (–A) ......................................................... Record depreciation of capitalized asset for 2012.

7,985.40

Lease Liability (-L) ...................................................................................... Interest Expense (E, -SE)............................................................................ Cash (-A) ............................................................................................. Made lease payment for 2012.

7,350.32 2,649.68

Depreciation Expense (E, –SE) ................................................................... Accumulated Depreciation (–A) ......................................................... Record depreciation of capitalized asset for 2013.

7,985.40

Lease Liability (-L) ...................................................................................... Interest Expense (E, -SE) ..................................................................... Cash (-A) ............................................................................................. Made lease payment for 2013.

7,938.32 2,061.68

Depreciation Expense (E, –SE) ................................................................... Accumulated Depreciation (–A) ......................................................... Record depreciation of capitalized asset for 2014.

7,985.40

Lease Liability (-L) ...................................................................................... Interest Expense (E, -SE)............................................................................ Cash (-A) ............................................................................................. Made lease payment for 2014.

8,573.36 1,426.64

Depreciation Expense (E, –SE) ................................................................... Accumulated Depreciation (–A) ......................................................... Record depreciation of capitalized asset for 2015.

7,985.40

Lease Liability (-L) ...................................................................................... Interest Expense (E, -SE)............................................................................ Cash (-A) ............................................................................................. Made lease payment for 2015.

9,260.00 740.00

7,985.40

10,000

7,985.40

10,000

7,985.40

10,000

7,985.40

10,000

7,985.40

10,000

Classifying the lease as an operating lease would give rise to both higher net income and a lower debt/equity ratio. By classifying the lease as an operating lease, net income would be reduced during 2011 by $10,000 [from part (a)] for rent expense. Alternatively, classifying the lease as a capital lease


would reduce net income by a total of $11,179.56 [from part (b)] for the interest expense associated with the lease and for the depreciation associated with the capitalized asset. Future obligations under operating leases are not disclosed in a company's financial statements as a liability. Consequently, an operating lease would not affect a company's total liabilities. On the other hand, the present values of future lease obligations are reported as liabilities under capital leases, which means that a capital lease results in increased liabilities compared to an operating lease. In addition, the differential effect of capital and operating leases on net income will affect total stockholders' equity through Retained Earnings. Specifically, the balance in Retained Earnings, and thus total stockholders' equity, will be higher by classifying the lease as an operating lease as opposed to classifying it as a capital lease. Therefore, QMart’s total liabilities would be lower and its stockholders' equity higher if the lease were classified as an operating lease rather than as a capital lease. This means that classifying the lease as an operating lease would yield a lower debt/equity ratio. At the end of the useful life both will be equal.

E11–23 = Rental Expense per Car  Number of Cars = $10,000  100 cars = $1,000,000

a. Annual Rental Expense

= $10,000 per Car  100 Cars x Present Value of an Ordinary Annuity Factor for i = 10%, n = 5 = $1,000,000  3.7908 from Table 5 in Appendix A = $3,790,800 Automobiles (+A).................................................................................... 3,790,800 Lease Liability (+L) ........................................................................... 3,790,800 Leased automobiles.

b. Present Value of Lease Payments

c.

Interest Expense

= Lease Obligation  10% = $3,790,800  10% = $379,080

Depreciation Expense

= = =

Cost of Automobiles ÷ 5 Years $3,790,800 ÷ 5 $758,160

Total Rental Expense

= Interest Expense + Depreciation Expense = $379,080 + $758,160 = $1,137,240

d. Classifying the lease as an operating lease would give rise to both higher net income and a lower debt/equity ratio. By classifying the lease as an operating lease, net income would be reduced during 2011 by $1,000,000 [from part (a)] for rent expense. Alternatively, classifying the lease as a capital lease would reduce net income by a total of $1,137,240 [from part (c)] for the interest expense associated with the lease and for the depreciation associated with the capitalized asset. Future obligations under operating leases are not disclosed in a company's financial statements as a liability. Consequently, an operating lease would not affect a company's total liabilities. On the other hand, the present values of future lease obligations are reported as liabilities under capital leases, which means that a capital lease results in increased liabilities compared to an operating lease. In


addition, the differential effect of capital and operating leases on net income will affect total stockholders' equity through Retained Earnings. Specifically, the balance in Retained Earnings, and thus total stockholders' equity, will be higher by classifying the lease as an operating lease as opposed to classifying it as a capital lease. Therefore, Tradeall's total liabilities would be lower and its stockholders' equity higher if the lease were classified as an operating lease rather than as a capital lease. This means that classifying the lease as an operating lease would yield a lower debt/equity ratio.


E11–23 Concluded e. Off-balance sheet financing refers to financing agreements that require future payments, yet are structured so that the financing arrangement does not meet any of the criteria for the financing arrangement to be reported as a liability. The substance of an operating lease is considered to be a rental agreement. This implies that a company does not incur an obligation under the lease until it actually uses the item being leased. Thus, the future obligations under the lease should not be reported as a liability. Alternatively, the substance of a capital lease is considered to be a purchase agreement. This implies that the company has, in substance, acquired an asset and that the lease is simply a note payable for the acquisition of that asset. Thus, the present value of the future cash outflows specified in the lease agreement should be reported as a liability. The difference between operating and capital leases provides a way for companies to engage in off-balance-sheet financing. That is, by structuring a lease agreement as an operating lease, the lessee can engage in off-balance sheet financing.

E11–24 a. Since the face value of the bank loan equals the proceeds of the loan (i.e., $149,388), the effective interest rate is equal to the stated interest rate. Therefore, the appropriate effective interest rate for Watts Motors for a ten-year borrowing arrangement is 12%. This rate should also be used for the lease. The annual lease payments would be an ordinary annuity for i = 12% and n = 10. Setting up the following formula and solving for the payment amount gives us the annual lease payment that would equate the two financing options. $149,388 = Lease Payment  Present Value of an Ordinary Annuity Factor for i = and n = 10 $149,388 = Lease Payment  5.65022 (from Table 5 in Appendix A) Lease payment = $26,439.32 (rounded)

12%

b. With the lease payment, Watts Motors would pay $26,439.32 at the end of each year for ten years. With the bank loan, Watts Motors would make interest payments of $17,926.56 ($149,388  12%) at the end of each year for ten years and a payment of $149,388 at the end of Year 10. The essential difference between the two financing arrangements is that a portion of every lease payment is applied against the outstanding principal balance while the annual payments under the bank loan do not reduce the principal balance. c.

Option 1 Building (+A) .............................................................................................. Notes Payable (+L) .............................................................................. Purchased a building. Option 2 Assets Acquired Under Capital Leases (+A) ............................................... Obligations Under Capital Leases (+L) ................................................ Acquired a building under a capital lease.

149,388 149,388

149,388 149,388

Option 3 Under the operating lease, the building would not be capitalized. Instead, on every lease payment date, Watts Motors would debit Lease Expense or Rent Expense for $26,439 and credit Cash for the same amount.


E11–24 Concluded d.

Interest Expensea

Payment $26,439.32 26,439.32

$17,926.56 16,905.03

Principal Reductionb

Principal $149,388.00 140,875.24 131,340.95

$8,512.76 9,534.29

a Interest Expense = Principal  Effective Interest Rate of 12% b Principal Reduction = Payment – Interest Expense

e. Present Value

= $26,439.32  Present Value of an Ordinary Annuity Factor for i = 12% and n = 8 = $26,439.32  4.96764 (from Table 5 in Appendix A) = $131,341.02

The present value of the future lease payments equals the amount reported on the balance sheet calculated in part (d).

E11–25 Present Value

= Present Value of Face Value + Present Value of Interest Payment = (Face Value  Present Value Factor) + (Periodic Interest Payment  Present Value of an Ordinary Annuity Factor)

Note 1 Since the proceeds (i.e., present value) equal the face value, we know that the effective rate equals the stated rate. Consequently, the effective rate for Note 1 is 8%. As proof: Present value (i = 8%, n = 6) PV of face value ($10,000  .63017 from Table 4 in Appendix A).............................. PV of interest payments [($10,000  8%)  4.62288 from Table 5 in Appendix A] ................ Total present value (i.e., proceeds) ...................................................... Note 2 $35,056 PV Factor PV Factor

$

6,301.70

3,698.30 $ 10,000.00

= ($100,000  Present Value Factor) + [($100,000  0%)  Present Value of an Ordinary Annuity Factor] PV factor = $35,056 ÷ $100,000 = .35056

Examining Table 4 in Appendix A (i.e., present value of $1 table) for n = 8, we find that the effective rate is 14%.


E11–25 Concluded Note 3 $922 = ($1,000  Present Value Factor) + [($1,000  7%)  Present Value of an Ordinary Annuity Factor] Since the proceeds (i.e., present value) are less than the face value, we know that the note was issued at a discount. Consequently, the effective rate must be more than the stated rate. Try i = 9% for n = 5: ($1,000  .64993 from Table 4 in Appendix A) + [($1,000  7%)  3.88965 from Table 5 in Appendix A)] = $649.93 + $272.28 = $922 (rounded) Therefore, the annual effective interest rate must be 9%. Bond 1 $11,635 = ($10,000  Present Value Factor) + [($10,000  3%)  Present Value of an Ordinary Annuity Factor] Since the proceeds are greater than the face value, we know that the bond was issued at a premium. Consequently, the effective rate is less than the stated rate. Try i = 2% for n = 20: ($10,000  .67297 from Table 4 in Appendix A) + [($10,000  3%)  16.35143 from Table 5 in Appendix A] = $6,729.70 + $4,905.43 = $11,635 (rounded) The effective rate per period is 2%. Since there are two interest periods per year, the annual effective interest rate is 4%. Bond 2 $54,323 = ($50,000  Present Value Factor) + [($50,000  4.5%)  Present Value of an Ordinary Annuity Factor] Since the proceeds (i.e., present value) are greater than the face value, we know that the bond was issued at a premium. Consequently, the effective rate must be less than the stated rate. Try i = 4% for n = 30: ($50,000  .30832 from Table 4 in Appendix A) + [($50,000  4.5%)  17.29203 from Table 5 in Appendix A] = $15,416.00 + $38,907.07 = $54,323 (rounded) The effective rate per period is 4%. Since there are two interest periods per year, the annual effective interest rate is 8%.


E11–26 a. Since the bonds have a face value of $1,000 and they are selling for 89.16, an individual bond would have a present value of $891.60 ($1,000 x 89.16%). For these bonds to be attractive to an investor who requires an annual rate of return of 12%, the present value of the bonds' future cash flows discounted using a discount rate of 6% semiannually must be greater than or equal to $891.60. If the present value is less, the bonds would not provide an annual rate of return of at least 12%. The present value of the bonds using a discount rate per six-month period of 6% is: Present value (i = 6%, n = 16) PV of maturity receipt ($1,000  .39365) ............................................................................ PV of interest receipts [($1,000  4%)  10.10590)............................................................. Total present value....................................................................................

$ 393.65 404.24 $ 797.89

Since the present value of the future cash flows is less than $891.60, the bonds are providing an annual rate of return of less than 12%. Thus, the bonds do not provide the required rate of return, and they should not be considered for investment purposes. b. The annual effective interest rate that would make an investor indifferent to purchasing the bonds at 89.16 would be 10%, which implies a six-month rate of 5%. As proof: Present value (i = 5%, n = 16) PV of maturity receipt ($1,000  .45811) ............................................................................ PV of interest receipts [($1,000  4%)  10.83777)............................................................. Total present value (i.e., proceeds) ...........................................................

$ 458.11 433.51 $ 891.62

At an annual effective rate of 10%, an investor would be indifferent to purchasing the bonds.

E11–27 a. Bonneville issued the debt at a fixed rate, meaning that its outlay for interest will not change even if market interest rates change. A drop in market rates would not lower the interest paid by Bonneville; therefore, such a drop would increase the liability on the books of the company. In effect, the company would be paying above-market rates for its debt due to the fixed nature of the contract. b. Bonneville could manage the risk of fluctuating interest rates (and debt values) by entering into a hedging agreement known as an interest rate swap. Under a swap arrangement, Bonneville would agree with a counter party, most likely a large commercial bank, to receive periodic payments at a fixed rate of interest while also agreeing to pay out periodic payments that are tied to a floating rate of interest (Treasury Bills, for example). If market rates increase over the time period, Bonneville will make larger periodic payments under the swap agreement, while a drop in market rates will reduce the amount that Bonneville remits in periodic interest payments; regardless of changes in market rates, Bonneville will receive the same fixed payment from the counter party.


The fixed payment received under the swap agreement will match what the stated rate of interest requires Bonneville to pay in the long term debt contract. The effect of the interest rate swap is to hedge against changes in debt values due to changes in market rates; the variable interest payments from the swap agreement effectively eliminate those market value changes.

PROBLEMS P11–1 a. The present value of the future cash flows of this note equals $20,000. Since the effective rate of 10% equals the stated rate of 10%, the note will be issued at par value. Consequently, the face value of this note would be $20,000. b. The present value of the future cash flows of this note equals $20,000. Since the effective rate of 10% exceeds the stated rate of 0%, the note will be issued at a discount. The task is to determine what amount of cash paid at the end of two years discounted at 10% would equal $20,000 today. In other words, $20,000 $20,000 Face value

= (Face Value  Present Value Factor for i = 10%, n = 2) = Face Value  .8264 (from Table 4 in Appendix A) = $24,202 (rounded)

Consequently, the face value of this note would be $24,202. c.

Note A Cash (+A) ................................................................................................... Notes Payable (+L) .............................................................................. Issued note payable for cash. Note B Cash (+A) ................................................................................................... Discount on Notes Payable (–L) ................................................................ Notes Payable (+L) .............................................................................. Issued note payable for cash.

d. Note A Interest Expense (E, –SE) ........................................................................... Cash (–A)............................................................................................. Made interest payment. *

20,000 20,000

20,000 4,202 24,202

2,000* 2,000

$2,000 = $20,000  10%

Notes Payable (–L)..................................................................................... Cash (–A)............................................................................................. Made principal payment on note payable. Note B Interest Expense (E, –SE) ...........................................................................

20,000 20,000

1,782*


Discount on Notes Payable (+L) .......................................................... Amortized discount on notes payable. * $1,782

1,782

= $4,202 Initial discount – $2,420 discount amortized during 2012

Notes Payable (–L)..................................................................................... Cash (–A)............................................................................................. Made principal payment on note payable.

24,202 24,202

P11–2 a. The bonds will be issued at a discount. The bond market has determined that purchasers of Hartl Enterprises' bonds should earn an annual return on their investment of 10%. However, Hartl Enterprises is offering interest equal to only 8%. Since the stated interest rate cannot be changed, the only way that the investors can earn their 10% return is to invest a smaller amount in Hartl Enterprises. They will still receive the same future cash flows. Consequently, the bonds will be issued at a price that allows the investors to earn a return of exactly 10% on their investment. b. Face value ............................................................................................... Present value (i = 5%, n = 20) PV of maturity receipt ($10,000  .3769 from Table 4 in Appendix A) ............................ PV of interest receipts ($400  12.4622 from Table 5 in Appendix A) ............................. Total present value................................................................................. Discount ................................................................................................. Cash (+A) ................................................................................................... Discount on Bonds Payable (–L) ................................................................ Bonds Payable (+L) ............................................................................. Issued bonds. c.

Interest Expense (E, –SE) ........................................................................... Discount on Bonds Payable (+L) ......................................................... Interest Payable (+L) ........................................................................... Accrued interest payable. a $218.85 b $18.85 c $200.00

= = = = =

$ 10,000

$ 3,769 4,985 $

8,754 1,246

8,754 1,246 10,000

218.85a 18.85b 200.00c

Book value  Effective rate per period  Portion of period outstanding $8,754  5%  3/6 Interest expense – Interest payable Face value  Stated rate per period  Portion of period outstanding $10,000  4%  3/6

d. Interest Expense (E, –SE) ........................................................................... Interest Payable (–L).................................................................................. Discount on Bonds Payable (+L) ......................................................... Cash (–A)............................................................................................. Made interest payment.

218.85 200.00 18.85 400.00


P11–3 a. L-T Debt/Equity Ratio

= Total Long-Term Liabilities ÷ Total Stockholders' Equity = $40,000 ÷ $100,000 = 0.4

b. Proceeds = Present Value of Future Cash Flows Discounted at 11% for 5 Periods = $40,000  .59345 (from Table 4 in Appendix A) = $23,738 If Manheim Corporation borrows this $40,000, its long-term debt/equity ratio would be .637 [($40,000 + $23,738) ÷ $100,000]. c.

Proceeds = Present Value of Future Cash Flows Discounted at 4% for 40 Periods = Present Value of the Face Value + Present Value of Interest Payments = ($40,000  .20829 from Table 4 in Appendix A) + [($40,000  5%)  19.79277 (from Table 5 in Appendix A)] = $8,332 + $39,586 = $47,918 If Manheim Corporation issues these bonds, its long-term debt/equity ratio would be .879 [($40,000 + $47,918) ÷ $100,000].

P11–4 a. Note A Face value ........................................................................................... Present value (i = 10%, n = 5) PV of face value ($20,000  .6209 from Table 4 in Appendix A) ........................ PV of interest receipts ($0  3.7908 from Table 5 in Appendix A) ............................... Total present value (i.e., proceeds) .................................................... Discount ............................................................................................. Note B Face value ........................................................................................... Present value (i = 10%, n = 8) PV of face value ($35,000  .4665 from Table 4 in Appendix A) ........................ PV of interest receipts ($2,800  5.3349 from Table 5 in Appendix A) ........................ Total present value (i.e., proceeds) .................................................... Discount .............................................................................................

$ 20,000

$ 12,418 0 $

12,418 7,582

$ 35,000

$ 16,328 14,938 31,266 $ 3,734


P11–4

Concluded

Note C Face value ........................................................................................... Present value (i = 4%, n = 20) PV of face value ($50,000  .4564 from Table 4 in Appendix A) ........................ PV of interest receipts ($2,000  13.5903 from Table 5 in Appendix A) ...................... Total present value (i.e., proceeds) .................................................... Discount/premium ............................................................................. b. Note A Cash (+A) ................................................................................................... Discount on Notes Payable (–L) ................................................................ Notes Payable (+L) .............................................................................. Issued notes payable for cash. Note B Cash (+A) ................................................................................................... Discount on Notes Payable (–L) ................................................................ Notes Payable (+L) .............................................................................. Issued notes payable for cash. Note C Cash (+A) ................................................................................................... Notes Payable (+L) .............................................................................. Issued notes payable for cash. c.

Interest Expense (E, –SE) ........................................................................... Cash (–A)............................................................................................. Incurred and paid interest.

d. Note B Interest Expense (E, –SE) ........................................................................... Discount on Notes Payable (+L) .......................................................... Cash (–A)............................................................................................. Incurred and paid interest. a $3,126.60 b $326.60 c

$2,800.00

$ 50,000

$ 22,820 27,181 50,000 $ 0

12,418.00 7,582.00 20,000.00

31,266.00 3,734.00 35,000.00

50,000.00 50,000.00

2,000.00 2,000.00

3,126.60a

= Book Value  Effective Rate per Period = ($35,000 – $3,734)  10% = Interest Expense – Interest Payment = Face Value  Stated Rate per Period = $35,000  8%

326.60b 2,800.00c


P11–4

Continued

Note C Interest Expense (E, –SE) ........................................................................... Cash (–A)............................................................................................. Incurred and paid interest.

2,000.00 2,000.00

e. Interest Expense (E, –SE) ........................................................................... 1,241.80* Discount on Notes Payable (+L) .......................................................... Amortized discount on notes payable. _________________ * $1,241.80 = Book Value  Effective Rate per Period = ($20,000 – $7,582)  10%

1,241.80

P11–5 a. The effective interest rate can be calculated in two ways. The first way is by solving for i in each of the following equations. Note A: Note B: Note C:

$37,566 = [$50,000  (1 + i)-3] $50,000 = [$50,000  (1 + i)-3] + {$5,000  [(1 - (1+ i)-3) ÷ i]} $45,027 = [$50,000  [(1 + i) -3] + {$3,000  [(1 - (1+ i)-3) ÷ i]}

The second way is by trial and error. Plug an interest rate into the equations until the right-hand side of the equation equals the left-hand side. The annual effective interest rate is 10%. b.

Interest Expense (Note A) Year 1 Year 2 Year 3 Total

c.

3,756.60a 4,132.26b 4,545.14c $ 12,434.00 $

Interest Expense (Note B) $

5,000.00 5,000.00 5,000.00 $ 15,000.00

Interest Expense (Note C) 4,502.70d 4,652.97e 4,817.33f $ 13,973.00 $

Interest Expense = Book Value at Beginning of Period  Effective Rate a $3,756.60 = Initial Book Value of $37,566  10% b $4,132.26 = ($37,566 + $3,756.60)  10% c $4,545.14 = ($50,000 – $37,566) – ($3,756.60 + $4,132.26) d $4,502.70 = Initial Book Value of $45,027  10% e $4,652.97 = [$45,027 + ($4,502.70 – $3,000.00)]  10% f $4,817.33 = ($50,000 – $45,027) – ($4,502.70 – $3,000.00) – ($4,652.97 – $3,000.00) + $3,000 Note A


Return

Expense

4,507.92a 5,048.88b 5,654.73c $ 15,211.52

Year 1 Year 2 Year 3 Total

$

Income

$

3,756.60 4,132.26 4,545.14 $ 12,434.00

$

751.32 916.61 1,109.59 $ 2,777.52

a $4,507.92 = $37,566  12% b $5,048.87 = ($37,566 + $4,507.92)  12% c

$5,654.73 = ($37,566 + $4,507.92 + $5,048.87)  12%

P11–5

Concluded

Note B Year 1 Year 2 Year 3 Total

Return

Expense

6,000.00a 6,720.00b 7,526.40c $ 20,246.40 $

Income

$

5,000.00 5,000.00 5,000.00 $ 15,000.00

$ 1,000.00 1,720.00 2,526.40 $ 5,246.40

a $6,000.00 = $50,000  12% b $6,720.00 = ($50,000 + $6,000)  12% c

$7,526.40 = ($50,000 + $6,000 + $6,720)  12%

Note C

Return

Expense

Income

Year 1 Year 2 Year 3 Total

5,403.24a

$ 4,502.70 4,652.97 4,817.33 $ 13,973.00

$

$

6,051.63b 6,777.82c $ 18,232.69

900.54 1,398.66 1,960.49 $ 4,259.69

a $5,403.24 = $45,027  12% b $6,051.63 = ($45,027 + $5,403.24)  12% c

$6,777.82 = ($45,027 + $5,403.24 + $6,051.63)  12%

d. Total Debt = Current Liabilities as of 12/31/12 + (Long-Term Liabilities as of 12/31/12 + Face Value of Notes Payable – Discount Balance) Total Stockholders' Equity = Stockholders' Equity as of 12/31/12 + Net Income Debt/Equity Ratio = Total Debt ÷ Total Stockholders' Equity

12/31/13 12/31/14 12/31/15

Debt/Equity (Note A)

Debt/Equity (Note B)

Debt/Equity (Note C)

4.270 4.277 4.271

4.516 4.279 3.972

4.418 4.278 4.086

e. Boyton must consider at least four factors in deciding which note to issue. First, the company must consider the income that can be earned from the proceeds. Since Note B provides the largest proceeds,


this note provides the highest net income. Second, the company must consider the cash outflow effects of each note. If the company did not have sufficient cash on hand to meet an interest or principal payment, it could be forced into bankruptcy. Since each note requires a payment at maturity of $50,000, the only difference between the notes is the periodic interest payments. In this case, Note A requires the lowest interest payments. Third, the company must consider the immediate effects on its debt/equity ratio. If the company has any existing debt with a debt covenant that specifies a maximum debt/equity ratio, one of the notes may cause the company to violate the debt covenant. In this case, Note A results in the lowest debt/equity ratio in the current year. Finally, the company must consider the trend in the debt/equity ratio over time. If the company needs or desires to issue additional debt in the future, it might be constrained by its future debt/equity ratio. Creditors might be wary of a company with too high of a debt/equity ratio. In this case, the decrease in the debt/equity ratio is greatest for Note B.

P11–6 a. The Amount of Interest Payments

= Face Value of Debt  Stated Interest Rate = $800,000  10% = $80,000

b. When the note payable was issued, the stated interest rate did not equal the effective interest rate; the effective interest rate exceeded the stated interest rate. Consequently, the proceeds from the note were less than face value, so that the entire loan to Rix Driving Range and Health Club would actually earn the effective interest rate on its money. The excess of the face value over the proceeds gave rise to the Discount on Notes Payable, and from Rix's viewpoint, this account effectively represents prepaid interest. Over the life of the note, this discount will be amortized to Interest Expense. Consequently, the difference between the balance in Interest Expense and the cash paid out for interest payments represents the amortization of the Discount on Notes Payable. c.

Interest Expense $95,000 Rate

= Book Value at Beginning of the Period  Effective Interest Rate = ($800,000 – $70,000)  Effective interest rate = 13% (rounded)

d. Interest Expense (E, –SE) ........................................................................... Discount on Notes Payable (+L) .......................................................... Cash (–A)............................................................................................. Incurred and paid interest.

95,000 15,000 80,000

P11–7 a. Face value .......................................................................................... Present value (i = 4%, n = 12) PV of cash payment at maturity ($20,000  0.6246 from Table 4 in Appendix A) ..................... PV of cash interest payments ($600  9.3851 from Table 5 in Appendix A) .......................... Total present value............................................................................ Discount on bonds............................................................................. b. Face value .......................................................................................... Present value (i = 4%, n = 11)

$ 20,000.00

$ 12,492.00 5,631.06 18,123.06 $ 1,876.94 $ 20,000.00


PV of cash payment at maturity ($20,000  0.6496 from Table 4 in Appendix A) ..................... PV of cash interest payments ($600  8.7605 from Table 5 in Appendix A) .......................... Total present value............................................................................ Discount on bonds.............................................................................

$ 12,992.00 5,256.30 $

18,248.30 1,751.70

The present value of the cash flows on these bonds as of December 31, 2012, using the effective interest rate on the date the bonds were originally issued, represents the book value of the bonds as of December 31, 2012. c.

The difference of $125.24 in present values from June 30, 2012 and December 31, 2012 represents the change in book value of these bonds for this six-month period. The change in book value would be captured by the amortization of the Discount on Bonds Payable account.

P11–7

Concluded

d. Interest Expense (E, –SE) ........................................................................... Discount on Bonds Payable (+L) ......................................................... Cash (–A)............................................................................................. Incurred and paid interest. a $724.92 b $124.92 c

$600.00

724.92a 124.92b 600.00c

= Book Value  Effective Rate per Period = $18,123.06  4% = Interest Expense – Interest Payment = Face Value  Stated Rate per Period = $20,000  3%

The amount of discount on bonds payable is essentially the same as the amount in part (c); the difference of 32¢ is due to rounding. Under the effective-interest method, bonds are carried on the balance sheet at their present value (based upon the effective rate at the initial date of issue) at that particular point in time. Hence, it makes no difference if one computes the present value of the cash outflows associated with bonds or applies the effective-interest method; both methods will yield essentially identical financial statements.

P11–8 a. To compute the amount of money that Ross Running Shoes must invest on June 30, 2012, the future cash flows must be discounted at the investment rate of 8%. Since the investment rate is an annual rate, and interest is paid semiannually, the rate must be adjusted to a six-month rate of 4%. Therefore, i = 4% and n = 6. Present Value

= Present Value of Face Value + Present Value of Interest Payments = ($10,000  .79031 from Table 4 in Appendix A) + [($10,000  5%)  5.24214 from Table 5 in Appendix A] = $7,903.10 + $2,621.07 = $10,524.17

b. Interest Expense (E, –SE) ........................................................................... Premium on Notes Payable (–L) ................................................................

420.97a 79.03b


500.00c

Cash (–A)............................................................................................. Incurred and paid interest. a $420.97 = Book Value  Effective Rate per Period = ($10,000 + $524.17)  4% b $79.03 = Interest Expense – Interest Payment c

c.

$500.00 = Face Value  Stated Rate per Period = $10,000  5%

Interest Expense (E, –SE) ........................................................................... Premium on Notes Payable (–L) ................................................................ Cash (–A)............................................................................................. Incurred and paid interest.

412.64a 87.36b 500.00c

a $412.64 = Interest Payment – Premium Amortization b $87.36 = Total Premium ÷ Number of 6-Month Periods = $524.17 ÷ 6 periods c

P11–8

$500.00 = Face Value  Stated Rate per Period = $10,000  5%

Concluded

d. Under the effective-interest method, the company will recognize interest expense during 2012 of $420.97 [from part (b)]. Under the straight-line method, the company will recognize interest expense during 2012 of $412.64 [from part (c)]. Thus, the straight-line method results in lower expenses and higher net income in the early periods of a note issued at a premium. e. Over the life of a note or bond, both the effective-interest and straight-line methods will amortize the entire discount or premium balance. Consequently, over the life of a note or bond, both methods will amortize exactly the same amount of discount or premium. As noted in part (d), for notes issued at a premium, the straight-line method will recognize lower interest expense than the effective-interest method in the early years of the note's life. The lower interest expense recognized under the straightline method will eventually have to be offset if both methods are to recognize the same amount of interest expense over the life of the note. Consequently, the straight-line method will have to recognize “relatively” higher interest expense and, hence, lower net income in the later years of a note issued at a premium.

P11–9 a. Note A 1/1/12 Present value (i = 6%, n = 3) PV of face value ($1,000  .8396) PV of interest payment ($100  2.6730) Total present value 12/31/13 Present value (i = 6%, n = 1) PV of face value

$

839.60

267.30 $ 1,106.90

12/31/12 Present value (i = 6%, n = 2) PV of face value ($1,000  .8900) PV of interest payment ($100  1.8334) Total present value

$

890.00

183.34 $ 1,073.34


($1,000  .9434) PV of interest payment ($100  .9434) Total present value Note B 1/1/12 Present value (i = 10%, n = 3) PV of face value ($1,000  .75132) PV of interest payment ($100  2.48685) Total present value 12/31/13 Present value (i = 10%, n = 1) PV of face value ($1,000  .90909) PV of interest payment ($100  .90909) Total present value

$

943.40

94.34 $ 1,037.74

$

751.32

248.69 $ 1,000.00

$

909.10

90.91 $ 1,000.00

12/31/12 Present value (i = 10%, n = 2) PV of face value ($1,000  .82645) PV of interest payment ($100  1.73554) Total present value

$

826.45

173.55 $ 1,000.00


P11–9

Continued

Note C 1/1/12 Present value (i = 10%, n = 3) PV of face value ($1,000  .75132) PV of interest payment ($60  2.48685) Total present value 12/31/13 Present value (i = 10%, n = 1) PV of face value ($1,000  .90909) PV of interest payment ($60  .90909) Total present value

$ 751.32 149.21 $ 900.53

12/31/12 Present value (i = 10%, n = 2) PV of face value ($1,000  .82645) PV of interest payment ($60  1.73554) Total present value

$ 909.09 54.55 $ 963.64

b. Interest Expense

Payment Amount

Disc./Prem. Amortization

Face Value

Note A 1/1/12 12/31/12 12/31/13 12/31/14

$66.42 64.40 62.26

$100.00 100.00 100.00

$33.58 35.60 37.74

$1,000.00 1,000.00 1,000.00 1,000.00

$106.90 $1,106.90 73.32 1,073.32 37.71 1,037.73 ($0.00) 1,000.00

Note B 1/1/12 12/31/12 12/31/13 12/31/14

$100.00 100.00 100.00

$100.00 100.00 100.00

$0.00 0.00 0.00

$1,000.00 1,000.00 1,000.00 1,000.00

$0.00 $1,000.00 0.00 1,000.00 0.00 1,000.00 0.00 1,000.00

Note C 1/1/12 12/31/12 12/31/13 12/31/14

$90.05 93.06 96.36

$60.00 60.00 60.00

$30.05 33.06 36.37

$1,000.00 1,000.00 1,000.00 1,000.00

$99.48 69.43 36.37 ($0.00)

Date

Disc./Prem. Balance

Book Value

$900.52 930.57 963.63 1,000.00

$ 826.45 104.13 $ 930.58


P11–9

Concluded

c. Date

Interest Expense

Payment Disc./Prem. Amount Amortization

Face Value

Disc./Prem. Balance

Book Value

Note A 1/1/12 12/31/12 12/31/13 12/31/14

$64.37 64.37 64.37

$100.00 100.00 100.00

$35.63 35.63 35.63

$1,000.00 1,000.00 1,000.00 1,000.00

$106.90 $1,106.90 71.27 1,071.27 35.63 1,035.63 0.00 1,000.00

Note B 1/1/12 12/31/12 12/31/13 12/31/14

$100.00 100.00 100.00

$100.00 100.00 100.00

$0.00 0.00 0.00

$1,000.00 1,000.00 1,000.00 1,000.00

$0.00 $1,000.00 0.00 1,000.00 0.00 1,000.00 0.00 1,000.00

Note C 1/1/12 12/31/12 12/31/13 12/31/14

$26.84 26.84 26.84

$60.00 60.00 60.00

$33.16 33.16 33.16

$1,000.00 1,000.00 1,000.00 1,000.00

$99.48 66.32 33.16 ($0.00)

$900.52 933.68 966.84 1,000.00

d. Compare parts (b) and (c) to part (a). The effective interest method maintains the net book value of the liability equal to the present value of the future cash flows of the liability throughout the liability's life. Alternatively, the straight-line method does not maintain this equality. Further, under the effective interest method, interest expense is always the same percentage of the outstanding debt throughout the life of the liability. This constant relationship arises because interest expense is computed as the book value times the effective interest rate, and since the effective interest rate is assumed to be constant, interest expense remains a constant percentage of the liability. The straight-line method does not result in this constant relationship between interest expense and the outstanding liability, as evidenced by the amounts reported under interest expense in part (c).

P11–10 a. Book Value of Debt

= Face Value of $500,000 + Premium Balance of $12,600 = $512,600

Cash Paid to Retire Debt

= Face Value  104% = $500,000  104% = $520,000

Loss = Excess of Cash Paid Over Book Value = $512,600 – $520,000 = $7,400 b. Cash Paid to Retire Debt

= Face Value  108% = $500,000  108% = $540,000


P11–10 Concluded Loss = Excess of Cash Paid Over Book Value = $540,000 – $512,600 = $27,400 c.

Ginny and Bill Eateries is required to make an interest payment on June 30, 2012 under the terms of the debt agreement. The entry to record this payment would be: Interest Expense (E, –SE) ........................................................................... Premium on Bonds Payable (–L) ............................................................... Cash (–A)............................................................................................. Incurred and paid interest.

15,378a 4,622b 20,000c

a $15,378 = Book Value  Effective Rate per Period = $512,600  3% b $4,622 = Interest Expense – Interest Payment c

$20,000 = Face Value  Stated Rate per Period = $500,000  4%

Book Value of Debt = Face Value + Premium Balance = $500,000 + ($12,600 – $4,622) = $507,978 Cash Paid to Retire Debt

= Face Value  110% = $500,000  110% = $550,000

Loss = Excess of Cash Paid Over Book Value = $550,000 – $507,978 = $42,022

P11–11 a. Face value ........................................................................................... Present value (i = 7%, n = 10) PV of face value ($5,000  .5083 from Table 4 in Appendix A) .......................... PV of interest payments ($300  7.0236 from Table 5 in Appendix A) ........................... Total present value (i.e., proceeds) .................................................... Discount ............................................................................................. Cash (+A) ................................................................................................... Discount on Bonds Payable (–L) ................................................................ Bonds Payable (+L) ............................................................................. Issued bonds.

$ 5,000.00

$ 2,541.50 2,107.08 4,648.58 $ 351.42 4,648.58 351.42 5,000.00


P11–11 Continued b. Interest Expense (E, –SE) ........................................................................... Discount on Bonds Payable (+L) ......................................................... Cash (–A)............................................................................................. Incurred and paid interest.

325.40a 25.40b 300.00c

a $325.40 = Book Value  Effective Rate per Period = $4,648.58  7% b $25.40 = Interest Expense – Interest Payment. c

c.

$300.00 = Face Value  Stated Rate per Period = $5,000  6%

As of June 30, 2014, the bonds have a remaining life of five six-month periods until they mature. Option 1: Repurchase the bonds through the bond market. Present value (i = 5%, n = 5) PV of face value ($5,000  .7835 from Table 4 in Appendix A) .......................... PV of interest payments ($300  4.3295 from Table 5 in Appendix A) ........................... Total present value (i.e., repurchase price) .......................................

$ 3,917.50 1,298.85 $ 5,216.35

Option 2: Repurchase the bonds using the call provision. Repurchase Price

= Face Value  103.5% = $5,000  103.5% = $5,175.00

In this case, Ficus Tree Farm would have to use less cash to redeem the bonds using the call provision than to repurchase them through the bond market. Consequently, the company should use the call provision to redeem the bonds. d. Assume that a company wishes to redeem all outstanding bonds prior to maturity. It is unlikely that it could accomplish this goal by repurchasing the bonds through the bond market. Some bondholders would simply be unwilling to sell the bonds. It is costly for bondholders to sell their bonds and reinvest. They incur transaction costs (i.e., brokerage fees, etc.) when selling investments. It is also timeconsuming (for example, the opportunity cost of researching new investment opportunities). Bondholders would also consider the tax implications of selling their bonds. If the bondholder would have to recognize any gains on the sale of the bond, these gains would be considered taxable income. To avoid these taxes, the bondholder may prefer to simply hold the bond. By exercising a call provision, a company can compel all bondholders to surrender their bonds. Consequently, if a company wishes to retire all outstanding bonds, the company will have to resort, at least partially, to exercising any relevant call provisions.


P11–11 Concluded e. Bonds Payable (–L) .................................................................................... Extraordinary Loss on Redemption (E, –SE) .............................................. Discount on Bonds Payable (+L) ......................................................... Cash (–A)............................................................................................. Redeemed bonds. *see table Date

Interest Expense

1/1/12 6/30/12 1/1/13 6/30/13 1/1/14 6/30/14

Payment Disc./Prem. Amount Amortization

$325.40 327.18 329.08 331.12 333.30

$300.00 300.00 300.00 300.00 300.00

$25.40 27.18 29.08 31.12 33.30

Face Value

5,000.00 380.35 205.35* 5,175.00

Disc./Prem. Balance

$5,000.00 5,000.00 5,000.00 5,000.00 5,000.00 5,000.00

Book Value

$351.42 $4,648.58 326.02 4,673.98 298.84 4,701.16 269.76 4,730.24 238.64 4,761.36 205.35 4,794.65

P11–12 a. Face value ........................................................................................ Present value (i = 5%, n = 8) PV of face value ($100,000  .67684 from Table 4 in Appendix A) ................. PV of interest payments ($3,000  6.46321 from Table 5 in Appendix A) ................... Total present value.......................................................................... Discount .......................................................................................... Date

Interest Expensea

Cash Paymentb

12/31/12 6/30/13 12/31/13 6/30/14 12/31/14 6/30/15 12/31/15 6/30/16

$4,353,70 4,421.39 4,492.45 4,567.08 4,645.43 4,727.70 4,814.09 4,904.79

$3,000.00 3,000.00 3,000.00 3,000.00 3,000.00 3,000.00 3,000.00 3,000.00

$ 100,000

$ 67,684 19,390

Amortized Discountc

$1,353.70 1,421.39 1,492.45 1,567.08 1,645.43 1,727.70 1,814.09 1,904.79

87,074 $ 12,926 Book Valued

$ 88,427.70 89,849.09 91,341.54 92,908.62 94,554.05 96,281.75 98,095.84 100,000.00

a Interest Expense = Book Value at the Beginning of the Period  Effective Rate per Period of 5% b Cash Payment = Face Value of $100,000  Stated Rate per Period of 3% c

Unamortized Discount = Unamortized Discount at the Beginning of the Period – Excess of Interest Expense Over the Cash Payment d Book Value = Face Value of $100,000 – Unamortized Discount


P11–12 Concluded b. Cash outflows Total interest payments

= =

$3,000  8 payments $24,000

Total principal payment

=

$100,000 on maturity of the bonds

Total cash outflow

= =

$24,000 + $100,000 $124,000

= =

Proceeds received upon issuing the bonds $87,073

Cash inflows Cash inflows

Therefore, cash outflows exceed cash inflows by $36,927. c.

Cash outflows Post-tax interest payments

= = =

[$3,000  (1 – tax rate)]  8 payments [$3,000  (1 – 34%)]  8 payments $15,840

Total principal payment

=

$100,000 on maturity of the bonds

Total cash outflow

= =

$15,840 + $100,000 $115,840

= =

Proceeds received upon issuing the bonds $87,073

Cash inflows Cash inflows

Therefore, cash outflows exceed cash inflows by $28,767. d. Cash outflows Individual post-tax interest payments = $3,000  (1 – tax rate) = $1,980 = $1,980  Present value of an ordinary annuity

Present value of post-tax payments = 5% and n = 8)

= $1,980  6.4632 from Table 5 in Appendix A = $12,797 Total principal payment

= $100,000 on maturity of the bonds

Present value of principal payment Total cash outflow

= $67,684 [from part (a)] = $12,797 + $67,684 = $80,481

Cash inflows Cash inflows

= =

Proceeds received upon issuing the bonds $87,073

Therefore, cash inflows exceed cash outflows by $6,592.

for i


P11–13 a. On the financial statements a capital lease is treated like the company had purchased the fixed assets. The asset and the related liability are recorded on the balance sheet and interest and depreciation are recorded on the income statement. An operating lease is treated like a recurring expense each month but nothing is recorded on the balance sheet. The amount of the lease payment is shown as an expense each month. b. A company may want to treat leases as operating leases because there is no debt that is recorded on the balance sheet. This treatment impacts a number of financial ratios (debt-to-equity, for example) and so may be to the company’s advantage to treat it like an operating lease. c.

total liability ÷ total asset ratio if Wal-Mart treats these leases as: currently recorded: $98 ÷ $163 = 60.1% if all leases are capital leases: $106.1 (98 + 8.1) ÷ $171.1 (163 + 8.1) = 62.0% $3.5 ÷ $5.5 = 63.6%, 63.6% of $12.8 billion = $8.1 billion

d. An analyst needs to be able to compare companies that use different methods for accounting for leases. If an analyst does not do this additional analysis there is a good chance that the analyst will be misled as to the relative performance of the companies. The more leases that the companies have on their books, the more this difference between operating leases and capital leases could affect the analysis of the companies. A review of “off-balance sheet financing” is always a prudent step in financial analysis.

P11–14 a. The initial balance sheet value of the equipment and the initial leasehold obligation both equal the present value of the lease payments. This amount can be determined in the following ways. Present value of lease payments

= FMV of equipment = $119,782

or Present value of lease payments

= Present value of lease payments = $30,000  Present value of an ordinary annuity factor for i = 8% and n = 5 = $30,000  3.99271 (from Table 5 in Appendix A) = $119,781.30


P11–14

Concluded

Balance Sheet Value of Leasehold Date Equipmenta Obligationb 1/1/11 $119,781.30 $119,781.30 12/31/11 95,825.04 99,363.80 12/31/12 71,868.78 77,312.91 12/31/13 47,912.52 53,497.94 12/31/14 23,956.26 27,777.78 12/31/15 (0.00) (0.00) Total

Interest Expensec

Depr. Expensed

Total Expense

$9,582.50 7,949.10 6,185.03 4,279.84 2,222.22 $30,218.70

$23,956.26 23,956.26 23,956.26 23,956.26 23,956.26 $119,781.30

$33,538.76 31,905.36 30,141.29 28,236.10 26,178.48 $150,000.00e

a Balance Sheet Value of Equipment = Value of Equipment on 1/1/11 – Accum. deprec. b Leasehold Obligation = Leasehold Obligation at Beginning of the Period – ($30,000

Lease Payment – Interest Expense for the Period) Interest Expense = Leasehold Obligation at Beginning of the Period  8% d Depreciation Expense = $119,781.30 ÷ 5 years e Total has penny discrepancy due to rounding to even cents throughout lease term. c

= Annual Rent Payments  Number of Years of the Lease = $30,000  5 years = $150,000

b. Total Rent Expense

c.

If the lease is treated as a capital lease, total expenses would be $150,000 [from part (a)]. If the lease is treated as an operating lease, total expenses would still be $150,000 [from part (b)]. Although total expenses would be the same under either approach, different expense accounts are affected under the two approaches. With a capital lease, the $150,000 is allocated between interest expense and depreciation expense, while with an operating lease, the entire $150,000 is allocated to rent expense.

P11–15 a. If the lease is treated as an operating lease, Thompkins Laundry would not have to report any liability associated with the lease. Therefore, its debt/equity ratio would be as follows. Debt/Equity Ratio

= = = =

Total Liabilities ÷ Stockholders' Equity (Current Liabilities + Long-Term Liabilities) ÷ Stockholders' Equity $30,000 ÷ $40,000 0.75

b. If the lease is treated as a capital lease, Thompkins Laundry would have to report a liability equal to the present value of the future lease payments. Therefore, its debt/equity ratio would be affected. Present value of lease payments

= $5,000  Present value of an ordinary annuity factor for i = 12% and n = 5 = $5,000  3.60478 (from Table 5 in Appendix A) = $18,023.90


Debt/equity ratio

P11–15

= ($30,000 + $18,023.90) ÷ $40,000

=

1.20

Concluded

c.

Rent Expense Operating lease Capital lease

$5,000.00 0.00

Interest Expense $

0.00 2,162.87

Depreciation ___Expense_ $

0.00 3,604.78

Total Expenses $5,000.00 5,767.65

d. There are two primary reasons why Thompkins Laundry might want to arrange the terms of the lease agreement so that the lease would be classified as an operating lease rather than as a capital lease. First, lease obligations under an operating lease are not disclosed on the face of the balance sheet. Consequently, operating leases are essentially off-balance-sheet financing and will not affect any existing debt covenants that are based on reported liabilities. Second, in this case the capital lease classification results in higher expenses and, hence, lower net income in 2012 than the operating lease classification. Decreased net income would adversely affect any contracts, such as the manager's incentive contract, written on the basis of reported net income. To avoid classifying this lease as a capital lease, Thompkins Laundry would have to arrange the terms so that the lease did not meet any of the criteria for capital leases. Consequently, the company would have to arrange the terms so that: (1) (2) (3) (4)

the present value of the lease payments is less than 90% of the fair market value of the leased property; the term of the lease is less than 75% of the leased property's life; the lessee does not have the right either during or at the expiration of the lease agreement to purchase the property from the lessor at a nominal amount; or ownership of the property is not transferred to the lessee from the lessor by the end of the lease term.

P11–16 a. Equipment (+A) ......................................................................................... Discount on Notes Payable (–L) ................................................................ Notes Payable (+L) .............................................................................. Purchased equipment in exchange for a note. b. Present Value $17,604

17,604 2,396 20,000

= Present Value of Maturity Payment + Present Value of Periodic Payments = ($20,000  Present Value Factor) + ($1,000  Present Value of an Ordinary Annuity Factor)

Since the present value of $17,604 is less than the face value, we know that the note was issued at a discount. Consequently, the effective rate is greater than the stated rate. We also know that the stated rate is 5% ($1,000 ÷ $20,000 face value). Try i = 6% for n = 5: ($20,000  .74726 from Table 4 in Appendix A) + ($1,000  4.21236 from Table 5 in Appendix A) = $14,945 + $4,212 = $19,158 Try i = 8% for n = 5 ($20,000  .68058 from Table 4 in Appendix A) + ($1,000  3.99271 from Table 5 in Appendix A) = $13,612 + $3,992 = $17,604


Therefore, the effective interest rate on the note is 8%.

P11–16 c.

Concluded 1,408a

Interest Expense (E, –SE) ........................................................................... Discount on Notes Payable (+L) .......................................................... Cash (–A)............................................................................................. Incurred and paid interest.

408b 1,000

a $1,408 = Book Value  Effective Rate per Period = ($20,000 – $2,396)  8% b $408 = Interest Expense – Interest Payment

d. 12/31/12 Net Book Value

= = =

Face Value – 12/31/12 Discount on Notes Payable $20,000 – ($2,396 – $408) $18,012

P11–17 a. Since the bonds are selling at par value, the effective interest rate must be equal to the stated interest rate of 9%. The effective interest rate is the sum of two components: a risk-free component and a risk premium. It is given in the problem that the risk-free rate is 7%, which implies that the risk premium on Hodge Sports, bonds must be the difference between the effective interest rate of 9% and the risk-free rate of 7%, or 2%. b. If the risk premium increased from 2% to 5%, the effective interest rate would increase to 12%. A single bond would now be worth $889.59 to you, as calculated below. (Remember that bonds usually have a face value of $1,000 and pay interest semiannually.) Present value (i = 6%, n = 10) Present value of face value ($1,000  .55839 from Table 4 in Appendix A) ............................... Present value of interest payments ($45  7.36009 from Table 5 in Appendix A) .................................. Total present value.................................................................................... c.

$ 558.39 331.20 $ 889.59

A decrease in the prime interest rate would probably result in a drop in the effective interest rate used to discount the future cash flows of Hodge Sports’ bonds. As the effective interest rate drops, the stated interest rate looks relatively more attractive to investors. Thus, demand for the bonds should increase, which, in turn, should drive up the selling price of the bonds. A single bond would now be worth $1,040.55, as calculated below. Present value (i = 4%, n = 10) Present value of face value ($1,000  .67556 from Table 4 in Appendix A) ............................... Present value of interest payments ($45  8.11090 from Table 5 in Appendix A) .................................. Total present value....................................................................................

$

675.56

364.99 $ 1,040.55


P11–18 a. The effective interest rate on the bonds is 8%. The future value of the bond payments are $2,000 (semi-annual interest payment based on the stated rate of 4%) for four periods and $100,000 (principal due at maturity); the present value is the purchase price of $92,994. The effective rate of 8% discounts the future values to the present value. (The general present value formula of 1/[(1 + r) to the nth] was used in this calculation.) b. Cash 2,000 Bond Investment 1,720 Interest Revenue 3,720 Receipt of interest payment on 11/30/2011 (3,720 = Eff. Rate per period of 4% X $92,994)

Cash 2,000 Bond Investment 1,789 Interest Revenue 3,789 Receipt of interest payment on 5/31/2012 (3,789 = Eff. Rate per period of 4% X [92,994 + 1,720]) c. On May 31, 2012 the book value of the investment is $96,503 (92,994 + 1,720 + 1,789). On the same date, if market interest rates are 6% the market value of the investment is $98,087 (PV of a $2,000 ordinary annuity, n=2, r = 3 plus PV of a single sum of $100,000, n = 2, r = 3).


ISSUES FOR DISCUSSION ID11–1 a. A debenture is an unsecured bond. That is, there is no collateral supporting the bond. Thus, should the company not repay the bonds, investors do not have security in any of the company's assets that could be sold to repay the bonds. For this reason, unsecured bonds are riskier than secured bonds. Investors are compensated for this increased risk on debentures through a higher return (i.e., effective interest rate). Accordingly, these bonds would be priced lower than a secured bond in the same company. b. There are three general reasons why a company would repurchase its outstanding debt. First, the company may no longer need the money it borrowed. By repurchasing the debt, the company could avoid incurring interest. Second, due to a decrease in interest rates, the company may have repurchased its debt with the intent of issuing new debt at the lower prevailing interest rates. Finally, the company may repurchase some of its debt in an effort to improve its balance sheet. This would generally be in an effort to improve some financial ratios specified in debt covenants. c.

Repurchasing debt would decrease both a company's liabilities (due to the amount of debt repurchased) and its assets (due to the cash paid out to repurchase the debt). For Sun Company, its stockholders' equity would also decrease because it paid out $957.50 for each bond when the book value of a bond was only $875. Thus, Sun Company would have a loss of $82.50 on each bond repurchased, which would decrease stockholders' equity through closing the loss into Retained Earnings.

d. Sun Company would not have recognized any loss if it had not repurchased its debt. Unless there is evidence to the contrary, such as a company repurchasing its debt, accountants assume that when a company issues debt, the debt will remain outstanding until it matures. This implies that changes in the market value of the debt are irrelevant to the financial position of the company as reported in its financial statements.

ID11–2 a. The stated interest rate affects only the magnitude of periodic interest payments. What is important to investors is the rate of return on their investments. Thus, if an investor is not in need of periodic cash payments, a non-interest-bearing obligation that provides a competitive rate of return is an attractive investment option. b. The rate that discounts $200 million due in eight years to a present value of $66.48 million is 14.75%. c.

If bonds have a stated rate, the company has to have sufficient cash flow to make the periodic interest payments. Thus, if a company does not expect to have sufficient cash flows to support periodic interest payments, it is to the company's advantage to issue bonds with a stated interest rate of zero.


ID11–2 Concluded d. To simplify the calculations, the effective interest rate of 14.75% [see part (b)] is rounded to 15%. 5% stated rate Present value of $200 million paid in 8 years $200 million  .32690 (from Table 4 in Appendix A) ................................... Present value of periodic interest payments ($200 million  5%)  4.48732 (from Table 5 in Appendix A) ...................... Issue price........................................................................................................ 18% stated rate Present value of $200 million paid in 8 years $200 million  .32690 (from Table 4 in Appendix A) ................................... Present value of periodic interest payments ($200 million  18%)  4.48732 (from Table 5 in Appendix A) .................... Issue price........................................................................................................

$

65,380,000

44,873,200 $ 110,253,200

$

65,380,000

161,543,520 $ 226,923,520

ID11–3 a. The effective interest rate is the interest rate that equates the undiscounted future cash flows with the present value of the future cash flows. For both alternatives, the undiscounted cash flows are only the fifteen annual payments of $6 million each, and the present value of both alternatives is the $45,636,480 price of the jet plane. The equation to equate the undiscounted future cash flows and the present value is as follows. $45,636,480 = $6,000,000  {[1– (1 + i)-15] ÷ i]} Solving for i mathematically or by trial and error indicates that the annual effective interest rate is 10%. b. Cash (+A) ........................................................................................ Note Payable (+L) .................................................................... Issued note payable.

45,636,480

Airplane (+A)................................................................................... Cash (–A).................................................................................. Purchased airplane.

45,636,480

Airplanes Capitalized Under Leases (+A) ........................................ Lease Liability (+L) ................................................................... Leased airplane.

45,636,480

c.

45,636,480

45,636,480

45,636,480

d. If Southwest Airlines borrows the necessary funds and then purchases the airplane, Southwest's fixed assets and liabilities would both increase by $45,636,480. In addition, Southwest would have to depreciate the airplane. The effect on the financial statements would be identical if Southwest leases the airplane, and the lease is considered to be a capital lease. e. Southwest Airlines would not have to prepare any journal entry when it signs the lease if the lease is considered to be an operating lease.


ID11–3 Concluded f.

Structuring the leasing arrangement as an operating lease would be an example of off-balance sheet financing. With an operating lease, the substance of the lease arrangement is that Southwest is renting the airplane from the Boeing Company. Thus, Southwest is not considered to have any obligation to Boeing until Southwest actually uses the airplane. As Southwest uses the airplane, Southwest should record rent expense. This means that Southwest would never report any liability on its balance sheet associated with the future payments required under the lease agreement. Thus, Southwest would be able to finance the "acquisition" of an airplane without having to report any liability associated with acquiring the airplane. Southwest would most likely engage in off-balance sheet financing in order to prevent an increase in the amount of reported debt. By limiting any increases in reported debt, the company decreases the chance it would violate any debt covenants that specify a maximum debt/equity ratio. In addition, reporting less debt would make Southwest appear to be a less risky investment option. Since the return a company must pay on investment capital is positively associated with the risks of the company, anything that would make Southwest appear to be less risky should decrease its cost of capital. The real question, however, is whether potential investors, in evaluating investment alternatives, focus solely on the debt reported on the balance sheet or, instead, focus on the company's obligations reported in the footnotes to the financial statements. The footnotes would usually disclose any major obligations under operating leases.

ID11–4 a. The current portion of Long Term Debt ($221 million) appeared in the Current Liabilities section of the balance sheet; the rest of the Long Term Debt, totaling $8,120 million, appeared in the long-term liabilities section of Johnson & Johnson’s balance sheet. b. A zero coupon debenture is a debt instrument that has a stated rate of interest of 0%. The debenture contract only requires the repayment of the face amount at maturity. However, because no company borrows at zero percent, the debentures are sold at a discount depending on the effective rate of interest. The zero coupon debenture that are due in 2020 carry an effective interest rate of 3.00, indicating that Johnson & Johnson did not receive the face value of the debentures at funding but will have to repay the face value at maturity (meaning that the company effectively is paying a 3% interest rate to borrow the money). c.

A bond contract with a stated rate of interest equal to the effective rate of interest will be sold at par (no discount or premium). The 2033, 2023, 2017, 2038 and 2018 debentures carry effective rates equal to their stated rates, therefore selling at par.

d. The 6.95% notes due in 2029 were issued with an effective rate of interest of 7.14%, which is in excess of the stated rate of 6.95%. Therefore, the notes were sold at a discount, meaning that the face amount is greater than the balance sheet amount of $293 million.


ID11–5 a. For the market value of the liability to increase so dramatically, market interest rates would have had to fall substantially. b. Loss on long term debt 1,400 Long term debt 1,400 c.

Loss on retirement of debt 1,400 Long term debt

8,300 Cash

9,700

d. Carrying the debt at fair market value would require the company to book the $1.4 billion loss, even if the debt were not retired at that market price. On the other hand, carrying the debt at book value using the effective interest rate method would show the $1.4 billion loss only if the debt were retired at current market prices. If the debt were not retired, the loss would not be recorded. ID11–6 a. A large amount of debt forces a company's management to place greater emphasis on generating cash so that it has sufficient cash to make the required interest and principal payments. Thus, a company may alter its operating, investing, and financing decisions to allow it to generate the cash it needs when it needs it. b. The massive borrowing activity during the 1980s would have manifested itself as increased liabilities on the companies' balance sheets. By analyzing different companies' current ratios and debt/equity ratios, which are measures of a company's solvency, potential investors may have been able to identify those companies that were taking on an excessive amount of debt. However, even this type of analysis may not have been sufficient to identify overly risky companies. Companies will often engage in off-balance sheet financing, such as structuring leasing arrangements as an operating lease. Companies are most likely to engage in off-balance sheet financing when they are close to violating existing debt covenants that specify a maximum debt/equity ratio or when the company already has a large amount of debt. Since, by definition, off-balance sheet financing does not show up on the balance sheet as a liability, it will not be reflected in either the current ratio or the debt/equity ratio. An alternative analysis strategy investors could have used was to examine the statement of cash flows to determine whether the company was consistently generating enough cash from operating activities to service its debt. This approach is good in that any cash payments associated with off-balance sheet financing will be reflected on the statement of cash flows. The negative aspect of this analysis approach is that the analysis cannot be adequately performed until the company is making interest and principal payments. By this time it may be too late! c.

A debenture is an unsecured bond. That is, there is no collateral supporting the bond. Thus, should the company not repay the bonds, investors in debentures, unlike investors in secured bonds, do not have security in any of the company's assets that could be sold to repay the bonds. In other words, in the event a company liquidates, the secured creditors are paid before the unsecured creditors. This means that if the company does not have sufficient cash available after liquidating to repay both the secured creditors and the holders of debentures, it is the latter group that will not receive full payment.


ID11–7 a.

Home Depot

Liabilities Total assets

$23 billion $41 billion

Liabilities/total assets ratio

0.56

b. If all leases are capital leases: Home Depot Liabilities + Lease liabilities Total liabilities Assets + Lease liabilities Total Assets

$23 billion 2.7 billion $25.7 billion

$15 billion $33 billion 0.45

Lowe’s $15 billion 3.6 billion $18.6 billion

$41 billion $33 billion 2.7 billion 3.6 billion $43.7 billion $36.6 billion

Adjusted liabilities/total assets ratio 0.59

c.

Lowe’s

0.51

Both companies saw their adjusted debt ratio increase. In both cases, the adjustment doesn’t make much of a difference since the companies already have relatively high ratios. However, adding the leases to the balance sheet does increase the leverage of the companies.

d. An analyst may wish to make the adjustment required above so he/she can make equal comparisons between large retailers. By making the adjustment, the analyst knows that differences due to accounting method choices or lease reporting choices have been removed.

ID11–8 a. A ratings agency is a supposedly independent expert charged with the responsibility of analyzing the risks associated with debt (and equity) securities. Once the risks have been analyzed and quantified, the ratings agency assigns a grade (a “rating”) to the security. The lower the risk, the higher the rating; the higher the risk, the lower the rating. b. Investors interested in purchasing the security will perform their own analysis of the underlying risks but will also look to the rating as guidance. If the ratings agency assigns a low rating (implying greater risk), the investor will demand a higher interest rate to compensate for the risks involved. On the other hand, if the agency assigns a high rating (implying less risk), the investor does not require as high a rate of interest. As discussed in this chapter, the rate demanded by the investor (the effective rate of interest) determines the price the investor will pay to purchase the security and receive the stated rate of interest. If the rate demanded by the borrower (due to the rating of the security), exceeds the stated rate in the security, the investor will purchase the security at a discount (that is to say, the price of the security will be below the face value). c.

The ratings agency should look at a number of areas, including: the income and credit history of the individual borrowers (the homeowners), the value of the homes pledged as collateral, the amount of downpayment made by the borrowers (the initial equity in the house), and the overall health of the housing market, both nationally and in the local markets where the loans were made.


ID11–9 a. The covenant limits the company’s borrowing capacity by stating that funded debt can be no more than three times EBITDA (a rough estimation of annual cash flow). Since EBITDA was $1,604 million, funded debt could be no more than $4,812 million. With existing debt at $3,505 million, the covenant limits additional debt to no more than $1,307 million. b. The creditors are trying to control the amount of debt that J.C. Penney has on its balance sheet by limiting that debt to a multiple of annual cash flow. The thinking is that the debt will be repaid from that cash flow, so a reasonable limit on debt would be one that correlates the amount of debt with the amount of cash flow the company generates. If the company wants to borrow more, it needs to generate more cash flow; if the company (due to a recession or to poor operations) generates less cash flow, then it will be allowed to borrow less. c.

If J.C. Penney violates the financial covenant, the creditors have the right to immediately call the loans, requiring the company to pay them off at once. Most likely, however, would be a renegotiation of the credit agreements between the creditors and the company. Contract issues such as interest rates, fees, and future financial covenants might be changed by the lenders who would feel that they are carrying more risk due to the covenant violation.

ID11–10 a. On the financial statements a capital lease (a lease that is “equivalent to purchasing an asset”) is treated like the company had purchased the fixed asset. The asset and the related liability are recorded on the balance sheet and interest and depreciation are recorded on the income statement. An operating lease (a lease in the nature of short-term hire) is treated like a recurring expense each month but nothing is recorded on the balance sheet. The amount of the lease payment is shown as an expense each month. The result is that a company with an operating lease will show higher net income (in early years) and a lower debt/equity ratio than a company with a capital lease. b. The rule passed in 1981 was unpopular for 2 primary reasons. The first was that it forced companies to capitalize some leases. This would have the impacts as described above. Capitalized leases would tend to lower net income and increase the debt/equity ratio. The second reason it was unpopular was because it was a relatively complex set of rules that had to be followed. These rules required a fair amount of time and effort for companies to implement. c.

Financial engineers have sought to keep debt off the balance sheet by structuring contracts in such a manner that the contract will qualify as an operational lease when the reality is that the fixed asset has all other characteristics of an asset that would normally be capitalized. This is done by making sure that the lease contract does not meet any of the four criteria that would force the company to capitalize the lease.

d. Mr. Holgate makes a good point. When two transactions, that are substantially the same, can be recorded in significantly different ways on the financial statements then there is a problem. This is confusing to users of the financial statements and can lead to non-productive decisions being made. Companies will structure transactions to work around accounting rules as opposed to structuring the transaction in the best way for the company.


ID11–11 a.

During the severe economic recession, the likelihood that companies would not be able to meet their obligations (the companies’ “default risk”) increased substantially. Investors, sensing this increased risk level, reacted by shunning debt from companies with heightened default risk. In the financial markets, this investor reaction increased the supply of corporate bonds and simultaneously decreased demand for these financial contracts. Prices of the debt fell considerably.

b.

A company that was able to purchase its own debt at a steep discount would be reducing its liabilities without having to pay full price to do so. Effectively, companies could decrease a dollar’s worth of debt by paying less than a dollar to do so.

c.

On the financial statements, any company retiring debt by paying less than the current carrying cost of the liabilities would experience a Gain on Retirement of Debt (along with a cash outflow and a reduction of the liability).

ID11–12 Bristol-Myers Squibb is concerned that changes in interest rates could adversely impact the company’s financial condition. To protect against this possibility—to manage its interest rate risk—the company has entered into interest rate swaps with another party. In essence, an interest rate swap allows the two parties to exchange interest payments. One party would have interest rates on debt that is locked in at a fixed rate, meaning that no matter what market rates of interest do, the fixed rate will not change. The other party to the swap agreement would have interest rates on debt that float with changes in market interest rates. The party with the fixed rate swaps interest payments with the party with floating rates, effectively giving the company the other rate arrangement.

ID11–13 a. The long-term debt / total asset ratio for Nike was 9.7% in 2009 ($1,279.5/$13,249.6) and 10.4% in 2008 ($1,295.9/$12,442.7). The ratio decreased only slightly over this time period. b. Interest rates range from 4.7% to 5.66% on the domestic long term issues and 1.5% to 4.3% on the Japaneses issues. c.

The Nike Logistics YK issues were denominated in Japanese yen, creating currency risk for the company. If exchange rates between the U.S. dollar and the Japanese yen move against the company, losses could occur.

d. The company is relying less and less on long-term debt. No additional long-term debt was issued in the last two years.


e. Footnote #8 indicates that the 2009 market value of debt obligations is $456.4 million versus a book value of $469.2 ($450.8 million market value versus $447.4 book value for 2008). This small disparity is driven by the difference between the stated rate of interest on the obligations and the company’s effective interest rate as determined by the debt markets. f.

Footnote #18 describes the company’s hedging activities. Nike has entered into deriviative financial contracts to protect itself against interest rate fluctuations. (The company also hedges against foreign exchange currency risks.)


CHAPTER 12 STOCKHOLDERS' EQUITY BRIEF EXERCISES BE12–1 a. 67.8% of net income was paid in dividends during the year ($1,533/$2,260). b. The issuance of common stock affected the basic accounting equation by increasing assets (cash) and increasing stockholders equity by the same amount (common stock and additional paid-in capital). c.

The purchase of treasury stock affected the basic accounting equation by reducing assets (cash) and reducing stockholders equity (treasury stock) by the same amount.

d. The total dollars distributed to the company’s shareholders during the year totaled $1,591 ($1,533 + $58). This is comprised of dividends paid and the treasury stock that was purchased. e. The balance in retained earnings as of the end of the year was $2,115. ($1,388 + $2,260 – $1,533).

BE12–2 a. The number of shares outstanding after the split would be 194 million shares (97 million x 2) and the price per share would be approximately $50 ($100/2). b. The company’s overall value or market capitalization is $9.7 billion ($50 x 194 million shares). The company’s overall market should not change simply because of the share split. The number of shares will double but the price per share will be cut in half. Sometimes company’s that announce a share split see their stock price rise because many investors see a share split as a positive sign from management but this shareholder action is inferring positive news that has not been announced.

BE12–3 a. During 2007 the company paid an average of $51.65 per share in its repurchase program ($1,756/34). b. During 2008 the company paid an average of $58.00 per share in its repurchase program ($1,044/18). c.

With no information regarding 2010 treasury purchases, the balance in the treasury stock account will be $24,198 ($25,398 - $1,200) .

EXERCISES

1


E12–1 a.,b.,c. Accounts

Effect on Account

Effect on Total Stockholders' Equity

Increase Increase N/A Increase Increase Increase Decrease Decrease Increase N/A

Increase

(6)

Common Stock Additional Paid-In Capital, C/S None Treasury Stock Common Stock Additional Paid-In Capital, C/S Retained Earnings Treasury Stock Additional Paid-In Capital, T/S None

(7)

Retained Earnings

(1) (2) (3) (4)

(5)

Increase

No effect Decrease No effect

Increase No effect

Increase

E12–2 a. Debt = Total Liabilities = $52,000 + $35,000 = $87,000 Contributed Capital

Earned Capital

= Preferred Stock + Common Stock + Additional Paid-In Capital, Preferred Stock + Additional Paid-In Capital, Common Stock – Treasury Stock = $50,000 + $80,000 + $50,000 + $100,000 – $80,000 = $200,000

= Retained Earnings = $113,000

The portions of Lamont's assets provided by debt, contributed capital, and earned capital are, therefore, 21.75%, 50%, and 28.25%, respectively. b. Debt/Equity

Debt/Equity

c.

= = = =

Total Liabilities ÷ Stockholders' Equity Total Liabilities ÷ (Contributed Capital + Earned Capital) $87,000 ÷ ($200,000 + $113,000) .278

= Total Liabilities ÷ Total Stockholders' Equity = (Total Liabilities + Contributed Preferred Capital) ÷ (Contributed Common Capital + Earned Capital – Treasury Stock) = ($87,000 + $50,000 + $50,000) ÷ ($80,000 + $100,000 + $113,000 – $80,000) = .878

Most states restrict the dollar amount of dividends to either the balance in Retained Earnings or the balance in Retained Earnings less any treasury stock. So in this case, Lamont Brothers, either would be restricted to $113,000 or $33,000, depending upon the state.


E12–3 (1)

No entry is necessary.

(2)

Cash (+A)............................................................................................... Common Stock (+SE)...................................................................... Additional Paid-In Capital, Common Stock (+SE) ........................... Issued common stock.

300,000

Cash (+A)............................................................................................... Preferred Stock (+SE) ..................................................................... Additional Paid-In Capital, Preferred Stock (+SE) .......................... Issued preferred stock.

3,750,000

Cash (+A)............................................................................................... Preferred Stock (+SE) ..................................................................... Issued preferred stock.

2,500,000

(3)

(4)

50,000 250,000

2,500,000 1,250,000

2,500,000

Effect of each event on basic accounting equation: Transaction

Assets

Liabilities

1. 2. 3. 4.

NE + + +

NE NE NE NE

Owner’s Equity NE + + +

Since par value of a share of stock has no relationship to market value, it has very little economic significance. At one time, par value was construed to be legal minimum capital to protect creditors in times of dissolution or bankruptcy, but over time the concept has lost its appeal as creditors have found better ways to protect themselves.


E12–4 a. Treasury Stock (–SE) .................................................................................. Cash (–A)............................................................................................. Acquired treasury stock. (Dollars in millions) b. Debt/Equity

c.

1,000 1,000

= Total Liabilities ÷ Total Stockholders' Equity

Before

= $1,048 ÷ $2,012 = 0.52

After

= $1,048 ÷ ($2,012 – $1,000) = 1.04

Earnings per Share

= Net Income ÷ Outstanding Shares

Before

= $94 ÷ 125.5 Shares = $0.75 per Share

After

= $94 ÷ (125.5 Shares Issued – 26 Treasury Shares) = $0.94 per Share

d. A company might choose to purchase treasury stock at year end in order to reduce the number of shares outstanding as of year-end. A smaller number of shares outstanding will increase the earnings per share (based on average # of shares outstanding during the year).

E12–5 a. (1)

(2)

(3)

(4)

(5)

Cash (+A) ................................................................................................ Common Stock (+SE) ....................................................................... Additional Paid-In Capital, Common Stock (+SE) ............................ Issued common stock.

500,000

Cash (+A) ................................................................................................ Preferred Stock (+SE) ...................................................................... Issued preferred stock.

60,000

Treasury Stock (–SE)............................................................................... Cash (–A) ......................................................................................... Repurchased treasury stock.

45,000

Cash (+A) ................................................................................................ Treasury Stock (+SE) ........................................................................ Additional Paid-In Capital, Treasury Stock (+SE) ............................. Reissued treasury stock.

18,000

Cash (+A) ................................................................................................ Additional Paid-In Capital, Treasury Stock (–SE) .................................... Retained Earnings (–SE) ......................................................................... Treasury Stock (+SE) ........................................................................ Reissued treasury stock.

5,000 3,000 7,000

125,000 375,000

60,000

45,000

15,000 3,000

15,000


E12–5

Concluded

b. Preferred stock ($8 par value, 5,000 shares outstanding) ........................ Common stock ($5 par value, 25,000 shares issued, 24,000 shares outstanding) ..................................................................... Additional paid-in capital .......................................................................... Retained earnings...................................................................................... Treasury stock ........................................................................................... Total stockholders' equity .........................................................................

$

60,000

125,000 375,000 493,000 (15,000) $ 1,038,000

E12–6 a. (1)

(2)

Treasury Stock (–SE) ...................................................................... Cash (–A) ................................................................................. Purchased treasury stock.

60,000

Cash (+A) ........................................................................................ Retained Earnings (–SE) ................................................................. Treasury Stock (+SE) ............................................................... Reissued treasury stock as compensation.

10,000 20,000

60,000

30,000

Note:

Depending upon the terms of the employees' compensation package, it may be more appropriate to debit Compensation Expense for $20,000 rather than debiting Additional PaidIn Capital for $20,000.

(3)

Cash (+A) ........................................................................................ Treasury Stock (+SE) ............................................................... Additional Paid-In Capital, Treasury Stock (+SE)..................... Reissued treasury stock.

33,000

Common stock........................................................................................... Additional paid-in capital (common and treasury stock) .......................... Retained earnings ($60,000 - $20,000 +$20,000) ..................................... Total stockholders' equity .........................................................................

$ 80,000 13,000 60,000 $153,000

30,000 3,000

b. A total of $3,000 of additional paid-in capital is attributable to treasury stock. This amount would be recorded in the account Additional Paid-In Capital, Treasury Stock. Under the cost method, this amount represents the amount of proceeds received in excess of the acquisition cost of the treasury stock reissued.

E12–7 a. Treasury Stock (–SE) .................................................................................. Cash (–A)............................................................................................. Purchased treasury stock.

2,850,000 2,850,000


E12–7

Concluded

b. Common stock........................................................................................... Additional paid-in capital, common stock ................................................. Retained earnings................................ ($4,500,000 +$350,000 - $50,000) Treasury stock ........................................................................................... Total stockholders' equity ......................................................................... c.

$

100,000 2,400,000 4,800,000 (2,850,000) $ 4,450,000

When common stock is initially issued, it is recorded at the value of the assets received. In this case Stuart Corporation received $25 per share for 100,000 shares of common stock, for a total of $2,500,000. This $2,500,000 was allocated between the accounts Common Stock and Additional Paid-in Capital, Common Stock. Under the cost method of accounting for treasury stock, acquiring treasury stock does not cause the amount recorded for the initial issue of common stock to be adjusted. The treasury stock is simply recorded at the value of the assets given up. So if the value of the assets given up to acquire the treasury stock exceeds the value of the assets received when the common stock was initially issued, then the Treasury Stock balance will exceed contributed capital.

E12–8 a. Book Value per Share

= Common Stockholders' Equity ÷ Common Shares Outstanding = $7,187 ÷ 735 Shares = $9.78 per Share

b. Book Value per Share

= [$7,187 + (50 Shares  $32)] ÷ 785 Shares = $11.19 per Share

c.

Book Value per Share

= [$7,187 + (50 Shares  $20)] ÷ 785 Shares = $10.43 per Share

d. Book Value per Share

= [$7,187 – (50 Shares  $32)] ÷ 685 Shares = $8.16 per Share

e. Book Value per Share

= [$7,187 – (50 Shares  $20)] ÷ 685 Shares = $9.03 per Share

f.

Issuing stock can either increase or decrease the book value of a company's common stock. Whether issuing stock increases or decreases the book value depends upon the issue price of the new stock. If new stock is issued at a price above the pre-issue book value, then issuing the stock increases the book value. Alternatively, if new stock is issued at a price below the pre-issue book value, then issuing the stock decreases the book value.

g. Purchasing treasury stock can either increase or decrease the book value of a company's common stock. Whether purchasing treasury stock increases or decreases the book value depends upon the acquisition price of the treasury stock. If treasury stock is acquired at a price above the pre-issue book value, then purchasing treasury stock decreases the book value. Alternatively, if treasury stock is acquired at a price below the pre-issue book value, then purchasing the stock increases the book value.


E12–9 a. Cash (+A) ................................................................................................... Preferred Stock (+SE) .......................................................................... Issued preferred stock.

300

b. Cash (+A) ................................................................................................... Common Stock (+SE) .......................................................................... Additional Paid-In Capital, Common Stock (+SE)................................ Issued common stock.

120

c.

30

Cash (+A) ................................................................................................... Treasury Stock (+SE) ........................................................................... Additional Paid-In Capital, Treasury Stock (+SE) ................................ Reissued treasury stock.

300

100 20

20 10

E12–10 =

Common Stock + Additional Paid - in Capital Number of Shares Issued

=

$10,000 + $25,000 2,000 *

=

$17.50

*Total Par Value of Common Stock

÷

Par Value per Share

=

Number of Shares Issued

$10,000

÷

$5

=

2,000

a. Issue Price per Share

b. Purchase Price of Treasury Stock

c.

=

Treasury Stock Carrying Value on Balance Sheet Number of Treasury Stock Purchased

=

$8,000 400

=

$20/Share

To acquire Timeco, Zielow issued 1,000* shares and the market price of Timeco at the time of acquisition was $28,000.** * $5,000 c/s ÷ $5 P/V ** $5,000 c/s + $23,000 AP/C

d. Since the common stock accounts is always credited with total par value of shares issued to stock option holders, the company issued 200 shares (i.e., $1,000 c/s ÷ $5 a share P/V). The stock options were exercised at a price of $9* a share. Most certainly the market price of Zielow’s shares would be more than $9 at that time. * ($1,000 c/s + $800 AP/C) ÷ 200 shares.


E12–10 Concluded e. Per Share Dividend Rate

=

Total Dividends Paid During 2012 Common Stock Outstanding

=

$3,520 2,600 *

=

$1.354 per share

* ($16,000 ÷ $5) – [(8,000 + 4,000) ÷ $20] In this problem it is assumed that during the year 2012, treasury-stock was acquired at $20 a share.

E12–11 =

Common Stock + Additional Paid - in Capital Number of Shares Issued

=

$8,000 + $32,000 8,000 shares *

=

$5 a share

*Number of Shares Issued

÷

Total Par Value of Common Stock

=

Par Value per Share of C/S

$8,000

÷

$8,000

=

$1

a. Issue Price per Share

b. Purchase Price of Treasury Stock

c.

=

Treasury Stock Carrying Value on Balance Sheet Number of Treasury Stock Purchased

=

$18,000 1500 , Shares

=

$12/Share

Since $3,000 worth of treasury stock was used to satisfy the stock options, at a price of $12 a share, a total of 250 shares were issued through stock options. Since retained earnings is debited at $2,750, it means that stock options were sold at par value or $1 a share resulting in the following entry. Cash (+A) ................................................................................................... Retained Earnings (–SE)............................................................................. Treasury Stock (+SE) ...........................................................................

250 2,750 3,000


E12–11 Concluded d. Per Share Cash Dividend Rate

=

Total Cash Dividends Number of Common Shares Outstanding

=

$3,500 6,750 *

=

$.519 per share

* 6,750 = 8,000 Shares – 1,500 Treasury Shares Purchased + 250 Treasury Reissued

E12–12 a. Only those shares that are both issued and outstanding are eligible to receive dividends. Since Enerson has 375,000 shares of common stock issued, but 50,000 of those shares are held in treasury, the total number of common shares outstanding and, hence, eligible for a dividend are 325,000 shares. b. Date of declaration Cash Dividend (–SE) .................................................................................. Dividend Payable (+L) ......................................................................... Declared cash dividend.

3,900,000 3,900,000

Date of record No journal entry is necessary. Date of payment Dividend Payable (–L) ................................................................................ Cash (–A)............................................................................................. Paid cash dividend.

3,900,000 3,900,000

E12–13 a. Each year the preferred stockholders are entitled to $5 for each share of preferred stock outstanding. Since 5,000 shares are outstanding, total dividends to preferred stockholders should be $25,000 per year. Year 2008

Dividends Declared $

0

2009

30,000

2010

80,000

2011 2012

15,000 40,000

Dividends to Preferred $

0 25,000 5,000 20,000 25,000 15,000 10,000 25,000

Dividends to Common $

(for 2008) (for 2009) (for 2009) (for 2010) (for 2011) (for 2011) (for 2012)

0 0 35,000 0 5,000


E12–13 Concluded b. Dividends in arrears are the dividends preferred stockholders are entitled to if and when the company's board of directors declares a dividend. The amount of dividends in arrears equals the cumulative total of dividends not declared and not paid to preferred stockholders in the current and past years. The amount of dividends in arrears at the end of each year would be as follows. December 31, 2008 December 31, 2009 December 31, 2010 December 31, 2011 December 31, 2012 c.

$25,000 $20,000 $0 $10,000 $0

Dividends in arrears should not be considered a liability. A liability represents the probable future sacrifice of assets. A company may choose to reinvest its profits back into the company, or the company may not be financially secure enough to pay a dividend. This uncertainty is only resolved when the company's board of directors actually declares a dividend. Preferred stockholders are only entitled to receive their dividend when the company declares a dividend. If the board of directors never declares a dividend, then the preferred stockholders are not entitled to receive one; thus, no liability exists until the dividend is actually declared.

E12–14 a. Stock Dividend (–SE).................................................................................. Common Stock (+SE) .......................................................................... Additional Paid-In Capital, Common Stock (+SE)................................ Declared and issued 2% stock dividend.

11,200* 960 10,240

* $11,200 = (10,000 shares issued – 2,000 shares in treasury)  2%  $70 per share b. No journal entry is necessary. However, the company should prepare a memorandum entry stating that the par value has decreased from $6 to $4 per share and that there are now 15,000 shares issued and 12,000 shares outstanding. c.

Stock Dividend (–SE).................................................................................. Common Stock (+SE) .......................................................................... Additional Paid-In Capital, Common Stock (+SE)................................ Declared and issued 10% stock dividend. * $64,000

64,000* 4,800 59,200

= (10,000 shares issued – 2,000 shares in treasury)  10%  $80

d. No journal entry is necessary. However, the company should prepare a memorandum entry stating that the par value has decreased from $6 to $3 per share and that there are now 20,000 shares issued and 16,000 shares outstanding.


E12–14 Concluded e. Ratio = (Common Stk. + Additional Paid-In Capital – Treasury Stock) ÷ Retained Earnings Prior to entries ($60,000 + $100,000 – $24,000) ÷ $60,000 = 2.27 After (a) [($60,000 + $960) + ($100,000 + $10,240) – $24,000] ÷ ($60,000 – $11,200) = 3.02 After (b) ($60,000 + $100,000 – $24,000) ÷ $60,000 = 2.27 After (c) [($60,000 + $4,800) + ($100,000 + $59,200) – $24,000] ÷ ($60,000 – $64,000) = .00* After (d) ($60,000 + $100,000 – $24,000) ÷ $60,000 = 2.27 * Since retained earnings account is –$4,000, no ratio can be computed. In a stock split the number of outstanding shares is simply “split” into smaller units, which require the corporation to distribute additional shares. However, in a stock dividend additional shares, usually expressed as a percentage of the outstanding shares, are issued to stockholders. Large stock dividends have essentially the same effect as stock splits.

E12–15 a. Option 1 Stock Dividend (–SE).................................................................................. Common Stock (+SE) .......................................................................... Additional Paid-In Capital, Common Stock (+SE)................................ Declared and issued 10% stock dividend. Option 2 Stock Dividend (–SE).................................................................................. Common Stock (+SE) .......................................................................... Additional Paid-In Capital, Common Stock (+SE)................................ Declared and issued 20% stock dividend.

42,500 5,000 37,500

85,000 10,000 75,000

When a company declares an ordinary stock dividend, the fair market value of the new shares issued is transferred from Retained Earnings, via the Stock Dividend account, to the contributed capital accounts. Since in most states dividends are restricted to the balance in Retained Earnings (or Retained Earnings less Treasury Stock), any decrease in its account balance decreases the amount of potential dividends. So from the stockholders' viewpoint a stock dividend is not attractive because it decreases potential future dividends. The primary reason that a company would declare an ordinary stock dividend is as a publicity gesture. Some companies take great pride in being able to "promote" the company's dividend-paying history. If a company finds itself short of cash and still wants to be able to claim that it paid a dividend, then the company can maintain its dividend-paying streak by declaring a stock dividend.


E12–15 Concluded b. Option 3 would have no effect on any of the account balances reported in the financial statements because a stock split does not affect a company's financial position. However, a stock split does affect both the par value per share of the company's common stock and the number of shares outstanding, and as a result earnings per share. In this particular case, the 2-for-1 stock split would result in the par value per share of Railway Shippers' common stock decreasing from $10 to $5 and in the number of common shares outstanding increasing from 5,000 shares to 10,000 shares. c.

Declaring a stock split does not inherently increase or decrease a company's value. After a stock split, a company's value is allocated over a larger number of shares, so each share is worth less. If each share is worth less, then each share should sell for a lower price. If a company's stock is trading at too high a price, the average investor will not be able to invest in the company. If the price of the company's stock was lowered, trading in the company's stock would be stimulated. Consequently, a company can lower the price of its stock and stimulate trading in its stock by declaring a stock split.

E12–16 a. Appropriating retained earnings serves to restrict a portion of retained earnings from the payment of future dividends. Appropriations of retained earnings usually arise for two reasons. First, a creditor may require the borrower to appropriate retained earnings. Such appropriations prevent the borrower from paying "excessive" dividends to the stockholders, thereby reducing the amount of cash available to repay creditors. Second, a company may decide to restrict future dividends and use the cash that would have otherwise been used to pay dividends to finance plant expansion and so forth. In this particular case, the company appropriated retained earnings for both of these reasons. Auditors would require that appropriations of retained earnings be disclosed in the financial statements because they affect the magnitude of a company's future dividends. The magnitude of future dividends is information that current and potential investors desire. Failure to adequately disclose such information could cause investors to lose money on their investments and, consequently, to sue the auditor. b. Common stock........................................................................................... Additional paid-in capital .......................................................................... Retained earnings Restricted ........................................................................................... Unrestricted........................................................................................ Total stockholders' equity ......................................................................... c.

XX XX 350,000 450,000 $ XX

The company can only declare a dividend equal only to the portion of retained earnings which exceed $500,000. Since the debt covenant requires a minimum balance of $500,000 in the retained earnings account, the board is constrained by that clause and could possibly declare dividends up to a maximum of $300,000.


PROBLEMS P12–1 a. Cash (+A) ................................................................................................... Preferred Stock (+SE) .......................................................................... Issued preferred stock. Debt/Equity

b. Debt/Equity

c.

100,000 100,000

= Total Liabilities ÷ Stockholders' Equity = $250,000 ÷ ($330,000 + $100,000) = .58 = ($250,000 + $100,000) ÷ $330,000 = 1.06

If management classifies the stock as stockholders' equity, then the company will not be in violation of its debt agreement. However, if management classifies the stock as debt, then the company will be in violation of its debt agreement. Since violating debt agreements can be quite costly to both the company and managers, managers have incentives to classify the stock as stockholders' equity. The terms of the preferred stock make it appear to be more similar to debt than to equity. For example, the stock has a specified rate, does not participate in the benefits of ownership (i.e., does not vote and does not participate in profits), and has a fixed life. Based upon these factors, it appears that the stock is in substance actually debt and should be classified as such. Auditors will typically be guided by generally accepted accounting principles (GAAP) in deciding how to report an item. However, in most cases, GAAP does not provide clear-cut guidance, and auditors must apply their judgment. In applying their judgment, auditors are likely to consider the costs and benefits of alternative reporting options to themselves, the financial statement users, and the company in deciding how to classify the item. Auditors are normally better off by having events recorded in the most conservative manner. In this case, the most conservative manner of recording the stock would be to classify it as debt. Classifying the stock in this way decreases the probability that financial statement users will suffer out-of-pocket losses and be able to sue the auditors. Consequently, the auditors would probably prefer that the stock be classified as debt. It should be noted that some securities are such a hybrid of debt and equity that GAAP requires such securities to be reported as neither debt nor equity. Instead, GAAP requires such securities to be reported on the balance sheet in a special section between long-term debt and stockholders' equity.

P12–2 a. The balance in the Common Stock account represents the number of shares of common stock issued times the par value per share. Since the balance of $300,000 represents 50,000 shares, the par value per share must be $6. b. Book Value = (Total Stockholders' Equity – Contributed Capital from Preferred Stockholders) ÷ Number of Common Shares Outstanding = [($840,000 – $300,000) - $50,000] ÷ 45,000 Shares = $10.89


P12–2 c.

Concluded

Since 50,000 shares have been issued, and only 45,000 shares are outstanding, 5,000 shares are held in treasury. The balance of $40,000 in treasury stock represents the cost of these 5,000 shares. Consequently, the average price of the treasury stock was $8.

d. If the company reissues the treasury stock at $10 per share, stockholders' equity would increase by $50,000 and the company would make the following entry. Cash (+A) ................................................................................................... Treasury Stock (+SE) ........................................................................... Additional Paid-In Capital, Treasury Stock (+SE) ................................ Reissued treasury stock.

50,000 40,000 10,000

Debt/equity ratio The company's debt/equity ratio is calculated as Total Debt ÷ Total Stockholders' Equity. This ratio would decrease, because the numerator would be unchanged, while the denominator would increase. Book value The book value is calculated as Total Common Stockholders' Equity ÷ Number of Common Shares Outstanding. In this case, the numerator would increase by $50,000, and the denominator would increase by the 5,000 shares reissued. The book value would now be $10.80 [($540,000 – $50,000 + $50,000) ÷ (45,000 + 5,000)]. Since the book value prior to the reissue was $10.89 (see part b), the company's book value would decrease. Earnings per share Earnings per share is calculated as Net Income ÷ Number of Common Shares Outstanding. If the company did not reissue the stock, its EPS would be $1 per share ($45,000 ÷ 45,000 shares). The reissue of treasury stock has no effect on net income, but it does increase the number of common shares outstanding. So the company's EPS would decrease. Its EPS would now be $0.90 per share [$45,000 ÷ (45,000 + 5,000)].

P12–3 (1)

(2)

(3)

Cash (+A) ................................................................................................ Common Stock (+SE) ....................................................................... Issued common stock.

500,000

Cash (+A) ................................................................................................ Common Stock (+SE) ....................................................................... Additional Paid-in Capital, Common Stock (+SE) ............................ Issued common stock.

400,000

Cash (+A) ................................................................................................ Common Stock (+SE) ....................................................................... Additional Paid-in Capital, Common Stock (+SE) ............................ Issued common stock.

300,000

500,000

10,000 390,000

100,000 200,000


P12–3 (4)

b.

Concluded

Cash (+A) ................................................................................................ Preferred Stock (+SE) ...................................................................... Issued preferred stock.

400,000 400,000

Par value has accounting significance only in that it is used to determine the amount that is allocated to the paid-in capital accounts associated with common stock or preferred stock accounts when stock is issued. Economically, par value has little significance. Par values were initially established to protect creditors. Par value represents the amount that stockholders are liable to creditors. However, because par values are usually very small amounts, such as $1.00 or even $0.01 and because debt covenants now exist to protect creditors, par value no longer has much economic significance.

P12–4 a. Dividends are paid only on the shares that are both issued and outstanding. In this case, 55,000 shares have been issued, but 8,000 of these shares are held as treasury stock. Thus, only 47,000 shares are eligible to receive a dividend. b. Date of declaration Cash Dividend (–SE) ................................................................................... Dividend Payable (+L) ......................................................................... Declared divided.

705,000 705,000

Date of record No journal entry is necessary. Date of payment Dividend Payable (–L) ................................................................................ Cash (–A)............................................................................................. Paid dividend. c.

Stock Dividend (–SE).................................................................................. Common Stock (+SE) .......................................................................... Additional Paid-in Capital, Common Stock (+SE) ................................ Declared and issued stock dividend.

705,000 705,000

235,000a 47,000b 188,000

a $235,000 is calculated as follows:

= Number of Shares Outstanding  10% = 47,000  10% = 4,700 Shares 2. Value of Dividend = $50 Fair Market per Share  4,700 Shares to Be Distributed b $47,000 = 4,700 shares to be distributed  par value of $10 per share 1. Number of Shares to Be Distributed

d. The overall impact of cash dividends is a decline in the Retained Earnings account. Since retained earnings is a part of equity, the debt/equity ratio will increase. Issuance of stock dividends results in no change in the overall stockholders’ equity of the company. However, the balance in the retained earnings account goes down and common stock and additional paid-in capital account balances go up. Thus, the debt/equity ratio remains unchanged after the issuance of stock dividends.


P12–4

Concluded

e. Stockholders would generally prefer a cash dividend over a stock dividend. Assume that you own 1,300 shares of Royal Company's common stock prior to any dividend. Since there are 47,000 shares outstanding, you own 3% of the company. In addition, since each share is worth $50, the total value of your investment is $65,000. Since the company's financial position would not be expected to improve or worsen simply from declaring a stock dividend, the total value of your investment should still be worth $65,000. Thus, it appears that receiving a stock dividend has not improved your wealth. To the extent that Royal Company cannot declare and pay dividends in excess of its balance in Retained Earnings, a stock dividend may even decrease your wealth. By declaring a stock dividend, Royal Company has capitalized part of Retained Earnings, which means that it will never be available for cash dividends. Alternatively, with a cash dividend you would receive something of value, namely cash, while still maintaining a 3% ownership interest in the company. The trade-off, however, is that the value of the company would decrease by the value of the cash dividend.

P12–5 a. Each preferred stockholder is entitled to 10% of the par value, or $5.00. Thus, the 15,000 preferred stockholders are entitled to a total of $75,000 in any particular year.

Year

Total Dividends

Preferred Dividends

2006 2007 2008 2009 2010 2011 2012

$ 65,000 100,000 70,000 50,000 125,000 110,000 99,000

$65,000 75,000 70,000 50,000 75,000 75,000 75,000

b. Year

Total Dividends

2006 2007

$ 65,000 100,000

2008 2009

70,000 50,000

2010

125,000

2011 2012

110,000 99,000

Preferred Dividends $65,000 10,000 75,000 70,000 5,000 45,000 30,000 75,000 75,000 75,000

(for 2006) (for 2006) (for 2007) (for 2008) (for 2008) (for 2009) (for 2009) (for 2010) (for 2011) (for 2012)

Common Dividends

Preferred per Share

Common per Share

$

0 25,000 0 0 50,000 35,000 24,000

$4.33 5.00 4.67 3.33 5.00 5.00 5.00

$0.00 0.50 0.00 0.00 1.00 0.70 0.48

Common Dividends

Preferred per Share

Common per Share

0 15,000

$4.33 5.67

$0.00 0.30

0 0

4.67 3.33

0.00 0.00

20,000

7.00

0.40

35,000 24,000

5.00 5.00

0.70 0.48

$


P12–6 a. The maximum cash dividend that Cotter Company could declare given its current financial position is $25,000. This amount represents the cash that the company currently has on hand. If the company wants to declare a larger cash dividend, it would either have to sell some of its assets or borrow cash, thereby changing its financial position. b. In most states a company can not legally declare a dividend that exceeds the balance in Retained Earnings. Since Cotter Company has a balance in Retained Earnings (after closing entries) of $288,000, this amount is the maximum amount that the company could legally declare as a stock dividend. To be considered an ordinary stock dividend, however, the stock dividend cannot exceed 20% to 25% of the common shares already outstanding. The company's common stock is currently selling for $40 per share, which means that the company could distribute an additional 7,200 common shares ($288,000 ÷ $40 per share) in a stock dividend. Since the 7,200 shares represent only 14.4% of the 50,000 shares already outstanding, the distribution of the 7,200 shares would be considered an ordinary stock dividend. c.

Cash Dividend Cash Dividend (–E)..................................................................................... Cash (–) ............................................................................................... Declared and paid cash dividend. Stock Dividend Stock Dividend (–E).................................................................................... Common Stock (+SE) .......................................................................... Additional Paid-In Capital, Common Stock (+SE)................................ Declared and issued stock dividend.

25,000 25,000

288,000 72,000 216,000

d. If Cotter Company sold its marketable securities, it would receive the market value of $50 per share. Since Cotter Company owns 2,500 shares, it would receive $125,000. Combining this $125,000 with the $25,000 of cash already on hand would allow Cotter Company to declare and pay a cash dividend of $150,000.

P12–7 a. Stevenson Enterprises would make the following journal entry for the stock dividend. Stock Dividend (–SE).................................................................................. Common Stock (+SE) .......................................................................... Additional Paid-In Capital, Common Stock (+SE)................................ Declared and issued 10% stock dividend. * $150,000 = (60,000 shares outstanding  10%)  $25 per share

150,000* 36,000 114,000


P12–7

Continued

Stockholders’ equity: Common stock ($6 par value, 650,000 shares authorized, 76,000 shares issued, 66,000 shares outstanding, and 10,000 shares held as treasury stock) ................................................ Additional paid-in capital (C/S) ................................................................. Retained earnings...................................................................................... Treasury stock ........................................................................................... Total stockholders' equity .........................................................................

$

456,000 639,000 545,000 (100,000) $ 1,540,000

b. If Stevenson Enterprises declares a 2-for-1 stock split, no journal entry is necessary. Stockholders’ equity: Common stock ($3 par value, 1,300,000 shares authorized, 140,000 shares issued, 120,000 shares outstanding, and 20,000 shares held as treasury stock) ................................................ Additional paid-in capital (C/S) ................................................................. Retained earnings...................................................................................... Treasury stock ........................................................................................... Total stockholders' equity ......................................................................... c.

$

420,000 525,000 695,000 (100,000) $ 1,540,000

If Stevenson Enterprises declares the stock dividend and then a stock split, the only journal entry the company would have to make would be the journal entry given in Part (a). Stockholders’ equity: Common stock ($3 par value, 1,300,000 shares authorized, 152,000 shares issued, 132,000 shares outstanding, and 20,000 shares held as treasury stock) ................................................ Additional paid-in capital (C/S) ................................................................. Retained earnings...................................................................................... Treasury stock ........................................................................................... Total stockholders' equity .........................................................................

$

456,000 639,000 545,000 (100,000) $ 1,540,000

d. Stevenson Enterprises does not need to prepare any journal entry for the stock split. If Stevenson Enterprises subsequently declares and pays a 10% stock dividend, the company would have to make the following entry. Stock Dividend (–SE).................................................................................. Common Stock (+SE) .......................................................................... Additional Paid-In Capital, Common Stock (+SE)................................ Declared and issued 10% stock dividend. a $150,000 b $36,000

150,000a

= [(60,000 shares initially outstanding + 60,000 shares issued in stock split)  10%]  $12.50 per share = 12,000 new shares issued  $3 par value per share

36,000b 114,000


P12–7

Concluded

Stockholders’ equity: Common stock ($3 par value, 1,300,000 shares authorized, 152,000 shares issued, 132,000 shares outstanding, and 20,000 shares held as treasury stock) ................................................ Additional paid-in capital (C/S) ................................................................. Retained earnings...................................................................................... Treasury stock ........................................................................................... Total stockholders' equity .........................................................................

$

456,000 639,000 545,000 (100,000) $ 1,540,000

P12–8 a.

2007

Dollar Amount / Shares

Exercised stock options $77,400 Sale of common stock Repurchase of stock

6,600

41,900 194,500

=

Average price $ 11.73

2,800 12,200

$313,800

$ 14.96 $ 15.94

21,600

$ 14.53

2008 Exercised stock options $ 35,900 Sale of common stock

26,800 $62,700

4,800 2,500 7,300

$ 7.48 $ 10.72 $ 8.59

b. The common stock shares were issued at a higher price than the options. Stock options typically have a life of 5-10 years. So it is very likely that the stock options that were exercised this year were granted in previous years. If the stock price has risen over the last few years then the options would have been granted at lower prices in previous years. The trend in the stock price over from 2007 to 2008 has been down rather significantly.


P12–9 a. (1)

(2)

(3)

(4)

Treasury Stock (–SE) ...................................................................... Cash (–A) ................................................................................. Purchased treasury stock.

1,000

Cash (+A) ........................................................................................ Preferred Stock (10%) (+SE) .................................................... Additional Paid-In Capital, 10% Preferred Stock (+SE) ........... Issued preferred stock.

3,800

Cash (+A) ........................................................................................ Treasury Stock (+SE) ............................................................... Additional Paid-In Capital, Treasury Stock (+SE)..................... Reissued treasury stock.

660

10% Preferred Cash Dividend (–SE) ............................................... Dividends Payable (+L) ............................................................ Declared cash and stock dividend.

750a

1,000

2,000 1,800

360 300

750

a Cumulative preferred dividends are paid first.

Since no dividends have been paid since 1998, dividends in arrears in the amount of $1,000 (10 years of 10% of par value) on the 10% cumulative Preferred stock must be paid before any dividends can be paid on any other shares. The entire $750 payment goes to the cumulative shares.

(5)

(6)

Dividends Payable (–L)................................................................... Cash (–A) ................................................................................. Paid and issued dividend.

750 750

No journal entry is needed. A memorandum entry should be made stating that the company's common stock now has a par value of $.50 per share and that 7,000 shares are now issued and 840 shares [after the events underlying entries (1) and (3)] are held in treasury.


P12–9

Concluded

b. Preferred stock (10%, $10 par value, cumulative) .................................... Preferred stock (12%, $10 par value, noncumulative) .............................. Common stock ($0.50 par value, 10,000 shares authorized, 7,000 shares issued, and 840 shares held in treasury) ....................... Additional paid-in capital: Preferred stock (10%) ......................................................................... Preferred stock (12%) ......................................................................... Common stock .................................................................................... Treasury stock .................................................................................... Retained earnings...................................................................................... Treasury stock ($5,750 + $1,000 - $360) ................................................... Total stockholders' equity ......................................................................... * $4,405

=

$

3,000 1,500 3,500

2,850 1,275 2,345 300 4,405* (6,390) $ 12,785

$4,256 – $750 (dividends declared) + $899 (net income)

P12–10 a. Number of Shares Issued

Average Issue Price

= Increase in Contributed Capital ÷ Number of Shares = (Increase in Par Value + Increase in Additional Paid-in Capital) ÷ 900 Shares = [($200,000 – $110,000) + ($150,000 – $35,000)] ÷ 900 Shares = $227.78 per Share

b. Number of Shares Issued

Average Issue Price

c.

= Increase in Par Value ÷ Par Value per Share = ($200,000 – $110,000) ÷ $100 per Share = 900 Shares

= ($900,000 – $750,000) ÷ $10 per Share = 15,000 Shares

= [($900,000 – $750,000) + ($465,000 – $298,000)] ÷ 15,000 Shares = $21.13 per Share

Treasury Stock (–SE) .................................................................................. Cash (–A)............................................................................................. Purchased treasury stock. Average Repurchase Price

110,000 110,000

= Total Repurchase Price ÷ Number of Shares Held in Treasury = $110,000 ÷ 5,000 Shares Held in Treasury = $22.00 per Share


P12–10 Concluded d. Book value equals total common stockholders' equity divided by the total number of common shares outstanding. To determine total common stockholders' equity prior to the acquisition of the treasury stock, the value of the 5,000 shares of treasury stock must be added back to total stockholders' equity, and the 5,000 shares of treasury stock must be considered to be outstanding. Thus, prior to the acquisition of the treasury stock, the book value of outstanding common shares was: Total Common Stockholders Equity ÷ Number of Common Shares Outstanding = [(Total Stockholders' Equity + Treasury Stock) – Preferred Stock – Additional Paid-In Capital, Preferred Stock] ÷ Number of Common Shares Outstanding = ($2,180,000 + $110,000) – $200,000 – $150,000) ÷ 90,000 Shares = $21.56 per Share After the repurchase of the treasury stock, the book value of outstanding shares was $21.53 per share ($1,830,000 ÷ 85,000 shares outstanding). Thus, repurchasing the treasury stock decreased book value per share by $0.03.

P12–11 a. Number of Shares Issued

b. Average Issue Price

c.

= Change in Total Par Value ÷ Par Value per Share = ($110,000 – $70,000) ÷ $10 per Share = 4,000 Shares

= Cash Received ÷ Number of Shares Issued = [Change in Total Par Value + Change in Additional Paid-In Capital, Common Stock] ÷ 4,000 Shares [from part (a)] = [($110,000 – $70,000) + ($625,000 – $500,000)] ÷ 4,000 Shares = $41.25 per Share

Ending Retained Earnings = Beginning Retained Earnings + Net Income – Dividends Declared – Treasury Stock Adjustments $975,000 = $250,000 + $2,000,000 – Dividends Declared – 0* Dividends declared = $1,275,000 * Since Additional Paid-in Capital, Treasury Stock increased during 2011, we know that, on average, Tracey Corporation reissued the treasury stock for more than what it paid for the stock. Since Retained Earnings is adjusted for treasury stock transactions only when the stock is reissued for less than its acquisition cost, there is no treasury stock adjustment to Retained Earnings during 2011.

d. Total dividends declared = $1,275,000 [from Part (c)] Dividends to Preferred Stockholders

= Dividends in Arrears + Current Dividends Dividends for 2010 + Dividends for 2011 = ($400,000  10%) + ($400,000  10%) = $80,000

Dividends to Common Stockholders

= Total Dividends – Dividends to Preferred = $1,275,000 – $80,000 = $1,195,000

=


P12–11 Concluded e. Treasury Stock (–SE) .................................................................................. Cash (–A)............................................................................................. Purchased treasury stock.

105,000

f.

90,000

Cash (+A) ................................................................................................... Treasury Stock (+SE) ........................................................................... Additional Paid-in Capital, Treasury Stock (+SE) ................................ Reissued treasury stock.

g. Reissue Price per Share

105,000

21,000 69,000

= Cash Received ÷ Number of Shares Reissued = $90,000 [from part (f)] ÷ 300 Shares* = $300.00 per Share

* The 300 shares reissued is computed as follows: 1. Cost of shares = $105,000 ÷ 1,500 shares = $70 per share 2. Number of shares reissued = Change in treasury stock balance ÷ $70 per share = ($105,000 – $84,000) ÷ $70 per share = 300 shares

P12–12 a. (1)

(2)

Cash (+A) ........................................................................................ Common Stock (+SE) ............................................................... Additional Paid-in Capital, Common Stock (+SE) .................... Issued common stock.

750,000

Stock Dividend (–SE) ...................................................................... Common Stock (+SE) ............................................................... Additional Paid-in Capital, Common Stock (+SE) .................... Declared and issued 10% stock dividend.

90,000*

300,000 450,000

30,000 60,000

* $90,000 = 50,000 shares outstanding  10%  $18 per share (3)

(4)

(5)

(6)

Cash (+A) ........................................................................................ Common Stock (+SE) ............................................................... Additional Paid-in Capital, Common Stock (+SE) .................... Issued common stock.

1,320,000

Treasury Stock (–SE) ...................................................................... Cash (–A) ................................................................................. Purchased treasury stock.

375,000

Cash (+A) ........................................................................................ Treasury Stock (+SE) ............................................................... Additional Paid-in Capital, Treasury Stock (+SE) ..................... Reissued treasury stock.

135,000

360,000 960,000

375,000

125,000 10,000

The company should make a memorandum entry stating that par value is now $2 per share and the total number of shares issued is now 345,000 shares, of which 315,000 shares are outstanding and 30,000 shares are held in treasury at an adjusted cost of $8.33 per share.


P12–12 Concluded (7)

Cash (+A) ........................................................................................ Treasury Stock (+SE) ............................................................... Additional Paid-In Capital, Treasury Stock (+SE)..................... Reissued treasury stock.

80,000 66,640* 13,360

* $66,640 = 8,000 shares  Adjusted acquisition price of $8.33 per share (8)

(9)

(10)

Cash Dividend (–SE) ....................................................................... Dividend Payable (+L) ............................................................. Declared a cash dividend.

50,000

Dividend Payable (–L) .................................................................... Cash (–A) ................................................................................. Paid cash dividend.

50,000

Cash (+A) ........................................................................................ Treasury Stock (+SE) ............................................................... Additional Paid-In Capital, Treasury Stock (+SE)..................... Reissued treasury stock.

30,000

50,000

50,000

16,660* 13,340

* $16,660 = 2,000 shares  Adjusted acquisition price of $8.33 per share b. Common stock ($2 par value, 1,000,000 shares authorized, 345,000 shares issued, and 20,000 shares held in treasury) .............. Additional paid-in capital: Common stock .................................................................................... Treasury stock .................................................................................... Retained earnings...................................................................................... Treasury stock ........................................................................................... Total stockholders' equity ......................................................................... a $480,000

b $166,700

$690,000 1,470,000 36,700 480,000a (166,700)b $ 2,510,000

= Cumulative net income – Cumulative declared dividends = ($400,000 + $100,000 + $100,000 + $20,000) – [$90,000 from entry (2) + $50,000 from entry (8)] = $375,000 [from entry (4)] – $125,000 [from entry (5)] – $66,640 [from entry (7)] – $16,660 [from entry (10)]

P12–13 a. Only investors of shares that are both issued and outstanding are eligible to vote to elect a board of directors. If the current board could reduce the number of shares held by investors other than themselves, then they would control a greater proportion of the shares outstanding and could decrease the probability that Vadar, Inc., would take over Edmonds. One way that the board could consolidate its ownership position is to reacquire the company's common stock from other investors.


P12–13 Concluded If the current board could consolidate its ownership so that it controlled 50% plus one share of the outstanding shares, they could completely block Vadar, Inc.'s takeover attempt. In this particular case, the current board members own 140,000 shares (35%  400,000 shares outstanding). To allow the board to completely control the company without the board members personally acquiring any additional shares, the total number of shares outstanding could not exceed 279,999 [(140,000 shares  2) – 1 share]. Since 260,000 shares are currently held by non-board investors, and the non-board members can control a total of only 139,999 shares (279,999 shares – 140,000 shares held by board members), the company would have to repurchase 120,001 shares of the company's stock. b. The company will need to pay $50 per share for 120,001 shares [from part (a)]. Consequently, the company will need at least $6,000,050 in cash. Edmonds currently has insufficient cash to repurchase all 120,001 shares. If the board wants to block Vadar's takeover attempt by repurchasing some of its common stock, the company will have to borrow additional cash. c.

Assets Cash Other current assets Noncurrent assets

$ 1,149,950* 4,200,000 8,220,000

Total assets

$ 13,569,950

Liabilities and Stockholders' Equity Liabilities $ 5,250,000 Common stock 8,000,000 Retained earnings 6,320,000 Less: Treasury stock (6,000,050) Total liabilities and stockholders' equity $ 13,569,950

* $1,149,950 = $3,150,000 cash on hand + $4,000,000 cash borrowed –$6,000,050 cash paid out for stock d. Debt/Equity Ratio = Total Liabilities ÷ Total Stockholders' Equity Before $1,250,000 ÷ ($8,000,000 in Common Stock + $6,320,000 in Retained Earnings) = .087 After $5,250,000 ÷ ($8,000,000 in Common Stock + $6,320,000 in Retained Earnings – $6,000,050 in Treasury Stock) = .631 The purchase of the treasury stock had a large effect on the company's financial position, as evidenced by the large increase in the company's debt/equity ratio. The actual purchase of the treasury stock reduced Edmonds' stockholders' equity by the value of the stock purchased. Since Edmonds financed the purchase by issuing debt, the purchase directly increased Edmonds' long-term debt by the $4,000,000 borrowed.


P12–14 a. Loss on Write-Down of Fixed Assets (Lo, –SE) .......................................... Fixed Assets (–A)................................................................................. Wrote down obsolete fixed assets. b. Alternative 1 Cash (+A) ................................................................................................... Current Assets (–A) ............................................................................. Fixed Assets (–A)................................................................................. Liquidated assets.

50,000 50,000

650,000 200,000 450,000

Cash Dividends (–SE) ................................................................................. Cash (–A)............................................................................................. Declared and paid cash dividend.

250,000

Liabilities (–L)............................................................................................. Cash (–A)............................................................................................. Repaid liabilities.

400,000

Alternative 2 Cash (+A) ................................................................................................... Current Assets (–A) ............................................................................. Fixed Assets (–A)................................................................................. Liquidated assets.

250,000

400,000

650,000 200,000 450,000

Cash Dividends (–SE) ................................................................................. Cash (–A)............................................................................................. Declared and paid cash dividend.

400,000

Liabilities (–L)............................................................................................. Cash (–A)............................................................................................. Repaid liabilities.

250,000

Alternative 3 Cash (+A) ................................................................................................... Current Assets (–A) ............................................................................. Fixed Assets (–A)................................................................................. Liquidated assets.

400,000

250,000

650,000 200,000 450,000

Treasury Stock (–SE) .................................................................................. Cash (–A)............................................................................................. Purchased treasury stock.

250,000

Liabilities (–L)............................................................................................. Cash (–A)............................................................................................. Repaid liabilities.

400,000

250,000

400,000


P12–14 Concluded Alternative 4 Cash (+A) ................................................................................................... Current Assets (–A) ............................................................................. Fixed Assets (–A)................................................................................. Liquidated assets. Treasury Stock (–SE) .................................................................................. Cash (–A)............................................................................................. Purchased treasury stock. c.

650,000 200,000 450,000

650,000 650,000

In the event of a liquidation, creditors have the first claim on the company's assets. Consequently, obligations to creditors should be settled in full before the stockholders receive any residual assets. In alternatives (2) and (4), the board of directors has proposed circumventing the law and has planned to distribute assets to the stockholders first and then distribute the residual assets, if any, to the creditors. The creditors could sue the company and probably force the stockholders to return the "excess" dividends, which would then be used to satisfy the creditors' claims.


ISSUES FOR DISCUSSION ID12–1 a. A 100% stock dividend effectively means that the number of shares will be doubled. Therefore, if 149.5 million shares were outstanding before the stock dividend, then 299 million shares were outstanding after the dividend. b. Stock Dividend (–SE)............................................................................... Common Stock (+SE) ....................................................................... Declared and issued stock dividend.

149,500a 149,500

a

Because the stock dividend can be considered a 2:1 stock split par value for the 149.5 million shares is used. c.

Total Value of Hershey

Post dividend Value/Share

= Number of Shares Outstanding  Market Value per Share = 149.5 Million Shares  $46 per share = $6.877 Billion = Total Value of Hershey ÷ Number of Shares Outstanding = $6.877 Billion ÷ 299 Million Shares = $23 per Share

d. Prior to the stock dividend, Mr. Jones owned .669% of Hershey (1 million shares ÷ 149.5 million shares outstanding), and Mr. Jones' investment was worth $46 million (1 million shares  $46 market value per share). After the stock dividend, Mr. Jones would own 2,000,000 shares, and the total number of shares outstanding would increase to 299 million shares. Thus, Mr. Jones would still own .669% of Hershey (2 million ÷ 299 million shares outstanding). Because the market price would be expected to drop to $23 per share [see part (c)], Mr. Jones' investment would still be worth approximately $46 million. Thus, neither the percentage of shares owned nor the total value of an equity investment would be expected to be affected by a stock dividend, provided that the market price decreases proportionately to the increase in the number of shares outstanding. e. As demonstrated in part d, a stock dividend does not represent an economic exchange between a corporation and its shareholders. A stock dividend merely splits the shareholders’ interests into smaller pieces. The total value of the company and its assets and liabilities remain unchanged. f.

A company may issue a stock dividend for several reasons. These reasons could include wanting to give the appearance of paying a dividend when there is limited cash on hand, wanting to decrease the market price of the shares outstanding, or wanting to expand the number of shares outstanding without selling new stock.

ID12–2 There are two general situations that would lead a company to cut its dividend. The first scenario is when a company is no longer able to generate enough cash to pay the dividend that it has been paying in the past. This usually reflects a downturn in the company’s business and the stock price does not act well after this announcement. The second scenario is when a company has been paying a dividend and is still generating enough cash to pay the dividend but decides not to. This is usually because the company feels that it has


very good investment opportunities and wants to use the money to pursue them. This is usually a positive sign and the stock price may rise.

ID12–3 Since the stock price of a company is affected by the expected future cash inflows due to dividend payments and capital appreciation, the price of Philip Morris (now known as Altria Group) stock rose in response. After the $6 billion stock buy-back is completed, the company will have fewer shares outstanding in the market, and it will increase the future EPS and the book value of the company’s stock. Further, the company is raising the quarterly dividend by almost 20%, which means higher-cash inflow to the existing shareholders. All of these are positive signs and would lead to an increase in the price of Philip Morris’s common stock. However, the company’s credit rating will go down, primarily due to the fact that after paying an increased dividend and executing a $6 billion stock buy-back plan, Philip Morris may be low on cash, which could affect its short-term solvency position. Therefore, the rating agencies would perceive Philip Morris as a risky company. The higher the risk, the lower the credit rating.

ID12–4 a. Companies could use their cash to invest in the latest technology to improve the productivity of their manufacturing operations. Companies could use their excess cash to perform research and development to find new products for their markets. Companies could use their excess cash to acquire other companies to take advantage of economies of scale as a larger entity with less competition. Companies could use their cash to repay debt to lower interest costs and improve their performance during cyclical downturns. If any of the above alternate uses of cash increase the profitability and cash flow of the company, the shareholders benefit through capital appreciation (an increased stock price, for example), as well as the potential increase in dividend payments made possible by the improved performance of the company. If the alternate use of cash also improves the company’s standing in its industry, and positions it for future success, the shareholders benefit from the improve long term prospects of the company. b. Fewer shares in the market would imply that the market capitalization per share is higher, driving up the stock price. If the company’s performance (profitability, cash flow, leverage, etc.) is not changed but there are fewer shares of ownership, then each share would command a higher price as the total value of the company is divided among the equity providers. Driving up the share price strictly by reducing the number of shares outstanding is an artificial change as the company itself is no different. c.

A company purchasing its own stock is in essence no different than an investor purchasing stock in the company. “Buy low, sell high” makes sense for any investor. If, due to current market levels, the stock price is high, there is always the risk that the buyer—the company in this case—would be buying high with the possibility that a market correction could drive the stock price down to low levels. The company, similar to any investor, would then be holding a stock with a market value below the cost paid to acquire the stock. If a company’s stock is at a historically low point, repurchasing stock does not pose the same risk. However, if the stock price is very high, the possibility exists that the company would be purchasing an asset that will drop in value.

ID12–5


a. A hybrid security carries some of the properties of debt and some of the properties of equity. A determination must be made as to which properties dominate the characteristics of the security. If it is determined that the hybrid carries mainly equity characteritists, then it should be reported in the Stockholders’ Equity section of the Balance Sheet and should not affect the Income Statement. In either case, the hybrid security will be treated in the Financing Activities section of the Statement of Cash Flow. On the other hand, if the debt characteristics dominate, then the security should be classified as a liability on the Balance Sheet and will affect the Income Statement. c.

Equity securities typically carry more risk than debt securities but consequently offer higher rates of return. If an investor was interested in accepting more risk than offered by a debt security, but required more of a return to justify that risk, then the investor might have an interest in a hybrid security. If the risk profile of a pure equity security was too great for the investor, but the returns offered by debt securities insufficient for the investor’s needs, then a hybrid security would offer a perfect compromise. A company might be interested in offering a hybrid security if it did not want to give up all that would be required by a traditional equity security or if it had no additional capacity to raise funds through traditional debt securities. An issue of preferred stock, for example, would allow the company to raise financing without giving up the voting rights associated with common stock; further, if the company could borrow no additional money from its banks and the capital markets, the preferred stock issue might be marketable as it would not carry the interest payments required each year under a debt issue.

c. Characteristics associated with debt include a higher priority toward receiving payments in case of a company liquidation and some type of fixed requirement of periodic payments. Characteristics associated with equity include a vote for the directors of the company and no fixed maturity date for the financial arrangement.

ID12–6 The case describes a typical scenario that usually occurs while enforcing the debt covenant restrictions. Many times such agreements or decrees explicitly specify how changes in the key performance indicators will be treated due to an unanticipated change in the accounting standards. In the current case it is not clear whether any such clause exists that deals with the effect of change in net worth due to an unforeseen accounting standard. From the perspective of an executive of Westinghouse, the company should not be forced to place the $325 million in the escrow account due to the following reasons: (1) Westinghouse’s net worth dipped below $1.9 billion due to a new method of accounting mandated by FASB. The decline was entirely due to factors beyond Westinghouse’s control. (2) The new method of accounting merely creates an additional expense and an additional liability on the books of the company. It does not impact the current cash flow situation. Therefore, there is no change in the economic situation of Westinghouse. Therefore, it should not be subjected to setting aside $325 million in an escrow account. From the perspective of a representative of the federal government, Westinghouse should place $325 million in an escrow account. The terms of the decree are very clear, which say that if the company’s net worth falls below $1.9 billion, it should place $325 million in an escrow account. As

P12–6 Concluded


a federal government representative, he has to ensure that the terms of the decree are complied with. From the perspective of a resident of Bloomington, Indiana, Westinghouse should place $325 million in an escrow account to build the incinerator to destroy the PCB-Contaminated materials. As a resident, one must be very concerned about the impact of PCB-contaminated materials on the health and safety of the town’s residents as well as the impact on the town’s environment. Several unanticipated factors may occur that may be out of the company’s control. These risks are inherent in doing business. The company has no right to transfer those risks to the community by not complying with the decree. Further, technically, the company was always liable for employee retirement costs, but it ignored that liability by adopting a liberal accounting policy of a pay-as-you-go basis. It is only fair that Bloomington, Indiana want to ensure that the company pays for the hazardous waste cleanup, irrespective of the games that the company can play with the accounting numbers.

ID12–7 a. An investor can make a return through 1) capital appreciation (the stock moving up in value), 2) receipt of cash dividends from the company, or 3) both capital appreciation and dividends. b. Companies that pay dividends are not necessarily under the same pressure to improve earnings and cash flow as companies who opt not to pay dividends. Investors who are confident they can rely on a steady stream of dividends are less likely to pressure management to take steps to continually boost future cash flows to drive the stock price ever higher. Companies, on the other hand, who reinvest all earnings in their operations must provide a return to their shareholders in the form of higher stock prices; these companies will be under the gun to continually improve operations, grow their businesses, and improvement cash flow and earnings in order to move the stock price. c.

Investors will monitor sales, profits and earnings-per-share on the income statement, as well as cash flow from operations on the statement of cash flow. In addition to financial statement clues, investors can look for industry information, such as market share and consumer surveys to predict future financial results for the company.

ID12–8 a. Book value of a stock is simply the equity on the balance sheet divided by the number of equity shares outstanding. This figure is driven by Generally Accepted Accounting Principles. Market value is the value placed on a share of stock by all the buyers and sellers in the free market. This figure is driven by company results, but it is also influenced by factors such as general macroeconomic trends, stock market trends, and very often the subjective mood of investors. b. The price-to-book ratio is the market price (per share) divided by the book value (per share). Prior to the buyout offer, the price-to-book ratio of Dow Jones was 6.0 ($36 per share divided by $6 per share). After the buyout offer, the ratio jumped to 9.8 ($58.60 per share divided by $6 per share).

ID12–8 Concluded


c.

Two reasons might explain the high offer for Dow Jones. First, News Corporation might see some specific synergies of operation between its media businesses and those of Dow Jones (including the Wall Street Journal and the popular WSJ website); the combined businesses might be worth more to News Corporation than they would be to other investors. Other investors might only see $36 in value based on the projected cash flow of Dow Jones, but News Corporation might see $60 in value because of future business plans combining and growing the two companies together. Secondly, many market watchers speculated that News Corporation offered such a premium for Dow Jones simply to scare off other potential buyers in order to make the deal close faster. And, in retrospect, since no other bidders emerged, the $60 strategy may have been a sound approach to the acquisition.

d. External events (such as technological change) can cloud the future for companies and, since stock prices are driven by valuing a company’s future, affect the stock valuation. When events beyond the control of management change the prospects for a business, the market will react by pricing in those expected changes. If the internet will drive advertisers away from traditional newspapers, the future cash flow of those newspapers will drop; the market reacts by simply pricing the stock today in anticipation of those lowered expectations.

ID12–9 a. “Net” proceeds mean the actual cash received by the company after commissions to underwriters and expenses are paid. “Gross” proceeds would equal all the cash paid by investors buying the offered shares. b. Dividing the proceeds by the number of shares issued indicates that the stock price has been climbing rapidly. The proceeds per share issued climbed from $68.29 to $287.72 and then leveled off at $268.05 for the three years shown. c.

Google has been one of the major success stories for companies going public in the last decade. As internet traffic has increased, Google has captured a dominant share in search engine activity. The company’s business model sells advertising to companies, allowing the advertiser to directly target internet users based on search requests. More searches will mean more advertising dollars to Google. Technological changes and a very profitable business model have brightened the future for Google. The stock market has reacted to those heightened expectations for cash flow and profits by driving the stock price higher.

ID12–10 a. Both Rhodia and Segway issued equity because of losses that would be evident on the Income Statement. The influx of funds would be evident in the Financing Activities section of the Statement of Cash Flow. The Balance Sheet would also show an immediate increase in the Cash asset, but that would soon be converted to other assets as the company employed the funds. b. When companies issue new stock, the concept of ownership dilution must always be considered. If Aventis wants to maintain its 15.3 percent ownership of Rhodia, it must purchase 15.3 percent of the new shares issued. A large equity stake in another company, such as what Aventis has with Rhodia, is purchased for strategic control reasons. If Aventis wants to continue to significantly influence the operations of Rhodia, it must maintain its large ownership share in the company. If new shares are sold to the public, and if Aventis does not purchase its pro rata share, its 15.3 percent ownership stake will be diluted to something lower and Aventis’ influence might follow suit.


The original equity providers of Segway might have a similar interest in maintaining their control position of that company. If they do, they will have to purchase the percentage of the $31 million offering that matches their current ownership percentage in the company. However, if the investors are not concerned with maintaining the amount of influence in the company, they do not need to buy into the new offering. c.

Since both companies are experiencing profitability problems, before purchasing additional equity in the companies, a sound investor would need to understand the steps taken by management to correct the profitability problems. To assure that “good money was not chasing bad money”, a sound investor would meet with management, discuss the causes of past problems, and come to a firm understanding of and agreement with the remediation steps taken to restore the companies to profitability. Without sustained profits, no business can remain viable. A sound investor would need to believe that the companies will soon be in a position to show that revenues exceed operating expenses.

ID12–11 a. A 4:1 stock split means that the corporation will replace every share of outstanding stock with 4 shares of the stock. Because the corporation is simply splitting the ownership shares into smaller pieces, declaring or issuing a stock split does not affect the dollar amounts Walt Disney would report in its financial statements. However, a 4:1 stock split would result in (1) the number of shares of common stock Disney had outstanding to increase by a factor of four and (2) the par value per share to drop to 25% of its pre-stock split value. b. The stock market often views the declaration of a stock split as a positive signal about the corporation. For example, the market often perceives a stock split as a signal from management that they believe that they can maintain the price of the stock despite the stock split. In addition, some companies increase their dividend per share soon after splitting their stock. Such positive signals, in turn, increase demand for the company's stock, which drives up the price of the stock. In addition, stock splits would be expected to cause the market price per share to drop once the stock split actually takes place to reflect the additional shares of stock outstanding. The lower market price would make the stock more affordable to more investors, which should increase demand for the stock and drive up the price. c.

Theoretically, the value of a company's stock equals the present value of the cash flows the stock market expects an investor to generate from an equity investment in the company. Any information that would cause the stock market to change its expectations--either positively or negatively—about the future cash flows that would be realized from an investment would result in a change in the price of the company's stock. Thus, if the market perceives the declaration of a stock split as positive new information about the company's future prospects, as suggested by the quote from The Wall Street Journal, then the market would be expected to use this information and increase the price of the company's stock.

ID12–12 a. When a stock price is discussed in terms of a multiple of earnings, say 45 times, it simply means that the market price per share is 45 times the size of the earnings per share. Or, every penny of earnings is currently valued by the market at 45 cents. This “multiple” is often cited as a basis for how over- or under-priced a stock is by the market. When stock markets highly value a stock, its multiple will be very high (say 45 times), but when markets turn cold to a stock, that multiple drops, meaning every penny of earnings is worth less to investors in the market. b. A volatile market will challenge a company that is intent on repurchasing its own shares. As with any investor, companies buying their own shares do not wish to overpay for their purchases. If a volatile market exists and stock prices are jumping all around, the company with a committed


buyback program will be paying many different prices (some high, some low) for the shares it repurchases. c. Stock repurchases will reduce the company’s cash balances (affecting both liquidity and solvency) and will lower the company’s equity base (increasing leverage). Subsequent stock re-issuances will increase cash balances and equity levels, but the reissue price will determine if those equity levels will be fully restored. If the stock is reissued for a lower price than the acquisition price, a permanent hit to equity will occur (adjusted through Additional Paid In Capital, assuming APIC balances are sufficient). If, on the other hand, the stock is re-issued for a higher price, then an increase in equity balances will occur. d. Borrowing to repurchase shares has a double effect on a company’s leverage. The borrowing, of course, increases the companies liabilities, while the Treasury Stock accounting decreases the company’s equity levels. Both treatments (+L, -SE) increase a company’s leverage. e. Reissuing Treasury Stock is considered to be a capital transaction and, as such, does not affect company’s income statement and its level of profitability.

a

ID12–13 a. Stock-based compensation expense is the expense related to the granting (and subsequent exercise) of stock options to employees. If an employee is given the option to purchase shares of company stock from the company, the cost of those shares to the company has to be included as a form of compensation expense, just as cash paid to an employee for services rendered is considered compensation expense. The calculation of the actual cost of the shares to the company is still subject of much debate and controversy, but the fact that options are a form of compensation is generally accepted. b. As the shares are issued (or re-issued from treasury), the statement of stockholders’ equity must reflect the change in shares outstanding. Since the options are now considered an expense, the income statement is also affected, as is the statement of cash flow (and balance sheet) for any cash received by the company when the options are exercised by the employee.

c.

If the options were not considered an expense, Microsoft’s net income would have been considerably higher in the years shown, so the expense is definitely material to the company. Net income would have been higher by 11.6%, 8.5% and 11.4% for 2009, 2008 and 2007, respectively, if the cost of the options had not been expensed.

d. Many companies resisted the expensing of options due to the negative effect the move would have on earnings. Many start-up and technology companies relied heavily on stock options for employee compensation. These companies knew that the large amount of options exercised by their employees would dramatically increase expenses and reduce earnings.

ID12–14 The Statement of Stockholders’ Equity for Emerson Electric for three years is shown at the end of Chapter 12 in the textbook. Under the Retained Earnings section, the dividend history for 2007 – 2009 can be seen.


The company has consistently paid cash dividends in each of the years. On a per share basis, the dividend payment is rising from 2007’s $1.05 per share to 2008’s $1.20 per share to 2009’s $1.32 per share. As a percentage of net income, dividends represented 39.2% (2007), 39.0% (2008), and 57.9% (2009). The large increase in percentage of earnings paid out in dividends in 2009 is driven by a slight increase in dividend dollars (up to $998 million) but more so by the drop in dollar profits (from $2.4 billion to $1.7 billion). The Treasury Stock transactions slowed in the last year (possibly due to the drop in profitability). In 2007, $849 million of shares were repurchased, but that amount grew to $1.1 billion in 2008; in the latest year the amount fell to $695 million. Even though the company is reissuing shares annually, due to stock option plans, due to the size discrepancy of repurchases versus re-issues, the net effect is to remove shares from the marketplace.

ID12–15 a. Companies can spend their cash balances on a number of areas: they can expand physical capacity, they can acquire other businesses, they can invest in new technologies, they can pay cash dividends, or they can use their cash to repurchase their own shares. Companies that repurchase shares are saying that the best use of cash to build value for the shareholder is to use that cash to reduce the number of shares in the market. Companies that do not repurchase their shares feel that there is a better use for cash balances, a better way to build value for the shareholder. b. HP and Apple have been very successful companies, using cash balances to acquire other businesses (HP) and to reinvest in new product launches (Apple). Bear Stearns, on the other hand, was forced into a fire sale, selling itself to another financial services firm in order to avoid a bankruptcy filing. c.

Companies can create wealth for shareholders by building and running successful businesses (leading to higher share value), by paying cash dividends from strong earnings, and by repurchasing stock from shareholders.

ID12–16 a. Net income is the difference between revenue and expenses for a given time period. Comprehensive income includes net income but also includes any other changes to shareholders’ equity (such as unrealized gains/losses on available-for-sale securities and changes to currency exchange rates). b. Comprehensive income is for a given time period, such as a year, while accumulated other comprehensive income is a running total of past comprehensive incomes. The difference between comprehensive income and accumulated other comprehensive income can be thought of as the difference between the income statement and the balance sheet: one is for a period of time while the other is an accumulation as of a point in time. c.

Under U.S. GAAP, the information must be provided as either a separate statement or part of the Statement of Shareholders’ Equity. Under IFRS, the information must be provided as a separate statement or an extension of the Income Statement.

ID12–17


a. As of 5/31/2009: Liabilities ÷ Total Assets = 34.4% Contributed Capital ÷ Total Assets = 24.5% Retained Earnings ÷ Total Assets = 41.1%

b. Class A Common Stock: 175 million shares authorized and 95.3 million issued and outstanding; Class B Common Stock: 750 million shares authorized and 390.2 million issued and outstanding c. Purchase outstanding stock Pay dividends (in millions)

2009 $649.2 $466.7

2008 $1,248.0 $ 412.9

2007 $985.2 $343.7

The amount of cash used to repurchase stock has decreased, while the amount of cash used to pay dividends has steadily increased each year. d. Preferred stock is considered a hybrid security, carrying some characteristics of debt and other characteristics of equity. Nike has opted to list the preferred stock balance of $0.3 million alongside its liabilities due to the dividend obligations of the stock (see footnote #10) and the lack of general voting rights associated with the stock. e. For the year ending 5/31/2009 Nike added back stock-based compensation expense of $170.6 million. This expense reduced earnings but has not yet cost the company cash, so the amount is added back in the computation of cash from operations. Footnote #11 outlines the expenses related to the granting of stock to employees; the company employs financial models to value the compensation granted to employees in the form of stock and deducts this value as an expense (even if no cash is currently involved in the compensation). f.

Over the time period, Nike’s stock has traded from $38.57 per share to $70.07 per share. 2009 dividends per share were $0.96 ($466.7/485.5), while 2008 dividends per share were $0.84 ($412.9/491.1).

g. Average exercised prices were $27.55 (2006), $33.45 (2007), and $35.70 (2008). h. Nike’s accumulated other comprehensive income as of 5/31/2009 was $367.5 million. This balance consists primarily of net gains on cash flow and investment hedges. Comprehensive income for the year ending 5/31/2009 was $116.1 million, compared to $1.487 billion for net income.


CHAPTER 13 THE COMPLETE INCOME STATEMENT BRIEF EXERCISES BE13–1 To accurately compare a company’s performance from one year to the next, non-recurring items such as debt retirements and accounting changes, as well as the activities of discontinued operations, should be excluded from results. An analyst reviewing a company’s performance would want to see those items that should occur in every year of a company’s operations, not the one-time, special expenses and gains. In Goodrich’s case, an analyst would compare 2008 and 2007, focusing on recurring revenues and expenses only; those items in the two years that could not be expected to happen in future years would be excluded from the analysis. A comparison of Goodrich’s operating results would then give the anlayst an idea toward the company’s future results.

BE13–2 ASSETS

=

Closure of Tampa Breweries: Write down of plant assets –113.7 Employee severence costs –19.4 Disposal costs

LIABILITIES +

= = =

Sale of St. Louis Cardinals: Sale of team (+cash, – assets) +95.3, –95.3 Gain on sale of team +54.7

=

+26.9

OWNERS’ EQUITY

–113.7 –19.4 –26.9

+54.7

BE13–3 Glancing only at Merck’s net income trend would give an analyst the impression that the company’s operations are growing more profitable. However, the improved results are due to special items that cannot be counted on to recur in future periods. The large increase in “other income” (from $2.3 billion in 2008 to $10.7 billion in 2009) contributes greatly to the company’s bottom line, but an analyst would certainly question if the $10 billion level will happen again in 2010. Other one-time charges that affect the profit trend line are the Vioxx settlement (debit expense, credit contigent liability until the cash is paid) and the restructuring charges (debit expense, credit liability and cash). The equity income from affiliates is a more difficult read for an analyst. Given Merck’s business strategy, long term investments (that exert significant influence but not control) in other companies will be a recurring part of the business; however, the equity income shown on the income statement does not represent a cash inflow to Merck as it is simply Merck’s share of the other company’s profitability.

1


BE13–3 Concluded An analyst would certainly be interested in computing the recurring revenues and expenses from Merck’s core business: (amount in billions) 2009 2008 2007 Sales $27.4 $23.8 $24.2 Materials, Production Expenses 9.0 5.6 6.1 Marketing, Administrative Expenses 8.5 7.4 7.6 R & D Expenses 5.8 4.8 4.9 Operating Income $ 4.1 $ 6.0 $ 5.6 Op Income/Sales 15% 25% 23% This analysis actually shows a declining trend in profitability based on the revenues and expenses that are recurring in nature. .

BE13–4 a. Comprehensive Income can be calculated as: Net Income $486 million Less: Unrealized Loss AfS Securities 4 million Less: Foreign Exchange Loss 10 million Comprehensive Income $472 million b. The balance for Accumulated Other Comprehensive Income is included in the Stockholders’ Equity section of the balance sheet, to reflect the changes in equity that do not originate from profitability. Comprehensive Income (the change to Accumulated Other Comprehensive Income) is reflected at the bottom of the Income Statement, below the calculation of Net Income. Finally, the Statement of Stockholders’ Equity will contain a column showing all the changes to Accumulated Other Comprehensive Income (with the ending balance matching what is shown on the Balance Sheet).


EXERCISES E13–1 a. Statement 1.

IS

b. Classification

c. Explanation

Other This is an example of a bookkeeping entry without an underlying economic event. Some are involuntary. N/A N/A Usual and frequent Wages are normal, recurring part of

voluntary, some are 2. N 3. N 4. IS operations. 5. IS Usual and frequent Bad debts are a normal, recurring part of operations. 6. IS Unusual or infrequent Sales of equipment are secondary to normal operations. 7. IS Unusual and infrequent Expropriation against a company occurs rarely in the U.S. 8. SE N/A 9. SE N/A 10. IS Unusual or infrequent Dividend Revenue is secondary to normal buying and selling activities of most businesses. 11. IS Usual and frequent Cost of goods sold is part of normal daily operations. 12. IS Usual and frequent Rent expense is part of normal operations.

E13–2 a.

1. 2. 3. 4. 5. 6. 7. 8.

1 4 5 5 4 5 1 1

9. 10. 11. 12. 13. 14. 15.

2 5 5 5 1 5 3

b. As the transactions move from 1 to 5, they move away from exchanges with owners toward exchanges with third parties involving the company's central, ongoing activities. Stockholders and creditors are considered providers of capital, and this financing provides capital that is necessary for the company to be able to operate. However, the company does not exist to draw capital from the equity and debt investors; instead, it exists to provide goods and/or services to other entities. Providing goods and/or services is considered the company's operations.

E13–3


a. (1) (2) (3) (4) (5)

Financing Operating Operating Financing Financing Smedley Company Income Statement For the Year Ended December 31, 2013 Fees earned ......................................................................................................... Expenses ............................................................................................................ Net income ..........................................................................................................

b. Comprehensive Income

$ 50,000 24,000 $ 26,000

= Change in Equity from Nonowner Sources = Revenues – Expenses + Gains – Losses + Cumulative Accounting Adjustments = $50,000 – $24,000 + 0 – 0 = $26,000

* Fees and expenses represent Smedley’s only nonowner changes in equity. c.

The two are equal because Smedley’s economic events/activities do not differ from those described in the income statement.

E13–4 a. (1)

(2)

(3)

(4)

Net income Maximum Dividend

= 20%  $182,800 = $36,560 Income before change in accounting method Maximum Dividend = 20%  $130,800 = $26,160 Income before extraordinary items Maximum Dividend = 20%  $108,800 = $21,760 Net operating income Maximum Dividend = 20%  $150,000 = $30,000

b. The bank requires restrictions on dividend payments to increase the probability that Morton Manufacturing will have sufficient cash to meet its interest and principal payments. More stringent restrictions on dividends decrease the amount that Morton Manufacturing can potentially pay out as dividends, thereby increasing the probability that the company will have sufficient cash to meet its obligations to the bank. In this particular case, basing the dividend restriction on net operating income provides the most stringent dividend restriction. Further, managers have greater ability to manipulate items disclosed after net operating income than those items disclosed before it. For example, the decision to sell assets and realize gains is completely at a manager's discretion, as is a change in accounting principle resulting in an increase in net income. Therefore, basing the dividend restriction on net operating income limits management's ability to manipulate the restriction.


E13–5 a. Losses from equity investments: AT&T invested excess cash in the equity of other companies; the value of these investments dropped during the course of the year; Loss from discontinued operations: AT&T closed down or sold off business units during the year; the business units had generated losses (expenses exceeded revenues), but those units are no longer operating as part of the company; Gain on disposition of discontinued operations: The discontinued business units were sold for a price that exceeded the book value of the units; Gain/Loss from accounting changes: AT&T made a one-time change to its accounting treatment of particular transactions and the change resulted in either a gain or loss.

b.

Reported net income

($ in billions)

Loss on equity investments Loss from discontinued operations Gain from sale of discontinued operations Gain from accounting changes

Year 1

Year 2

Year 3

$7.7

($13)

$1.9

7.5

0.4 4.0

(1.3)

14.5

0.01

(1.3)

(0.9)

Loss from accounting changes Operating income

0.01

(.02) 0.9

$17 .0

$1.5

$1.9

Each of these special items are not part of operating income, which represents the recurring activities of the business. These special charges are one time gains or losses and so can mislead the users of financial statements as to the future prospects of the company. The schedule above shows that the operating income of AT&T dropped significantly from Year 1 levels, but that the performance between Year 2 and Year 3 is more consistent than shown by the net income figures. Adjusting net income for the nonrecurring items gives a clearer picture of the company’s performance; business has changed significantly from Year 1 levels, but performance in the last two years has been more consistent.

E13–6 a. Cash (+A) ................................................................................................... Liabilities (–L)............................................................................................. Assets (–A) .......................................................................................... Gain on Disposal of Business Segment (Ga, +SE) ............................... Sold business segment.

625,000 1,400,000 1,850,000 175,000


E13–6 Concluded Gain on Disposal of Business Segment (–Ga, –SE) .................................... Income Tax Liability (+L) ..................................................................... Recognized income tax liability on disposal of business segment.

61,250* 61,250

$61,250 = Gain on disposal of $175,000  Tax rate of 35%

Income From Operations of Discontinued Segment (–Ga, –SE)................ Income Tax Liability (+L) ..................................................................... Recognized income tax benefit on net loss of discontinued segment. *

5,250* 5,250

$5,250 = Net income of $15,000  Tax rate of 35%

b. Discontinued operations: Income from operations of discontinued segment (net of tax expense of $5,250) ................................................................. Gain on disposal of business segment (net of $61,250 in taxes) ....... Discontinued operations .............................................................

$ 9,750 113,750 $ 123,500

E13–7 a.

Mallory Services Income Statement For the Year Ended December 31, 2012 Income from continuing operations (before taxes) ................... Income tax expense .................................................................... Income from continuing operations........................................... Discontinued operations: Income earned by discontinued segment (net of tax expense of $4,200)......................................................... $ 7,800b Gain on disposal of discontinued segment (net of tax expense of $13,300)....................................................... 24,700c Discontinued operations .................................................. Net income ................................................................................. _________________ a $332,500 = $950,000  35%

$ 950,000 332,500a $ 617,500

32,500 $ 650,000

b Income = Revenues – Expenses  (1 – tax rate)

$7,800 = ($35,000 – $23,000  (1– 35%) c Gain = Sale price – Book value  (1– tax rate)

$24,700 = [$105,000 – ($93,000 – $26,000)]  (1 – 35%) Earnings persistence reflects the extent to which a particular dollar of earnings can be expected to continue in the future. A user would expect the income from continuing operations of $617,500 to continue in the future, but interpret the income and gain from the discontinued division to reflect a one-time increase in earnings that is not expected to be realized in the future.


E13–7 Concluded b.

Mallory Services Income Statement For the Year Ended December 31, 2012 Income from continuing operations (before taxes) ............................................ Income tax expense ............................................................................................. Net income ..........................................................................................................

$ 962,000a 336,700b $ 625,300

______________________________

a Income from continuing operations = Income excluding clerical division + Activities

of clerical division $962,000 = $950,000 + ($35,000 – $23,000) b $336,700 = $962,000  35%

A user would expect the entire amount of net income to persist in the future. No items are separated out as one-time unusual and/or infrequent. c.

Sharon Sowers faces a number of trade-offs as she decides whether to complete the sale in 2012 or 2013. First, income is already positive and strong in 2012. The gain from the sale increases net income by only $24,700. If Mallory Services expects a weaker 2013, it may be more beneficial to the income statement to wait until 2013 in order to offset poorer performance. Also, by selling in 2012, recurring income from operations isn’t as large as it could be. Users would interpret the financial statements as $617,500 persisting into the future versus $625,300 if the division isn’t sold until 2013. On the other hand, Sharon may receive bonuses on net income and may prefer to receive the bonus now or may even need the gain caused by the disposal to increase net income to some minimum amount for a bonus. Also Mallory Services may face debt convenants that need to be covered by a larger amount of income. Finally, Susan may be able to use the cash from the sale to invest in alternative projects that immediately earn a higher return than produced by the clerical division.

E13–8 a.

Carmich Industries Income Statement For the Year Ended December 31, 2012 Income from continuing operations (before taxes) ................... Income tax expense .................................................................... Income from continuing operations........................................... Discontinued operations: Income earned by discontinued segment (net of tax expense of $8,750)......................................................... $ 16,250b Gain on disposal of discontinued segment (net of tax expense of $45,850)....................................................... 85,150c Discontinued operations .................................................. Net income ................................................................................. a $675,500 = $1,930,000  35%

= Revenues – Expenses  (1 – tax rate) $16,250 =($145,000 – $120,000  (1 – 35%)

b Income

$ 1,930,000 675,500a $ 1,254,500

101,400 $ 1,355,900


c

E13–8

Gain = Sale price – Book value  (1 – tax rate) $85,150 = [$350,000 – ($437,000 – $218,000]  (1 – 35%)

Concluded Earnings persistence reflects the extent to which a particular dollar of earnings can be expected to continue in the future. A user would expect the income from continuing operations of $1,254,500 to continue in the future, but interpret the income and gain from the discontinued division to reflect a one-time increase in earnings that is not expected to be realized in the future.

b. Carmich Industries Income Statement For the Year Ended December 31, 2012 Income from continuing operations (before taxes) ......................................... Income tax expense .......................................................................................... Net income .......................................................................................................

$ 1,955,000a 684,250b $ 1,270,750

a Income from continuing operations = Income excluding clerical division + Activities of clerical

division $1,955,000 = $1,930,000 + $25,000 b $684,250 = $1,955,000  35%

A user would expect the entire amount of net income to persist in the future. No items are separated out as one-time, unusual, and/or infrequent. c.

Rob Blandig faces a number of trade-offs as he decides whether to complete the sale in 2012 or 2013. First, income is already positive and strong in 2012. The gain from the sale increases net income by only $85,150, only 6.7%,. If Carmich Industries anticipates a weaker 2013, it may be more beneficial to the income statement to wait until 2013 in order to offset poorer performance. Also, by selling in 2012, recurring income from operations isn’t as large as it could be. Users would interpret the financial statements as $1,254,500 persisting into the future versus $1,270,750 if the division isn’t sold until 2013. Also, Rob and other top management receive a 20% bonus on income from continuing operations. This means that top management earns an additional 20% on the income from the division if it is not sold in 2012. On the other hand, Carmich Industries may face debt convenants that need to be covered by a larger amount of income. Also, Rob may be able to use the cash from the sale to invest in alternative projects that immediately earn a higher return than produced by the chemical division and increase income and his bonus even higher next year.


E13–9 a. Net Sales is the revenue generated by selling products/services to customers. Operating expenses are those charges that make the revenue possible. Operating income is the profitability associated with the basic (recurring) operations of the company. Net cost of debt is the interest expense net of any interest income. Income taxes is the expense related to the taxes incurred on the company’s profits. Net income of equity-accounted affiliates is Group Danone’s share of the income of companies in which Danone owns a significant (but not controlling) portion; this income statement item is a non-cash addition to profitability. Net income from continuing operations is the bottom-line profitability, after taxes, of all the operations of the company that will recur in the following fiscal period (2008, in this case). Net income from discontinued operations is the profitability from companies owned during 2007 but sold or closed by the end of the year; in other words, the discontinued operations will not recur in 2008 for Group Danone. b. During 2007 Group Danone sold or closed operations that generated 3.3 million euros in profitability; this level of earnings represented 76% of Danone’s total profits for 2007, as the company’s continuing operations (those that will occur again in 2008) earned only 1.0 million euros. An analyst interested in evaluating the future performance of Group Danone would look to the continuing operations as the guide for future results, not the total operations; the 3.3 million euros earnings will not recur for Group Danone in 2008 and beyond.

E13–10 a. Loss on Destruction of Inventory (Lo, –SE)................................................ Inventory (–A) ..................................................................................... Recognized inventory loss from earthquake.

2,325,000

b. Extraordinary Loss on Destruction of Inventory (Lo, –SE) ......................... Inventory (–A) ..................................................................................... Recognized inventory loss from earthquake.

2,325,000

Income Tax Liability (–L) ............................................................................ Extraordinary Loss on Destruction of Inventory (–Lo, +SE) ................ Recognized income tax benefit on inventory loss from earthquake.

813,750*

2,325,000

2,325,000

813,750

* $813,750 = Loss of $2,325,000  Tax rate of 35% c.

If the earthquake is considered both an unusual and infrequent event, then the loss should be classified as an extraordinary loss. However, if the loss is considered unusual or infrequent, but not both, then it should be classified as other revenues and expenses. If the loss is considered both usual and frequent, then it should be disclosed as part of operations. Since Paxson Corporation's plant is located in San Francisco, then the magnitude of the earthquake would have to be considered. To be considered an extraordinary loss, the events that gave rise to the loss must be both unusual in nature and occur infrequently. In San Francisco, minor earthquakes are not infrequent; they are expected to occur occasionally. Consequently, if this loss was due to a minor earthquake, it would not be considered extraordinary. Since earthquakes are not usual (i.e., part of the company's normal operations), the loss should not be considered as part of operations. Instead, the loss should be classified as an other expense. Alternatively, if the damage was due to a major earthquake, it might qualify as an extraordinary loss. Major earthquakes, such as the 1906 and 1989 earthquakes that devastated the Bay area, are both unusual and infrequent.


d. If Paxson’s plant was located in Miami, Florida, then any earthquake is considered both unusual and infrequent and would qualify as an extraordinary loss.

E13–11 a. If a lawsuit is considered unusual but not infrequent, then it would be classified under other expenses and losses. Consequently, the loss from the lawsuit would be used to compute net income from continuing operations. Bonus = 12%  [($800,000 – $480,000)  (1 – Tax Rate)] = 12%  [$320,000  (1 – 35%)] = $24,960 b. If the loss from the lawsuit is considered extraordinary, then the loss would not be used to compute net income from continuing operations. Bonus = 12%  [$800,000  (1 – Tax Rate)] = 12%  [$800,000  (1 – 35%)] = $62,400 c.

Gain not considered extraordinary Bonus = 12%  [($800,000 + $480,000)  (1 – Tax Rate)] = 12%  [$1,280,000  (1 – 35%)] = $99,840 Gain considered extraordinary Bonus = 12%  [$800,000  (1 – Tax Rate)] = 12%  [$800,000  (1 – 35%)] = $62,400

d. Income figures are often used as the basis for awarding incentive compensation to managers, or included in ratios which are, in turn, incorporated into debt covenants. If the income figures included in incentive contracts or in debt covenants stipulate between income from continuing operations and net income, then the decision of whether or not to classify an event as extraordinary or not can have economic consequences. As demonstrated in parts (a) through (c), how a loss or gain is classified can have a profound effect on the magnitude of a manager's bonus.

E13–12 a. Income from continuing operations ........................................ Disposal of business segment* ................................................ Extraordinary loss ..................................................................... Changes in accounting method ................................................ Net earnings per share .............................................................

$ 0.73 0.33 (0.46) 0.40 $ 1.00

b. Income from continuing operations ........................................ Disposal of business segment* ................................................ Extraordinary loss ..................................................................... Changes in accounting method ................................................ Net earnings per share .............................................................

$ 0.44 0.20 (0.28) 0.24 $ 0.60


c.

Income from continuing operations ........................................ Disposal of business segment* ................................................ Extraordinary loss ..................................................................... Changes in accounting method ................................................ Net earnings per share .............................................................

$ 0.37 0.16 (0.23) 0.20 $ 0.50

* The EPS disclosure for the disposal of the business segment includes both the income from the disposed segment and the gain on the sale of the disposed segment.

E13–13 a. Rothrock Consolidated Income Statement For the Year Ended December 31, 2012 Revenue: Operating revenues ............................................................... $ 87,000 Total revenue......................................................................... Expenses: Operating expenses ............................................................... Other revenue ............................................................................ Income from continuing operations (before taxes) ................... Income tax expense .................................................................... Income from continuing operations........................................... Discontinued operations: Income earned by discontinued segment (net of taxes of $1,050) ................................................................... $ 1,950 Loss on disposal of discontinued segment (net of tax benefit of $7,350) .......................................................... (13,650) Discontinued operations ....................................................... Income before extraordinary items ........................................... Extraordinary loss (net of tax benefit of $1,750) ....................... Income before change in accounting method ........................... Effect of change in accounting method (net of taxes of $4,375) ..................................................................... Net income .................................................................................

$ 87,000 32,500 5,200 $ 59,700 20,895 $ 38,805

(11,700) $ 27,105 (3,250) $ 23,855 8,125 $ 31,980

b. Rothrock Consolidated Statement of Retained Earnings For the Year Ended December 31, 2012 Retained earnings, January 1, 2012..................................................................... Plus: Net income.................................................................................................. Less: Dividends declared ..................................................................................... Retained earnings, December 31, 2012 ..............................................................

$ 72,000 31,980 (18,000) $ 85,980


E13–14 a. Watson Company Income Statement For the Year Ended December 31, 2012 Sales revenues ............................................................................ Cost of goods sold ...................................................................... Gross profit ................................................................................. Operating expenses: Administrative expenses ....................................................... $ Depreciation expense ............................................................ Selling expenses..................................................................... Total expenses.................................................................. Income from operations ............................................................. Other revenues and expenses: Rent revenue ......................................................................... $ Loss on sale of fixed assets .................................................... Total other revenues and expenses ................................. Income from continuing operations (before taxes) ................... Income taxes .............................................................................. Income from continuing operations........................................... Extraordinary loss (net of tax benefit of $70,700) ..................... Net income .................................................................................

$1,385,000 475,000 $ 910,000 100,000 250,000 189,000 539,000 $ 371,000 360,000 (105,000) 255,000 $ 626,000 219,100 $ 406,900 (131,300) $ 275,600

b. Watson Company Statement of Retained Earnings For the Year Ended December 31, 2012 Retained earnings, January 1, 2012..................................................................... Plus: Net income.................................................................................................. Less: Dividends .................................................................................................... Retained earnings, December 31, 2012 .............................................................. c.

$ 847,000 275,600 (460,000) $ 662,600

If Watson Company had no tax liability as of January 1, 2012, and made no tax payments during 2012, then the company's tax liability as of December 31, 2012 would equal the sum of the intraperiod tax allocations. Watson Company incurred $219,100 in taxes associated with income from continuing operations. The extraordinary loss provided a tax benefit, thereby reducing income taxes by $70,700. Therefore, the company's total tax liability as of December 31, 2012 would be $148,400.


E13–15 a. Kennington Company Income Statement For the Year Ended December 31, 2012 Income from continuing operations (after taxes) ...................... Loss on lawsuit (net of tax benefit of $21,700) .......................... Income from continuing operations........................................... Gain on disposal of discontinued segment (net of tax expense of $8,750)............................................... $ 16,250 Extraordinary loss on early retirement of debt (net of tax benefit of $13,300) .............................................. (24,700) Net income .................................................................................

$ 235,000 (40,300) $ 194,700

(8,450) $ 186,250

Maximum dividends Kennington can declare = Income from continuing operations  15% $29,205 = $194,700  15% b. Kennington Company Income Statement For the Year Ended December 31, 2012 Income from continuing operations (after taxes) ...................... Gain from sale of short-term investment (net of tax expense of $8,750)............................................... Income from continuing operations........................................... Gain on disposal of discontinued segment Extraordinary loss on lawsuit, (net tax benefit of $21,700)................................................... $ (40,300) Extraordinary loss on early retirement of debt (net of tax benefit of $13,300) .............................................. (24,700) Net income .................................................................................

$ 235,000 16,250 $ 251,250

(65,000) $ 186,250

Maximum dividends Kennington can declare = Income from continuing operations  15% $37,687.50 = $251,250  15%


E13–16 a. Madigan International Income Statement For the Year Ended December 31, 2012 Income from continuing operations (before taxes) ................... Gain on sale of subsidiary ........................................................... $ 42,000 Gain due to change in accounting principle ............................... 25,000 Income from continuing operations........................................... Income tax expense .................................................................... Income from continuing operations........................................... Extraordinary items: Loss of inventory due to earthquake, (net tax benefit of $18,550) ............................................. $ (34,450) Loss due to write-off of accounts receivable, (net tax benefit of $13,300) ............................................. (24,700) Extraordinary items............................................................ Net income .................................................................................

$ 865,000 67,000 $ 932,000 326,200a $ 605,800

(59,150) $ 546,650

a$326,200 = $932,000  35%

b. Madigan’s accountants are not presenting income statement items in accordance with generally accepted accounting principles. Instead, they are presenting all the gain items as part of income from continuing operations and all the loss items as extraordinary. The intention is to represent to investors and other users a higher income from continuing operations than actually exists. The greater is income from continuing operations, the more income investors and users expect will exist in the future. The gain on the sale of the subsidiary should be reported as a discontinued operation separately after income from continuing operations. The gain from the change in accounting principles should be reported separately after income from continuing operations. The loss due to the earthquake may be extraordinary as reported by Madigan, depending on the location of Madigan and the magnitude of the earthquake. The loss due to a write-off of accounts receivable, however, occurs in the ordinary course of business and should not be reported as an extraordinary item.


E13–16 Concluded The income statement that is consistent with GAAP follows. Madigan International Income Statement For the Year Ended December 31, 2012 Income from continuing operations (before taxes) ................... Loss due to write-off of accounts receivable ............................. Income from continuing operations........................................... Income tax expense .................................................................... Income from continuing operations........................................... Gain on discontinued operations, (net of tax expense of $14,700)............................................. $ 27,300 Extraordinary loss on inventory due to earthquake, (net of tax benefit of $18,550) .............................................. (34,450) Gain due to change in accounting principle, (net of tax expense of $8,750)............................................... 16,250 Net income .................................................................................

$ 865,000 (38,000) $ 827,000 289,450a $ 537,550

9,100 $ 546,650

a $289,450 = $827,000  35%

c. Madigan’s original treatment makes future income look more favorable. Stock prices tend to follow a multiple of earnings, so higher future income would lead to a higher future stock price.

E13–17 a. 2011

2012

2013

2014

2015

Net Income 250,000 -200,000* 50,000 250,000 -200,000* 50,000 250,000 -200,000* 50,000 250,000 -200,000* 50,000 250,000 -200,000* 50,000

Balance Sheet Value

800,000**

600,000

400,000

200,000

0

*straight-line depreciation expense = $1,000,000/5 years **Book Value = Cost – Accumulated Depreciation


E13–17 Concluded b. Net Income Balance Sheet Value 2011 250,000 -400,000* (150,000) 600,000** 2012 250,000 -240,000* 10,000 360,000 2013 250,000 -144,000* 106,000 216,000 2014 250,000 - 86,400* 163,600 129,600 2015 250,000 -129,600* 120,400 0 *double declining balance depreciation expense = 2 x Book Value/5 years **Book Value = Cost – Accumulated Depreciation c.

Net Income Balance Sheet Value 2011 250,000 -210,000* 40,000 790,000** 2012 250,000 -168,000* 82,000 622,000 2013 250,000 -190,000* 60,000 432,000 2014 250,000 - 205,000* 45,000 227,000 2015 250,000 -227,000* 23,000 0 *depreciation expense = change in market value **Book Value = Cost – Accumulated Depreciation

d. All three approaches yield the same total profitability over five years (each method totals $250,000 in profitability), but the methods allocate those profits differently. The straight-line method produces equal profits of $50,000 per year, while the double-declining balance method results in lower net income in the earlier years (due to the acceleration of the depreciation expenses) and higher net income in the later years. Finally, the market value approach ties the fleet expense to the drop in market value of the fleet assets, producing a fluctuating annual net income (that nonetheless totals the same $250,000 over the life of the fleet).


PROBLEMS P13–1 a. 1. 2. 3. 4. 5. 6. 7. 8. 9.

Operating Operating Operating Financing Operating Operating Operating Financing Operating & Investing

b. 1. 2. 3. 4. 5. 6. 7. 8. 9.

Normal and recurring Normal and recurring Other revenues and expenses Issues and payments of debt Normal and recurring Other revenues and expenses Changes in accounting principles Exchanges with stockholders The sale of the securities would be classified as purchases, sales, and exchanges of assets, and the loss would be classified as other revenues and expenses.

c.

Items (2), (3), (5), (6), (7), and (9) would all be disclosed on the income statement as follows. (2)

The $500,000 would be disclosed as operating revenues, and the $375,000 would be disclosed as cost of goods sold, which is an operating expense.

(3)

Minor earthquakes are not infrequent in San Francisco, although they are unusual. Thus, the $100,000 would be disclosed as part of other revenues and expenses.

(5)

The $143,000 would be disclosed as operating expenses.

(6)

Lawsuits are unusual, but it appears that the company is frequently sued. Thus, the $10,000 would be disclosed as part of other revenues and expenses.

(7)

The effect of the change in accounting methods would be disclosed, net of any tax effect, after extraordinary items and before net income.

(9)

The amount of the loss would be disclosed as part of other revenues and expenses.


P13–2 a. Bonus = 25%  (Income from Operations Before Interest Expense – Interest Expense) = 25%  [$1,200,000 – ($1,000,000  8% Interest Rate)] = $280,000 b. Bonus = 25%  (Income from Operations Before Interest Expense – Interest Expense) = 25%  ($1,200,000 – $0) = $300,000 c.

The decision whether to finance the plant expansion through an equity issue or through a debt issue is worth $20,000 to the managers because the managers will receive an additional $20,000 in bonuses if the company issues equity instead of debt. Although issuing equity is in the best interest of the managers, this option may not be in the best interest of the existing stockholders. It is possible that issuing additional stock would dilute the ownership interests of the existing stockholders. Further, the interest payments on debt are tax deductible. Hence, issuing debt would reduce cash outflows for taxes.

d. If interest expense was not considered to be an operating expense, the managers' bonus would be the same whether the company issued debt or equity. In either case the bonus would be 25% of $1,200,000, or $300,000. In this situation, the managers would, hopefully, base their decision on factors that are more relevant to the stockholders.

P13–3 a. 1. 2. 3. 4. 5. 6. 7.

Financing Operating Financing Operating Investing Financing Investing & Financing

b. Raleigh Corporation Income Statement For the Year Ended December 31, 2012 Fees earned ................................................................................ Expenses: Wage expense ....................................................................... $ 125,000 Supplies expense ................................................................... 35,000 Depreciation expense ............................................................ 80,000 Miscellaneous expense.......................................................... 75,000 Total expenses.................................................................. Net income .................................................................................

$ 580,000

315,000 $265,000


P13–3 c.

Concluded

Comprehensive Income

= Change in Equity from Nonowner Sources = Revenues – Expenses + Gains – Losses + Cumulative Accounting Adjustments = $580,000 – $315,000 + 0 – 0 = $265,000

* Fees and expenses represent Raleigh’s only nonowner changes in equity. The two measures are equal because Raleigh’s economic events/activities do not differ from those described in the income statement.

P13–4 a. Sales Sales returns Cost of goods sold Dividends Rent expense Wages payable Loss on sale of food services division Loss incurred by food services division Depreciation expense Cumulative effect on income of change in depreciation methods Gain on land appropriated by government Insurance expense Inventory Administrative expenses Prepaid insurance Gain on sale of short-term investments

Income Statement Yes Yes Yes No Yes No Yes segment Yes segment Yes

Classification . Usual and frequent Usual and frequent Usual and frequent N/A Usual and frequent N/A Disposal of business

Yes method

Change in accounting

Yes Yes No Yes No Yes

Unusual and infrequent Usual and frequent N/A Usual and frequent N/A Unusual or infrequent

Disposal of business Usual and frequent


P13–4

Concluded

Note:

The loss on sale of food services division and the loss incurred by food services division would be classified as a disposal of a business segment only if the food services division meets the requirements of being a business segment as defined under GAAP. If the food services division does not meet these requirements, then the loss on sale of food services division would be classified as part of other revenues and expenses. The loss incurred by food services division would be broken down into its components (i.e., revenues and expenses) and reported as part of operating revenues and expenses.

b. Crozier Industries Income Statement For the Year Ended December 31, 2012 Revenue: Sales revenue......................................................................... $ 977,000 Less: Sales returns ................................................................. (9,000) Gain on sale of short-term investments ................................ 142,000 Total revenue.............................................................................. Expenses: Cost of goods sold ................................................................. $ 496,000 Rent expense ......................................................................... 90,000 Depreciation expense ............................................................ 100,000 Insurance expense ................................................................. 12,000 Administrative expenses ....................................................... 109,000 Total expenses.................................................................. Income from continuing operations (before taxes) ................... Income taxes .............................................................................. Income from continuing operations........................................... Discontinued operations: Loss on operation of discontinued segment (net of tax benefit of $3,500)............................................... $ (6,500) Loss on disposal of discontinued segment (net of tax benefit of $700).................................................. (1,300) Discontinued operations .................................................. Income before extraordinary items ........................................... Extraordinary gain on appropriated land (net of taxes of $32,200) ................................................................... Income before effect of accounting changes ............................. Cumulative effect of change in accounting principle (net of tax benefit of $45,500) .............................................. Net income .................................................................................

$ 1,110,000

$

807,000 303,000 106,050 196,950

$

(7,800) 189,150

$

59,800 248,950

$

(84,500) 164,450

$

Sales revenues and all the expenses (cost of goods sold, rent, depreciation, insurance, and administrative) are expected to persist into the future. The gain on the sale of short-term investments is expected to occur infrequently, as a secondary activity to the primary business operations. The losses due to the discontinued operations loss and its sale are one-time amounts, not expected to persist into the future. The extraordinary gain on appropriated land is both unusual and infrequent and is not expected to occur again. Finally, the cumulative effect of a change in accounting principle may occur, but only infrequently. Future income statements should not be affected by many accounting changes.


P13–5 a. 1. Hurricanes are unusual in that they are not part of a company's normal operations; however, they occur relatively frequently in Florida. Thus, this loss probably should not be classified as an extraordinary item. Instead, it should be disclosed gross of taxes as part of other revenues and expenses. 2. A loss on the disposal of a business segment is not considered an extraordinary item. This item should be disclosed net of taxes as a separate item on the income statement after income from continuing operations, but before extraordinary items. The classification of this item is the disposal of a business segment. 3. This loss appears to be both unusual and infrequent; consequently, it should be classified as an extraordinary item on the income statement. Extraordinary items should be disclosed net of any tax effect. 4. Writing off an open account receivable as uncollectible should not be disclosed on the income statement. Under GAAP, a company should use the allowance method to account for bad debts. With the allowance method, write-offs of uncollectible accounts affect only balance sheet accounts. 5. Floods are unusual in that they are not part of a company's normal operations. Although Arizona does get flooding, particularly flash floods, the flooding would probably be considered to be infrequent. Consequently, this loss should be classified as an extraordinary item on the income statement. Extraordinary items should be disclosed net of any tax effect. b. Extraordinary items: Loss to employee destruction (net of tax benefit of $78,750) ........................... 146,250 Loss due to flood (net of tax benefit of $31,500) ............................................... 58,500 Total Loss on Extraordinary Items $204,750

P13–6 a. Income from continuing operations (before taxes) Income taxes Income from continuing operations Extraordinary loss (net of tax benefit of $47,250) Net income b. Income tax expense

$ $ $

850,000 297,500 552,500 (87,750) 464,750

= Income taxes on net income generated during 2012 = Income taxes on income from continuing operations + Income tax effect of extraordinary loss = $297,500 tax expense [from part (a)] – $47,250 tax benefit [from part (a)] = $250,250


P13–6

Concluded

c. MTM Company Statement of Retained Earnings For the Year Ended December 31, 2012 Retained earnings, January 1, 2012.................................................................... Plus: Net income................................................................................................. Less: Dividends declared during 2012 ................................................................ Retained earnings, December 31, 2012 .............................................................

$ 1,259,000 464,750 (175,000) $ 1,548,750

d. The balance in the Income Tax Liability account would equal the beginning balance plus the entire amount owed to the government for income taxes as of December 31, 2012 less tax payments made during 2012. MTM has two sources of income taxes during 2012. The first source is the income tax on income from continuing operations in the amount of $297,500 [from part (a)]. The second source is the income tax benefit on the extraordinary loss in the amount of $47,250 [from part (a)]. Therefore, MTM owes the beginning balance of $70,000 plus the total of the two sources of 2012 taxes of $250,250 less the 2012 tax payments of $200,000. Therefore, the tax liability balances of December 31, 2012 equals $120,250.

P13–7 a. This allows the user of the financial statements to try and make estimates of the future potential results of the company. Discontinued operations will not impact future revenues and expenses so this gives more information to the user. Continuing operations gives users of the financial statements the best information for estimating the company’s future performance. b. An estimate of the number of shares outstanding is calculated by taking net earnings ($3,544.2) and dividing it by the basic net earnings per share ($2.87) or 1,235 million shares. c.

The distinction between basic net earnings per share and diluted net earnings per share is that basic is calculated using the weighted average number of shares outstanding and diluted is calculated using the outstanding weighted average number of shares plus the number of shares that would be outstanding if all convertible instruments were converted to common shares. The most common financial instruments that are convertible to common shares are stock options, preferred stock and some bonds.

d. An estimate of the number of shares that would be outstanding if all other instruments were converted to common shares is calculated by taking net earnings ($3,544.2) divided by diluted net earnings per share ($2.83) or 1,252 million shares.

P13–8 a.

The readers of the financial statements are interested in what operations and profitability are likely to continue into the following year. Line items such as discontinued operations and extraordinary events will not occur in the following year and thus should be separated from continuing operations that will be likely to recur in the upcoming year.

b.

Earnings were $1,362 million and earnings per share were $1.08, therefore the number of shares would be $1,362/$1.08 = 1,261.111 million.


P13–8 Concluded c.

Earnings per share represents the net income that can be attached to each share that is currently outstanding. If, however, additional shares become outstanding (due to, for example, the conversion of preferred shares into common shares), then the net income needs to be spread among a larger number of shares. Diluted EPS shows the net income spread among the total number of shares that could be outstanding.

d.

Earnings were $1,362 million and diluted earnings per share were $1.07, therefore the number of shares would be $1,362/$1.07 = 1,272.897 million.

P13–9 a. 1. Extraordinary losses are disclosed on the income statement net of the tax benefit after discontinued operations but before the effect of changes in accounting methods. 2. Extraordinary gains are disclosed on the income statement net of taxes after discontinued operations but before the effect of changes in accounting methods. 3. The sale of inventory would be disclosed on the income statement (gross of taxes) as part of operating revenues. The associated cost of inventory sold would be disclosed on the income statement (gross of taxes) as part of operating expenses. 4. The loss on disposal of a business segment would be disclosed on the income statement net of taxes as part of discontinued operations, which is disclosed after income from continuing operations but before extraordinary items. 5. The income effect due to change in accounting method would be disclosed net of the tax effect on the income statement as the last item before net income. 6. Advertising expense would be disclosed on the income statement (gross of taxes) as part of operations expenses. 7. Income earned by the disposal of a business segment would be disclosed on the income statement net of taxes as part of discontinued operations, which is disclosed after income from continuing operations but before extraordinary items. b. Extraordinary items: Extraordinary loss (net of tax benefit of $87,500) ............................... Extraordinary gain (net of taxes of $19,250)........................................ Discontinued operations: Loss on disposal of business segment (net of tax benefit of $35,000) ........................................................... Income earned by disposed business segment (net of taxes of $52,500) .................................................................... Effect of change in accounting method (net of taxes of $28,000) ....................................................................

(162,500) 35,750

(65,000) 97,500 52,000


P13–9 Concluded c.

Income from continuing operations ................................... Disposal of business segment ............................................. Extraordinary items ............................................................. Effect of changes in accounting method ............................. Net earnings per share ........................................................ a $0.16

= = b $(0.63) = =

$ 3.00 0.16a (0.63)b 0.26 $ 2.79

(Income from disposed segment – Loss on disposal) ÷ 200,000 shares ($97,500 – $65,000) ÷ 200,000 shares (Extraordinary gain – Extraordinary loss) ÷ 200,000 shares ($35,750 – $162,500) ÷ 200,000 shares

P13–10 Microbiology Labs Income Statement For the Year Ended December 31, 2012 Sales revenue......................................................................... Cost of goods sold ................................................................. Gross profit ............................................................................ Operating expenses ............................................................... Income from operations ........................................................ Loss on sale of office equipment ........................................... Income from continuing operations (before taxes) .............. Income taxes ......................................................................... Income from continuing operations...................................... Disposal of business segment: Gain on disposal of assets (net of taxes of $87,500) ......$ Income from operations of disposed segment (net of taxes of $52,500) ........................................................ Disposal of business segment ..................................... Income before extraordinary items ...................................... Extraordinary items: Loss on retirement of bonds (net of tax benefit of $28,000) ...................................................................... Loss due to insect infestation (net of tax benefit of $280,000) .................................................................... Extraordinary losses .................................................... Net income ............................................................................

$ 10,000,000 2,500,000 $ 7,500,000 750,000 $ 6,750,000 (60,000) $ 6,690,000 2,341,500 $ 4,348,500 162,500 97,500* 260,000 $ 4,608,500

$

(52,000) (520,000) (572,000) $ 4,036,500

Earnings per share: Income from continuing operations.................................................................... Disposal of business segment.............................................................................. Extraordinary items .............................................................................................

$ 2.17 0.13 (0.29)


Net earnings per share ........................................................................................ * $97,500

$ 2.01

= [Discontinued segment's sales – Discontinued segment's cost of goods sold – Discontinued segment's operating expenses  (1 – Tax rate of

35%)

The objectives of financial accounting are to allow prediction of future cash flows. Earnings numbers are useful because they reflect changes in a company’s resources. Some measures such as income from operations reflect amounts expected to persist in the future from the normal business operations. Other measures, such as income from continuing operations, reflect an infrequent or unusual item with less persistence. Finally, extraordinary items, disposal gains and losses, and cumulative effects of accounting method changes have low persistence and are less useful for prediction of future cash flows.

P13–11 a. The sale of the credit card division (disposal of a business segment) would be shown on the income statement after net income from continuing operations; the segment would be separated into two line items, the income from the discontinued operations up to the date of disposal and the gain from the disposal (both shown net of tax). The restructuring charges would be shown in “Other revenues and expenses”, on the income statement after the calculation of net operating income. b. Sears’ performance over the two-year period is better measured when the one-time, non-recurring items are removed from the analysis. The profitability of $309 million is not as strong as it appears, because it includes a gain from the sale of a division (that will not recur in the following year); likewise, the following year’s profits of $53 million should actually be viewed more positively because the restructuring expense (which is presumably a non-recurring item) dragged down that year’s bottom line.

P13–12 a. Recognized income and expense under IFRS is similar to comprehensive income under U.S. GAAP. b. The SORIE would be presented as: Net profit Fair value gains(losses) on available-for-sale securities and currency translation gains(losses) Total recognized income and expense c.

1,500 (1,266) 234

The statement of shareholders’ equity will show the effects from currency and securities transactions that affect equity but do not affect profitability. (The balance sheet will also show the end result in the shareholder equity section.)


P13–13 a. Adjusting journal entries (1)

(2)

Inventory (ending) (+A) ................................................................. Cost of Goods Sold (E, –SE) ............................................................ Purchases (–A) ........................................................................ Inventory (beginning) (–A) ...................................................... Recorded cost of inventory sold.

480,000 737,000

Bad Debt Expense (E, –SE) ............................................................. Allowance for Doubtful Accounts (–A) ................................... Estimated bad debt expense.

25,000*

* (3)

(4)

(5)

25,000

$25,000 = $75,000 – $50,000 Balance in Allowance for Doubtful Accounts

Depreciation Expense (E, –SE) ....................................................... Accumulated Depreciation (–A).............................................. Depreciated fixed assets.

85,000

Insurance Expense (E, –SE) ............................................................ Prepaid Insurance (–A) ........................................................... Recognized expiration of insurance policy.

20,000

Interest Expense (E, –SE) ............................................................... Interest Payable (+L) ............................................................... Incurred, but did not pay, interest.

25,000*

* (6)

750,000 467,000

20,000

25,000

$500,000  10% = $50,000 less 25,000 Already Recognized

Income Tax Expense (E, –SE) ......................................................... Income Tax Liability (+L) ......................................................... Accrued income taxes on continuing operations. *

85,000

$21,000 = =

21,000* 21,000

Income from continuing operations  Tax rate of 35% {($1,256,000 + $76,000) – [$737,000 + $100,000 + $255,000 + ($25,000 + $25,000) + $25,000 + $85,000 + $20,000]}  35%

Income Tax Liability (–L) ............................................................................ Extraordinary Loss (–Lo, +SE) ............................................................. Recognized income tax benefit from extraordinary loss.

12,250* 12,250

* $12,250 = Extraordinary loss of $35,000  Tax rate of 35% Income Tax Liability (–L) ............................................................................ Cumulative Loss from Accounting Change (–Lo, +SE) ........................ Recognized income tax benefit from accounting change.

21,000 21,000


P13–13 Continued Closing entries (c1) Income Summary ........................................................................... Sales ............................................................................................... Gain on Sale of Land ..................................................................... Cost of Goods Sold ................................................................. Administrative Expenses......................................................... Selling Expenses ...................................................................... Interest Expense .................................................................... Bad Debt Expense .................................................................. Depreciation Expense ............................................................. Insurance Expense .................................................................. Income Tax Expense ............................................................... Extraordinary Loss .................................................................. Cumulative Loss from Accounting Change ............................. Closed revenues and expenses into Income Summary. (c2)

(c3)

22,750 1,256,000 76,000 737,000 100,000 255,000 50,000 25,000 85,000 20,000 21,000 22,750 39,000

Retained Earnings ......................................................................... Income Summary .................................................................... Closed Income Summary into Retained Earnings.

22,750

Retained Earnings .......................................................................... Dividends ................................................................................ Closed Dividends into Retained Earnings.

135,000

22,750

135,000


P13–13 Concluded b. Laidig Industries Income Statement For the Year Ended December 31, 2012 Revenues: Sales ....................................................................................... $ 1,256,000 Gain on sale of fixed assets ................................................... 76,000 Total revenues .................................................................. Expenses: Cost of goods sold ................................................................. $ 737,000 Administrative expenses ....................................................... 100,000 Selling expenses..................................................................... 255,000 Depreciation expense ............................................................ 85,000 Bad debt expense .................................................................. 25,000 Insurance expense ................................................................. 20,000 Interest expense .................................................................... 50,000 Total expenses.................................................................. Income from continuing operations (before taxes) ................... Income taxes .............................................................................. Income from continuing operations........................................... Extraordinary loss (net of tax benefit of $12,250) ..................... Income effect due to change in accounting principle (net of tax benefit of $21,000) .............................................. Net loss .......................................................................................

$ 1,332,000

1,272,000 $ 60,000 21,000 $ 39,000 (22,750)

$

(39,000) (22,750)

Earnings per share: Income from continuing operations ........................................... Extraordinary loss ....................................................................... Income effect due to change in accounting principle ................ Net earnings per share ...............................................................

$ 0.20 (0.11) (0.20) $(0.11)

c. Laidig Industries Statement of Retained Earnings For the Year Ended December 31, 2012 Retained earnings, January 1, 2012.................................................................. Net loss ........................................................................................................... Less: Dividends declared .................................................................................. Retained earnings, December 31, 2012 ...........................................................

$ 673,000 (22,750) (135,000) $ 515,250

d. The objectives of financial accounting are to allow prediction of future cash flows. Earnings numbers reflect changes in a company’s resources. Some measures such as income from continuing operations are highly persistent, reflecting amounts expected to persist in the future due to normal business operations. Net income includes an extraordinary loss and income from a change in accounting principle, items that have low persistence and are less useful for the prediction of future cash flows.


ISSUES FOR DISCUSSION ID13–1 a. The net income and comprehensive income for CVS are very similar due to the relatively low dollar amount of changes to equity that did not also affect profitability. Caterpillar, on the other hand, saw a significant drop ($3,788 million) from net income to comprehensive income due to losses from foreign exchange, pension obligations and securities holdings. b. Caterpillar operates globally, so it must deal with foreign currency exposure; as shown, if exchange rates move against the company, hits to equity occur. CVS is mainly focused on its domestic retail operations, so it does not have this same exposure. Caterpillar has negotiated labor contracts with its production employees that specify some defined benefit pension obligations for the company, which bring into effect market and interest rate risks that can create equity losses for the company. CVS employs a different type of workforce and does not offer its employees the same type of retirement benefits. c. An analyst would focus on the companies’ profit levels, but would also focus on what other factors could harm the company’s equity cushion. A certain level of expertise and understanding in foreign exchange markets and trends would be required for an analyst following Caterpillar; an analyst focused on CVS would not be as concerned with the FX markets. Similarly, an understanding of pension accounting and assumptions appears more important for an analyst following Caterpillar than it would for someone following CVS.

ID13–2 a. Tightened credit markets will mean less volume in debt issuances. Since each issue is rated by an agency, the revenue of the ratings agencies will decline as the number of debt issuances declines. b. The income statement will be directly affected with a drop in revenue and a consequent drop in profits. Stockholders’ equity will be affected due to the reduced net income shown on the income statement. To balance the lowered equity, the ratings agencies will either reduce asset balances or increase liabilities. c.

It is very possible that the companies will see less volume in the future, even if the credit markets return to their previous levels. It is possible that we will see a restructuring in the markets, with less emphasis placed on the ratings assigned by outside agencies. Given the fact that the agencies assigned inaccurate grades on the mortgage-backed securities, and the fact that these inflated grades caused investors to lose principal, it is very possible that investors will look to a different mechanism to help understand risk. This potential would permanently depress revenue and profit levels for the ratings agencies.

ID13–3 a. For an event to be classified as extraordinary, the event must have been both unusual in nature and infrequent in occurrence.


b. If the eruptions continue periodically, then such eruptions would probably not be viewed as being infrequent in occurrence. Thus, any losses resulting from future eruptions probably would not be considered to be extraordinary. c.

The entire loss would have been $69.231 million. That is, the after-tax loss of $36 million divided by (1 – tax rate of 48%). The journal entries would be: Extraordinary Loss from Volcano Eruption (Lo, –SE) ............................ Timberland (–A) ............................................................................. Recognized extraordinary loss from volcano eruption.

69,231,000

Income Tax Liability (–L) ....................................................................... Extraordinary Loss from Volcano Eruption (–Lo, +SE) ................... Recognized tax benefit from volcano eruption.

33,231,000

69,231,000

33,231,000

ID13–4 a. Standard & Poor's may have viewed the charge as different from previous years and felt including it would distort comparisons from year to year. Value Line may have felt that the exclusion of the charge would be a distortion of the income for the year. b. The income statement is comprised of several categories. Revenues and expenses that are both usual and frequent are classified as operations. Revenues and expenses that are either usual or frequent, but not both, are classified as other revenues and expenses and considered part of continuing operations. Following continuing operations are special items. Such items, in order of their disclosure, are discontinued operations, extraordinary items (which are both unusual and infrequent), and the cumulative effect of accounting changes. All items listed after continuing operations are disclosed net of the associated tax effect. c.

All items listed on a company's income statement are important in that these items affect a company's actual financial position and/or the amounts reported on the balance sheet for assets and liabilities. Thus, financial analysts do not care how or where the information is recorded as long as the information is available. Alternatively, the location of items on the income statement can have important economic consequences. For example, if a company's management receives incentive compensation based on income, it is important whether the incentive compensation contract defines income as income from operations, income from continuing operations, or net income. Whether something is classified as an operating expense, an other expense, or an extraordinary loss, therefore, would affect management's incentive compensation, depending on how income is defined. In addition, stockholders are interested in a company's earning power, i.e., the company's ability to generate net assets on an ongoing basis. If the company were to include items as part of income from continuing operations that were not expected to continue on into the future, stockholders would be receiving distorted information that could cause them to misvalue the company's stock. Thus, the classification of items on the income statement could affect the magnitude of management's incentive compensation as well as the valuation of the entire company.

ID13–5 a. Analysts are many times looking for companies that will be the dominant company in different business categories. The internet has provided an alternative method for selling goods and services to the public. Since this is a new industry many analysts were not sure how much market share this channel


would take away from the traditional retailing industry. As a result analysts were trying to pick the future winners in this new industry channel. Since few of these companies are profitable analysts had to develop alternative measures to try and evaluate the relative and absolute performance of these companies. b. The specific effect that this FASB rule had on the income statements on companies like Amazon.com was to increase the cost of good sold and decrease the gross profit percentage. These companies lobbied against this change because these companies were promoting the idea that their gross profit percentages were much higher than traditional retailers. After this change it became very apparent that their cost structure was not a competitive advantage against traditional retailers. At that point their stock prices started to decline. c.

There is no impact on the reported cash flows of these companies as a result of this accounting change. This change moves where these costs are reported on the income statement but has no impact on the physical operations of the company.

d. Theoretically, the price of a company's stock equals the present value of the cash flows the stock market expects a stockholder in that company to realize from holding that stock. Thus, the stock price of these companies should not be impacted by this accounting change. While the cash flow of a company will not change as a result of this accounting change it does impact the expectations of investors as to the future cash flows of these companies. This change highlighted information in a way that reduced investors future estimates of the profitability or cash flow of these companies.


ID13–6 a. Although it could be argued that lawsuits are a normal part of conducting business in the United States and that lawsuits may not be that infrequent, the settlement was probably disclosed as an extraordinary loss. The determining factor in each case would be if the event was infrequent and unusual, if the event is both of these then it should be reported as an extraordinary. Extraordinary items should be disclosed net of the associated tax effect. With a tax rate of 34% and a loss of $909.5 million, Kodak would receive a tax benefit of $309.23 million. Thus, Kodak should have reported a net loss of $600.27 million on its 1990 income statement for the settlement. b. The patent infringement case is an example of a contingency. SFAS No. 5, "Accounting for Contingencies" states that a contingency is "an existing condition, situation, or set of circumstances involving uncertainty as to possible gain or loss to an enterprise that will ultimately be resolved when one or more future events occur or fail to occur." In this particular case, Polaroid had a gain contingency and Kodak had a loss contingency. Under the guidelines set forth in SFAS No. 5, the most that Polaroid could have done in its 1989 annual report was to disclose the lawsuit in a footnote. Alternatively, Kodak could have either disclosed nothing about the lawsuit, disclosed the lawsuit in a footnote, or recorded a loss and associated liability. The appropriate course of action depended upon (1) whether it was remote, reasonably possible, or probable, given the information available in 1989, that Kodak would eventually lose the lawsuit and (2) whether the amount of the loss could be reasonably estimated in 1989. If the probability that Kodak would eventually lose the lawsuit was remote, then Kodak could ignore the lawsuit for financial reporting purposes. If the probability of the loss was reasonably possibly or if the amount of the loss could not be reasonably estimated, Kodak should have disclosed the lawsuit in a footnote only. Finally, if both the probability of the loss was probable and the amount of the loss could be reasonably estimated, Kodak should have recorded a loss and associated liability. Due to the magnitude of the case and the associated publicity, it is doubtful that Kodak ignored the lawsuit for financial reporting purposes. In addition, if Kodak knew that it was probable that it would lose the lawsuit, it probably would have settled the case out of court. Thus, Kodak most likely did not think it was probable that it would lose the lawsuit, and therefore only disclosed the lawsuit in a footnote. c.

There are at least two reasons why Kodak's stock increased in value. First, the settlement provided unexpected "good" news about Kodak. The stock market may have expected Kodak to lose the lawsuit and have to pay out close to the amount being asked for by Polaroid. The fact that Kodak had to pay considerably less than expected caused the stock market to positively reevaluate the prospects of investing in Kodak. Second, Kodak reported a large increase in operating earnings. Stockholders are interested in a company's earning power. That is, stockholders are interested in a company's ability to generate net assets from operations on an ongoing basis. The large increase in Kodak's operating earnings provided information to the stock market that Kodak has strong earning power. Alternatively, the lawsuit settlement is a one-time payout that should not adversely affect Kodak's ongoing operations, although it may have some short-term effects on Kodak's earning power.


ID13–7 a. Net earnings from discontinued operations – is shown net of tax as an irregular item on the income statement. Research and development costs – are shown as an expense below cost of goods sold and before income from continuing operations. Foreign currency translation – recognized gains and losses are shown gross on the income statement as an other gain or loss. Any unrecognized gains or losses are included in comprehensive income. Net gain on disposal of business – is shown net of tax below income from continuing operations on the income statement. Provision for restructuring – is shown gross on the income statement in the other gain or loss section. Gain on the sale of assets – are shown gross in the other gain or loss section of the income statement. Cost of products sold – is shown gross in the cost of goods section of the income statement. Litigation charges – are shown gross in the other gains and losses on the income statement. Equity in net income of affiliates— is shown gross on the income statement and before income from continuing operations. b. The items that would be expected to be persistent or be repeated in future years are research and development costs, foreign currency translation, cost of products sold and equity income from affiliates. Each of these items is an expense (or gain) that Bristol-Myers Squibb would expect to incur in future years in the normal operation of the business. The other items are not expected to occur again in future years.

ID13–8 a. Federal Express should report this as an extraordinary gain on its income statement. This assumes that aircraft being destroyed by fire is both unusual and infrequent. This is a one-time impact on the wealth of Federal Express and would not be expected to persist in the future. b.

Motorola would report the gains related to these asset sales below the gross profit line in a section called “Other Gains and Losses”. The sale of assets is not Motorola’s primary business and so these gains should not be shown in the revenue section at the top of the income statement. At the same time these sales are not extraordinary items since they are not unusual and infrequent. These sales represent one-time increases in wealth to the company and would not be expected to persist into the future. c.

Owens-Corning would report the $68 million restructuring charge below the income from operations line, the $11 million of equity in net income of affiliates in the revenue section, and the $15 million accounting change below the income from operations line. Both the restructuring charge and the accounting change would not be expected to persist in the future but the equity in net income of affiliates would be expected to persist in the future.

d. Owens-Corning would report the $875 million charge below the income from operations line on the income statement. The class action lawsuits related to asbestos are (as a group) unusual and infrequent and should not be reported in income from operations. This is a one-time reduction in the wealth of OCF and would not be expected to persist into the future.


ID13–9 a. Rising raw material costs would be reflected in Cost of Goods Sold on the income statement, as well as in Inventory (Raw Materials) on the balance sheet. Currency fluctuations would be reflected in comprehensive income under currencey translation adjustments. A gain would be reflected on the income statement under “other gains/expenses”. Increased costs to introduce new models would be reflected on the income statement under operating expenses (engineering, marketing, etc.). b. Rising material costs and costs to introduce new models might be expenses that will permanently affect BMW’s business model. c.

Comprehensive income measures all changes in a company’s equity due to nowowner transactions. Foreign currency adjustments (as seen by global operations such as BMW) and certain holding gains/losses are not thought to affect a company’s profits but do indeed affect the amount of equity shown on the financial statements. Comprehensive income captures all nonowner changes to equity.

d. Analysts will review a number of factors in addition to a company’s earnings, including: macroeconomic events, overall stock market pricing, a company’s guidance about future earnings, management changes, and a company’s market share.

ID13–10 a. Volkswagen and Carrefour both define operating income to come before (to not include) income from affiliated companies, while Sony includes the equity income from affiliates in its operating income. Carrefour includes interest (finance) costs in its calculation of operating income, while Volkswagen and Sony both deduct interest expense after operating income is calculated. b. It is possible that the financing function for Carrefour is more aligned with that company’s daily operations of its business. It is possible that Sony considers the work of its affiliated companies to be so closely aligned with its own core operations that it includes the equity income from those affiliates in the calculation of operating profits. c. An analyst following all three of these companies would strive for consistency in the analysis of the companies’ profits. An analyst would reconstruct the three income statements to treat all the line items in the same manner; for example, the analyst may follow VW’s lead and take the affiliate income and interest expense for Carrefour and Sony and move them lower on the income statement so that they are not included in the operating profit line.

ID13–11


a. The most significant non-operating income statement item for Nike is impairment; in the year ending 5/31/2009, Nike charged over $400 million against earnings to lower the carrying cost of intangible assets (including Goodwill). These charges did not occur in fiscal 2007 or 2008. In addition, the company booked a $195 million restructuring expense (that did not occur in previous years and would not be likely to recur in 2010) and showed an unusually large other income addition. In total, these one-time items in 2009 decreased net income by $507.8 million. b. Nike’s EPS increased from $2.96 in 2007 to $3.80 in 2008 and then decreased to $3.07 in 2009. If all potential shares are converted to outstanding, EPS would drop from 2009’s $3.07 to $3.03. c.

Other comprehensive income items are shown in the Statement of Stockholders’ Equity. For the year ending 5/31/2009, comprehensive income was $1,602.8 million, $116.1 million higher than net income. The difference was due to net gains on cash flow and investment hedges and losses from foreign currency translation adjustments.


CHAPTER 14 THE STATEMENT OF CASH FLOWS BRIEF EXERCISES BE14–1 a. Depreciation expense is shown as an adjustment to net income to calculate cash flow. Depreciation expense is added back to net income because it is a non-cash expense. This means that it is deducted in the calculation of net income but there is no cash expenditure related to depreciation. b. Net income plus depreciation does not equal net cash provided by operating activities because all of the changes in current assets and current liabilities are also shown in the cash provided by operating activities section. Increases in current asset accounts represent a use of cash and increases in current liabilities present a source of cash. c.

The estimated net change in current assets and current liabilities is $2.9 billion. This is calculated by taking $14.6 billion net cash provided by operating activities minus (net income of $8.3 billion plus $3.4 billion of depreciation and amortization).

BE14–2 During 2008 Pier One collected $1,305.1 million from its customers. This can be calculated as follows: Accounts receivable, beginning balance Add: 2008 Sales Less: Accounts receivable, ending balance Cash collected during 2008

$ 24.2 million 1.3 billion 19.1 million $1,305.1 million

BE14–3 a. Cost of inventories purchased during 2008 equals: Inventory, ending balance Add: 2008 Cost of sales Less: Inventory, beginning balance Inventory purchased in 2008

$316 million 957 million 412 million $861 million

b. Cash payments made to suppliers during 2008 equals: Accounts payable, beginning balance Add: 2008 inventory purchases Less: Accounts payable, ending balance Cash payments to suppliers made in 2008

BE14–4 1

$

$

106 million 861million – 81 million 886 million


a. Cash from operations Cash from investing Cash from financing Change in cash

Agilent $ 756 (399) (774) $ (417)

AMD $ (692) (27) 220 $ (499)

b. AMD is generating cash from financing activities—that is, the company is raising cash from equity and/or debt issuances. Agilent, on the other hand, is using cash to return to shareholders and/or repay debt. Both companies are investing cash in long-term assets. Finally, Agilent is generating cash from operations, while AMD’s core business is losing cash balances. Overall, the net change in cash is similar for both companies, but AMD is only generating cash through financing, while Agilent is increasing cash balances from its daily operations. c.

Cash from operations exceeds the net income(loss) figure due to depreciation, other non-cash expenses and the change to operating accounts (such as receivables, payables). The difference in Cash from Financing Activities tells you that AMD is having to supplement its cash balance deficits (from operations) through borrowings and/or equity issuances while Agilent is able to use the positive cash generated by its operations to pay dividends and/or repurchase shares and/or reduce debt.

BE14–5 a.

In 2007 profit was below operating cash flows because the change in operating accounts (such as receivables and payables) acted as a source of cash balances. Receivables decreased and/or payables increased, increasing cash balances as the cash was freed up from the operating account line(s). In 2008, when profits exceeded operating cash flows, the change in account balances reversed from what happened in the prior year. The operating accounts in 2008 were a use of cash, with receivables increasing and/or payables decreasing (which tied up cash in the operating account line).

b. In 2007 the company used its impressive cash flow from operations and combined it with the proceeds from debt and equity issuances to fund a rather large investment in long term assets. In 2008, the company sold off some long term assets and used that cash, along with the amount generated from operations, to pay dividends, repurchase shares, and/or reduce debt.

EXERCISES


E14–1 1. 2. 3. 4. 5. 6. 7. 8. 9. 10.

Investing activity Financing activity Operating activity Financing activity Investing activity Investing activity (for the cash paid for the building) Financing activity (for the mortgage payable) Financing activity (for the principal payment) Operating activity (for the interest payment) Operating activity Financing activity Operating activity

E14–2 1.

Not included on the statement of cash flows because it does not affect cash. As proof, the entry for this event would be: Allowance for Doubtful Accounts (+A) .................................. Accounts Receivable (–A) ............................................... Wrote off uncollectible accounts.

XX XX

2.

Investing activity

3.

Financing activity

4.

Any gain or loss would be adjusted out of operating activities. The exchange of notes for the building would have no net impact on the statement of cash flows because it does not affect cash. As proof, the entry for this event would be: Notes Receivable (+A) ........................................................... Accumulated Depreciation: Building (+A) ............................. Building (–A) ................................................................... Sold a building in exchange for a five year note.

5.

XX

Not included on the statement of cash flows because it does not affect cash. As proof, the entry for this event would be: Dividend (–SE)........................................................................ Dividend Payable (+L) ..................................................... Declared a dividend.

6.

XX XX

XX XX

Not included on the statement of cash flows because it does not affect cash. As proof, the entry for this event would be: Bonds Payable (–L) ................................................................ Common Stock (+SE)....................................................... Retired bonds with common stock.

XX XX


E14–2

Concluded

7.

Investing activity

8.

Not included on the statement of cash flows because it does not affect cash. As proof, the entry for this event would be: Dividend (–SE)........................................................................ Common Stock (+SE)....................................................... Additional Paid-in Capital, Common Stock (+SE) ............ Declared and issued a stock dividend.

9.

XX XX XX

Not included on the statement of cash flows because it does not affect cash. As proof, the entry for this event would be: Depreciation Expense (E, –SE) ............................................... Accumulated Depreciation (–A)...................................... Depreciated fixed assets.

XX XX

10.Operating activity 11.

Not included on the statement of cash flows because it does not affect cash. As proof, the entry for this event would be: Inventory (+A) ........................................................................ Accounts Payable (+L) ..................................................... Purchased inventory on account.

XX XX

12.

Operating activity

13.

Any gain or loss would be included in operating activities only to adjust Net Income. The balance of the transaction would not be included on the statement of cash flows because it does not affect cash. As proof, the entry for this event would be: Land (+A)................................................................................ Accumulated Depreciation: Building (+A) ............................. Building (–A) ................................................................... Exchanged a building for land.

14.

Financing activity

15.

Operating activity

XX XX XX


E14–3 Company AAA BBB CCC DDD EEE

Cash Provided (Used) by Operations Investments 320 (178) 219 (450) 197 (414) 120 (130) 290 (120)

Financing $(180) 190 80 430 (100)

Net Increase (Decrease) $ (38) (41) (137) 420 70

AAA This company appears to be following a policy of maintaining a relatively constant cash balance. The company also appears to be relying primarily on operating activities to provide cash to finance the acquisition of nonoperating assets and to finance the repayment of debt and/or acquisition of treasury stock. BBB Similar to AAA, this company appears to be following a policy of maintaining a relatively constant cash balance. This company also appears to be using cash from operating activities and from borrowings to purchase nonoperating assets. CCC CCC appears to be using large amounts of cash generated in both the current and prior periods to acquire nonoperating assets. This company also appears to disdain borrowing money. DDD DDD appears to be following a policy of acquiring large amounts of cash in the current period. The company acquired this cash primarily through borrowings. EEE EEE appears to be acquiring moderate amounts of cash in the current period through operating activities. Similar to Company AAA, Company EEE used some of this cash to acquire nonoperating assets and to repay existing debts and/or to acquire treasury stock.

E14–4 Kraft Foods – Cash from Investments ($1,239): the company was able to generate cash from operations and used most of this increase in cash to repay debt, pay dividends, and/or repurchase equity. The company also used cash to purchase long-term assets. Kellogg’s – Net Change in Cash $79: the company was able to generate cash from operations and used that cash to reinvest in long-term assets and to repay debt, pay dividends, and/or repurchase equity. General Mills– Cash from Operations $1,650: the company was able to generate a significant amount of cash from operations and used that cash to reinvest in long-term assets and to repay debt, pay dividends, and/or repurchase equity.


E14–5 Assets 1. Depreciation expense

–170,000

2. Issue of common stock

+180,000

=

=

3. Purchase IBM stock

375,000 (securities) –375,000 (cash)

4. Purchase insurance

+27,000 (ppd insurance) –27,000 (cash)

5. Purchase of building

Liabilities

=

+ Owners’ Equity –170,000 +180,000

+200,000 (building) – 40,000 (cash) = +160,000 (mortgage)

1. Under the direct method, depreciation expense is not included on the statement of cash flows because it does not affect cash. However, under the indirect method, depreciation expense is included under operating activities as an adjustment to net income to arrive at net cash flows from operating activities. 2. Issuance of common stock, $180,000, increases cash. The $180,000 would be included in the financing activities section. 3. Purchase of marketable securities, $375,000, decreases cash. Marketable securities are nonoperating assets for most businesses. Consequently, the purchase of these securities for cash would be disclosed in the investing activities section. 4. Prepaid insurance, $27,000, decreases cash. The $27,000 would be included in the operating activities section. 5. Down payment on building, $40,000, decreases cash. The $40,000 would be included under investing activities. (The mortgage would be disclosed in the footnotes to the financial statements or in a supplemental schedule to the statement of cash flows.)

E14–6 a.

1. 2. 3. 4. 5. 6. 7. 8.

Merchandise Inventory and Accounts Payable Prepaid Insurance Unearned Sales Revenue, Accounts Receivable and Allowance for Doubtful Accounts Rent Payable and Prepaid Rent Dividends Receivable Wages Payable Supplies Inventory Interest Payable, Prepaid Interest, Premium on Bonds Payable, and Discount on Bonds Payable. Note that if a company does not use a Premium on Bonds Payable account for bonds issued at a premium or a Discount on Bonds Payable account for bonds issued at a discount, it would be necessary to use the Bonds Payable account to analyze cash flows associated with interest.


9. 10.

E14–6

Unearned Rent Accumulated Depreciation

Concluded

b. Sales Revenue 1. Unearned Sales Revenue would increase when a company collects cash from a customer in advance of providing goods or services. Since this amount would not be reflected in Sales Revenue until the company provided goods or services, the amount of the increase in Unearned Sales Revenue would have to be added to accrual-basis sales. 2. Accounts Receivable would increase when a company makes credit sales. The amount of the increase would be reflected in Sales Revenue, yet the company would not have collected any cash. Thus, the amount of the increase in Accounts Receivable would have to be deducted from accrualbasis sales. 3. An increase in Allowance for Doubtful Accounts by itself has no effect on accrual-basis sales. However, one must analyze the account to determine whether the company had any write-offs or recoveries of previously written off accounts during the year. Write-offs would be deducted from accrual-basis sales, while recoveries would be added to accrual-basis sales to arrive at cash collections from sales. Cost of Goods Sold Adjusting the balance in Cost of Goods Sold for changes in Merchandise Inventory and Accounts Payable will convert COGS from the accrual-basis to the cash outflow for inventory. An increase in Merchandise Inventory means that the company purchased more inventory than it sold during the year. An increase in the balance of Merchandise Inventory is added to COGS because it is assumed that the company paid cash for the inventory it acquired during the year. However, an increase in the balance of Accounts Payable indicates that this assumption is not valid. That is, an increase in Accounts Payable implies that the company purchased some of its inventory on account rather than for cash. Thus, the increase in Accounts Payable has to be deducted from COGS to offset the increase in Merchandise Inventory added to COGS that did not really reflect cash outflows. Interest Expense 1. An increase in Interest Payable would be deducted from accrual-basis interest expense because the increase implies that the company incurred interest expense, but has not yet paid it. Thus, the balance in Interest Expense overstates the amount of cash disbursed during the year for interest. 2. An increase in Prepaid Interest implies that the company disbursed cash during the year to cover future interest expense. Since the cash disbursement is not reflected in the current period's interest expense, the increase in Prepaid Interest would be added to accrual-basis interest expense to arrive at cash outflows associated with interest. 3. An increase in Discount on Bonds Payable does not affect interest expense per se; it indicates that the company issued additional bonds at less than face value. However, the net increase in the discount is comprised of two components. First, the discount balance increases for the discount associated with the new bonds issued. Second, the discount balance decreases for the amount of the discount balance amortized during the accounting period. Since the amount of the amortized discount flows into interest expense, the amount of the discount amortized would be deducted


from accrual-basis interest expense. Consequently, the discount account would have to be analyzed in depth to determine the magnitude of these two components. Similarly, the balance in Premium on Bonds Payable would have to be analyzed in depth.

E14–7 a.

Hamilton Direct method Cash collections from customers Cash paid for inventory Cash paid for other expenses Net cash flow from operating activities Indirect method Net income Adjustments: Depreciation expense Net cash flow from operating activities

$

$

900,000 (400,000) (200,000) 300,000

Watson $

$

900,000 (400,000) (200,000) 300,000

$ 250,000

$ 200,000

50,000 $ 300,000

100,000 $ 300,000

b. Cash flows from operating activities measures all the cash inflows and cash outflows associated with a company's operating assets and liabilities. Alternatively, net income measures the inflows and outflows of operating assets and liabilities, not just the cash associated with operating assets and liabilities. Thus, cash flows from operating activities can differ from net income due to items affecting net income that do not affect cash and due to cash flows that do not affect net income. In this particular case, the difference is due to depreciation expense. Depreciation is the systematic allocation of the cost of fixed assets. Since depreciation is simply the allocation of the asset's cost, depreciation does not affect cash flows. The indirect method more clearly shows that net income is different from cash flows because this method explicitly reconciles the difference between the two. c.

Disagree. Many people think that depreciation expense represents a fund established to finance future acquisitions of fixed assets. If this were true, it would follow that companies using accelerated depreciation methods would have more cash available than companies that use straight-line depreciation. However, as demonstrated in part (a), depreciation expense has absolutely no effect on the cash flows from operating activities. Both companies have the same cash flows, even though each company uses a different method to compute depreciation. One must remember that depreciation is simply the allocation of the net cost of fixed assets. Cash flows associated with fixed assets arise when fixed assets are acquired or sold, not when the cost of the fixed asset is allocated to expenses.

E14–8 a. 1.

2.

3.

Cash (+A) ........................................................................................ Contributed Capital (+SE) ....................................................... Owner contributed capital.

20,000

Cash (+A) ........................................................................................ Notes Payable (+L) .................................................................. Borrowed money from the bank.

60,000

Property, Plant, & Equipment (+A) ................................................ Cash (–A) ................................................................................. Purchased long-lived assets.

25,000

20,000

60,000

25,000


4.

E14–8 5.

6.

7.

Inventory (+A) Cash (–A) ................................................................................. Accounts Payable (+L) ............................................................. Purchased inventory.

40,000 25,000 15,000

Continued Cash (+A) ........................................................................................ Accounts Receivable (+A) .............................................................. Sales (R, +SE) ........................................................................... Made sale.

20,000 60,000

Cost of Goods Sold (E, –SE) ............................................................ Inventory (–A) ......................................................................... Recognized cost of inventory sold.

25,000

Operating Expenses (E, –SE) .......................................................... Payable to Bank (–L) ...................................................................... Dividend (–SE)................................................................................ Cash (–A) ................................................................................. Made cash disbursements.

18,000 5,000 2,000

Operating Expenses (E, –SE) .......................................................... Operating Expenses Payable (+L) ............................................ Incurred, but did not pay, expenses.

15,000

80,000

25,000

25,000

15,000

b. Tony’s Business Income Statement For the Year Ended December 31, 2012 Sales ........................................................................................................... Cost of goods sold ..................................................................................... Operating expenses ................................................................................... Net income ................................................................................................

$ 80,000 (25,000) (33,000) $ 22,000

Tony's Business Statement of Retained Earnings For the Year Ended December 31, 2012 Beginning retained earnings balance: January 1, 2012 ............................. Plus: Net income........................................................................................ Less: Dividends .......................................................................................... Ending retained earnings, December 31, 2012 .........................................

$

0 22,000 (2,000) $ 20,000


E14–8

Continued Tony's Business Balance Sheet December 31, 2012

Assets Cash ....................................................... Accounts receivable .............................. Inventory ............................................... Property, plant & equipment ................

Total assets............................................ c.

$

25,000 60,000 15,000 25,000

$ 125,000

Liabilities & Stockholders' Equity Accounts payable...................... $ 15,000 Operating expenses payable .. 15,000 Payable to bank ........................ 55,000 Contributed capital ................... 20,000 Retained earnings ..................... 20,000 Total liabilities and stockholders' equity ................ $ 125,000

Cash Beginning balance Owner's contribution Proceeds from bank loan Proceeds from sale

0 20,000 Purchase of fixed assets 25,000 60,000 Purchase of inventory 25,000 20,000 Operating expenses 18,000 Principal payment 5,000 Dividend payment 2,000

Ending balance

25,000

Tony's Business Statement of Cash Flows For the Year Ended December 31, 2012 Cash from operating activities: Cash collections from sales ..................................................... Cash paid for inventory ........................................................... Cash paid for expenses ............................................................ Net cash increase (decrease) due to operating activities .......................................................................... Cash from investing activities: Purchase of fixed assets ..........................................................

$ 20,000 (25,000) (18,000) $ (23,000)

(25,000)


Cash from financing activities: Proceeds from owner's contribution ...................................... Proceeds from bank loan ........................................................ Principal repayment on debt ................................................... Payment of dividend ............................................................... Net cash increase (decrease) due to financing activities .......................................................................... Net increase (decrease) in cash....................................................

$ 20,000 60,000 (5,000) (2,000) 73,000 $ 25,000

Beginning cash balance, January 1, 2012 ..................................... Ending cash balance, December 31, 2012....................................

E14–8

0 $ 25,000

Concluded

d. Tony's Business Statement of Cash Flows For the Year Ended December 31, 2012 Cash from operating activities: Net income ................................................................ Adjustments: Increase in accounts receivable ........................... Increase in inventory............................................ Increase in accounts payable ............................... Increase in operating expense payable................ Total adjustments ........................................... Net cash increase (decrease) due to operating activities ...........................

$ 22,000 $ (60,000) (15,000) 15,000 15,000 (45,000) $ (23,000)

Cash from investing activities: Purchase of fixed assets ............................................ Cash from financing activities: Proceeds from owner's contribution ........................ Proceeds from bank loan .......................................... Principal repayment on debt ..................................... Payment of dividend ................................................. Net cash increase (decrease) due to financing activities .............................................. Net increase (decrease) in cash......................................

(25,000)

$ 20,000 60,000 (5,000) (2,000) 73,000 $ 25,000

Beginning cash balance, January 1, 2012 ....................... Ending cash balance, December 31, 2012......................

0 $ 25,000

E14–9 a.

1.

2.

Cash (+A) ........................................................................................ Common Stock (+SE) ............................................................... Issued common stock.

6,000

Inventory (+A) ................................................................................

6,000

6,000


Accounts Payable (+L) ............................................................. Purchased inventory on account. 3.

4.

6,000

Equipment (+A) .............................................................................. Cash (–A) ................................................................................. Purchased equipment.

5,000

Cash (+A) ........................................................................................ Accounts Receivable (–A) ....................................................... Collected cash from customers.

10,000

5,000

10,000


E14–9 5.

6.

7.

8.

9.

10.

Continued Accounts Payable (–L).................................................................... Cash (–A) ................................................................................. Made payment to suppliers.

5,000

Dividend (–SE)................................................................................ Cash (–A) ................................................................................. Declared and paid cash dividend.

2,000

Rent Expense (E, –SE) .................................................................... Prepaid Rent (+A)........................................................................... Cash (–A) ................................................................................. Disbursed cash for rent.

6,000 6,000

Cash (+A) ........................................................................................ Accounts Receivable (+A) .............................................................. Sales (R, +SE) ........................................................................... Made sales.

65,000 35,000

Miscellaneous Expenses (E, –SE) ................................................... Cash (–A) ................................................................................. Incurred and paid miscellaneous expenses.

40,000

Cash (+A) ........................................................................................ Marketable Securities (–A) ..................................................... Gain on Sale of Marketable Securities (Ga, +SE) .................... Sold marketable securities.

25,000

b.

Cash (B.B.) (1) (4) (8) (10)

25,000 6,000 10,000 65,000 25,000

(E.B.)

67,000

(3) (5) (6) (7) (9)

5,000 5,000 2,000 12,000 40,000

5,000

2,000

12,000

100,000

40,000

20,000 5,000


E14–9

Concluded

c. Driftwood Shipbuilders Statement of Cash Flows For the Year Ended December 31, 2012 Cash flows from operating activities: Cash collections from customers ........................................ Cash payments for rent ....................................................... Cash payments for miscellaneous expenses ....................... Cash payments for inventory .............................................. Net cash increase due to operating activities ................

$ 75,000 (12,000) (40,000) (5,000)

Cash flows from investing activities: Proceeds from sale of marketable securities ...................... Purchase of equipment ....................................................... Net cash increase due to investing activities .................

$ 25,000 (5,000)

Cash flows from financing activities: Proceeds from issue of common stock ............................... Dividend payment ............................................................... Net cash increase due to financing activities ................. Net increase in cash ................................................................. Beginning cash balance, January 1, 2012 ................................. Ending cash balance, December 31, 2012 ...............................

$ 18,000

20,000

$

6,000 (2,000) 4,000 $ 42,000 25,000 $ 67,000

E14–10 Insurance 2012 Ending prepaid insurance $7,000 Insurance purchases Wages 2012 Ending wages payable $6,000 Wages paid

= 2012 Beginning prepaid insurance + Insurance purchases during 2012– 2012 Insurance expense = $4,200 + Insurance purchases – $3,000 = $5,800

= 2012 Beginning wages payable + 2012 Wage expense – Wages paid during 2012 = $0 + $8,500 – Wages paid = $2,500


E14–11 a. 2012 Ending machinery $45,000 Machinery purchased

= 2012 Beginning machinery + Cost of machinery purchased during 2012 – Cost of machinery sold during 2012 = $20,000 + Machinery purchased – $8,000 = $33,000

b. When the machinery was sold during 2012, Dylan’s Toys, would prepare the appropriate entry using the following format. Cash (+A) ............................................................... Accumulated Depreciation (+A) ............................ Machinery (–A) ............................................... Gain on Sale of Machinery (Ga, +SE) ..............

XX XX XX XX

We can find the cash collected for the sale of the machinery by first calculating the other three amounts. Machinery It is given in the exercise that the cost of the machinery sold was $8,000. Gain on Sale of Machinery It is given in the exercise that the gain on the sale was $2,000. Accumulated Depreciation 2012 Ending accumulated depreciation

$15,000 Accumulated depreciation on items sold

= 2012 Beginning accumulated depreciation + 2012 Depreciation expense – Accumulated depreciation on items sold = $10,000 + $7,000 – Accumulated depreciation on items sold = $2,000

From the entry given above, Cash = Cost of machinery sold + Gain on sale of machinery – Accumulated depreciation on machinery sold = $8,000 + $2,000 – $2,000 = $8,000 c.

Cash (+A) ................................................................................................... Accumulated Depreciation (+A) ................................................................ Machinery (–A) ................................................................................... Gain on Sale of Machinery (Ga, +SE) .................................................. Sold machinery.

8,000 2,000 8,000 2,000

E14–12 Cash Receipts 965 Change in payables (‘07-‘08)

Cash Payments

12/31/07 accounts receivable

$

$ 138

2008 Sales

44,564 2008 cost of sales

34,451

12/31/08 accounts receivable

(887) Change in inventory (‘08-’07)

(67)


2008 cash receipts

$44,642 2008 cash payments

$ 34,522


E14–13 1. Direct method The first step in calculating the cash flows from operating activities is to calculate the cash inflows and outflows associated with each income statement account. These calculations are given below. Cash collections from customers: Cash collections from customers

Cash paid for inventory: a. Inventory purchased 2012 Ending inventory $9,000 Net purchases

= 2012 Beginning inventory + Net inventory purchased during 2012 – 2012 Cost of goods sold = $11,000 + Net purchases – $30,000 = $28,000

b. Disbursements for inventory 2012 Ending accounts payable

$3,000 Payments Cash paid for wages: 2012 Ending wages payable $1,800 Wages paid Cash paid for advertising: 2012 Ending prepaid advertising $3,000 Advertising paid

= Sales – Ending accounts receivable + Beginning accounts receivable + Ending deferred revenues – Beginning deferred revenues = $48,000 – $4,000 + $5,000 + $0 – $3,000 = $46,000

= 2012 Beginning accounts payable + Net inventory purchased during 2012 – Payments for inventory during 2012 = $4,000 + $28,000 – Payments = $29,000

= 2012 Beginning wages payable + 2012 Wage Expense – Wages paid during 2012 = $900 + $4,000 – Wages paid = $3,100

= 2012 Beginning prepaid advertising + Advertising paid – 2012 Advertising expense = $1,200 + Advertising paid – $1,000 = $2,800

Cash flows from operating activities: Cash collections from customers........................................................ Payments for inventory ...................................................................... Payments for wages ........................................................................... Payments for advertising .................................................................... Net cash increase due to operating activities ....................................

$ 46,000 (29,000) (3,100) (2,800) $ 11,100


E14–13 Concluded 2. Indirect method Cash flows from operating activities: Net income ............................................................................ Adjustments: Depreciation ..................................................................... Decrease in accounts receivable ...................................... Decrease in inventory ....................................................... Increase in wages payable ................................................ Increase in prepaid advertising ........................................ Decrease in deferred revenues ........................................ Decrease in accounts payable .......................................... Total adjustments........................................................ Net cash increase due to operating activities ..............................................................

$ 11,000 $

2,000 1,000 2,000 900 (1,800) (3,000) (1,000) 100 $ 11,100

E14–14 Grimes Pools Statement of Cash Flows For the Year Ended December 31, 2012 Cash from operating activities: Cash collections from sales ..................................................... Cash paid on operating liabilities ............................................ Cash paid for expenses ............................................................ Net cash increase (decrease) due to operating activities ..........................................................................

$ 35,000 (6,000) (34,000) $ (5,000)

Cash from investing activities: Proceeds from sale of nonoperating assets ............................ Cash from financing activities: Proceeds from issuing debt ..................................................... Payment of dividends .............................................................. Repurchase of contributed capital .......................................... Net cash increase (decrease) due to financing activities .......................................................................... Net increase (decrease) in cash.................................................... Beginning cash balance, January 1, 2012 ..................................... Ending cash balance, December 31, 2012....................................

8,000a

$

2,000b (3,000) (4,000)c (5,000) $ (2,000)

$

6,000 4,000

a Proceeds from sale of nonoperating assets equals the decrease in nonoperating assets. Since no gain

on sale of assets or loss on sale of assets is reported, one must assume that the book value of the assets sold equaled the proceeds from the sale. b Proceeds from issue of debt = Increase in nonoperating liabilities c Repurchase of contributed capital = Decrease in contributed capital


E14–15 Romora Supply House Statement of Cash Flows For the Year Ended December 31, 2012 Cash from operating activities: Cash collections from sales ..................................................... Cash from decrease in noncash operating assets ................... Cash from increase in operating liabilities .............................. Cash paid for expenses ............................................................ Net cash increase (decrease) due to operating activities ..........................................................................

$ 64,000 5,000 5,000 (61,000) $ 13,000

Cash from investing activities: Purchase of nonoperating assets ............................................ Cash from financing activities: Proceeds from contributed capital.......................................... Payment on debt ..................................................................... Payment of dividends .............................................................. Net cash increase (decrease) due to financing activities .......................................................................... Net increase (decrease) in cash....................................................

(4,000)a

$

3,000b (2,000)c (3,000)

Beginning cash balance, January 1, 2012 ..................................... Ending cash balance, December 31, 2012....................................

(2,000) $ 7,000 5,000 $ 12,000

a Purchase of nonoperating assets = Increase in nonoperating assets b Proceeds from contributed capital = Increase in contributed capital c

Payment on nonoperating debt = Decrease in nonoperating liabilities

E14–16 Accrual-Basis Sales 2012 Sales = Cash inflows from sales made during 2012 + 2012 Ending accounts receivable – 2012 Beginning accounts receivable – 2012 Ending deferred revenue + 2012 Beginning deferred revenue = $65,000 + $3,000 – $9,000 – $4,000 + $1,000 = $56,000 Accrual-Basis COGS 2012 COGS = Cash disbursements during 2012 for inventory – 2012 Ending inventory + 2012 Beginning inventory + 2012 Ending accounts payable – 2012 Beginning accounts payable = $40,000 – $18,000 + $10,000 + $7,000 – $3,000 = $36,000


E14–16 Concluded Accrual-Basis Wage Expense 2012 Wage expense = Cash disbursements during 2012 for wages + 2012 Ending wages payable – 2012 Beginning wages payable = $6,000 + $2,100 – $1,300 = $6,800 Accrual-Basis Advertising Expense 2012 Advertising expense = Cash disbursement during 2012 for advertising – 2012 Ending prepaid advertising + 2012 Beginning prepaid advertising = $1,000 – $5,000 + $8,000 = $4,000 Depreciation Expense 2012 Depreciation expense

= 2012 Ending accumulated depreciation – 2012 Beginning accumulated depreciation = $8,000 – $5,000 = $3,000 Schlee and Associates Income Statement For the Year Ended December 31, 2012

Sales .............................................................................................................. Cost of goods sold ........................................................................................ Wage expense .............................................................................................. Advertising expense ..................................................................................... Depreciation expense ................................................................................... Net income ...................................................................................................

$ 56,000 (36,000) (6,800) (4,000) (3,000) $ 6,200

E14–17 L.L. Beeno Operating Section – Statement of Cash Flows (Direct Method) For the Year Ended December 31, 2012 Cash from operating activities: Cash receipts from revenues ................................................... Cash payments for inventory .................................................. Cash payments for wages........................................................ Cash payments for insurance .................................................. Cash payments for interest ..................................................... Cash payments for taxes ......................................................... Net cash increase (decrease) due to operating activities ..........................................................................

$ 45,900 (26,400) (5,100) (3,900) (1,600) _(1,200) $

7,700


E14–17 Concluded

L.L. Beeno Operating Section – Statement of Cash Flows (Indirect Method) For the Year Ended December 31, 2012 Cash flows from operating activities: Net income ............................................................................ Adjustments: Depreciation expense ....................................................... Increase in accounts receivable ........................................ Decrease in inventory ....................................................... Decrease in prepaid insurance ......................................... Decrease in accounts payable .......................................... Increase in wages payable ................................................ Total adjustments........................................................ Net cash increase due to operating activities ..............................................................

$ $

5,500

3,300 (1,100) 300 300 (1,700) 1,100 2,200 $

7,700

E14–18 Martland Stores Operating Section – Statement of Cash Flows (Direct Method) For the Year Ended December 31, 2012 Cash from operating activities: Cash receipts from revenues ................................................... Cash payments for inventory .................................................. Cash payments for wages........................................................ Cash payments for insurance .................................................. Cash payments for interest ..................................................... Net cash increase (decrease) due to operating activities ..........................................................................

$ 97,500 (59,200) (16,500) (8,900) _(2,100) $ 10,800


E14–18 Concluded

Martland Stores Operating Section – Statement of Cash Flows (Indirect Method) For the Year Ended December 31, 2012 Cash flows from operating activities: Net income ............................................................................ Adjustments: Depreciation expense ....................................................... Decrease in accounts receivable ...................................... Decrease in inventory ....................................................... Decrease in prepaid insurance ......................................... Decrease in accounts payable .......................................... Increase in wages payable ................................................ Total adjustments........................................................ Net cash increase due to operating activities ..............................................................

$ (3,600) $

5,700 1,500 5,300 300 (500) 2,100 14,400 $ 10,800

E14–19 Mako Retail Operating Section – Statement of Cash Flows (Direct Method) For the Year Ended December 31, 2012 Cash from operating activities: Cash receipts from revenues ................................................... Cash payments for inventory .................................................. Cash payments for wages........................................................ Cash payments for rent ........................................................... Cash payments for interest ..................................................... Cash payments for taxes ......................................................... Net cash increase (decrease) due to operating activities ..........................................................................

$ 108,700 (58,800) (13,000) (8,400) (3,600) _ (4,400) $ 20,500


E14–19 Concluded Mako Retail Operating Section – Statement of Cash Flows (Indirect Method) For the Year Ended December 31, 2012 Cash flows from operating activities: Net income ............................................................................ Adjustments: Depreciation expense ....................................................... Loss on sale of equipment ................................................ Increase in accounts receivable ........................................ Decrease in inventory ....................................................... Decrease in prepaid rent .................................................. Decrease in accounts payable .......................................... Increase in wages payable................................................ Decrease in interest payable ............................................ Increase in unearned revenue .......................................... Total adjustments........................................................ Net cash increase due to operating activities ..............................................................

$ 11,200 $

6,200 4,200 (2,200) 600 600 (3,400) 2,200 (700) _ 1,800 9,300 $ 20,500

E14–20 Steeler and Jones Operating Section – Statement of Cash Flows (Direct Method) For the Year Ended December 31, 2012 Cash from operating activities: Cash receipts from revenues ................................................... Cash payments for inventory .................................................. Cash payments for wages........................................................ Cash payments for rent ........................................................... Cash payments for interest ..................................................... Cash payments for taxes ......................................................... Net cash increase (decrease) due to operating activities ..........................................................................

$ 86,300 (50,600) (15,500) (11,400) (1,700) _(4,800) $

2,300


E14–20 Concluded Steeler and Jones Operating Section – Statement of Cash Flows (Indirect Method) For the Year Ended December 31, 2012 Cash flows from operating activities: Net income ............................................................................ Adjustments: Depreciation expense ....................................................... Gain on sale of equipment ............................................... Decrease in accounts receivable ...................................... Decrease in inventory....................................................... Increase in prepaid rent ................................................... Decrease in accounts payable .......................................... Decrease in wages payable............................................... Increase in interest payable ............................................. Decrease in unearned revenue ........................................ Total adjustments........................................................ Net cash increase due to operating activities ..............................................................

$

9,300

$

5,700 (5,200) 1,100 1,500 (400) (5,400) (2,300) 200 _ (2,200) (7,000) $

2,300

E14–21 Harbaugh Auto Supply Operating Section – Statement of Cash Flows (Direct Method) For the Year Ended December 31, 2012 Cash from operating activities: Cash receipts from sales revenue............................................ Cash receipts from service revenues....................................... Cash receipts from interest revenue ....................................... Cash payments for inventory .................................................. Cash payments for wages........................................................ Cash payments for rent ........................................................... Cash payments for interest ..................................................... Cash payments for taxes ......................................................... Net cash increase (decrease) due to operating activities ..........................................................................

$ 47,000 32,400 9,600 (26,700) (19,000) (16,500) (5,600) (10,300) $ 10,900


E14–21 Concluded Harbaugh Auto Supply Operating Section – Statement of Cash Flows (Indirect Method) For the Year Ended December 31, 2012 Cash flows from operating activities: Net income ............................................................................ Adjustments: Depreciation expense ....................................................... Gain on sale of investments ............................................. Increase in accounts receivable ........................................ Increase in inventory ........................................................ Decrease in interest receivable ........................................ Increase in prepaid rent ................................................... Decrease in accounts payable .......................................... Decrease in wages payable............................................... Increase in interest payable ............................................. Decrease in taxes payable ................................................ Decrease in unearned revenue ........................................ Total adjustments........................................................ Net cash increase due to operating activities ..............................................................

$ 24,300 $ 11,500 (13,200) (500) (2,900) 300 (1,200) (2,600) (1,400) 600 (1,200) (2,800) (13,400) $ 10,900

E14–22 Standard Center Manufacturing Operating Section – Statement of Cash Flows (Direct Method) For the Year Ended December 31, 2012 Cash from operating activities: Cash receipts from sales revenue............................................ Cash receipts from service revenue ........................................ Cash receipts from interest revenue ....................................... Cash payments for inventory .................................................. Cash payments for wages........................................................ Cash payments for rent ........................................................... Cash payments for interest .....................................................

$ 73,500 25,800 9,500 (14,600) (28,800) (24,200) (7,500)


Cash payments for taxes ......................................................... Net cash increase (decrease) due to operating activities ..........................................................................

_(3,400) $ 30,300

E14–22 Concluded Standard Center Manufacturing Operating Section – Statement of Cash Flows (Indirect Method) For the Year Ended December 31, 2012 Cash flows from operating activities: Net income ............................................................................ Adjustments: Depreciation expense ....................................................... Loss on sale of investments .............................................. Decrease in accounts receivable ...................................... Decrease in inventory ....................................................... Decrease in interest receivable ........................................ Increase in prepaid rent ................................................... Increase in accounts payable ........................................... Decrease in wages payable............................................... Decrease in interest payable ............................................ Decrease in taxes payable ................................................ Decrease in unearned revenue ........................................ Total adjustments........................................................ Net cash increase due to operating activities ..............................................................

E14–23 An estimate of net cash from operations would be: Income before tax Less: Tax Plus: Provision for amortization Less: Net gains on sales of assets Plus: Provision for impairment Less: Net increase in working capital Cash from operations

2,214 (624) 1,946 (219) 642 (964) 2,995

$2,800 $

4,300 17,900 6,000 1,000 2,200 (2,700) 3,900 (400) (300) (2,000) _(2,400) 27,500 $30,300


Taxes are deducted because they represent an operating cash outflow, while the non-cash charges for amortization and impairment are added back. The sale of assets is an investing activity, so it should not be included in the operating section, while the increase in working capital represents a use of cash and is therefore deducted.

PROBLEMS P14–1 a., b., and c. Transaction 1. 2. 3. 4. 5. 6. 7. 8. 9. 10. 11.

Section Operating N/A Operating Investing Operating Financing N/A Financing N/A Financing Investing N/A

Inflow

Outflow

Amount

X

$ 60,000

X

40,000 94,000 15,000 75,000

X X X X

150,000

X

475,000 100,000

X

P14–2 a., b., and c. Transaction 1. 2. 3. 4. 5. 6. 7. 8. 9.

Section Operating Investing Operating N/A Operating Investing Operating Financing Financing Operating

Inflow X

Outflow X X

Amount $ 52,000 12,000 30,000

X X X

10,000 90,000 45,000 50,000 40,000 25,000

X X X


10. 11. 12. 13.

Financing N/A N/A N/A

X

300,000

P14–3 a. Transaction 1. 2. 3. 4. 5. 6. 7. 8. 9. 10. 11. 12. 13.

b.

1. 2. 3. 4. 5.

6. 7. 8. 9. 10. 11. 12.

Cash Affected Yes Yes Yes Yes Yes Yes No Yes Yes No No No No

Type of Effect Provided Used Used Provided Provided Provided Used Used

Dollar Amount $ 1,200 13,000 9,000 7,000 2,500 3,000 7,000 5,000

Investing Investing Operating Financing Operating (for the interest) Investing or Operating (for the principal) depending on how the company acquired the note. If it was accepted in a sales transaction, it could be argued that collecting the principal is an operating activity. If the company acquired the note in any other way, the collection of the principal would be considered an investing activity. Operating N/A Financing Operating N/A N/A N/A


13.

N/A

P14–4 The most notable similarity among all three companies is that they have generated increasing amounts of cash from their operating activities. Amazon.com has used cash to invest in long-term assets in two of the three years shown, while Priceline and eBay both used cash to invest in long-term assets in each of the years shown. The three companies also look very similar from a financing perspective. Priceline, Amazon and eBay typically use cash in their financing activities, which indicates either dividends have been paid, debt has been retired, or equity has been repurchased. The only difference was in 2007, when Priceline and Amazon generated cash from financing activities, meaning the firms issued equity and/or debt to raise cash.


P14–5 Transaction

Effect on Cash

1.

$50,000

2.

(55,000)

3.

100,000

Section of Statement

Explanation

Operating inventory activity. Operating expenses. Investing

Operations is defined in terms of Cash payments for operating Sale of a nonoperating asset.

Note: If the company uses the indirect method to prepare its statement of cash flows, the $15,000 loss on sale of fixed assets would be included in the operating activities section as an adjustment to net income to arrive at net cash flow from operating activities. 4. 5. 6.

(70,000) (500,000) (100,000)

Financing Financing Operating

7. 8.

No effect (500,000)

N/A Investing in equity securities,

9.

$202,000

Financing

10.

No effect

Operating

Dividend payment. Stock repurchase. Cash payment associated with operating assets. No effect on cash. Cash payment for an investment a nonoperating asset. Issuing debt provides financing to the company. Net income would be lower but would be offset by an increase in rent payable. The net of these two is a zero effect on cash.

P14–6

a. Cash from operating activities Cash from investing activities Cash from financing activities Increase (decrease) in cash Cash balance at beginning of year Cash balance at end of year Missing values

2012

2011

2010

$

X 160 (150) $ X 86 $ 176

$

(202) X 280 (24) X 86

$

$

$

(102) 110

$

80 90

$ $

$ $

X (500) 900 110 0 X (290) 110

b. Ruttman Enterprises began operations during 2010. As part of its start-up phase, the company appears to have issued large amounts of stock and/or debt to finance the acquisition of nonoperating assets, to cover the cash used by operating activities during the start-up phase, and to provide a cash surplus for future years. During 2011, the company once again issued large amounts of stock and/or debt to finance the acquisition of nonoperating assets and to cover the cash used by operating activities. However, the company acquired only enough additional financing to meet these needs; it did not acquire an excessive amount of capital through financing activities. By 2012 the company was generating cash from its operating activities and was able to dispose of some of its nonoperating assets. These cash inflows were then used to retire some of the company's debt and/or to reacquire some of its outstanding stock.


P14–7 a.

2008 Cash provided (used) by operating activities Cash provided (used) by investing activities Cash provided (used) by financing activities Increase (decrease) in cash Cash balance at beginning of year

2007

$14,591 $9,606 (13,711) (9,123) (2,020) (5,590) (1,140) (5,107) 11,293 16,400

Cash balance at end of year

$

10,153

2006 $11,353 (2,787) (6,077) 2,489 13,911

$ 11,293

$ 16,400

b. Cash from operations has been consistently strong over the three-year period. H-P uses this cash flow to invest in its growing business (negative cash from investing activities in all three years) and is still able to return cash to shareholders and repay any debt (negative cash from financing activities in all three years). The company’s strong cash flow from operations allows it to continue to fund its growth and to return cash to the sources of funding (debt and equity providers).

P14–8 a. Case 1: Based on the $820,000 beginning balance in the Buildings account and the purchase during 2012 of a building for $60,000, one would expect the Buildings account to have a balance of $880,000 at the end of 2012. The fact that its balance is only $750,000 implies that Webb Industries must have sold a building that originally cost $130,000. Similarly, based on the beginning balance of $80,000 in the Accumulated Depreciation: Buildings account and the $40,000 of depreciation taken on the building during 2012, one would expect the Accumulated Depreciation: Buildings account to have a balance of $120,000 at the end of 2012. The fact that its balance is only $100,000 implies that the accumulated depreciation associated with the building that Webb Industries sold during 2012 must have been $20,000. This information is summarized in the following T accounts. Buildings B.B. Purchase

820,000 60,000

E.B.

750,000

Sale

Accumulated Depreciation X

Sale

Y

B.B. Depr. Exp.

80,000 40,000

E.B.

100,000

X = Cost of building sold = $130,000 Y = Accumulated depreciation on building sold = $20,000 Case 2: Based on the $380,000 beginning balance in the Equipment account and the sale during 2012 of equipment that originally cost $50,000, one would expect the Equipment account to have a balance at the end of 2012 of $330,000. The fact that its balance is $500,000 implies that Webb Industries must have purchased some equipment for $170,000. Similarly, based on the beginning balance of $85,000 in the Accumulated Depreciation: Equipment account and the $15,000 of depreciation taken on the equipment during 2012, one would expect the Accumulated Depreciation: Equipment account to have a balance at the end of 2012 of $100,000. The fact that its balance is only $75,000 implies that the accumulated depreciation associated with the equipment that Webb Industries sold during 2012 must have been $25,000. This information is summarized in the following T accounts.


P14–8 Continued Equipment B.B. Purchase

380,000 X

E.B.

500,000

Sale

Accumulated Depreciation 50,000

Sale

Y

B.B. Depr. Exp.

85,000 15,000

E.B.

75,000

X = Cost of equipment purchased = $170,000 Y = Accumulated depreciation on equipment sold = $25,000 Case 3: Based on the $250,000 beginning balance in the Land account and the sale of land during 2012, one would expect the balance in the Land account to decrease. The fact that its balance is still $250,000 at the end of 2012 implies that (1) Webb Industries must have purchased some land during 2012 and (2) the cost of the land purchased exactly equaled the original cost of the land that was sold. Since a gain on the sale of land equals the excess of the proceeds over the cost of the land, it can be inferred that the land that Webb Industries sold during 2012 originally cost $225,000 (i.e., proceeds of $300,000 less gain of $75,000). Thus, the cost of the land that Webb Industries purchased during 2012 was $225,000. This information is summarized in the following T account. Land B.B. Purchase

250,000 Y

E.B.

250,000

Y = Cost of land purchased = $225,000

Sale

X

X = Cost of land sold = $225,000

Case 4: In exchange between two independent parties, one would expect the fair market value of the item given up by one of the parties to equal the fair market value of the item that party is to receive. Thus, it is probably safe to assume that the fair market value of the new building is $600,000. Based on the $820,000 beginning balance in the Buildings account and the acquisition during 2012 of a new building for $600,000, one would expect the Buildings account to have a balance of $1,420,000 at the end of 2012. The fact that its balance is only $750,000 implies that Webb Industries must have disposed of a building during 2012 that originally cost $670,000. Similarly, based on the beginning balance of $80,000 in the Accumulated Depreciation: Buildings account and the $40,000 of depreciation taken on buildings during 2012, one would expect the Accumulated Depreciation: Buildings account to have a balance at the end of 2012 of $120,000. The fact that its balance is only $100,000 implies that the accumulated depreciation associated with the building that Webb Industries sold during 2012 must have been $20,000. Further, based on the $250,000 beginning balance in the Land account and the disposal during 2012 of land that had a book value of $150,000, one would expect the Land account to have a balance of $100,000 at the end of 2012. The fact that its balance is $250,000 implies that Webb Industries must have acquired some land during 2012 that cost $150,000. This information is summarized in the following T accounts.


P14–8 Continued Buildings B.B. Purchase

820,000 600,000

E.B.

750,000

Sale

Accumulated Depreciation X

Sale

Y

B.B. Depr. Exp.

80,000 40,000

E.B.

100,000

X = Cost of building sold = $670,000 Y = Accumulated depreciation on building sold = $20,000 Land B.B. Purchase

250,000 Z

E.B.

250,000

Sale

150,000

Z = Cost of land purchased = $150,000 b. Case 1: Proceeds from sale

Note:

= Cost of building sold – Related accumulated depreciation = $130,000 – $20,000 = $110,000

This solution assumes that there was no gain or loss on the sale of the building, since no such information was given in the problem.

In the statement of cash flows for 2012, Webb Industries would report the following items under cash flows from investing activities. Proceeds from sale of building Purchase of building Case 2: Proceeds from sale

$ 110,000 (60,000)

= Book value of equipment sold + Gain on sale = ($50,000 – $25,000) + $5,000 = $30,000

In the statement of cash flows for 2012, Webb Industries would report the following items under cash flows from investing activities. Proceeds from sale of equipment Purchase of equipment

$

30,000 (170,000)

Case 3: In the statement of cash flows for 2012, Webb Industries would report the following items under cash flows from investing activities. Proceeds from the sale of land

$300,000


Purchase of land

(225,000)

P14–8 Concluded Case 4: Since Webb Industries exchanged land for a building, this transaction did not affect cash and would not be disclosed in the body of the statement of cash flows. However, Webb Industries did collect $650,000 from the building it sold ($670,000 cost of building sold – $20,000 accumulated depreciation on the building), which Webb Industries would report under cash flows from investing activities.

P14–9 Total number of shares issued during 2012

Number of shares issued for cash

= Change in balance of common stock account ÷ Par value per share of common stock = ($128,000 – $100,000) ÷ $1 per share = 28,000 shares

= Total number of shares issued – (Shares issued as stock dividend + Shares issued in exchange for land) = 28,000 shares – [(100,000 shares outstanding on 1/1/12  20%) + 6,000 shares exchanged for land] = 28,000 shares – (20,000 shares + 6,000 shares) = 2,000 shares

Cash received = Change in common stock account due to issue of common stock for cash + Change in additional paid-in capital, common stock account due to issue of common stock for cash = (2,000 shares  $1 par value per share) + [($95,000 – $12,000) – $40,000a + $12,000b] = $2,000 + $31,000 = $33,000 a $40,000 represents the additional paid-in capital from the 20% stock dividend. The company distributed

20,000 shares, and the fair market value at the time was $3 per share. One dollar was allocated to the Common Stock account, and the remaining $2 was allocated to the Additional Paid-in Capital, Common Stock account. b $12,000 represents the additional paid-in capital from exchanging stock for land.

P14–10 a. Ending accounts receivable

2011 $95,000 Gross sales 2012

= Beginning accounts receivable + (Gross sales – Sales returns) – Cash collections

= $0 + (Gross sales – $20,000) – $350,000 = $465,000


$150,000 Gross sales

= $95,000 + (Gross sales – $25,000) – $500,000 = $580,000

P14–10 Concluded b. 2012 Ending inventory $110,000 Net purchases

= 2012 Beginning inventory + Net purchases of inventory during 2012 – 2012 Cost of goods sold = $130,000 + Net purchases – $375,000 = $355,000

2012 Ending accounts payable

$115,000 Cash payments c.

Ending prepaid insurance

= 2012 Beginning accounts payable + Net purchases of inventory during 2012 – Cash payments for inventory during 2012 = $105,000 + $355,000 – Cash payments = $345,000 = Beginning prepaid insurance + Insurance purchased during the year – Insurance expense

2011 Ending prepaid insurance

= $0 + $65,000 – $35,000 = $30,000

2012 Ending prepaid insurance

= $30,000 + $90,000 – $50,000 = $70,000

P14–11 Accrual sales

= = =

Collections from customers + Increase in accounts receivable $26,000 + $3,000 $29,000

Accrual COGS

= Payments to suppliers – Increase in inventory + Increase in accounts payable = $13,000 – $3,000 + $1,000 = $11,000

Accrual operating expenses

= Payments for expenses – Decrease in accrued payables = $10,000 – $2,000 = $8,000 Battery Builders, Inc. Income Statement

Sales .................................................................................................................. Cost of goods sold ............................................................................................ Depreciation expense ....................................................................................... Other operating expenses ................................................................................ Gain on sale of equipment ............................................................................... Net income .......................................................................................................

$ 29,000 (11,000) (3,000) (8,000) 2,000 $ 9,000


P14–12 a. Pendleton Enterprises Statement of Cash Flows from Operating Activities For the Years Ended December 31, 2010, 2011, and 2012 Cash collections from customers and sales* ............................. Cash payments for expenses and to suppliers** ....................... Net cash increase (decrease) due to operating activities ................................................................................ *Revenues minus increase in Accounts Receivable **Expenses minus increase in Accounts Payable

$

2012 9,000 (12,000)

2011 $ 13,000 (8,000)

2010 $ 4,000 (4,000)

$

(3,000)

$

$

5,000

0

b. Pendleton Enterprises Statement of Cash Flows from Operating Activities For the Years Ended December 31, 2010, 2011, and 2012 Cash collections from customers and sales ............................... Cash payments for expenses and to suppliers ........................... Net cash increase (decrease) due to operating activities ................................................................................

$

2012 9,000 (12,000)

2011 $ 9,000 (8,000)

2010 $ 8,000 (4,000)

$

(3,000)

$

$

1,000

4,000

c. Pendleton Enterprises Statement of Cash Flows from Operating Activities For the Years Ended December 31, 2010, 2011, and 2012 Cash collections from customers and sales ............................... Cash payments for expenses and to suppliers ........................... Net cash increase (decrease) due to operating activities ................................................................................

2012 $ 9,000 (12,000)

2011 $ 13,000 (11,000)

2010 $ 4,000 (1,000)

$

$

$

(3,000)

2,000

3,000

d. Managers can manipulate cash flows from operating activities by manipulating the timing of cash collections and cash payments associated with operating activities. By comparing parts (b) and (c) with part (a), it is obvious that these types of manipulations offset themselves in the next period. So if a manager wants to continue manipulating cash flows from operating activities, the manager will have to manipulate the timing of cash inflows and outflows every year.


P14–13 Watson and Holmes Detective Agency Statement of Cash Flows – Direct Method For the Year Ended December 31, 2012 Cash flows from operating activities: Cash collections from customers ............................................ Cash paid for inventory ........................................................... Cash paid for interest .............................................................. Cash paid for other expenses .................................................. Net cash increase (decrease) due to operating activities .......................................................................... Cash flows from investing activities: Purchase of long-lived assets .................................................. Net cash increase (decrease) due to investing activities .......................................................................... Cash flows from financing activities: Cash paid for dividends ........................................................... Proceeds from issuance of common stock .............................. Net cash increase (decrease) due to financing activities .......................................................................... Net increase in cash ..................................................................... Beginning cash balance, January 1, 2012 ..................................... Ending cash balance, December 31, 2012.................................... * $35,500

$ 34,500* (23,000) (2,800) (9,000) $

$

(300)

(1,000) (1,000)

$

(700) 6,000

$

5,300 4,000

6,000 $ 10,000

= $42,000 revenues – $5,000 increase in accounts receivable + $500 increase in allowance for doubtful accounts – $2,000 bad debt expense – $1,000 decrease in deferred revenues


P14–13 Concluded Watson and Holmes Detective Agency Statement of Cash Flows – Direct Method For the Year Ended December 31, 2012 Cash flows from operating activities: Net income ................................................................ Adjustments: Depreciation......................................................... Increase in allowance for doubtful accounts ....... Decrease in accounts payable .............................. Decrease in discount on note payable ................. Increase in accounts receivable ........................... Decrease in inventory .......................................... Decrease in deferred revenues ............................ Total adjustments ........................................... Net cash increase (decrease) due to operating activities.............................

$ 2,000 $

2,000 500 (1,000) 200 (5,000) 2,000 (1,000)

Cash flows from investing activities: Purchase of long-lived assets .................................... Net cash increase (decrease) due to investing activities ............................................ Cash flows from financing activities: Cash paid for dividends ............................................. Proceeds from issuance of common stock ................ Net cash increase (decrease) due to financing activities ............................................ Net increase in cash .......................................................

(2,300) $

(300)

$ (1,000) (1,000) $

(700) 6,000

$

Beginning cash balance, January 1, 2012 ....................... Ending cash balance, December 31, 2012......................

5,300 4,000

6,000 $ 10,000

P14–14 a.

1.

2.

3.

4.

Cash (+A) ........................................................................................ Common Stock (+SE) ............................................................... Issued common stock.

60,000

Inventory (+A) ................................................................................ Accounts Payable (+L) ............................................................. Purchased inventory on account.

20,000

Prepaid Rent (+A)........................................................................... Cash (–A) ................................................................................. Prepaid rent.

7,000

Furniture (+A) ................................................................................ Cash (–A) ................................................................................. Long-Term Note Payable (+L) ................................................. Purchased furniture.

30,000

60,000

20,000

7,000

20,000 10,000


P14–14 Continued 5a.

5b.

6.

7.

8.

9.

10.

11.

12.

Accounts Receivable (+A) .............................................................. Sales (R, +SE) ........................................................................... Made sales on account.

35,200

Cost of Goods Sold (E, –SE) ............................................................ Inventory (–A) ......................................................................... Recognized cost of inventory sold.

8,800

Accounts Payable (–L).................................................................... Cash (–A) ................................................................................. Made payment to suppliers.

10,000

Cash (+A) ........................................................................................ Accounts Receivable (–A) ....................................................... Collected cash from customers.

12,000

Miscellaneous Expenses (E, –SE) ................................................... Cash (–A) ................................................................................. Incurred and paid miscellaneous expenses.

10,000

Depreciation Expense (E, –SE) ....................................................... Accumulated Depreciation (–A).............................................. Depreciated furniture.

5,000

Interest Expense (E, –SE) ............................................................... Accrued Interest Payable (+L) ................................................. Incurred, but did not pay, interest.

1,000

Dividends (–SE) .............................................................................. Dividends Payable (+L) ............................................................ Declared dividends.

3,000

Rent Expense (E, –SE) .................................................................... Prepaid Rent (–A) .................................................................... Used portion of prepaid rent.

3,000

35,200

8,800

10,000

12,000

10,000

5,000

1,000

3,000

3,000

b. ISS, Inc. Income Statement For the Year Ended December 31, 2012 Sales .................................................................................................................. Cost of goods sold ............................................................................................ Depreciation expense ....................................................................................... Rent expense .................................................................................................... Interest expense ............................................................................................... Miscellaneous expenses ................................................................................... Net income .......................................................................................................

$ 35,200 (8,800) (5,000) (3,000) (1,000) (10,000) $ 7,400


P14–14 Continued ISS, Inc. Statement of Retained Earnings For the Year Ended December 31, 2012 Beginning retained earnings balance: January 1, 2012 .................................... Plus: Net income............................................................................................... Less: Dividends ................................................................................................. Ending retained earnings balance: December 31, 2012...................................

$

0 7,400 (3,000) $ 4,400

ISS, Inc. Balance Sheet December 31, 2012 Assets Cash .............................................. Accounts receivable ..................... Inventory ...................................... Prepaid rent ................................. Furniture ...................................... Accumulated depreciation ...........

$ 25,000 23,200 11,200 4,000 30,000 (5,000)

Total assets...................................

$ 88,400

Liabilities & Stockholders' Equity Accounts payable ...................... Accrued interest payable .......... Dividend payable....................... Long-term note payable............ Common stock .......................... Retained earnings ..................... Total liabilities and stockholders' equity ................

$ 10,000 1,000 3,000 10,000 60,000 4,400 $ 88,400

ISS, Inc. Statement of Cash Flows – Indirect Method For the Year Ended December 31, 2012 Cash flows from operating activities: Net income ..................................................................... Adjustments: Depreciation ............................................................ Increase in accounts receivable............................... Increase in inventory ............................................... Increase in prepaid rent .......................................... Increase in accounts payable .................................. Increase in accrued interest payable....................... Total adjustments ............................................. Net cash increase (decrease) due to operating activities ....................... Cash flows from investing activities: Purchase of furniture ..................................................... Net cash increase (decrease) due to investing activities ................................................ Cash flows from financing activities: Proceeds from issuance of common stock..................... Net cash increase (decrease) due to financing activities ............................................... Net increase (decrease) in cash ............................................ Beginning cash balance, January 1, 2012 .............................. Ending cash balance, December 31, 2012 ............................

$ $

7,400

5,000 (23,200) (11,200) (4,000) 10,000 1,000 (22,400) $ (15,000) $ (20,000) (20,000) $ 60,000 60,000 $ 25,000 0 $ 25,000


P14–14 Concluded c.

Working capital

Current ratio

= Current assets ÷ Current liabilities = $63,400 ÷ $14,000 = 4.53

d. Working capital

Current ratio

= Current assets – Current liabilities = ($25,000 + $23,200 + $11,200 + $4,000) – ($10,000 + $1,000 + $3,000) = $63,400 – $14,000 = $49,400

= ($63,400 – $10,000 cash) – ($14,000 – $10,000 accounts payable) = $53,400 – $4,000 = $49,400 = $53,400 ÷ $4,000 = 13.35

Net cash flow used by operating activities

= ($15,000) from part (b) + ($10,000) = ($25,000)

P14–15 a. Marketing revenue 2012 Ending accounts receivable $150,000 Cash collections

= 2012 Beginning accounts receivable + 2012 Marketing revenue – Cash collections during 2012 = $105,000 + $1,000,000 – Cash collections = $955,000

Salary expense Since no related balance sheet account exists as of December 31, 2011 or as of December 31, 2012, it is safe to assume that the entire balance in Salary Expense was paid in cash. Therefore, the cash paid for salaries equals $250,000. Supplies expense 2012 Ending office supply inventory

$75,000 Office supply purchases

= 2012 Beginning office supply inventory + Office supplies purchased during 2012 – 2012 Office supplies expense = $85,000 + Supplies purchased – $175,000 = $165,000

Depreciation expense Depreciation is the allocation of the cost of a fixed asset. Depreciation does not provide cash or use cash; hence, the cash flow associated with depreciation is zero. Insurance expense 2012 Ending prepaid insurance

= 2012 Beginning prepaid insurance + Insurance purchased during 2012 – 2012 Insurance expense


$50,000 Insurance purchases

= $10,000 + Insurance purchased – $60,000 = $100,000


P14–15 Concluded Rent Expense 2012 Ending rent payable $20,000 Rent paid

= 2012 Beginning rent payable + 2012 Rent expense – Rent paid during 2012 = $8,000 + $120,000 – Rent paid = $108,000

This method would be similar to directly computing net cash flow from operating activities since the actual cash flow for each component of operating activities is being computed. Under the indirect method, net cash flow from operating activities is computed by adjusting net income. b. Net income ................................................................................... Adjustments: Depreciation expense .............................................................. Decrease in office supplies inventory ..................................... Increase in rent payable .......................................................... Increase in accounts receivable .............................................. Increase in prepaid insurance ................................................. Total adjustments .............................................................. Net cash increase due to operating activities .............

$ 295,000 $ 100,000 10,000 12,000 (45,000) (40,000) 37,000 $ 332,000

This method is similar to the indirect method since net income is adjusted for the change in each operating account on the balance sheet to arrive at cash flows from operating activities. The magnitudes of these adjustments are the same as in part (a). However, in part (a) each of these adjustments was related to an individual operating activity to determine the actual cash inflow or actual cash outflow associated with the individual operating activity; whereas in part (b) each adjustment is related to net income to arrive at the overall net cash flow associated with all operating activities. c.

Operating Cash Flows Income Statement

Adjustments

Marketing revenue Salary expense Office supplies exp.

$ 1,000,000 (250,000) (175,000)

Depreciation exp. Insurance expense Rent expense

(100,000) (60,000) (120,000)

Net income

$

295,000

Increase in accts. receivable None Decrease in office supplies inventory Depreciation expense Increase in prepaid insurance Increase in rent payable Cash provided by operating activities

Direct Method $(45,000) 0

$

10,000 100,000 (40,000) 12,000

955,000 (250,000) (165,000) 0 (100,000) (108,000)

$

332,000


P14–16 Direct method Bower Manufacturing Industries Statement of Cash Flows For the Year Ended December 31, 2012 Cash flows from operating activities: Cash collections from sales and accounts receivable.............. Cash paid to suppliers for inventory ....................................... Cash paid for wages................................................................. Cash paid for supplies.............................................................. Cash paid for interest .............................................................. Net cash decrease due to operating activities ...................

$ 90,000a (90,000) (12,000) (5,000) (5,000)

Cash flows from investing activities: Proceeds from sale of fixed assets .......................................... Proceeds from sale of marketable securities .......................... Net cash increase due to investing activities .....................

$ 125,000b 51,000c

$

(22,000)

176,000

Net cash flow from financing activities ........................................ Net increase in cash .....................................................................

0 $ 154,000

Beginning cash balance, January 1, 2012 ..................................... Ending cash balance, December 31, 2012....................................

593,000 $ 747,000

a Cash collections include changes in both Accounts and Notes Receivable. b Proceeds from sale of fixed assets is computed as follows:

Cost of machinery and equipment sold ............................................... Less: Related accumulated depreciation.............................................. Book value of machinery and equipment sold ..................................... Less: Loss on sale of fixed assets (per income statement) ................... Proceeds from sale of fixed assets ....................................................... c Proceeds from sale of marketable securities is computed as follows: Decrease in marketable securities ....................................................... Less: Loss on sale of marketable securities (per income statement) ........................................................................................ Proceeds from sale of marketable securities .......................................

$ 150,000 15,000 $ 135,000 10,000 $ 125,000

$ 55,000 4,000 $ 51,000


P14–16 Concluded Indirect method Bower Manufacturing Industries Statement of Cash Flows For the Year Ended December 31, 2012 Cash flows from operating activities: Net income ..................................................................... Adjustments: Decrease in inventory .............................................. Depreciation ............................................................ Decrease in discount on bonds payable .................. Decrease in supplies inventory................................ Loss on sale of fixed assets ...................................... Loss on sale of marketable securities ...................... Increase in accounts receivable............................... Increase in notes receivable .................................... Decrease in accounts payable ................................. Total adjustments ............................................. Net cash decrease due to operating activities ................................................... Cash flows from investing activities: Proceeds from sale of fixed assets ................................. Proceeds from sale of marketable securities ................. Net cash increase due to investing activities ..........

$

37,000

$ 25,000 30,000 5,000 2,000 10,000 4,000 (50,000) (50,000) (35,000) (59,000) $

(22,000)

$ 125,000 51,000 176,000

Net cash flow from financing activities ................................. Net increase in cash ..............................................................

0 $ 154,000

Beginning cash balance, January 1, 2012 .............................. Ending cash balance, December 31, 2012 ............................

593,000 $ 747,000


P14–17 Direct method Price Restaurant Supply Company Statement of Cash Flows For the Year Ended December 31, 2012 Cash flows from operating activities: Cash collections from customers ............................................ Cash paid to suppliers for inventory ....................................... Cash paid for interest .............................................................. Net cash decrease due to operating activities ................... Cash flows from investing activities: Proceeds from sale of plant equipment .................................. Purchase of plant equipment .................................................. Net cash increase due to investing activities ..................... Cash flows from financing activities: Proceeds from common stock issue........................................ Payment of dividends .............................................................. Net cash increase due to financing activities ..................... Net increase in cash ..................................................................... Beginning cash balance, January 1, 2012 ..................................... Ending cash balance, December 31, 2012....................................

$ 165,000 (199,000) (13,000) $

$

(47,000)

90,000a (25,000)a 65,000

$

65,000b (30,000)c $

35,000 53,000

120,000 $ 173,000

a Explanation of activity involving plant equipment:

Ending plant equipment $275,000 Equipment purchased

= Beginning plant equipment + Equipment purchased – Equipment sold = $350,000 + Equipment purchased – $100,000 = $25,000

Proceeds from sale of equipment

= Book value of assets sold + Gain on the sale = [(Asset cost – Accumulated depreciation on asset sold) + Gain on the sale] = [($100,000 – $20,000) + $10,000] = $90,000

b Proceeds from issue of common stock

c

Dividends

= Increase in common stock + Increase in additional paid-in capital (common stock) = $35,000 + $30,000 = $65,000

= Beginning retained earnings + Net income – Ending retained earnings = $204,000 + $37,000 – $211,000 = $30,000


P14–17 Concluded Indirect method Price Restaurant Supply Company Statement of Cash Flows For the Year Ended December 31, 2012 Cash flows from operating activities: Net income ........................................................... Adjustments: Depreciation.................................................... Decrease in accounts receivable ..................... Decrease in prepaid insurance ........................ Increase in accounts payable .......................... Increase in inventory....................................... Decrease in premium on bonds payable ........ Gain on sale of plant equipment..................... Total adjustments ...................................... Net cash decrease due to operating activities .......................... Cash flows from investing activities: Proceeds from sale of plant equipment ............... Purchase of plant equipment ............................... Net cash increase due to investing activities ....................................... Cash flows from financing activities: Proceeds from common stock issue..................... Payment of dividends ........................................... Net cash increase due to financing activities ....................................................... Net increase in cash .................................................. Beginning cash balance, January 1, 2012 .................. Ending cash balance, December 31, 2012.................

$ $

37,000

12,000 5,000 10,000 1,000 (100,000) (2,000) (10,000) (84,000) $

$

(47,000)

90,000 (25,000) 65,000

$

65,000 (30,000)

$

35,000 53,000

$

120,000 173,000


P14–18 a. 2011 Cash collections from customers: Sales for cash .................................................................... Cash collections from accounts receivable ...................... Total cash collections from customers ...................... Cash payments for: Salary ................................................................................ Advertising........................................................................ Administrative expenses .................................................. Janitorial expense ............................................................. Supplies ............................................................................ Total cash paid for expenses ..................................... Net cash increase due to operating activities .......................

$ 3,633,500a 4,936,500b $ 8,570,000c $ 1,649,000d 710,000 832,000 120,000 232,000e (3,543,000) $ 5,027,000

a $3,633,500 = 2011 sales of $5,590,000  65% b Cash collections = Sales on account + Decrease in accounts receivable

= ($5,590,000  35%) + $2,980,000 = $4,936,500 c An alternative approach to calculating the $8,570,000 is to add the decrease in Accounts Receivable during 2011 of $2,980,000 to the 2011 sales of $5,590,000. d $1,649,000 = Salary expense of $1,794,000 – Salary payable of $145,000 e $232,000 = Supplies expense of $299,000 – Supply inventory payable of $67,000 2012 Cash collections from customers: Sales for cash ......................................................................... Cash collections from accounts receivable ........................... Total cash collections from customers ........................... Cash payments for: Salary ..................................................................................... Advertising............................................................................. Administrative expenses ....................................................... Janitorial expense .................................................................. Supplies ................................................................................. Total cash paid for expenses .......................................... Net cash increase due to operating activities ............................

$ 1,491,750a 2,803,500b $

4,295,250c

$

(4,228,000) 67,250

$ 2,145,000d 705,000e 898,000 132,000 348,000f

= 2012 sales of $5,967,000  25% = Sales on account – Increase in accounts receivable = ($5,967,000  75%) – $1,671,750 c This amount can also be calculated as 2012 sales of $5,967,000 less the increase in Accounts Receivable of $1,671,750 during 2012. d $2,145,000 = Salary expense of $2,025,000 – Ending salary payable of $25,000 + Beginning salary payable of $145,000 e $705,000 = Advertising expense of $755,000 – Advertising payable of $50,000 a $1,491,750 b $2,803,500


f

$348,000

= Supplies expense of $281,000 + Beginning supply inventory payable of $67,000

P14–18 Concluded b. The first thing that must be explained to the stockholders is the nature of dividends. Dividends are paid out of assets, not out of net income. If a company has insufficient assets or has alternative uses for its assets, it will be unable to declare a cash dividend. Although net income is a measure of the net assets that have flowed into the company during the year from operations, these net assets may be in a form, such as inventory or accounts receivable, that cannot be easily distributed to stockholders. Thus, net income provides only a low-quality indication of potential dividends. A better indication of a company's potential ability to declare and pay cash dividends is the net cash flows from operating activities. In this particular case, 2011 was a good year for the Lynch Engineering Firm. The company generated $597,000 in net income and $5,027,000 in cash flows from operating activities [see part (a)]. The large cash flows are due primarily to the collection of receivables that were outstanding at the beginning of the year and were not associated at all with 2011 net income. 2012 was also a good year with respect to net income; the company generated income of $638,000. However, the company realized only a net cash flow from operating activities of $67,250. The low cash flows, in comparison to 2011's net cash flows, are due to (1) a larger proportion of sales being on account in 2012 and (2) customers not paying their accounts promptly. As a board member, you felt that the 2012 cash flows were insufficient to both fund future operations and pay a dividend.

P14–19 a. Original entries 1a. Cash (+A) ...................................................................................... Common Stock (+SE) ............................................................. Additional Paid-In Capital, Common Stock (+SE) .................. Issued common stock. 1b.

2.

3.

750,000 750,000

Cash (+A) ...................................................................................... Preferred Stock (+SE) ............................................................ Additional Paid-In Capital, Preferred Stock (+SE) ................. Issued preferred stock.

102,000

Fixed Assets (+A) .......................................................................... Cash (–A) ............................................................................... Purchased fixed assets.

750,000

Cash (+A) ...................................................................................... Bonds Payable (+L) ................................................................ Premium on Bonds Payable (+L) ........................................... Issued bonds.

29,200a

a b

4.

1,500,000

100,000 2,000

750,000

20,000b 9,200

$29,200 = 20 bonds  $1,000 face value per bond  146% $20,000 = 20 bonds  $1,000 face value per bond

Land (+A) ...................................................................................... Common Stock (+SE) .............................................................

40,000 15,000


Additional Paid-In Capital, Common Stock (+SE) .................. Purchased land in exchange for common stock.

25,000


P14–19 Continued 5a.

5b.

6a.

6b.

7.

8.

9a.

9b.

9c.

10.

11.

12.

Inventory (+A) .............................................................................. Accounts Payable (+L) ........................................................... Purchased inventory on account.

2,000,000

Accounts Payable (–L) .................................................................. Cash (–A) ............................................................................... Made payment to suppliers.

1,075,000

Cash (+A) ...................................................................................... Sales (R, +SE) ......................................................................... Made sales.

2,050,000

Cost of Goods Sold (E, –SE) .......................................................... Inventory (–A) ....................................................................... Recognized cost of inventory sold.

875,000

Prepaid Insurance (+A) ................................................................ Cash (–A) ............................................................................... Purchased two-year insurance policy.

80,000

Marketable Securities (+A) .......................................................... Cash (–A) ............................................................................... Purchased marketable securities.

250,000

Accounts Receivable (+A) ............................................................ Sales (R, +SE) ......................................................................... Made sales on account.

880,000

Cost of Goods Sold (E, –SE) .......................................................... Inventory (–A) ....................................................................... Recognized cost of inventory sold.

490,000

Cash (+A) ...................................................................................... Accounts Receivable (–A)...................................................... Collected cash from customers.

500,000

Miscellaneous Expenses (E, –SE) ................................................. Cash (–A) ............................................................................... Incurred and paid miscellaneous expenses.

500,000

Dividends (–SE) ............................................................................ Dividends Payable (+L) .......................................................... Declared dividends.

100,000

Interest Expense (E, –SE) ............................................................. Premium on Bonds Payable (–L) .................................................. Cash (–A) ............................................................................... Incurred and paid interest.

1,460a 140

_____________________

a b

2,000,000

1,075,000

2,050,000

875,000

80,000

250,000

880,000

490,000

500,000

500,000

100,000

$1,460 = Book value of $29,200  Effective rate per six-month period of 5% $1,600 = Face value of $20,000  Stated rate per six-month period of 8%

1,600b


P14–19 Continued Adjusting entries (a) Depreciation Expense (E, –SE) ..................................................... Accumulated Depreciation (–A) ............................................ Depreciated fixed assets. * (b)

(c)

(d)

(e)

140,000

$140,000 = ($750,000 Cost – $50,000 Salvage value) ÷ 5 year useful life

Insurance Expense (E, –SE) .......................................................... Prepaid Insurance (–A).......................................................... Expiration of a portion of insurance policy.

20,000

Unrealized Loss on Marketable Securities (Lo, –SE) .................... Allowance for Unrealized Loss on Marketable Securities (–A) ................................................... Adjusted short-term portfolio to LCM.

25,000

Miscellaneous Expenses (E, –SE) ................................................. Miscellaneous Payables (+L) ................................................. Incurred, but did not pay, miscellaneous expenses.

75,000

Bad Debt Expense (E, –SE) ........................................................... Allowance for Doubtful Accounts (–A) ................................. Estimated bad debts.

70,400*

* (f)

140,000*

20,000

25,000

75,000

70,400

$70,400 = Credit sales of $880,000  Uncollectible percentage of 8%

Loss on Inventory Write-down (Lo, –SE)...................................... Inventory (–A) ....................................................................... Adjusted inventory to LCM.

Cash B.B. (1a) (1b) (3) (6a) (9c)

0 1,500,000 102,000 29,200 2,050,000 500,000

E.B.

1,524,600

Marketable Securities

(2) (5b) (7) (8) (10) (12)

750,000 1,075,000 80,000 250,000 500,000 1,600

Accounts Receivable B.B. (9a)

0 880,000

E.B.

380,000

(9c)

5,000

Allow. for Unr. Loss on M. S.

B.B. (8)

0 250,000

B.B. (c)

0 25,000

E.B.

250,000

E.B.

25,000

Allowance for D. A.

500,000

5,000

Inventory

B.B. (e)

0 70,400

B.B. (5a)

0 2,000,000

E.B.

70,400

E.B.

630,000

(6b) (9b) (f)

875,000 490,000 5,000


P14–19 Continued Prepaid Insurance B.B. (7)

Land

0 80,000 (b)

E.B.

B.B. (4)

0 40,000

B.B. (2)

0 750,000

E.B.

40,000

E.B.

750,000

20,000

60,000

Accumulated Depreciation

Accounts Payable

B.B. (a)

0 140,000

E.B.

140,000

(5b)

Dividends Payable

E.B.

925,000

E.B.

75,000

Premium on Bonds Pay.

E.B.

100,000

E.B.

20,000

(12)

APIC: Preferred Stock

140

B.B. (3)

0 9,200

E.B.

9,060

Common Stock

B.B. (1b)

0 100,000

B.B. (1b)

0 2,000

B.B. (1a) (4)

0 750,000 15,000

E.B.

100,000

E.B.

2,000

E.B.

765,000

Retained Earnings

B.B. (1a) (4)

0 750,000 25,000

E.B.

775,000

(c4)

100,000

B.B. (c3)

E.B.

0 2,050,000 880,000

B.B. (6b) (9b)

0

E.B.

628,140

B.B. (11)

E.B.

B.B. (e)

0 875,000 490,000

B.B. (10) (d)

(c2) E.B.

0

E.B.

Depreciation Expense

0

B.B. (f)

E.B.

0

20,000

0

Unrealized loss on Mkt. Sec. B.B. (c)

(c2)

0 20,000 (c2)

E.B.

0 5,000

140,000

0

70,400

Loss on Inv. Write-down

0 140,000

575,000

Insurance Expense B.B. (b)

(c2)

0 500,000 75,000

1,365,000

0 70,400

1,460

100,000

0

Bad Debt Expense

0 1,460

0 100,000

Miscellaneous Expenses

0

Interest Expense

(c2)

0 728,140

Cost of Goods Sold

(c2)

(c2)

Dividends

(c4)

2,930,000

0

0 75,000

0 20,000

E.B.

E.B.

B.B. (d)

B.B. (3)

B.B. (6a) (9a)

B.B. (a)

0 2,000,000

0 100,000

Sales

E.B.

B.B. (5a)

B.B. (11)

APIC: Common Stock

B.B. (12)

1,075,000

Miscellaneous Payables

Bonds Payable

Preferred Stock

(c1)

Fixed Assets

0 25,000

5,000

(c2) E.B.

0

25,000


P14–19 Continued b. Closing entries (c1) Sales ............................................................................................. Income Summary .................................................................. Closed revenues into Income Summary. (c2)

(c3)

(c4)

2,930,000 2,930,000

Income Summary ......................................................................... Miscellaneous Expenses ....................................................... Cost of Goods Sold ................................................................ Interest Expense ................................................................... Bad Debt Expense ................................................................. Depreciation Expense ........................................................... Insurance Expense ................................................................ Loss on Inventory Write-down ............................................. Unrealized Loss on Marketable Securities ............................ Closed expenses into Income Summary.

2,201,860

Income Summary ......................................................................... Retained Earnings ................................................................. Closed Income Summary into Retained Earnings.

728,140

Retained Earnings ........................................................................ Dividends .............................................................................. Closed Dividends into Retained Earnings.

100,000

575,000 1,365,000 1,460 70,400 140,000 20,000 5,000 25,000

728,140

100,000

c. Mick’s Photographic Equipment Income Statement For the Year Ended December 31, 2011 Sales revenue .......................................................... Cost of goods sold ................................................... Gross profit ............................................................. Operating expenses: Depreciation expense....................................... Bad debt expense ............................................. Insurance expense............................................ Miscellaneous expenses ................................... Total operating expenses .......................... Income from operations ......................................... Interest expense...................................................... Loss on write-down of inventory ............................ Unrealized loss on marketable securities ............... Net income ..............................................................

$

2,930,000 1,365,000 $ 1,565,000

$

140,000 70,400 20,000 575,000 $

$

805,400 759,600 (1,460) (5,000) (25,000) 728,140


P14–19 Continued Mick’s Photographic Equipment Balance Sheet December 31, 2011 Assets Current assets: Cash .............................................................................................. Marketable securities (net of allowance for unrealized losses of $25,000) ................................................ Accounts receivable (net of allowance for doubtful accounts of $70,400) ............................................................. Inventory ...................................................................................... Prepaid insurance ......................................................................... Total current assets ............................................................... Land…. ............................................................................................... Fixed assets (net of accumulated depreciation of $140,000) ............. Total assets.......................................................................................... Liabilities and Stockholders' Equity Current liabilities: Accounts payable ......................................................................... Miscellaneous payables ............................................................... Dividends payable ........................................................................ Total current liabilities........................................................... Bonds payable (including associated premium of $9,060) ................. Stockholders' equity: Preferred stock ............................................................................. Common stock.............................................................................. Additional paid-in capital: Preferred stock ...................................................................... Common stock ....................................................................... Retained earnings......................................................................... Total stockholders' equity ..................................................... Total liabilities and stockholders' equity .............................................

55

$ 1,524,600 225,000 309,600 630,000 60,000 $ 2,749,200 40,000 610,000 $ 3,399,200

$

925,000 75,000 100,000 $

$

1,100,000 29,060

100,000 765,000 2,000 775,000 628,140 $

2,270,140 3,399,200


P14–19 Continued d. Direct method Mick’s Photographic Equipment Statement of Cash Flows For the Year Ended December 31, 2011 Cash flows from operating activities: Cash collections from customers ........................................ Cash paid for inventory ....................................................... Cash paid for insurance ....................................................... Cash paid for interest .......................................................... Cash paid for miscellaneous expenses ................................ Net cash increase (decrease) due to operating activities ...................................................................... Cash flows from investing activities: Purchase of marketable securities ...................................... Purchase of fixed assets ...................................................... Net cash increase (decrease) due to investing activities ...................................................... Cash flows from financing activities: Proceeds from bond issue ................................................... Proceeds from preferred stock issue .................................. Proceeds from common stock issue.................................... Net cash increase (decrease) due to financing activities ...................................................................... Net increase in cash ................................................................. Beginning cash balance, January 1, 2011 ................................. Ending cash balance, December 31, 2011................................

$

2,550,000 (1,075,000) (80,000) (1,600) (500,000) $

$

893,400

(250,000) (750,000) (1,000,000)

$

29,200 102,000 1,500,000

$

1,631,200 1,524,600

$

0 1,524,600


P14–19 Concluded Indirect method Mick’s Photographic Equipment Statement of Cash Flows For the Year Ended December 31, 2011 Cash flows from operating activities: Net income ..................................................................... Adjustments: Increase in accounts receivable ................................ Increase in inventory ................................................. Increase in prepaid insurance ................................... Decrease in premium on bonds payable ................... Increase in accounts payable .................................... Increase in miscellaneous payables .......................... Depreciation expense ................................................ Bad debt expense ...................................................... Unrealized loss on mkt. securities ............................. Total adjustments ................................................ Net cash increase (decrease) due to operating activities .................................... Cash flows from investing activities: Purchase of marketable securities ................................. Purchase of fixed assets ................................................. Net cash increase (decrease) due to investing activities ................................................. Cash flows from financing activities: Proceeds from bond issue .............................................. Proceeds from preferred stock issue ............................. Proceeds from common stock issue .............................. Net cash increase (decrease) due to financing activities ................................................. Net increase in cash .............................................................. Beginning cash balance, January 1, 2011 .............................. Ending cash balance, December 31, 2011 ............................

$ 728,140 $ (380,000) (630,000) (60,000) (140) 925,000 75,000 140,000 70,400 25,000 165,260 $

893,400

$ (250,000) (750,000) (1,000,000)

$

29,200 102,000 1,500,000

$

1,631,200 1,524,600

$

0 1,524,600


ISSUES FOR DISCUSSION ID14–1 a. Quality of earnings and earnings persistence have to do with the sustainability of the earnings as well as nearness of the earnings to the cash generated by operating activities. The indirect method of preparing the statement of cash flows is especially helpful in spotting the noncash adjustments to net income. This information helps investors identify the companies that use aggressive accounting choices, since the magnitude of such adjustments, in general, would be higher in these cases. b. In the case of Mattel, the changes in accounts receivable were certainly positive, which indicated that the majority of the sales were on credit with no contributions to cash. The change in the inventory could have been attributed to either: 1. matching the old production and currents sales or 2. purchasing new inventory on credit. Similarly, an increase in deferred taxes and translation gains would indicate no related cash effect except an adjustment to earnings due to (i.e., changes in tax rates) and foreign currency exchange rates. c.

Wall Street firms employ various models to identify over- and under-valued stocks. The choice of accounting policies by a firm is just one variable in the whole equation. Therefore, the choice of accounting policy may have some explanatory power, but alone it would not provide sufficient information to differentiate among the stocks.

ID14–2 a. The Washington Post will use the equity method to record the accounting transactions related to the other companies because it holds “significant influence” over those companies. The equity method means that the Washington Post will record its ownership percentage times the companies’ net income as equity income on the Washington Post’s income statement; if the other companies lose money, the Washington Post will show a net loss from equity investments on its income statement (its ownership percentage times the companies’ net losses). If the companies pay dividends, then the Washington Post would record this amount as an increase in cash and a reduction to the investment account. b. Losses in these investments are shown as losses on the income statement of the Washington Post. However, these are non-cash losses, so these losses are added back to the net income of the Washington Post to determine the cash changes for the year. (The treatment is similar to that of depreciation expense, another non-cash deduction to earnings.) c.

“Net of distributions” means the amount of income or loss that did not involve the exchange of cash between the Washington Post and the investment companies. The companies may have paid cash dividends to their owners, including the Washington Post, so these cash payments are deducted from the non-cash losses that are added back to net income to determine the Post’s operating cash flow.

d. The transactions that led to these disclosures were 1) the sale of fixed assets previously owned by the Washington Post at a loss (proceeds less than book value) and 2) the lowering of carrying value of fixed assets to a new (impaired) market value. These amounts would also appear on the income statement. The losses are added to net income in the calculation of cash from operations because the effect, if any, on cash is an investment activity (long term assets) and should therefore be shown in a different section of the statement of cash flows.


ID14–3 a. The capitalization and matching process is not very useful for assessing the cash available to a company because disbursing cash and consuming an item are two very different concepts. In order to measure performance, it is necessary to match the costs that were incurred to generate benefits against the benefits. Under accrual accounting, costs should be capitalized if they are expected to provide benefits in the future. Thus, when a company disburses cash to acquire an item expected to provide a future benefit, the company should capitalize the cost of the item as an asset. As the company uses the item over time to help generate revenue, the item will be expensed and matched against the revenue it helped generate. Thus, through the process of capitalizing costs and then expensing them when the cost helps generate revenue, revenues and expenses are closely matched. In this manner, the capitalization and matching process make it difficult to assess a company’s cash performance. b. Solvency refers to a company’s ability to pay its obligations as they come due, and earning power refers to a company’s ability to generate net assets through operating activities. The two concepts are closely related in that a company will not remain solvent if it is unable to generate net assets through operations. That is, a company cannot stay in business indefinitely if its operations fail to generate sufficient net assets to allow the company to pay back debt or pay dividends. Similarly, a company will not have very good earning power if it is not solvent. If a company is having solvency problems, it will have to divert cash away from operations and toward paying creditors. Information that is useful for assessing a company’s solvency can be found in both the balance sheet and statement of cash flows. The balance sheet lists the company’s obligations and the assets currently available to the company to use to pay off the liabilities. In addition, the balance sheet indicates which liabilities are expected to mature in the near future and which assets are more liquid. The statement of cash flows provides information about where the company is getting its cash and how it is using it. Of particular interest is the net cash generated or used by operating activities. The income statement is the primary source to assess a company’s earning power. Net income represents the net assets the company generated during the year from operations. By comparing net income to certain balance sheet amounts, such as average stockholders’ equity or average total assets, one can assess how effectively a company is using its assets to generate returns. c.

Bankruptcies usually imply that a company was unable to pay its debts. Because cash is the most common medium of exchange in the United States, creditors expect to receive interest and principal payments in cash. Thus, a wave of bankruptcies implies that companies were having cash flow problems, which, in turn, increases interest in assessing nonbankrupt companies’ cash flows to avoid other problem loans.

ID14–4


a. SuperValu paid out cash when it acquired fixed and intangible assets. Depreciation and amortization of these assets simply represent the allocation of the assets' cost to particular accounting periods. There is no cash outflow or inflow associated with depreciation and amortization. SuperValu is adding depreciation and amortization back to net income to arrive at net cash flow from operating activities because it uses the indirect method to report cash flows from operating activities. With the indirect method, a company begins with net income and then reconciles (i.e., explains) why net income is different from cash flows from operating activities. Since depreciation and amortization reduced SuperValu's net income but did not have any effect on cash, SuperValu needs to add depreciation and amortization back to net income to adjust net income to net cash flow from operating activities. b. Impairment charges are recorded when the company determines that the carrying value of an asset is greater than that asset’s fair value. If an asset has changed from the time of acquisition, or if business conditions have changed the effectiveness of that asset, the company will determine that the asset is overvalued. The charge is added back to net income for the same reason SuperValu added back depreciation and amortization. That is, recording the impairment expense reduces net income but has no effect on cash. c.

The “gain on the sale of assets” is subtracted from net earnings and “loss on sale” is added back because both of these items are non-recurring in nature and so the intent is to eliminate these from the calculation of cash from operating activities. (The cash flow related to the sale of assets belongs in the Investing section.) To reverse these out from net income, they have to be offset by the opposite of how they affect net income on the income statement. Therefore losses are added back and gains are subtracted away.

d. Receivables have been a consistent source of cash for SuperValu. The change in accounts receivable has been a decline each year, which is freeing up the company’s cash. Less receivables (as compared to the prior year) indicate more cash flowing into the company. Inventory, on the other hand, grew in 2008 and 2009, using up company cash balances. Of greater impact on the company’s cash position has been the reduction of accounts payables, draining cash from SuperValu. e. One of the first tipoffs that there may be some problems with the quality of earnings is if cash provided by operating activities is lower than net income. This is not the case with SuperValu and so there is no indication that earnings are being managed by the timing of transactions. SuperValu has shown significantly higher cash from operations than net income in all three years. Even if the large impairment charge in 2009 is adjusted out, earnings would have been less than cash from operations.

ID14–5 Shortcuts to “cash flow”, such as adding back depreciation and amortization (non-cash expenses) to net income, do not take into consideration all of the changes that affect cash. The indirect statement of cash flow starts by adding back these non-cash expenses, but the statement continues with the cash flow adjustments. Changes to working accounts are also included in the calculation to come up with the operating cash flow number. The most prominent example of the shortcoming of simply adding depreciation expense to earnings to calculate cash flow is the case of slowing receivables. If a company is growing rapidly because of additional credit sales, net income should also grow (holding all other factors

ID14–5 Concluded


constant). However, if those credit sales were made to companies without the ability or willingness to pay, net income will overstate the company’s ultimate cash position. As receivables become stagnant and build up on the balance sheet, the statement of cash flow would deduct this increase from net income to arrive at the cash flow number. The shortcut of simply adding back depreciation to net income will not reflect the problem with collections. Not including the changes to the current accounts (such as accounts receivable) runs the risk that the reader of the financial statements will not get a true picture of the company’s cash position.

ID14–6 a. The annual depreciation expense is decreasing, which would imply that the company has less long lived assets that require annual cost allocation. The company appears to not be investing in its fixed asset base, with long term assets becoming fully depreciated and not being replaced with new assets. b. The issuance of stock to employees as a form of compensation does not require the outlay of cash. There is a theoretical cost to the company in the form of the opportunity cost of not being able to sell those shares on the open market, but the granting of shares does not directly cost the company cash. Therefore, the expense associated with issuing those shares is a non-cash charge, and similar to depreciation expense, is added back to net income in the calculation of changes in cash balances. c. Inventory increased substantially in 2006, drainiing the company of $49 million of cash. In 2007 and 2008, however, inventory levels declined slightly with the lower investment in this operating account freeing up $7.8 million and $0.2 million of cash. d. The asset impairment charges are non-cash expenses (debit expense, credit asset to lower the carrying value of the asset and decreases earnings/equity) and, similar to depreciation expense, are added back to net income in the calculation of changes in cash balances. The sale of the subsidiary in 2006 resulted in a gain, which increased net income; however, the cash inflow from the sale is dealt with in the investing section of the statement of cash flows and the gain, which increased net income, needs to be backed out of the calculation of operating cash flows.

ID14–7 The statement of cash flows for Danone lists the company’s various sources of net income with an adjustment for non-cash charges, such as depreciation expense (similar to the setup in U.S. GAAP for the indirect format). Then the statement deals with changes in operating accounts that affect operating cash flow, again in a similar approach to U.S. GAAP. After totaling operating cash flow (from income, non-cash expenses and other changes, and working capital adjustments), Danone lists its investing activities (mainly the purchase and sale of long-term assets). Finally, sources and uses of debt and equity capital are totaled in the financing section. Of note, the company lists changes to marketable securities (short term, operating investments) in its financing section. The changes in cash from the three areas are tabulated to show the overall increase/decrease to company cash balances from the previous year. Company cash management practices differed greatly from 2007 to 2008. In 2007 the company acquired other businesses, using 12 billion euros; this significant investment was funded with operating surpluses and, more importantly, with current and long term borrowings (“financial liabilities”). In 2008, however, investing activities were significantly lower, allowing the company to use some of its excess cash flow to pay down liabilities and increase dividend payments to shareholders.

ID14–8


a. Over the three-year time period Eli Lilly has seen a change to its cash management profile. In 2006 the company augmented its strong operating cash flows with cash inflows from the sale of long term assets, allowing it repay debt and return cash to shareholders. But in 2007 and 2008, the company became a significant purchaser of long term assets, using $4.3 and $7.3 billion dollars of cash in its investing activities. The large investments reduced the company’s ability in 2007 to return cash to shareholders and actually required the company to turn to financing sources in 2008 due to the large investments. b. More than likely the significant change in investing cash means that the company has been acquiring new businesses and expanding existing businesses, all in an effort to build for the future. Cash from operations has been strong (even with the loss shown on the 2008 income statement), but the company must see that its pipeline of pharmaceutical products needs to be enhanced so that future operations are as cash lucrative as have been past operations. The significant investment in 2008 required the company to generate cash from financing sources, a distinct change from previous years.

ID14–9 a. Starbucks has been incredibly consistent in its generation of cash from operating activities over the time period. It has used this cash to open additional stores (cash from investing activities) and has still had excess balances allowing it to repay debt and return cash to shareholders. b. The heavy use of cash for investing signals this profile as one of a growing company. Starbucks differs from many other growth companies in that it does not have to look to financing sources (debt and/or equity) to fund the investments; the operations are strong enough to provide the funding for the growth. As discussed below, the rate of growth will be an issue in the future; in fact, cash used for investing is significantly lower in 2009 than it was in previous years. c.

At this point the company is quite strong. Fast growth can often hurt companies, but Starbucks is not leveraging up its balance sheet or diluting its existing owners to fund the growth. The consistent cash flow from operations given the economic climate of 2007-2009 is quite impressive.

d. At some point, the company’s growth will slow, meaning that the use of cash for investing will not be as much a drag for the company. If operations remain strong, the company might divert the cash (that in other years would have gone to new stores) to shareholders in the form of larger dividends or increased share repurchases. On the negative side, if the operations face increased competition (from competitors such as McDonald’s, from new coffee chains or from substitution products if the coffee fad fades), cash flow could decrease and limit the company’s ability to grow and return cash to its financiers.

ID14–10


a. Nike’s major sources of cash are cash from operations. This cash is primarily used to fund financing activities as the company continues to repurchase shares and pay dividends to stockholders; cash is also used to fund additions to property, plant and equipment. b. Depreciation and amortization expenses, stock-based compensation expenses and an impairment charge (to lower the carrying value of Goodwill due to the Umbro acquisition) appear to be the largest transactions that were non-cash in nature. c.

The gain on divestitures in 2008 is shown as a negative number in the operating section because the sale of long term assets is an investing activity and the associated cash inflow is accounted for in the investing section (see “Proceeds from divestitures”). The gain increased profits and therefore needs to be backed out in the calculation of operating cash flow.

d. In each of the three years shown, accounts receivable have grown, acting as a drain (a “use”) on company cash balances. The growth in receivables can indicate that some of the company’s sales growth (14% in 2008, 3% in 2009) is coming in the form of receivables that might not be collected, or from receivables that will be very slow to be collected. It is possible that the company’s top line in sales contains receivables that will not convert in the near future to cash collections. e. Over the three-year period, Nike’s strong operating cash flow has been able to fund the company’s capital spending (additions to property, plant & equipment) and fund a very strong stock buyback program, as well as a growing dividend. From the perspective of a Nike shareholder, the company’s strong operations have been able to fund the future asset base of the company (which will keep operations strong) while returning significant funds to the owners.


Appendix A The Time Value of Money

MULTIPLE CHOICE QUESTIONS 1.

Flores Company borrowed $10,000 at 10% interest for 5 years. Which statement is true? a. Regardless of when the note is repaid, total interest over the loan period is the same. b. Interest expense is the same regardless of the compounding periods. c. The amount to be repaid is the same regardless of whether the principal is repaid $2,000 per year, or as a sum at the end of five years. d. Interest is less if simple interest is used than if compound interest is used. Ans: D BT: AP Difficulty: Moderate TOT: 1 min. AACSB: Analytic AICPA BB: Critical Thinking AICPA FN: Measurement

2.

How much is interest revenue for 90 days on an 8%, 180-day note receivable with a face value of $25,000? a. $1,800 b. $500 c. $300 d. $400 Ans: B BT: AN Difficulty: Easy TOT: 1 min. AACSB: Analytic AICPA BB: Critical Thinking AICPA FN: Measurement

3.

How much is interest revenue for 30 days on an 8%, 90-day note receivable with a face value of $6,000? a. $40 b. $50 c. $90 d. $60 Ans: A BT: AN Difficulty: Easy TOT: 1 min. AACSB: Analytic AICPA BB: Critical Thinking AICPA FN: Measurement

A-1


A-2

Test Bank – Appendix - The Time Value of Money

4.

The following computation took place: $20,000 divided by the future value of a 12-year, 4% ordinary annuity What question will this computation answer? a. How much must be invested now so that equal payments can be withdrawn at the end of each year for 12 years? b. How much must be invested now so that $20,000 is accumulated by the end of the 12th year? c. How much will be available at the end of the12th year if a payment of $20,000 is deposted now? d. How much must be deposited at the end of every year so that $20,000 is available at the end of 12 years? Ans: D BT: AP Difficulty: Easy TOT: 1 min. AACSB: Analytic AICPA BB: Critical Thinking AICPA FN: Measurement

5.

Interest is compounded annually. What is the total amount of interest on a $7,000 note payable at the end of five years at 8%? a. $3,000 b. $3,285 c. $7,000 d. $3,791

Ans: B BT: AN Difficulty: Moderate TOT: 2 min. AACSB: Analytic AICPA BB: Critical Thinking AICPA FN: Measurement

6.

Which timing of payments is true for an ordinary annuity? a. All payments occur at the beginning of the first year. b. Payments begin immediately and occur once per year on the last day of each year. c. Payments occur at the end of each period. d. Payments occur at the beginning of each period. Ans: C BT: C Difficulty: Easy TOT: 1 min. AACSB: Analytic AICPA BB: Critical Thinking AICPA FN: Measurement

7.

Interest is compounded quarterly on a $10,000 note payable for 1 year at 12%. How much is total interest on the note? a. $1,338 b. $1,286 c. $1,255 d. $1,506 Ans: C BT: AN Difficulty: Moderate TOT: 2 min. AACSB: Analytic AICPA BB: Critical Thinking AICPA FN: Measurement


Test Bank – Appendix - The Time Value of Money

8.

A-3

You need to calculate the present value of an amount at 10% compounded quarterly for 2 years. What interest factor will you use? a. 10% for 4 periods b. 2.5% for 8 periods c. 20% for 2 periods d. 10% for 8 periods Ans: B BT: AP Difficulty: Easy TOT: 1 min. AACSB: Analytic AICPA BB: Critical Thinking AICPA FN: Measurement

9.

Sierra Capital wants to accumulate $100,000 at the end of 10 years to fund retirement benefits for its accountant. Annual deposits will be made into a special account earning 6%, beginning at the end of year 1. To calculate the amount of the equal deposits, use the a. future value of a annuity due. b. present value of a single amount. c. future value of an ordinary annuity. d. present value of an annuity. Ans: C BT: AP Difficulty: Easy TOT: 1 min. AACSB: Analytic AICPA BB: Critical Thinking AICPA FN: Measurement

10.

Malcom Corp. will deposit $10,000 annually at the end of each year for five years. Malcom will earn 6%. How much will be accumulated at the end of the 5 years? a. $65,321 b. $70,399 c. $50,000 d. $56,371 Ans: D BT: AN Difficulty: Moderate TOT: 2 min. AACSB: Analytic AICPA BB: Critical Thinking AICPA FN: Measurement

11.

Calculate the future value of equal semiannual payments of $9,000 at 12% compounded semiannually for 4 years. The answer is a. $43,014. b. $55,888. c. $89,077. d. $114,757. Ans: C BT: AN Difficulty: Moderate TOT: 2 min. AACSB: Analytic AICPA BB: Critical Thinking AICPA FN: Measurement

12.

Present value is a. how much today’s money will be worth in the future. b. the amount of money that must be invested now to produce a known future value. c. always larger than the future value. d. the total cost of interest over several years. Ans: B BT: C Difficulty: Easy TOT: 1 min. AACSB: Analytic AICPA BB: Critical Thinking AICPA FN: Measurement


A-4

Test Bank – Appendix - The Time Value of Money

13.

How much would you deposit today in a savings account that earns 10%, in order that you can make equal annual withdrawals of $1,200 each at the end of each of the next 15 years? a. $5,013 b. $9,127 c. $19,800 d. $18,000 e. $38,127 Ans: B BT: AN Difficulty: Moderate TOT: 2 min. AACSB: Analytic AICPA BB: Critical Thinking AICPA FN: Measurement

14.

Miracle Corporation wants to withdraw $60,000 from a savings account at the end of each year for ten years beginning one year from now. The savings earns 10% and is compounded annually. Which one of the following reflects the correct procedure to determine the required initial investment at the beginning of the first year? a. $60,000 times the present value of a 10-year, 10% ordinary annuity. b. $60,000 divided by the future value of a 10-year, 10% ordinary annuity. c. $60,000 times the future value of a 10-year, 10% ordinary annuity. d. $6,000 divided by the present value of a 10-year, 10% ordinary annuity. Ans: A BT: AP Difficulty: Moderate TOT: 1 min. AACSB: Analytic AICPA BB: Critical Thinking AICPA FN: Measurement

15.

An annuity due and an ordinary annuity have equal payments, the same interest rates, and the amount of time between the payments is equal. Which statement is true? a. The present value of the annuity due is less than the present value of the ordinary annuity. b. The future value of the annuity due is less than the future value of the ordinary annuity. c. The future value of the annuity due is equal to the future value of the ordinary annuity. d. The present value of the annuity due is greater than the present value of the ordinary annuity. Ans: D BT: AP Difficulty: Moderate TOT: 1 min. AACSB: Analytic AICPA BB: Critical Thinking AICPA FN: Measurement

16.

An amount is deposited for five years at 6% and is compounded semi-annually. Which interest rate and periods will be used to determine the present value? a. 8% for 5 periods b. 3% for 10 periods c. 3% for 2.5 periods d. 8% for 10 periods Ans: B BT: AP Difficulty: Easy TOT: 1 min. AACSB: Analytic AICPA BB: Critical Thinking AICPA FN: Measurement


Test Bank – Appendix - The Time Value of Money

17.

A-5

To determine how much must be deposited in the bank today so that you can withdraw 6 annual payments beginning one year from now, which interest factor will you need? a. Future value of an ordinary annuity of 1 b. Future value of an annuity due of 1 c. Present value of an ordinary annuity of 1 d. Present value of an annuity due of 1 Ans: C BT: AP Difficulty: Easy TOT: 1 min. AACSB: Analytic AICPA BB: Critical Thinking AICPA FN: Measurement

18.

On July 1, 2009, Roseland Inc. purchased land for a new manufacturing facility at a price of $750,000. However, the seller is financing the transaction and equal quarterly payments will be made starting today, July 1, 2009. The last semi-annual payment will be made on December 31, 2028. The applicable interest rate is 8%. How much is each semi-annual payment? a. b. c. d.

$35,365 $36,435 $37,893 None of the above

Ans: B BT: AN Difficulty: Moderate TOT: 2 min. AACSB: Analytic AICPA BB: Critical Thinking AICPA FN: Measurement Calculations: n = 40; I = 4% [$750,000/20.58448 (table 6)] = $36,435

19.

Carter Holding Co. intends to purchase a new accounting system, including hardware, software and a complete package of services needed to get the new system up and running. Carter has four options for paying for the new system. Which of the four options is the least costly if the applicable interest rate is 12%? a. b. c. d.

Make a lump sum payment of $100,000 today Make 10 annual payments of $16,000, starting today Make 40 quarterly payments of $4,000, starting today Make one lump sum payment of $150,000 four years from today

Ans: C BT: AN Difficulty: Moderate TOT: 2 min. AACSB: Analytic AICPA BB: Critical Thinking AICPA FN: Measurement Calculations: a. b. c. d.

(no calculations required) $100,000 n = 10; I = 12% [$16,000 * 6.32825 (table 6)] = $101,252 n = 40; I = 3% [$4,000 * 23.80822 (table 6)] = $95,233 n = 4; I = 12% [$150,000 * .63552 (table 4)] = $95,328


A-6

Test Bank – Appendix - The Time Value of Money

20.

Jim Hall invested $12,000 at 8% annual interest and left the money invested without withdrawing any of the interest for 15 years. At the end of the 15 years, Jim withdrew the accumulated amount of money. What amount did Jim withdraw, assuming the investment earns compounded interest? a. b. c. d.

$14,400 $38,066 $26,400 $15,783

Ans: B BT: AN Difficulty: Moderate TOT: 2 min. AACSB: Analytic AICPA BB: Critical Thinking AICPA FN: Measurement Calculations: n = 15; I = 8% $12,000 x 3.17217 (table 1) = $38,066

21.

Karla Simpson invested $15,000 at 10% annual interest and left the money invested without withdrawing any of the interest for 15 years. At the end of the 15 years, Karla decided to withdraw the accumulated amount of money. Karla has found the following values in various tables related to the time value of money. Present Value of 1 for 15 periods at 10% Future Value of 1 for 15 periods at 10% Present Value of an Annuity of 1 for 15 periods at 10% Future Value of an Annuity of 1 for 15 periods at 10%

0.23939 4.17725 7.60608 31.77248

Which factor would she use to compute the amount she would withdraw, assuming that the investment earns interest compounded annually? a. b. c. d.

0.23939 4.17725 7.60608 31.77248

Ans: B BT: AN Difficulty: Moderate TOT: 2 min. AACSB: Analytic AICPA BB: Critical Thinking AICPA FN: Measurement


Test Bank – Appendix - The Time Value of Money

22.

A-7

Thomas Young invested $15,000 at 10% annual interest and left the money invested without withdrawing any of the interest for 15 years. At the end of the 15 years, Thomas decided to withdraw the accumulated amount of money. Thomas has found the following values in various tables related to the time value of money. Present Value of 1 for 15 periods at 10% Future Value of 1 for 15 periods at 10% Present Value of an Annuity of 1 for 15 periods at 10% Future Value of an Annuity of 1 for 15 periods at 10%

0.23939 4.17725 7.60608 31.77248

To the closest dollar, which amount would he withdraw, assuming that the investment earns interest compounded annually? a. $18,591 b. $114,091 c. $47,659 d. $62,659 Ans: D BT: AN Difficulty: Moderate TOT: 2 min. AACSB: Analytic AICPA BB: Critical Thinking AICPA FN: Measurement

23.

Rowan and Lisa Sharp invested $10,000 in a savings account paying 5% annual interest when their son, Jeremy, was born. They also deposited $500 on each of his birthdays until he was 20 (including his 20th birthday). Rowan and Lisa have obtained the following values related to the time value of money to help them with their planning process for their compounded interest decisions. Present Value of 1 for 20 periods at 5% Future Value of 1 for 20 periods at 5% Present Value of an Annuity of 1 for 20 periods at 5% Future Value of an Annuity of 1 for 20 periods at 5%

0.37689 2.65330 12.46221 33.06595

To the closest dollar, how much was in the savings account on his 20th birthday (after the last deposit)? a. b. c. d.

$53,066 $43,066 $30,000 $26,533

Ans: B BT: AN Difficulty: Moderate TOT: 2 min. AACSB: Analytic AICPA BB: Critical Thinking AICPA FN: Measurement


A-8

Test Bank – Appendix - The Time Value of Money

24.

Harrison Marshall borrowed $65,000 on June 1, 2009. This amount plus accrued interest at 8% compounded annually is to be repaid on June 1, 2022. Harrison has obtained the following values related to the time value of money to help him with his financing process and compounded interest decisions. Present Value of 1 for 13 periods at 8% Future Value of 1 for 13 periods at 8% Present Value of an Annuity of 1 for 13 periods at 8% Future Value of an Annuity of 1 for 13 periods at 8%

0.36770 2.71962 7.90378 21.49530

To the closest dollar, how much will Harrison have to repay on June 1, 2022? a. $132,600 b. $310,707 c. $116,375 d. $176,775 Ans: D BT: AN Difficulty: Moderate TOT: 2 min. AACSB: Analytic AICPA BB: Critical Thinking AICPA FN: Measurement

25.

Stranton Company is considering investing in an annuity contract that will return $40,000 annually at the end of each year for 12 years. Stranton has obtained the following values related to the time value of money to help in its planning process and compounded interest decisions. Present Value of 1 for 12 periods at 9% Future Value of 1 for 12 periods at 9% Present Value of an Annuity of 1 for 12 periods at 9% Future Value of an Annuity of 1 for 12 periods at 9%

0.35554 2.81267 7.16073 20.14072

To the closest dollar, what amount should Stranton Company pay for this investment if it earns a 9% return? a. $497,066 b. $592,507 c. $805,629 d. $286,429 Ans: D BT: AN Difficulty: Moderate TOT: 2 min. AACSB: Analytic AICPA BB: Critical Thinking AICPA FN: Measurement


Test Bank – Appendix - The Time Value of Money

26.

A-9

Everett Corporation issues a 8%, 9-year mortgage note on January 1, 2009, to obtain financing for new equipment. Land is used as collateral for the note. The terms provide for semiannual installment payments of $131,600. The following values related to the time value of money were available to Everett to help them with their planning process and compounded interest decisions. Present Value of 1 for 9 periods at 8% Present Value of 1 for 18 periods at 4% Future Value of 1 for 9 periods at 8% Future Value of 1 for 18 periods at 4% Present Value of an Annuity of 1 for 9 periods at 8% Present Value of an Annuity of 1 for 18 periods at 4% Future Value of an Annuity of 1 for 9 periods at 8% Future Value of an Annuity of 1 for 18 periods at 4%

0.50025 0.49363 1.99900 2.02582 6.24689 12.65930 12.48756 25.64541

To the closest dollar, what were the cash proceeds received from the issuance of the note? a. $822,091 b. $947,520 c. $1,665,964 d. $1,643,363 Ans: C BT: AN Difficulty: Moderate TOT: 2 min. AACSB: Analytic AICPA BB: Critical Thinking AICPA FN: Measurement


A-10

Test Bank – Appendix - The Time Value of Money

27.

Gaynor Company is considering purchasing equipment. The equipment will produce the following cash flows: Year 1, $25,000; Year 2, $45,000; Year 3, $60,000. Below is some of the time value of money information that Gaynor has compiled that might help them in their planning and compounded interest decisions.

Present Value of 1 Future Value of 1 Present Value of an Annuity of 1 Future Value of an Annuity of 1

1 period, 11% 0.90090 1.11000 0.90090 1.00000

2 periods, 11% 0.81162 1.23210 1.71252 2.12000

3 periods, 11% 0.73119 1.36763 2.44371 3.37440

Gaynor requires a minimum rate of return of 11%. To the closest dollar, what is the maximum price Gaynor should pay for the equipment? a. $117,117 b. $102,917 c. $165,253 d. $246,209 Ans: B BT: AN Difficulty: Moderate TOT: 2 min. AACSB: Analytic AICPA BB: Critical Thinking AICPA FN: Measurement

28.

Clarkson Corporation earns 12% on an investment that will return $900,000, 7 years from now. Below is some of the time value of money information that Clarkson has compiled that might help in planning compounded interest decisions. Present Value of 1 for 7 periods at 12% Future Value of 1 for 7 periods at 12% Present Value of an Annuity of 1 for 7 periods at 12% Future Value of an Annuity of 1 for 7 periods at 12%

0.45235 2.21068 4.56376 10.08901

To the closest dollar, what is the amount Clarkson should invest now to earn this rate of return? a. $198,961 b. $407,115 c. $756,000 d. $410,738 Ans: B BT: AN Difficulty: Moderate TOT: 2 min. AACSB: Analytic AICPA BB: Critical Thinking AICPA FN: Measurement


Test Bank – Appendix - The Time Value of Money

29.

A-11

Turner Company is considering an investment, which will return a lump sum of $450,000 four years from now. Below is some of the time value of money information that Turner has compiled that might help in planning compounded interest decisions. Present Value of 1 for 4 periods at 10% Future Value of 1 for 4 periods at 10% Present Value of an Annuity of 1 for 4 periods at 10% Future Value of an Annuity of 1 for 4 periods at 10%

0.68301 1.46410 3.16986 4.64100

To the closest dollar, what amount should Turner Company pay for this investment to earn a 10% return? a. $270,000 b. $180,000 c. $307,355 d. $356,609 Ans: C BT: AN Difficulty: Moderate TOT: 2 min. AACSB: Analytic AICPA BB: Critical Thinking AICPA FN: Measurement

30.

Mitch has been offered three different contracts for a service he provides. Contract 1: $9,000 received at the beginning of each year for ten years, compounded at a 6 percent annual rate. Contract 2:

$9,000 received today and $20,000 received ten years from today. The relevant interest rate is 12 percent.

Contract 3:

$9,000 received at the end of Years 4, 5, and 6. The relevant annual interest rate is 10 percent.

What is the present value of Contract 1? a. $66,240.81 b. $118,627.11 c. $70,215.21 d. $125,744.76 Solution: C Contract 1 Present value = $9,000  Present value factor for an annuity due for i = 6% and n = 10 = $9,000  7.80169 (from Table 6) = $70,215.21

Ans: C BT: AN Difficulty: Moderate TOT: 2 min. AACSB: Analytic AICPA BB: Critical Thinking AICPA FN: Measurement


A-12

Test Bank – Appendix - The Time Value of Money

31.

Mitch has been offered three different contracts for a service he provides. Contract 1: $9,000 received at the beginning of each year for ten years, compounded at a 6 percent annual rate. Contract 2:

$9,000 received today and $20,000 received ten years from today. The relevant interest rate is 12 percent.

Contract 3:

$9,000 received at the end of Years 4, 5, and 6. The relevant annual interest rate is 10 percent.

What is the present value of Contract 2? a. $9,337.13 b. $71,117.00 c. $29,000.00 d. $15,439.40 Solution: D Contract 2 Present value = $9,000 + ($20,000  Present value factor for i = 12% and n = 10) = $9,000 + ($20,000  0.32197 from Table 4) = $15,439.40

Ans: D BT: AN Difficulty: Difficult TOT: 2 min. AACSB: Analytic AICPA BB: Critical Thinking AICPA FN: Measurement

32.

Mitch has been offered three different contracts for a service he provides. Contract 1: $9,000 received at the beginning of each year for ten years, compounded at a 6 percent annual rate. Contract 2:

$9,000 received today and $20,000 received ten years from today. The relevant interest rate is 12 percent.

Contract 3:

$9,000 received at the end of Years 4, 5, and 6. The relevant annual interest rate is 10 percent.

What is the present value of Contract 3? a. $18,497.15 b. $16,815.56 c. $24,619.68 d. $22,381.52 Solution: B Contract 3 Present value = ($9,000  Present value factor for an ordinary annuity for i = 10% and n = 3)  Present value factor for i = 10% and n = 3 = ($9,000  2.48685 from Table 5)  0.75131 from Table 4 = $16,815.56

Ans: B BT: AN Difficulty: Difficult TOT: 2 min. AACSB: Analytic AICPA BB: Critical Thinking AICPA FN: Measurement


Test Bank – Appendix - The Time Value of Money

33.

A-13

Morgan is considering entering into a contract to sell a building on January 1 in exchange for a note. The note pays a lump sum payment of $300,000 in ten years and ten annual payments of $2,500 beginning on the date of sale (January 1). If the annual interest rate is 10 percent, what is the total present value of the contract? a. $159,489.92 b. $132,559.55 c. $131,023.42 d. $155,505.55 Solution: B (1) Present value of annual receipts: Value = $2,500  Present value factor for an annuity due for i = 10% and n = 10 = $2,500  6.75902 (from Table 6) = $16,897.55 (2) Present value of lump-sum receipt: Value = $300,000  Present value factor for i =10% and n = 10 = $300,000  0.38554 (from Table 4) = $115,662.00 (3) Total present value: Value = $16,897.55 + $115,662.00 =

$132,559.55

Ans: B BT: AN Difficulty: Moderate TOT: 2 min. AACSB: Analytic AICPA BB: Critical Thinking AICPA FN: Measurement

34.

Kaitlin is contemplating investing in Cocoa Beach Tans. She estimates that the company will pay the following dividends per share at the end of the next four years and that the current price of the company’s common stock, which is $100 per share, will remain unchanged. Dividends

Year 1 $6

Year 2 $7

Year 3 $8

Year 4 $9

If Kaitlin wants to earn 12 percent on her investment and plans to sell the investment at the end of the fourth year, how much would she be willing to pay for one share of common stock? (Round all calculations to the nearest cent.) a. $130.00 b. $119.07 c. $85.90 d. $82.00 Solution: C Present value = Present value of dividends + Present value of proceeds = [($6  0.89286 from Table 4) + ($7  0.79719 from Table 4) + ($8  0.71178 from Table 4) + ($9  0.63552 from Table 4)] + ($100  0.63552 from Table 4) = $5.36 + $5.58 + $5.69 + $5.72 + $63.55 = $85.90 Ans: C BT: AN Difficulty: Moderate TOT: 2 min. AACSB: Analytic AICPA BB: Critical Thinking AICPA FN: Measurement


A-14

Test Bank – Appendix - The Time Value of Money

MATCHING QUESTIONS 1.

For each of the following situations in A through D, indicate the abbreviation of the table that should be used to solve for the solution requested. Place the abbreviation of the respective table in the space provided. You may use each table more than once or not at all. PVOA PVAD PV FVOA FVAD FV

Tables Present value of an ordinary annuity Present value of an annuity due Present value of a sum Future value of an ordinary annuity Future value of an annuity due Future value of a sum

_______A. How much would an investor deposit today in order to withdraw $12,000 at the beginning of each of the next four years, assuming that the first payment is withdrawn one year from today? _______B. If interest rates are compounded semi-annually, how much will a company accumulate in three years after making six equal semi-annual payments of $15,000 each? The first payment will be made today. _______C. If interest rates are compounded monthly, how much can a company withdraw per month for 6 months beginning one month from now if $100,000 is deposited today? _______D. You want to buy a house for $200,000 and finance it with interest compounded monthly. If it is financed over a 12-year period, what will be the amount of each annual payment, the first of which will be due at the beginning of the first year? Solution: A. PVOA B. FVAD C. PVOA D. PVAD KP 1 BT: K Difficulty: Moderate TOT: 5 min. AACSB: Analytic AICPA BB: Critical Thinking AICPA FN: Measurement SHORT PROBLEMS 1.

How much must be invested now to receive $10,000 per year for ten years if the first $10,000 is received today and the rate is 10%? Solution: n = 10; I = 10; $10,000  6.75902 [table 6] = $67,590. KP 1 BT: K Difficulty: Moderate TOT: 3 min. AACSB: Analytic AICPA BB: Critical Thinking AICPA FN: Measurement


Test Bank – Appendix - The Time Value of Money

2.

A-15

Ocean Corporation purchased a machine with a cash price of $35,000. Payments will be made at the end of every quarter for 30 payments beginning at the end of each quarter. The machine was financed at 12%. How much is each payment? Solution: n = 30; I = 3% ($35,000/19.60044) [table 5] = $1,786 KP 1 BT: K Difficulty: Moderate TOT: 3 min. AACSB: Analytic AICPA BB: Critical Thinking AICPA FN: Measurement

3.

You deposited $4,000 per year annually starting on January 1, 2008 in a bank account which earns 10%. How much will accumulate by December 31, 2011, the date of the final payment? Solution: n = 4; I = 10% ($4,000  5.105) [table 3] = $20,420 KP 1 BT: K Difficulty: Moderate TOT: 4 min. AACSB: Analytic AICPA BB: Critical Thinking AICPA FN: Measurement

4.

Middlesex Enterprises plans to issue $120,000 of 10-year, 6% bonds. The effective yield at the time of issuance is 8%. A. How much will the bonds sell for in the market if interest is paid annually? B. How much will the bonds sell for in the market if interest is paid semi-annually? Solution: A. Present value of $120,000 for 10 periods at 8%: $120,000  .46319 [table 4] = Present value of $7,200 annuity for 10 periods at 8%: $7,200  6.71008 [table 5] = Total B. Present value of $120,000 for 20 periods at 4%: $120,000  .45639 [table 4] = Present value of $3,600 annuity for 20 periods at 4%: $3,600  13.59033 [table 5] = Total

$55,582 48,312 $103,894

$54,767 48,925 $103,692

KP 1 BT: K Difficulty: Difficult TOT: 6 min. AACSB: Analytic AICPA BB: Critical Thinking AICPA FN: Measurement


A-16

Test Bank – Appendix - The Time Value of Money

5.

The phone rings. You answer, “Hello.” Is this Billy Bob?” “Yes, it is.” “This is Ed McMahon. Congratulations! You have just won the Just Kidding Clearing House sweepstakes! How would you like us to pay you?” You ponder over the best choice of accepting your winnings: 1. Equal payments of $250,000 at the end of each year for twenty years 2. A lump-sum payment of $2,400,000 today 3. A lump-sum payment of $100,000 today and payments of $400,000 at the end of every year for 10 years All earnings can be invested at 10 percent. Make a choice of one of the three options. Show calculations. Solution: 1. PV of ordinary annuity at I = 10 and n = 20 $250,000 x 8.51356 [table 5]=

$2,128,390

2. PV of sum of $2,400,000 =

$2,400,000

3. PV of $100,000 = PV of ordinary annuity at I= 10 and n=10 $400,000 x 6.14457 [table 5] =

$

100,000 2,457,828

2,557,828

Select choice 3. KP 1 BT: K Difficulty: Difficult TOT: 8 min. AACSB: Analytic AICPA BB: Critical Thinking AICPA FN: Decision Modeling

6.

Calculate the contract price of equipment that requires 20 annual payments of $5,000 at the end of each year, beginning one year after the purchase contract is signed. The interest rate expressed in the loan is 7%. Solution: Present value of $5,000 annuity for 20 periods at 7% $5,000 x 10.59401 [table 5] = $52,970 KP 1 BT: K Difficulty: Moderate TOT: 3 min. AACSB: Analytic AICPA BB: Critical Thinking AICPA FN: Measurement

SHORT ESSAY QUESTIONS 1.

Explain the concept of the "time value of money." Solution: The time value of money concept states than a dollar in the future is worth less than a dollar at present. This concept assigns value to money over time. Cash flows received are worth more if received now, rather than later. Cash flows paid are best if deferred for as long as possible. KP 1 BT: K Difficulty: Easy TOT: 3 min. AACSB: Analytic, Communication AICPA BB: Critical Thinking AICPA FN: Measurement


Test Bank – Appendix - The Time Value of Money

2.

A-17

How does inflation affect the value of money over time? Solution: Today's dollar will buy more goods than a future dollar will buy. In periods of rising prices (inflation) prices of today's goods are less than the prices for the same goods in the future. Inflation is the secondary reason why one would prefer a dollar today to a dollar in the future. KP 1 BT: K Difficulty: Easy TOT: 3 min. AACSB: Analytic, Communication AICPA BB: Critical Thinking AICPA FN: Measurement

3.

How does an annuity due differ from an ordinary annuity? Solution: An ordinary annuity assumes the first payment, or cash flow occurs at the end of the first period. An annuity due assumes the first payment occurs at the beginning of the first period. When determining the amount of interest on debt, the cost of interest is more for an annuity due because the balance accumulated is larger. KP 1 BT: K Difficulty: Easy TOT: 3 min. AACSB: Analytic, Communication AICPA BB: Critical Thinking AICPA FN: Measurement

4.

Why is present value not used more liberally on financial statements? Solution: Present value calculations require that both future cash flows and future interest rates be predicted. Predicting the future cash flows associated with an asset or liability with any degree of confidence is almost impossible. Likewise, the development of interest rate assumptions can vary widely and result in significant differences between present value calculations. Therefore, financial statement presentations typically avoid the subjectivity of present values and opt for historical cost, fair market value, replacement costs, and net realizable values. KP 1 BT: K Difficulty: Easy TOT: 3 min. AACSB: Analytic, Communication AICPA BB: Critical Thinking AICPA FN: Measurement


Appendix B Quality of Earnings Cases: A Comprehensive Review

MULTIPLE CHOICE QUESTIONS

1.

The following chart presents the cash flow profiles of four companies. All four companies are in the same industry and are comparable in size. Based on this limited information, which company likely has the weakest quality of earnings? Company 1 Net cash from investing Negative activities Net cash from operating Positive activities Net cash from financing Positive activities a. b. c. d.

Company 2 Positive

Company 3 Negative

Company 4 Positive

Negative

Positive

Positive

Negative

Negative

Positive

Company 1 Company 2 Company 3 Company 4

Solution: B because Company 2 is apparently selling off long-term assets to finance operating cash flow losses and payments to capital providers as well as to deemphasize its negative cash flows from operations. Ans: B BT: AP Difficulty: Moderate TOT: 2 min. AACSB: Analytic AICPA BB: Critical Thinking AICPA FN: Measurement

A-1


B-2

Test Bank – Appendix B - The Quality of Earnings

2.

The following information is available on four different companies. Assume that there is no salvage value on the equipment. All companies operate in the same industry and use similar processes and equipment.

Equipment Depreciation per year

Morton $2,000,000 $250,000

Starburst $500,000 $62,500

Ames $1,500,000 $30,000

Summers $1,000,000 $100,000

Based on this limited information, which company likely has the weakest quality of earnings? a. b. c. d.

Morton Starburst Ames Summers

Solution: C because Ames’ depreciation is out of line with the other companies indicating a 50 year life, which is highly unlikely. Ans: C BT: AN Difficulty: Moderate TOT: 2 min. AACSB: Analytic AICPA BB: Critical Thinking AICPA FN: Measurement

3.

The following information is presented from the financial statement of four companies that operate in the same industry, use similar processes, and are comparable in size.

Company 1 Company 2 Company 3 Company 4

Sales 2009

Sales 2010

$4,000,000 $3,800,000 $4,200,000 $4,300,000

$4,500,000 $4,200,000 $3,900,000 $4,000,000

Accounts Receivable 2009 $400,000 $400,000 $420,000 $410,000

Accounts Receivable 2010 $450,000 $390,000 $350,000 $530,000

Based on this limited information, which company likely has the weakest quality of earnings? a. b. c. d.

Company 1 Company 2 Company 3 Company 4

Solution D, accounts receivable changes are not in line with changes in sales, which are declining from 2009 to 2010. Ans: D BT: AN Difficulty: Moderate TOT: 3 min. AACSB: Analytic AICPA BB: Critical Thinking AICPA FN: Measurement


Test Bank – Appendix B - The Quality of Earnings

4.

B-3

The following information is presented from the financial statement of four companies that operate in the same industry, use similar processes, and are competitors in the same market.

Company 1 Company 2 Company 3 Company 4

Sales 2009 $4,000,000 $3,800,000 $4,200,000 $4,300,000

Sales 2010 $4,500,000 $4,300,000 $3,900,000 $4,000,000

Inventory 2009 $8,000,000 $7,500,000 $8,400,000 $8,500,000

Inventory 2010 $9,000,000 $9,990,000 $7,350,000 $7,440,000

Based on this limited information, which company likely has the weakest quality of earnings? a. b. c. d.

Company 1 Company 2 Company 3 Company 4

Solution B, inventory changes are not in line with changes in sales. Ans: B BT: AN Difficulty: Moderate TOT: 3 min. AACSB: Analytic AICPA BB: Critical Thinking AICPA FN: Measurement


B-4

Test Bank – Appendix B - The Quality of Earnings

5.

The following information is presented from the financial statement of four companies that operate in the same industry, use similar processes, are similar in size, and are competitors in the same market. The data cover Years 2009 to 2012. (in millions)

Company 1 Company 2 Company 3 Company 4 (in millions)

Company 1 Company 2 Company 3 Company 4

Advertising Expenses 2009 $2,010 $2,555 $2,400 $2,200

Advertising Expenses 2010 $2,271 $2,510 $2,400 $2,024

Advertising Expenses 2011 $2,612 $3,160 $2,400 $1,850

Advertising Expenses 2012 $3,056 $3,600 $2,400 $1,744

Research and Development 2009 $4,000,000 $3,800,000 $4,200,000 $4,300,000

Research and Development 2010 $4,000,000 $4,000,000 $4,640,000 $4,000,000

Research and Development 2011 $4,000,000 $4,200,000 $5,336,000 $3,500,000

Research and Development 2012 $4,000,000 $4,400,000 $6,200,000 $2,440,000

Based on this limited information, which company likely has the weakest quality of earnings at Year 4 or 2012? a. b. c. d.

Company 1 Company 2 Company 3 Company 4

Solution: D because there is a reduction of discretionary expenditures for R & D and advertising. Ans: D BT: AN Difficulty: Moderate TOT: 4 min. AACSB: Analytic AICPA BB: Critical Thinking AICPA FN: Measurement


Test Bank – Appendix B - The Quality of Earnings

6.

B-5

When looking at the statement of comprehensive income in the 2009 annual reports of four similar companies in the same industry, you find the following: Company 1 Company 2 Company 3 Company 4

$5 million for litigation charges $5 million provision for restructuring $5 million for research and development $5 million charge for disposal of a segment

Which company has an expense item that is likely to be persistent in terms of earnings? a. b. c. d.

Company 1 Company 2 Company 3 Company 4

Solution C, because items that are expected to be persistent in terms of earning are those transactions like research and development costs, that are expected to be repeated in future years during the normal operation of the business. The other items, like a charge for disposal of a business segment, a provision for restructuring, and litigation charges, are all items that are not expected to occur again in future years. Ans: C BT: AN Difficulty: Easy TOT: 1 min. AACSB: Analytic AICPA BB: Critical Thinking AICPA FN: Measurement

7.

The following information is available on four different companies. All companies operate in the same industry, are of similar size, and use similar processes and equipment. Price/Earnings Ratio 2008 2009 2010

Company 1

Company 2

Company 3

Company 4

10.0 4.0 5.0

9.9 10.8 11.5

7.0 11.0 10.9

11.0 10.0 9.0

Industry Average 9.8 10.7 11.4

Based on this limited information, which company likely has the highest quality of earnings at the end of the three year period? a. Company 1 b. Company 2 c. Company 3 d. Company 4 Solution: B Company 2 is consistent with the industry averages and also shows no P/E ratios below the industry average or declining ratio trends. Ans: B BT: AN Difficulty: Moderate TOT: 2 min. AACSB: Analytic AICPA BB: Critical Thinking AICPA FN: Measurement


B-6

Test Bank – Appendix B - The Quality of Earnings

8.

The following information was taken from the 2009 annual reports of four different companies in the same industry.

Net income before taxes Income tax expense Current Deferred Total Effective tax rate

Company 1 $2,754

Company 2 $1,097

Company 3 1,630

Company 4 1,585

1,107 (104) $1,003 36%

465 (43) $422 39%

520 (184) $1,110 36%

535 (188) $1,050 36%

Based on this limited data, which company appears to be more conservative and have stronger earning power? a. b. c. d.

Company 1 Company 2 Company 3 Company 4

Solution: Based on the information provided in this problem, the conservatism ratio of each company can be computed, indicating that the lower the ratio, the more conservative the company. The higher conservatism ratio indicates that management is more aggressive with its tax policies and results in higher future tax liabilities. Company 1’s conservatism ratio is the lowest and it therefore has stronger earning power. Conservatism Ratio

=

Reported Income Before Taxes ÷ Taxable Income

Company 1 ............................................................................................................ Taxable Income = $1,107 ÷ 36% = $3,075 Conservatism Ratio = $2,754 ÷ $3,075 = 0.90 Company 2 ............................................................................................................ Taxable Income = $465 ÷ 39% = $1,192 Conservatism Ratio = $1,097 ÷ $1,192 = 0.92 Company 3 ............................................................................................................ Taxable Income = $520 ÷ 36% = $1,444 Conservatism Ratio = $1,630 ÷ $1,444 = 1.13 Company 4 ............................................................................................................ Taxable Income = $535 ÷ 36% = $1,486 Conservatism Ratio = $1,585 ÷ $1,486 = 1.07

Ans: A BT: AN Difficulty: Difficult TOT: 4 min. AACSB: Analytic AICPA BB: Critical Thinking AICPA FN: Measurement


Test Bank – Appendix B - The Quality of Earnings

B-7

SHORT PROBLEMS 1. The net income amounts for Box and Wood, Inc. over a four-year period is as follows: 2007 $42,000

2008 $48,000

2009 $46,000

2010 $58,000

After further examination of the financial report, you note that Box and Wood, Inc. made accounting method changes in 2008 and 2010, which affected net income in those periods. In 2008, the company changed depreciation methods. This change increased the book value of its fixed assets in each subsequent year by $10,000. In 2010, the company adopted a new inventory method that increased the book value of the inventory by $18,000. Requirements: a. Calculate the effect of each of these changes on net income in the year of the change. b. Prepare a chart that compares net income across the four-year period, assuming that Box and Wood, Inc. made no accounting changes. How would your assessment of the company’s performance change after you learned of the accounting method changes? c. What principle of financial accounting makes it difficult to make such changes? Describe the conditions under which Box and Wood, Inc. would be allowed to make changes in their accounting methods. Solution: a.

(1) During 2008 the company changed depreciation methods. This change resulted in an increase of the book value of the assets versus if no change in accounting method had occurred. In other words, the depreciation expense went down by the same amount, i.e., $10,000. A decrease in the depreciation expenses would increase the net income by the same amount, i.e., $10,000. (2) During 2010 the company changed its method of inventory valuation, which also increased the book value of the inventory. Since the cost of inventory is allocated either to the cost of goods sold account or to the ending inventory account, this change implies that the Cost of Goods Sold decreased by $18,000. This would also increase the net income by $18,000. Overall it seems the company is having a bad year and is attempting to use liberal accounting policies to paint a “rosy” picture of the operations.

b.

2007

Net income as reported $ 42,000 Effect of depreciation change 0 Effect of inventory change 0 Adjusted net income $ 42,000

2008

2009

2010

$ 48,000 (10,000) 0 $ 38,000

$ 46,000 (10,000) 0 $ 36,000

$ 58,000 (10,000) (18,000) $ 30,000


B-8

Test Bank – Appendix B - The Quality of Earnings

The adjusted net income figures indicate that if the company had not changed accounting methods, it would have reported declining profits. In fact, the company would have reported net income of only $30,000 in 2010. The reported net income figures have been enhanced with accounting techniques rather than by sound economic health. Consequently, the company's performance would be viewed less positively. BT: AN Difficulty: Difficult TOT: 10 min. AACSB: Analytic, Communication AICPA BB: Critical Thinking AICPA FN: Reporting

2.

You have just been hired as a loan officer for Coastline Bank and Trust. Seaton Industries and Martin Company have both applied for $125,000 nine-month loans. It is the strict policy of the bank to have only $1,350,000 outstanding in unsecured loans at any point in time. Since the bank currently has $1,210,000 in unsecured loans outstanding it will be unable to grant loans to both companies. The bank president has given you the following selected information from the companies’ loan applications.

Cash Accounts receivable Inventory Prepaid expenses Total current assets Noncurrent assets Total assets

Seaton Industries Martin Company $ 15,000 $ 160,000 215,000 470,000 305,000 195,000 180,000 10,000 715,000 835,000 1,455,000 1,875,000 $ 2,170,000 $ 2,710,000

Current liabilities Long-term liabilities Contributed capital Retained earnings Total liabilities and shareholders’ equity

$

285,000 950,000 790,000 145,000

$

325,000 875,000 910,000 600,000

$ 2,170,000

$ 2,710,000

Net credit sales Cost of goods sold

$ 1,005,000 755,000

$ 1,625,000 960,000

Required: Assume that all account balances on the balance sheet are representative of the entire year. Based on this limited information, which company would you recommend to the bank president as the better risk for an unsecured loan? Support your answer with any relevant analysis, including examination of the current ratio, quick ratio, receivables turnover, and inventory turnover. Solution: As a loan officer, there should be concern about the potential borrower’s ability to meet its debts as they come due. Since both companies are requesting only nine-month loans, you should be interested in the potential borrowers' short-term solvency. Therefore, you would examine their current ratios and quick ratios. Further, you would consider the effect of the potential loan on these ratios. The current ratio is calculated as current assets divided by current liabilities.


Test Bank – Appendix B - The Quality of Earnings

Seaton: Martin:

B-9

$715,000 ÷ ($285,000 + $125,000) = 1.74 $835,000 ÷ ($325,000 + $125,000) = 1.86

It appears that both companies have more than sufficient current assets to meet their current obligations, including the new loan. However, some current assets, such as prepaid expenses and inventory, are not near-cash assets. Thus, a better measure of a potential borrower's ability to meet its current obligations is the quick ratio. This ratio is calculated as the sum of cash, marketable securities, and accounts receivable divided by current liabilities. Again, the effect of the new loan should be considered. Seaton: Martin:

($15,000 + $215,000) ÷ ($285,000 + $125,000) = .561 ($160,000 + $470,000) ÷ ($325,000 + $125,000) = 1.400

Based on the quick ratio, Martin appears to be a much better risk than Seaton. Martin has approximately 2.5 times more near-cash assets available than Seaton to meet its current obligations. Therefore, Martin does not have to rely as heavily on converting other assets to cash as Seaton does to meet its obligations. The company that can most readily convert its inventory and receivables to cash might be the better risk. Two possible measures of a company's ability to generate cash from its receivables and inventory are the turnover and number-of-days ratios. Receivables turnover is calculated as net credit sales divided by average accounts receivable, and the number of days for receivables is calculated as 365 divided by the receivables turnover. Receivables turnover: Seaton: $1,005,000 ÷ $215,000 = 4.67 Martin: $1,625,000 ÷ $470,000 = 3.46 Number of days: Seaton: 365 ÷ 4.67 = 78.16 Martin: 365 ÷ 3.46 = 105.49 These ratios indicate that Seaton, on average, collects its receivables 27 days quicker than Martin. Therefore, Seaton can more easily convert its receivables to cash than Martin can. Inventory turnover is calculated as cost of goods sold divided by average inventory, and the number of days is calculated as 365 divided by inventory turnover. Inventory turnover: Seaton: $755,000 ÷ $305,000 = 2.48 Martin: $960,000 ÷ $195,000 = 4.92 Number of days: Seaton: 365 ÷ 2.48 = 147.18 Martin: 365 ÷ 4.92 = 74.19 These ratios bode well for Martin. Martin sells its inventory, on average, 73 days sooner than Seaton sells its inventory. This difference implies that Martin generates more sales which, in turn, implies that it generates more accounts receivable. Although Martin does not turn over its receivables as often as Seaton, it has a larger amount of receivables to turn over. Thus, Martin potentially has more assets that can easily be converted into cash than Seaton. Based upon Martin's superior quick ratio and potential ability to generate cash from its


B-10

Test Bank – Appendix B - The Quality of Earnings

larger receivables base, the recommendation should be that the bank grants the loan to Martin. BT: AN Difficulty: Difficult TOT: 12 min. AACSB: Analytic, Communication AICPA BB: Critical Thinking AICPA FN: Reporting

3.

Parton Company began operation on January 1, 2008. The initial investment by the owners was $100,000. The following information was extracted from the company’s records. Net income

2008 2009 2010 2011

$51,000 49,000 51,500 50,500

Dec. 31 Dec. 31 Cost of Shareholders’ Inventory Goods Sold Equity $100,000 $200,000 $1,200,000 290,000 255,000 1,350,000 315,000 320,000 1,395,000 510,000 365,000 1,400,000

Required: a.

Compute the return on equity for each year. (Assume a $0 inventory for January 1, 2008). Has the company been effective at managing the capital provided by the equity owners? b. Does the information about inventory and the cost of goods sold indicate any reason for the trend in return on equity? Support your answer with any relevant ratios. Solution: a. Return on Equity = Net Income ÷ Average Stockholders' Equity 2008: $51,000 ÷ [($100,000 + $100,000) ÷ 2] = 0.51 or 51% 2009: $49,000 ÷ [($100,000 + $290,000) ÷ 2] = 0.25 or 25% 2010: $51,500 ÷ [($290,000 + $315,000) ÷ 2] = 0.17 or 17% 2011: $50,500 ÷ [($315,000 + $510,000) ÷ 2] = 0.12 or 12% It appears that the additional capital provided by the owners has not been used to generate net income. The company's net income has been relatively constant from 2008 to 2011. If the company had been effective at using the additional capital, the company's net income should have increased, and return on equity should have been relatively constant or increasing over time. However, if the company has used the additional capital for long-term projects, such as a new product, these projects may not generate any net income for several years. Once these projects begin generating income, the company's return on equity may increase to more appropriate levels. Therefore, the effectiveness of the company at using the owners' capital cannot be adequately evaluated without additional information. b. It appears that the company has overinvested in inventory. The inventory turnover and the days' supply of inventory for each year are:

Inventory turnover Days' supply

2008

2009

2010

2011

12.00 30.42

5.93 61.55

4.85 75.26

4.09 89.24


Test Bank – Appendix B - The Quality of Earnings

Inventory turnover = Cost of Goods Sold / Average Inventory 2008: ($1,200,000 / ($0 + 200,000) = 12.00 2009: ($1,350,000 / ($200,000 + 255,000) = 5.93 2010: ($1,395,000 / ($255,000 + 320,000) = 4.85 2011: ($1,400,000 / ($320,000 + 365,000) = 4.09

B-11


B-12

Test Bank – Appendix B - The Quality of Earnings

Days’ supply of inventory = 365 days/ Inventory turnover ratio 2008: 365 / 12.00 = 30.42 days 2009: 365 / 5.93 = 61.55 days 2010: 365 / 4.85 = 75.26 days 2011: 365 / 4.09 = 89.24 days These ratios indicate that the company went from having one month's supply of inventory on hand to having almost three months of inventory on hand. It appears that the company has more inventory on hand than is warranted, given demand for the inventory. The company could reduce inventory on hand and invest the proceeds in income-producing assets such as marketable securities. Such a move would make the company more profitable and provide owners a greater return on their investments. This change in investment policy would increase the company's return on equity.

BT: AN Difficulty: Difficult TOT: 12 min. AACSB: Analytic, Communication AICPA BB: Critical Thinking AICPA FN: Measurement

4.

The following selected financial information was obtained from the 2010 financial reports of Roper Designs and Turner Industries:

Interest expense Extraordinary gain (net of $320,000 taxes) Net income (including extraordinary item)

Roper $100,000 --610,000

Turner $175,000 1,300,000 1,675,000

Current liabilities Bonds payable Mortgage payable Common stock Additional paid-in capital Retained earnings Total liabilities and shareholders’ equity

140,000 725,000 1,490,000 500,000 215,000 290,000 $3,360,000

25,000 0 405,000 600,000 325,000 515,000 $1,870,000

Required: a. Assume that you are considering purchasing the common stock of one of these companies. (Since you have limited data, assume that the beginning balance sheet amounts equal ending balance sheet amounts for total assets and stockholders’ equity.) Based on this information, which company has a higher return on equity? Would your conclusion be different if the impact of the extraordinary item had not been included in net income? Should the extraordinary item be considered? Why or why not? b. Which company uses leverage more effectively? Does your answer change if you do not consider the impact of the extraordinary item on net income? Solution: a. Roper: Turner:

Return on Equity = Net Income ÷ Average Stockholders' Equity $610,000 ÷ [($1,005,000 + $1,005,000) ÷ 2] = 0.607 or 60.7% $1,675,000 ÷ [($1,440,000 + $1,440,000) ÷ 2] = 1.163 or 116.3%


Test Bank – Appendix B - The Quality of Earnings

B-13

Based on return on equity, Turner is almost twice as efficient as Roper at managing the shareholders’ capital. If unusual items were not considered, return on equity for each company would be: Roper: Turner:

$610,000 ÷ [($1,005,000 + $1,005,000) ÷ 2] = 0.607 or 60.7% ($1,675,000 – $1,300,000) ÷ [($1,440,000 + $1,440,000) ÷ 2] = 0.260 or 26.0%

Turner now appears to be considerably worse than Roper at managing the stockholders' capital. Including unusual items in calculating return on equity does provide a more complete measure of how efficiently a company managed its stockholders' equity in the current year. However, since unusual items are, by definition, items that occur infrequently, these items do not indicate a company's continued ability to efficiently manage the stockholders' capital. Thus, unusual items probably should not be used to calculate return on equity. b. Financial leverage indicates how effectively a company uses debt for the benefit of stockholders. Financial leverage equals return on equity less return on assets. Thus, return on assets must be calculated before calculating financial leverage. Return on Assets = (Net Income + Interest Expense (net of tax)) ÷ Average Total Assets Roper: ($610,000 + $100,000) ÷ [($3,360,000 + $3,360,000) ÷ 2] = 0.211 or 21.1% Turner: ($1,675,000 + $175,000) ÷ [($1,870,000 + $1,870,000) ÷ 2] = 0.989 or 98.9% Financial Leverage = Return on Equity – Return on Assets Roper: Turner:

0.607 – 0.211 = 0.396 1.163 – 0.989 = 0.174

From this analysis, Roper is approximately twice as effective as Turner at using debt to generate returns for its stockholders. If unusual items are not considered, the return on assets for each company would be: Roper: ($610,000 + $100,000) ÷ [($3,360,000 + $3,360,000) ÷ 2] = 0.211 Turner: [($1,675,000 – $1,300,000) + $175,000] ÷ [($1,870,000 + $1,870,000)÷2]= 0.294 Therefore, the financial leverage of the two companies would be: Roper: Turner:

0.607 – 0.211 = 0.396 0.260 – 0.294 = –0.034

If extraordinary items are not considered, Turner has negative financial leverage. That means that Turner is not generating a large enough return on its debt to even cover the interest expense. Thus, Turner is using debt to the detriment of its stockholders. It appears, therefore, that extraordinary items can affect the conclusions one draws when analyzing a company and its quality of earnings.

BT: AN Difficulty: Difficult TOT: 12 min. AACSB: Analytic, Communication AICPA BB: Critical Thinking AICPA FN: Measurement


B-14

Test Bank – Appendix B - The Quality of Earnings

5.

Carlton Electronics posted net income of $500,000 in 2009, compared with a loss of $100,000 in 2008. Over $200,000 of the 2009 profit was due to a problem with faulty approximation in its Toledo operations. The problem occurred when a tax liability had been accrued in prior years assuming a higher tax rate that was actually in effect when the taxes were paid. Required: How do you interpret this in terms of quality of earnings? How can a change in expected tax rates lead to a positive effect on reported earnings? Does the $200,000 represent an increase in overall wealth of the company? Solution: Working through an accounting adjustment due to a change in the tax rates of the Toledo operations, Carlton earned $200,000, rather than through its operations. A change in the expected tax rate can lead to a positive effect on reported earnings due to the fact that tax liability was accrued at a higher tax rate and eventually paid at a lower tax rate. However, the quality of earnings and overall wealth of the company is not increased by this transaction. It seems that Carlton estimated its tax liability to be higher, and it had to pay much less lower taxes at the end of the year. This is similar to a change of an accounting estimate.

BT: AN Difficulty: Easy TOT: 4 min. AACSB: Analytic, Communication AICPA BB: Critical Thinking AICPA FN: Measurement


Test Bank – Appendix B - The Quality of Earnings

B-15

6. Xenon, a major defense contractor, was faced with huge liabilities and feared violation of debt covenants. Therefore, Xenon declared Chapter 11 bankruptcy protection. Under Chapter 11, a company continues to operate but is protected from creditors while it tries to work out a reorganization plan. At that time the company’s management chose to take several significant charges under bankruptcy proceedings, including a $1 million liability not required by GAAP, but that better reflected its commitments to employees. Required: Why would Xenon’s management have chosen to take these charges at this time? Solution: As part of the Chapter 11 reorganization, Xenon can negotiate new credit agreements. These credit agreements contain debt covenants, most likely related to the amount of debt Xenon can have. If Xenon waited until after emerging from Chapter 11, the $1 million liability along with other charges might have caused the company to violate the amount of allowable debt as specified in its debt agreements. Such a violation could have forced the company back into Chapter 11 or to renegotiate its debt agreements at less favorable terms. By recording the charges prior to negotiating its new debt agreements, the new charges would be considered by creditors in creating the debt covenants. Thus, this liability would not place Xenon into an automatic violation of its debt covenants. Reasons for taking several significant charges while under bankruptcy proceedings include: (1) The significant charges would adversely affect Xenon’s reported results of operations and financial position, and Xenon may have been trying to extract more favorable settlement terms from its creditors by demonstrating weakened performance and financial position. (2) Xenon may have been positioning itself to show improved performance once it emerged from Chapter 11. By taking the charges now, not only does Xenon avoid having to reduce its earnings in the future, but it also reduces the company's earnings so much that its earnings can only increase next year. This latter strategy is known as "taking a bath."

BT: AP Difficulty: Moderate TOT: 5 min. AACSB: Analytic, Communication AICPA BB: Critical Thinking AICPA FN: Reporting

7.

You are reviewing the annual report for Mega City Electronics. You noticed that Mega City cut its dividend last year and the stock price when up. Required: Explain how a dividend cut could lead to an increased stock price. Solution: The most likely scenario is that the company has been paying a dividend and is still generating enough cash to pay the dividend but decides not to. This is usually because the company feels that it has very good investment opportunities and wants to use the money to pursue them. This is usually a positive sign and the stock price may rise.

BT: AP Difficulty: Easy TOT: 3 min. AACSB: Analytic, Communication AICPA BB: Critical Thinking AICPA FN: Reporting


APPENDIX A THE TIME VALUE OF MONEY EXERCISES EA–1 Time Periods (Years) Compound Interest Rates

5

10

15

5%

$150  1.27628 = $191.44

$150  1.62889 = $244.33

$150  2.07893 = $311.84

10%

$150  1.61051 = $241.58

$150  2.59374 = $389.06

$150  4.17725 = $626.59

15%

$150  2.01136 = $301.70

$150  4.04556 = $606.83

$150  8.13706 = $1,220.56

EA–2 Time Periods (Years) Compound Interest Rates

5

10

15

5%

$10,000 = $7,835.26 1.05^5

$10,000 = $6,139.15 1.05^10

$10,000 = $4,810.17 1.05^15

10%

$10,000 = $6,209.21 1.10^5

$10,000 = $3,855.44 1.10^10

$10,000 = $2,393.92 1.10^15

15%

$10,000 = $4,971.76 1.15^5

$10,000 = $2,471.85 1.15^10

$10,000 = $1,228.94 1.15^15

The above problem has also been attempted in an alternate way to demonstrate the use of formulas.

1


EA–3 Time Periods (Years) Compound Interest Rates

5

10

15

5%

$150  5.52563 = $828.84

$150  12.57789 = $1,886.68

$150  21.57856 = $3,236.78

10%

$150  6.10510 = $915.77

$150  15.93743 = $2,390.61

$150  31.77248 = $4,765.87

15%

$150  6.74238 = $1,011.36

$150  20.30372 = $3,045.56

$150  47.58041 = $7,137.06

EA–4 Time Periods (Years) Compound Interest Rates

5

10

15

5%

$150  5.80191 = $870.29

$150  13.20679 = $1,981.02

$150  22.65749 = $3,398.62

10%

$150  6.71561 = $1,007.34

$150  17.53117 = $2,629.68

$150  34.94973 = $5,242.46

15%

$150  7.75374 = $1,163.06

$150  23.34928 = $3,502.39

$150  54.71747 = $8,207.62

EA–5 Time Periods (Years) Compound Interest Rates

5

10

15

5%

$10,000  4.32948 = $43,294.80

$10,000  7.72173 = $77,217.30

$10,000  10.37966 = $103,796.60

10%

$10,000  3.79079 = $37,907.90

$10,000  6.14457 = $61,445.70

$10,000  7.60608 = $76,060.80

15%

$10,000  3.35216 = $33,521.60

$10,000  5.01877 = $50,187.70

$10,000  5.84737 = $58,473.70

2


EA–6 Time Periods (Years) Compound Interest Rates

5

10

15

5%

$10,000  4.54595 = $45,459.50

$10,000  8.10782 = $81,078.20

$10,000  10.89864 = $108,986.40

10%

$10,000  4.16987 = $41,698.70

$10,000  6.75902 = $67,590.20

$10,000  8.36669 = $83,666.90

15%

$10,000  3.85498 = $38,549.80

$10,000  5.77158 = $57,715.80

$10,000  6.72448 = $67,244.80

EA–7 a. ($50  .85734) + ($100  .68058) = $42.87 + $68.06 = $154.15 b. ($100  3.31213) = $331.21 = $385.24 c.

+ +

($100  .54027) $54.03

($60  .68058) + ($60  .63017) = $40.83 + $37.81 = $192.39

d. ($90  .58349) = $52.51 = $146.15

+ ($80  .54027) + $43.22

+ ($60  .58349) + ($60  .54027) + + $35.01 + $32.42 +

+ ($90  .54027) + ($90  .50025) + $48.62 +

($100  .46319) $46.32

$45.02

EA–8 a. ($50  .85734) + = $42.87 = $157.61

($100  .68058) + $68.06

b. ($100  3.57710) = $357.71 = $416.06

+ +

c.

+ +

($80  .58349) $46.68

($60  .73503) + ($60  .68058) + ($60  .63017) + = $44.10 + $40.83 + $37.81 = $207.78

($60  .58349) + $35.01

($100  .58349) $58.35

d. ($90  .63017) + ($90  .58349) = $56.72 + $52.51 = $157.85

+ ($90  . 54027) + $48.62

3

+ ($100  .50025) + $50.03


EA–9 a. Dollar amount

= $25,000  Future value factor for i = 10% and n = 4 = $25,000  1.46410 (from Table 1) = $36,603

Dollar amount

= $36,603  Future value factor for i = 12% and n = 3 = $36,603  1.40493 (from Table 1) = $51,425

Dollar amount

= $51,425  Future value factor for i = 15% and n = 5 = $51,425  2.01136 (from Table 1) = $103,434

b. Ben should not accept $36,000 for $25,000 at the end of 4 years. Why not? Because if he invests the initial $25,000 at 10 percent per annum compounded annually, he will have a total of $36,603, $603 more than the amount the person offered him.

EA–10 a. Dollar amount

= ($40,000  Present value factor for an ordinary annuity factor for i = 10% and n = 10) + ($500,000  Present value factor for i = 10% and n = 10) = ($40,000  6.14457 from Table 5) + ($500,000  .38554 from Table 4) = $245,782.80 + $192,770.00 = $438,552.80

b. There are two different ways to calculate the dollar amount. The two ways are shown below. Dollar amount

= ($40,000  Present value factor for an annuity due for i = 10% and n + ($500,000  Present value factor for i = 10% and n = 10) = ($40,000  6.75902 from Table 6) + ($500,000  .38554 from Table 4) = $270,360.80 + $192,770.00 = $463,130.80

Dollar amount

= $40,000 + ($40,000  Present value factor for an ordinary annuity factor for i = 10% and n = 9) + ($500,000  Present value factor for i = 10% and n = 10) = $40,000 + ($40,000  5.75902 from Table 5) + ($500,000  .38554 from Table 4) = $40,000 + $230,360.80 + $192,770.00 = $463,130.80

4

= 10)


EA–11 Option 1 Present value = = = Option 2 Present value =

$500,000  Present value factor for an ordinary annuity for i = 10% and n = 20) $500,000  8.51356 (from Table 5) $4,256,780

$4,500,000

Option 3 Present value = $1,000,000 + [($2,100,000  Present value factor for an ordinary annuity for i = 10% and n = 3)  Present value factor for i = 10% and n = 4] = $1,000,000 + [($2,100,000  2.48685 from Table 5)  .68301 from Table 4] = $1,000,000 + $3,566,941 = $4,566,941 Option 3 should be chosen because it has the highest present value. In other words, if receiving the equivalent amounts for each of the 3 payment patterns, alternative 3 would yield the largest payout today.

EA–12 Ordinary Annuity

a. $700  2.48685 (from Table 5) $700  2.73554 (from Table 6)

$1,740.80

b. $700 + ($700  1.73554 from Table 5) ($700  1.10000 from Table 1) + $700 + ($700  .90909 from Table 4)

1,914.88

c.

Annuity Due

$1,914.88

2,106.36

($700  1.10000 from Table 1) + $700 + ($700  .90909 from Table 4) $700  2.31000 (from Table 3) + $700

2,106.36 2,317.00

d. $700  3.31000 (from Table 2) $700  3.64100 (from Table 3)

2,317.00 2,548.70

e. The present value is the value of future cash flows at the current point in time. Thus, the values in Part (a) represent the present value of the two different annuities. f.

The future value is the value of future cash flows at a future point in time. Since the ends of Periods 1, 2, and 3 are all in the future, the value of the cash flows at those points in time all qualify as future values.

g. Annuity due is most valuable. The present value of annuity due is $174.08 more than the present value of ordinary annuity. In other words, if we were to receive $700 each year for the next 3 years, the payment pattern of the annuity due (payment to be received at the beginning of each year) should be

5


more preferable to us than the payment pattern of the ordinary annuity (payment to be received at the end of each year).

EA–13 a. Option 1 Present value = $240,000 Option 2 Present value = $500,000  Present value factor for i = 12% and n = 8 = $500,000  .40388 (from Table 4) = $201,940 Option 3 Present value = $600,000  Present value factor for i = 12% and n = 10 = $600,000  .32197 (from Table 4) = $193,182 Option 4 Present value = $50,000  Present value factor for an annuity due for i = 12% and n = 6 = $50,000  4.60478 (from Table 6) = $230,239 b. By computing the present value of each option's future cash flows, the cost of each option is comparable. Since Option 3 has the lowest present value, it appears to the best deal for Dunn Drafting Company. c.

Option 1: Present value = $240,000 Option 2: Present value = $500,000  Present value factor for i = 8% and n = 8 = $500,000  .54027 (from Table 4) = $270,135 Option 3: Present value = $600,000  Present value factor for i = 8% and n = 10 = $600,000  .46319 (from Table 4) = $277,914 Option 4: Present value = $50,000  Present value factor for an annuity due for i = 8% and n = 6 = $50,000  4.99271 (from Table 6) = $249,636 Option 1 now minimizes the present value of future cash flows. Thus, it appears that Option 1 is now the best option for Dunn Drafting Company.

6


EA–14 a. Since the Croziers plan to invest a lump sum today and then withdraw the money in the form of an annuity, two steps are required to determine how much the Croziers must invest today to pay for Ryan's college education. The first step is to calculate how much money they will need fifteen years from now when Ryan enters college to make the four payments at the beginning of each year Ryan is in college (i.e., the value of the annuity). The second step is to calculate how much they would have to invest now so that it will grow to the value calculated in the first step over the next fifteen years. The calculations are shown below. Present value of college expenses fifteen years in the future: Value = $40,000  Present value factor for an annuity due for i = 10% and n = 4 = $40,000  3.48685 (from Table 6) = $139,474.00 Present value of college expenses today: Value = $139,474.00  Present value factor for i = 10% and n = 15 = $139,474.00  .23939 (from Table 4) = $33,389.00 b. The present value of fourteen annual payments must equal the present value of $33,389 calculated in part (a). By using the following formula, the amount of the annual payments can be calculated. Present value = Annuity payment  Present value factor for an ordinary annuity for i =10% and n = 14 $33,389 = Annuity payment  7.36669 (from Table 5) Annuity payment = $4,532.43 c.

Current investment Present value of college expenses fifteen years in the future: Value = $40,000  Present value factor for an annuity due for i = 8% and n = 4 = $40,000  3.57710 (from Table 5) = $143,084 Present value of college expenses today: Value = $143,084  Present value factor for i = 8% and n = 15 = $143,084  .31524 (from Table 4) = $45,106 Annuity payment Present value = Annuity payment  Present value factor for an ordinary annuity for i = 8% and n = 14 $45,106 = Annuity payment  8.24424 (from Table 5) Annuity payment = $5,471.21

7


EA–15 a. ($30,000  .46319) + ($30,000  .42888) = $13,895.70 + $12,866.40 = $49,706.40

+ ($30,000  .39711) + $11,913.30

+ +

($30,000  .36770) $11,031.00

+ +

($30,000 x .46884) $14,065.20

b. $49,706.40  6.24689 = $7,956.98 c.

($30,000  .55839) + ($30,000  .52679) = $16,751.70 + $15,803.70 = $61,529.70

+ ($30,000  .49697) + $14,909.10

The yearly installment under 6% will be $61,529.70  6.8017 = $9,046.22

8


PROBLEMS PA–1 The price that Christie is willing to pay for the stock is comprised of two components: the present value of the dividends she expects to receive from holding the investment and the present value of the proceeds she will receive when she sells the investment. The total present value is calculated as follows.

Present value

= Present value of dividends + Present value of proceeds = [($5  .89286 from Table 4) + ($6  .79719 from Table 4) + ($7  .71178 from Table 4) + ($8  .63552 from Table 4)] + ($100  .63552 from Table 4) = $4.46 + $4.78 + $4.98 + $5.08 + $63.55 = $82.85

PA–2 a. Investment 1 Future value

Investment 2 Future value

= ($1,000  Future value factor for an ordinary annuity for i = 10% and n = 5)  Future value factor for i = 12% and n = 5 = ($1,000  6.10510 from Table 2)  1.76234 from Table 1 = $10,759.26 = $3,000  Future value factor for an ordinary annuity for i = 15% and n = 7 = $3,000  11.06680 from Table 2 = $33,200.40

Therefore, Wharton's total investment at the end of ten years will equal $43,959.66. b. Current investment

= = =

Future value  Present value factor for i = 12% and n = 10 $43,959.66  .32197 from Table 4 $14,153.69

Therefore, Wharton would have to invest $14,153.69 for ten years earning 12% compounded annually to have an amount equivalent to the two investments.

PA–3 a. Contract 1 Present value

Contract 2 Present value

Contract 3 Present value

= $8,000  Present value factor for an annuity due for i = 6% and n = 10 = $8,000  7.80169 (from Table 6) = $62,413.52 = $8,000 + ($20,000  Present value factor for i = 12% and n = 10) = $8,000 + ($20,000  .32197 from Table 4) = $14,439.40 = ($8,000  Present value factor for an ordinary annuity for i = 10% and n = 3)  Present value factor for i = 10% and n = 3 = ($8,000  2.48685 from Table 5)  .75131 from Table 4 9


= $14,947.16

10


PA–3 Concluded b. (1) Equivalent values at the end of Year 5: Contract 1 Present value

= ($8,000  Future value factor for an annuity due for i = 6% and n = 5) ($8,000  Present value factor for an annuity due for i = 6% and n = 5 = ($8,000  5.97532 from Table 3) + ($8,000  4.46511 from Table 6) = $47,802.56 + $35,720.88 = $83,523.44

+

Proof: $83,523.44  .74726 = $62,413 = Present value of Contract 1 in Part (a) Contract 2 Present value

= ($8,000  Future value factor for i = 12% and n = 5) + ($20,000  Present value factor for i = 12% and n = 5) = ($8,000  1.76234 from Table 1) + ($20,000  .56743 from Table 4) = $14,098.72 + $11,348.60 = $25,447.32

Proof: $25,447.32  .56743 = $14,439 = Present value of Contract 2 in Part (a) Contract 3 Present value

= ($8,000  Future value factor for i = 10% and n = 1) + $8,000 + ($8,000  Present value factor for i = 10% and n = 1) = ($8,000  1.10000 from Table 1) + $8,000 + ($8,000  .90909 from Table 4) = $24,072.72

Proof: $24,072.72  .62092 = $14,947 = Present value of Contract 3 in Part (a) (2)

Equivalent values at the end of Year 10:

Contract 1 Present value

= $8,000  Future value factor for an annuity due for i = 6% and n = 10 = $8,000  13.97164 from Table 3 = $111,773.12

Proof: $111,773.12  .55839 = $62,413 = Present value of Contract 1 in Part (a) Contract 2 Present value

= = = =

($8,000  Future value factor for i = 12% and n = 10) + $20,000 ($8,000  3.10585 from Table 1) + $20,000 $24,846.80 + $20,000.00 $44,846.80

Proof: $44,846.80  .32197 = $14,439 = Present value of Contract 2 in Part (a) Contract 3 Present value

= ($8,000  Future value factor for an ordinary annuity for i = 10% and  Future value factor for i = 10% and n = 4 = ($8,000  3.31000 from Table 2)  1.46410 from Table 1 = $38,769.37 11

n = 3)


Proof: $38,769.37  .38554 = $14,947 = Present value of Contract 3 in Part (a)

12


PA–4 Option 1 Present value Option 2 Present value

Option 3 Present value

= $25,000 = $60,000  Present value factor for i = 9% and n = 8 = $60,000  .50187 (from Table 4) = $30,112.20 = $5,000 + ($27,000  Present value factor for i = 9% and n = 3) + ($20,000  Present value factor for i = 9% and n = 20) = $5,000 + ($27,000  .77218 from Table 4) + ($20,000  .17843 from Table 4) = $5,000 + $20,848.86 + $3,568.60 = $29,417.46

Hartney should accept bonus option 2 because it has the highest present value. In other words, in terms of today’s dollars, bonus option #2 gives Hartney the most amount of money.

PA–5 a. Value

= = = =

b. Value

=

= = = c.

Value

=

= = = d. Value

= = = =

$5,000 + ($10,000  Present value factor for an ordinary annuity for i = 10% and n = 5) + ($15,000  Present value factor for i = 10% and n = 5) $5,000 + ($10,000  3.79079 from Table 5) + ($15,000  .62092 from Table 4) $5,000 + $37,908 + $9,314 $52,222 ($5,000  Future value factor for i = 10% and n = 2) + ($10,000  Future value factor for i = 10% and n = 1) + $10,000 + ($10,000  Present value factor for an ordinary annuity for i = 10% and n = 3) + ($15,000  Present value factor for i = 10% and n = 3) ($5,000  1.21000 from Table 1) + ($10,000  1.10000 from Table 1) + $10,000 + ($10,000  2.48685 from Table 5) + ($15,000  .75131 from Table 4) $6,050 + $11,000 + $10,000 + $24,869 + $11,270 $63,189 ($5,000  Future value factor for i = 10% and n = 4) + ($10,000  Future value factor for an ordinary annuity for i = 10% and n = 4) + [($10,000 + $15,000)  Present value factor for i = 10% and n = 1)] ($5,000  1.46410 from Table 1) + ($10,000  4.64100 from Table 2) + ($25,000  .90909 from Table 4) $7,321 + $46,410 + $22,727 $76,458 ($5,000  Future value factor for i = 10% and n = 5) + ($10,000  Future value factor for an ordinary annuity for i = 10% and n = 5) + $15,000 ($5,000  1.61051 from Table 1) + ($10,000  6.10510 from Table 2) + $15,000 $8,053 + $61,051 + $15,000 $84,104

13


PA–5 Concluded Proof: Value of each equivalent value today Option 1 1. 2. 3. 4.

$52,222  1.00000 $63,189  0.82645 $76,458  0.68301 $84,104  0.62092

Option 2

Option 3

$52,222 $52,222 $52,222 $52,222

PA–6 Present values a. Value = $10,000 b. Value

= = =

$2,000  Present value factor for an ordinary annuity for i = 8% and n = 8 $2,000  5.74664 from Table 5 $11,493.28

c.

Value

= = =

$5,000  Present value factor for an annuity due for i = 8% and n = 3 $5,000  2.78326 from Table 6 $13,916.30

d. Value

= = =

$3,000  Present value factor for an ordinary annuity for i = 8% and n = 5 $3,000  3.99271 from Table 5 $11,978.13

e. Value

= = =

$25,000  Present value factor for i = 8% and n = 7 $25,000  .58349 from Table 4 $14,587.25

f.

Value

= = =

$3,000  Present value factor for an ordinary annuity for i = 8% and n = 2 $3,000  1.78326 from Table 5 $5,349.78

g. Value

= = =

$4,000  Present value factor for an annuity due for i = 8% and n = 3 $4,000  2.78326 from Table 6 $11,133.04

Future values a. Value = = =

$10,000  Future value factor for i = 8% and n = 4 $10,000  1.36049 from Table 1 $13,604.90

b. Value

= = =

$2,000  Future value factor for an ordinary annuity for i = 8% and n = 8 $2,000  10.63663 from Table 2 $21,273.26

c.

= = =

$5,000  Future value factor for an annuity due for i = 8% and n = 3 $5,000  3.50611 from Table 3 $17,530.55

Value

Option 4

14


PA–6 Concluded

= =

($3,000  Future value factor for an ordinary annuity for i = 8% and n = 5)  Future value factor for i = 8% and n = 5 ($3,000  5.86660 from Table 2)  1.46933 from Table 1 $25,859.91

e. Value

=

$25,000

f.

= = =

$3,000  Future value factor for an ordinary annuity for i = 8% and n = 2 $3,000  2.08000 from Table 2 $6,240.00

d. Value

Value

=

g. Value = $4,000  Future value factor for an annuity due for i = 8% and n = 3 = $4,000  3.50611 from Table 3 = $14,024.44

PA–7 a. To determine whether the offer of $110,000 today is a good deal, the future cash flows must be converted into equivalent values in present dollars (i.e., present values). The contract specifies two types of future cash flows: $2,000 at the beginning of each year for ten years and a lump-sum receipt of $300,000 in ten years. The present value of the two types of cash flows are calculated below. (1) Present value of annual receipts: Value = $2,000  Present value factor for an annuity due for i = 10% and n = 10 = $2,000  6.75902 (from Table 6) = $13,518.04 (2) Present value of lump-sum receipt: Value = $300,000  Present value factor for i =10% and n = 10 = $300,000  .38554 (from Table 4) = $115,662.00 (3) Total present value: Value = $13,518.04 + $115,662.00 = $129,180.04 Since the present value of the future cash flows exceeds $110,000, it would not be wise for Joy Don Corp. to accept $110,000 in cash today in place of the note. By accepting the cash of $110,000 now, Joy would be worse off by more than $19,000. b. As the discount rate increases, the present value of future cash flows decreases. Since the present value of the future cash flows discounted at 10% exceeds $110,000, the discount rate at which Joy Don would be wise to accept $110,000 in cash instead of the note must be greater than 10%. Try i = 12%. Present value

= ($2,000  6.32825 from Table 6) + ($300,000  .32197 from Table 4) = $12,656.50 + $96,591.00 = $109,247.50

15


With a discount rate of 12%, the present value of the future cash flows is slightly less than $110,000, which implies that Joy Don would be better off accepting $110,000 in cash today rather than accepting the note.

16


APPENDIX B QUALITY OF EARNINGS CASES: A COMPREHENSIVE REVIEW CASE 1: LIBERTY MANUFACTURING The analysis that can be done on Liberty is limited because there is no access to macroeconomic information, industry data, or the company’s financial statements from the past several years. The following comments are based solely on the information provided and must be viewed as limited for that reason.

Ratio Analysis at Face Value Liberty’s reported profits of $159,000 in 2011 appear to be relatively high. Return on equity is 20 percent ($159,000/$799,000), return on assets is approximately 6.6 percent ($159,000/ $2,399,000), return on sales is 14.5% ($159,000/$1,100,000). [Note: ratios are calculated without balance sheet averages so that two years can be analyzed.] The company turns over its inventory, on average, about every 90 days (4 times per year). Moreover, the significant decrease in the inventory balance probably increased the company’s turnover from the previous year. These measures suggest that Liberty has relatively strong earning power. There is also preliminary evidence that Liberty’s solvency position is satisfactory. The company’s current ratio increased slightly from 2.5 in 2010 to 2.6 in 2011, and the quick ratio increased to a strong 2.2. There is some concern, however, about Liberty’s accounts receivable management. Average days receivable is approximately 141, indicating that the company turns over its receivables only 2.59 times per year. This relatively slow turnover will slow down cash collections and may reduce the company’s solvency position. The debt/equity (total liabilities/stockholders’ equity) ratio represents another source of concern, because it indicates that the company is carrying a much higher percentage of liabilities as of the end of 2011. Specifically, the ratio increased from 1.24 to 2.00 during 2011. The increase was caused primarily by increases in the outstanding accounts payable balance and a $600,000 increase in long-term liabilities. However, the interest coverage ratio is 3.2 [($199,000+$90,000)/$90,000], suggesting that the company’s profits were more than enough to cover the interest charges on its long-term debt. In summary, the ratio analysis conducted above suggests that Liberty’s earning power and solvency positions are reasonable. Significant debt has been added to its capital structure recently, but the company’s earnings power appears to be sufficient to cover the necessary debt payments if receivables are collected on a timely basis.

Earnings Persistence A closer look at the income statement reveals some very troubling aspects about Liberty. First, many of the “gains” reported on the income statement are not essential to the operations of the business. A realized gain on the sale of short-term investments accounted for $80,000 of the net profit; $40,000 was generated from Packer, a company whose stock price fell from $5.00 to $4.00 as of the end of 2011; $25,000 came from a translation gain because the dollar fell relative to the Swiss franc. The exchange rate between Swiss francs and dollars fell from 2:1 to 1.6:1 [200,000/($100,000 + $25,000)] as of December 31, 2011. Book gains recognized on the sale of land and the plant accounted for an additional $14,000. Without these 1


items Liberty’s operating income before taxes is an unimpressive $20,000, only 3.6 percent of sales. Most of these items cannot be counted on to continue in the future, and only a small portion of Liberty’s earning power can be considered persistent.

Quality of Earnings The quality of Liberty’s earning number is also questionable. The allowance for uncollectibles account dropped dramatically as a percentage of outstanding accounts receivable (from 6.7 percent to only 1.8 percent). This drop suggests Liberty’s management is protecting the income statement from bad debt expense. Second, with respect to short-term investment activity, it appears as if Liberty sold its shares of Fredericks in order to report a $80,000 realized gain on the income statement (the unrealized gains go to the balance sheet rather than the income statement). This is evident since it repurchased the shares within 4 days of the sale of the original shares. Third, Liberty reduced its inventory significantly between 2010 and 2011. Reviewing the inventory footnote shows that Liberty reduced its LIFO layers in 2011. This had the impact of increasing income by $210,000. This is calculated by taking the current cost per unit ($16) and subtracting the inventory carrying cost for the items that were sold out of inventory. During 2011 Liberty sold 50,000 units and purchased 25,000 units during the year. Therefore, 25,000 units were sold out of inventory. These 25,000 units had a carrying value as follows: 10,000 units @ $12 each, 10,000 units @ $6 each, and 5,000 units @ $2 each. Since the replacement cost is higher than the carrying cost of these units, a one-time increase in income is created. The total amount of this increase was $210,000 ([($16$12)*10,000] +[ ($16-$6)*10,000] + [($16-$2)*5,000]).

Recommendation Based on this analysis, I would not recommend that the applicant be considered further for a loan. Its solvency position is weak, and its reported earnings contain a number of items that are not expected to persist in the future. In addition, the analysis suggests that the quality of earnings is low and can’t be relied upon.


CASE 2: MICROLINE CORPORATION MEMO Date: January 2012 To: Sharon Sonneborn Re: Evaluation of Microline Corporation Microline is a potential acquisition candidate for Mega Industries. As such, Mega Industries requires an assessment of Microline’s financial condition and performance. I have prepared an evaluation of Microline’s solvency, earnings power, and earnings quality. The amounts in parentheses are stated in thousands of dollars. Microline’s reported 2011 profits of $3.25 million appear to be high, particularly in comparison with 2010’s results. Profits in 2011 are over three times the level of profits in 2010. Return on equity has increased from 4% ($900/$22,500) in 2010 to almost 14% ($3,250/$23,750). Profits have also grown faster than assets, as the return on assets has increased to 6% ($3,250/$51,250) from only 2% ($900/$42,500) in 2010. While sales are increasing, profits are increasing even faster. The return on sales has increased from 2% ($900/$52,500) in 2010 to 5% ($3,250/$60,000) in 2011. Finally, inventory is turning over about 5 2/3 times ($34,000/avg. inventory of $6,000) a year or every 64 days. This is important given that Microline’s inventory may be subject to obsolescence as suggested by its 2010 inventory write-down. These results indicate that Microline’s earnings power is strong. However, without industry data or data on Microline’s past financial statements, the assessment may be limited. With respect to solvency, Microline also appears to be strong. In 2011, Microline’s current ratio of 1.27 ($16,500/$13,000) increased from 0.94 ($11,750/$12,500) in 2010. The quick ratio also increased significantly from 0.46 in 2010 to 0.72 in 2011. The increases in these ratios are due to an increase in current assets rather than a decrease in current liabilities. One reason for the improvement in the current ratio is that in 2011 Microline issued a long-term note that contains a debt covenant requiring a current ratio greater than one. The increases in the receivables, inventory, and short-term investments in 2011 may all have been done in order to improve this ratio and meet the current ratio requirement. The result, however, may be overstated receivables. Receivables may not be turning over as quickly as they could be. While still a strong 11.9 ($60,000/average gross receivables of $5,050) or about every 31 days, the receivables turnover has likely decreased as a result of the current ratio requirement. Due to the issuance of additional notes totaling $7,000, Microline’s total liabilities to equity ratio increased from 0.89 ($20,000/$22,500) in 2010 to 1.16 ($27,500/$23,750) in 2011. This means that Microline is financed with slightly more debt than equity. Based on its reported profits, the interest coverage ratio is strong at 5.75 ($4,750 + $1,000/$1,000) in 2011 from 3.8 ($1,700 + $600/$600) in 2010. However, since the two new notes equal to $7,000 were issued during 2011, 2011’s interest expense is not reflective of expected interest expense in 2012. If both notes are at an 8% rate, interest expense will be almost $500 higher in 2012. Without additional income, the interest coverage ratio will decrease significantly and possibly indicate solvency problems. Although the earnings power and solvency indicators suggest that Microline is financially strong and has improved significantly in 2011, several issues suggest that its 2011 earnings may not be expected to persist in the future and that its earnings power is artificially high. First, Microline’s 2011 profits have been increased by the sale of land and certain short-term investments. For example, Microline sold land for a $1.5 million gain and immediately repurchased a similar parcel of land. Another $500,000 of income was


generated by unrealized and realized gains on its short-term investments. Without these gains, Microline’s 2011 net income would have been only $1.25 million or only slightly greater than its 2010 profits. Second, its investment in Ellery, Inc. produced income of $1.25 million in 2011. Finally, a favorable fluctuation in the Canadian dollar produced a gain of $500,000. None of these items represent income from Microline’s normal operations. Without all of them, Microline would have had $3.75 million less in income and would have, in fact, reported a loss for 2011. In addition to poor earnings persistence, earnings quality also may be poor for several reasons. First, the sales on the land and short-term investments appear to have been executed solely for the gains achieved. Immediately after the sale of the land, similar land was purchased back. In essence, Microline transferred excess cash into gains on the income statement. In addition to a stronger income statement, these sales may have been motivated by the bonuses Microline’s management receives if return on equity exceeds 10%. Without these gains on the sale of land and investments, reported income would only be about $1.25 million or 5% of equity. The 2010 results indicate no gains or losses from sales of land, equipment, or investments. Clearly, management entered into these transactions for their financial statement effects, as they were not in the shareholders’ best interest. Second, although Microline’s sales have increased 14% and its bad debts allowance has increased 14%, consistent with its policy, the increase in receivables suggests that it may have more bad debts than allowed for. Finally, selling and administrative expenses have increased 30% while sales have only increased 14%. This suggests that some expenses or losses incurred may be hidden in this line item (for example, the bonus earned by management in 2010). Overall, Microline appears to be strong. However, closer examination reveals that Microline’s own operations contribute little to its net income. A limited portion of the 2011 net income is expected to persist. In addition, a number of actions by Microline suggest that the earnings quality is poor.


CASE 3: TECHNIC ENTERPRISES AND SONAR-SUN, INC. MEMO To: Recommendation Team Re: Analysis of Technic Enterprises and Sonar-Sun, Inc. To support Timken Brother’s buy/sell recommendation with respect to Technic Enterprises and Sonar-Sun, I have analyzed their financial statements along three dimensions—solvency, earning power and persistence, and earnings quality. Solvency and Liquidity Position Technic Enterprises 2011 2010 Current ratio Quick ratio Liabilities/equity Receivables turnover

2.14 1.92 2.19 2.44

2.50 1.64 1.26

Sonar-Sun, Inc. 2011 2010 1.04 0.39 1.82 9.39

1.16 0.36 1.59

Technic Enterprises is experiencing decreasing liquidity and solvency. Its current ratio fell between 2010 and 2011, while the quick ratio increased. Current liabilities almost doubled between 2010 and 2011 at the same time inventory balances fell, creating the fall in the current ratio. Increases in current assets other than inventory, however, have helped Technic avoid a dangerously low current ratio. At 2.44, Technic’s receivables turnover is fair. For improved liquidity, receivables should be turning over more quickly than every 150 days. Recently, however, the receivables balance has grown while its bad debts allowance has not, suggesting that part of the receivables balance may be overstated. Another indicator of Technic’s declining solvency is its increasing liabilities to equity ratio. Even excluding current liabilities, the debt to equity ratio is a high 1.54. This means more debt finances Technic than equity. Part of the increase is due to the issuance of a new note for $18 million without sufficient increases in income or equity issuances. Sonar-Sun, Inc. is also experiencing a decreasing current ratio. However, unlike Technic’s, Sonar-Sun’s ratio is dangerously low. At the end of 2010, its current ratio was only 1.16. By the end of 2011, it had fallen even further to 1.04. Given that most of Sonar-Sun’s current assets are in the form of inventory, it is not surprising that the quick ratio is also extremely low, at 0.39 in 2011. These ratios are extremely weak and suggest that Sonar-Sun may soon face a liquidity crisis. Current liabilities have increased almost 50% while current assets have increased by only 30%. Despite additional common stock issued of $5,000,000, debt increased when a $5,000,000 note was issued to buy Wallingford Atlantic. This has only further weakened an already poor solvency position. Unlike Technic, however, Sonar-Sun does maintain a very strong receivables turnover ratio of 9.39, or turns over every 38 days. While Sonar-Sun does not separately report on its allowance accounts, there are no signs that it is experiencing collection problems. While Technic paid almost 50% of its 2011 net income in the form of a cash dividend to its investors, Sonar-Sun paid almost 100% of its 2011 net income as cash dividends. Given their weak solvency positions, neither payment appears to be prudent. Summary: Given its declining liquidity and solvency and potential additional liabilities, based on a scale of 1 (very weak) to 10 (very strong), I rate Technic’s solvency position a 4. Technic’s liquidity position is strong;


however, it appears to be increasing its debt, spending its cash on investments not immediately necessary for improving its financial position, and not significantly extinguishing its current obligations. Since it is not yet in default, I did not believe a lower rating appropriate. Also, since its receivables turnover is fair, obtaining cash from its operations does not seem to be a problem yet, but could be if improvements are not made. In contrast to Technic, Sonar-Sun has an even weaker liquidity position. Sonar-Sun has a balance sheet consisting mainly of inventory and debt. While its solvency is not as weak as Technic’s, it is nevertheless weak. Sonar’s choice to pay a large cash dividend rather than improve its solvency position is also not a favorable indicator. Its strong receivables turnover ratio, however, suggests it has the ability to generate cash more quickly than Technic. Sonar needs to improve its liquidity position and work to reduce its debts. Based on these considerations, I give Sonar a 3 on liquidity and solvency. It is lower than the rating for Technic because Sonar is in more immediate liquidity danger. Earning Power and Persistence

ROA ROE 0.25 Return on Sales Times interest earned Inventory turnover

Technic Enterprises

Sonar-Sun, Inc.

0.09 0.36 0.17 2.53 3.37

0.13 0.12 5.64 1.75

In 2011, Technic Enterprises performed well relative to its level of assets, equity, and sales. Each of these ratios suggests Technic earns profits efficiently. Further, at 3.37 or every 108 days, its inventory turnover is good. Further, the reduction in inventory has helped Technic achieve a good inventory turnover ratio. However, given the nature of its inventory and a recent history of a write-down, Technic would face less risk of obsolescence if it turned over its inventory even faster. Technic’s times-interest-earned ratio is fair and should improve as Technic retires its debt. Based on these indicators, it appears that Technic’s earnings power is strong. However, closer examination of the income statement and notes suggests otherwise. First, net income benefited by a $750,000 income effect from a change in the inventory costing method. Second, examination of miscellaneous gains and losses reveals various gains due to sales of investments, buildings, and land. Together the gains from these realized and unrealized gains amounted to $4,140 or 87% of the net income before the accounting change. In addition, a favorable translation gain added another $750,000 to income. Together these items account for all of Technic’s earnings. There is reason to be concerned because none of these items is essential to Technic’s operations or can be expected to persist in the future. Accounting methods cannot be changed annually nor can sales of assets occur frequently. Finally, the translation gain may easily turn into losses next year. Given that the net income would have decreased to almost nothing without these items unrelated to the normal business, earning power is extremely poor. Clearly, without the income from these other activities, Technic may have trouble meeting its obligations. Similar to Technic, the profitability ratios suggest that Sonar-Sun has a good level of earning power. All three earning power ratios (ROA, ROE, and ROS) are strong. In addition, the times-interest-earned ratio suggests Sonar is financially strong and is utilizing its assets effectively. The only weakness is in Sonar’s inventory turnover ratio. At 1.75, it turns over its inventory only every 209 days. This low rate of turnover risks Sonar for significant inventory obsolescence. Given its weak solvency position, the ability to generate sales quickly hurts Sonar’s financial position. Further, this long period is resulting in inventory obsolescence and unnecessary write-off losses, such as the $5,000,000 reduction recorded in 2011. In contrast to Technic’s foreign currency translation gain on foreign currency, Sonar-Sun’s net income includes a loss of


$1,500,000 due to the unfavorable translation of the peso. With the exception of the write-downs of inventory and equipment equal to $5,000,000, which are part of normal operations, most of Sonar’s income of $20,700,000 should persist in the future. Only $4,000,000 of the $20,700,000 was earned by EDM Suppliers, an affiliate not under control of Sonar. Note that unlike Technic, Sonar does not have other revenue and expense items hidden in a miscellaneous income statement items. Compared with Technic, operating expenses for Sonar appear to be larger (45% of sales versus 24% of sales). Disaggregation of the general operating expenses may provide information about the content of these expenses. This would help users decide whether these expenses are expected to persist in the future or not. Finally, Sonar’s equity investments and affiliate investment appear to be performing well. In 2011, Sonar earned $4,000,000 on its $12,000,000 affiliate investment (beginning balance) in EDM Suppliers. Technic’s investment in Lehmon Financial Services, however, has not done as well. After purchasing the shares at $300 each, Lehmon is now selling for $25.50 per share. This investment returns only $1,200 out of a $15,000 investment. Given liquidity and solvency problems, this investment may not have been good use of its assets. Summary: Given the portion of income generated from activities outside its normal operations, Technic does not appear to have good earnings power or persistence of positive income. Based on these factors, I rate Technic a 3 on earning power and persistence. In contrast, Sonar-Sun earned positive income in 2011 mostly due to its own primary operating activities. Also, some of its losses or expenses are not expected to persist in the future. For these reasons, Sonar-Sun earns a 7. Earnings Quality Technic Enterprises has presented its income statement and made decisions that favor the presentation of its performance. First, hidden within miscellaneous gains and losses is an inventory write-down of $450,000. This is an ordinary part of business and should be reported in the operations section. Further, a significant number of other gains are included in this miscellaneous category that are not an ordinary part of business nor are expected to persist in the future. For example, the sale of land appears to have been motivated in order to recognize a gain. Immediately after the sales, Technic repurchases 7 or the 10 parcels sold. Similar activity occurred with respect to its building and short-term investment. Second, operating expenses are a significant $7,200,000, but are not described further. Clearly, this expense may contain some questionable items. Third, Technic has adopted a change in an accounting method that results in a favorable credit to its income. Fourth, depreciation policies are liberal up to 40 years in order to reduce depreciation expense each year. Fifth, allowances for uncollectible accounts may not be sufficient. The reported allowance decreased in half even though the receivables balance almost doubled. Unlike Technic, Sonar’s accelerated depreciation methods result in an earnings amount that reflects expenses in the most conservative way. Neither Technic nor Sonar appear to manage their short-term investment portfolios for reporting purposes. Further, Technic uses FIFO inventory cost flow assumption, while Sonar uses a LIFO assumption. The result is lower income for Sonar. Also, Sonar reports a bad debt allowance of 4.4%, compared with only 2% for Technic, again suggesting that the quality of Sonar’s reported income is excellent and that of Technic is poor. Summary: Sonar-Sun’s earnings quality is not overly inflated by items at the discretion of management. Estimated expenses do not appear to be understated and appear to be of good quality, earning Sonar-Sun a rating of 8. Technic has significant income statement items that suggest reported income is artificially high. For this reason, I rate Technic’s earning quality a 5.


CASE 4: AVERY CORPORATION Evaluation of the CEO’s letter Immediately below is a chart containing financial ratios computed on the financial statements of Avery Corporation. The following discussion assesses earning power and solvency and, in so doing, addresses the specific questions posed by Sellers. The solution concludes by summarizing the evaluation of the CEO’s letter to the shareholders. Financial Ratios for Avery Corporation Return on equity Return on assets Earnings per share Return on sales Interest coverage Current ratio Quick ratio Receivables turnover (days) Receivables turnover (times) Inventory turnover (days) Inventory turnover (times) Debt/equity

2011 .062 .051 1.83* .076 3.54 3.79 1.95 110 3.32 302 1.21 .70

2010 .078 .059 3.58 .068 2.79 6.22 4.19 89 4.12 228 1.60 .78

2009

4.23 2.04

1.75

*EPS is $0.27 if preferred dividends are excluded from the numerator

Earning Power Assessment Although reported net income appears to have increased from 2010 to 2011 ($5,367 to $7,049), earning power dropped off substantially. First, three return ratios (ROE, ROA, and EPS) using reported net income show a decrease of earning power since 2010. All three have dipped, and EPS has decreased by almost 50 percent. (The EPS decrease is due primarily to the addition of 2.35 million outstanding shares—1 million for exercised stock options, 1 million for the acquisition of Buckeye, and .35 million for the stock dividend). Second, return on sales and interest coverage have shown considerable increases. However, there is reason to be concerned because the increase in both ratios occurred primarily because 2011 net income includes a gain on the sale of equipment ($6 million) and special service revenue ($8 million), neither of which are essential to the operations of Avery and can be expected to persist in the future. (The $6 million gain on the sale of equipment involved equipment with a cost of $20 million and accumulated depreciation of $6 million being sold for $20 million in cash.) Moreover, inventory turnover slowed to only 1.21 times per year, indicating that there are a number of slow-moving inventory items, and additional inventory write-offs may be necessary in the future. Proper income statement disclosure, which is not followed by Avery, should highlight net income from operations, excluding the inventory write-down, income from the affiliate, the gain on the sale of equipment, and special service revenue, for both 2010 and 2011. Avery’s net income from operations (before interest and taxes) for 2011 and 2010 would then be $6 million ($21.8 million – $1.8 million – $6 million – $8 million) and $11.55 million ($13.05 million – $1.5 million), respectively. It seems, therefore, that “persistent earnings” decreased considerably from 2010 to 2011, and in that time period the company issued 2.35 million common shares, further diluting its outstanding common stock. This disclosure format would also clearly disclose that the company’s operating sales actually decreased from $120 million in 2010 to $115 million in 2011. Moreover, the $5,000 inventory write-off, claimed to


be “extraordinary” by Tennenden, would not normally be considered as such under generally accepted accounting principles. Rather, it is a normal and recurring risk of business operations.

Solvency Assessment The solvency, activity, and capitalization ratios point to a potential solvency problem. The current and quick ratios have decreased from 2010 and 2011,while receivables turnover has slowed (suggesting that there may be collection problems). In addition, if the nonrecurring items are eliminated from reported net income, Avery’s interest coverage ratio plummets. It appears that profits are not sufficient to cover the company’s annual interest payments, despite Tennenden’s claim that “our profits are more than adequate to cover debt interest payments.” A closer look at Avery’s receivable and payable activity indicates that cash collections from receivables totaled $97 million ($35 million + $115 million -$8 million Unearned Revenues – $45 million), while sales totaled $115 million. This combination increased net accounts receivable by $9.95 million for the second year in a row. Cash payments for accounts payable totaled $67.7 million ($8.2 million + $75.5 million – $16 million), while purchases totaled $75.5 million. This combination increased payables by $7.8 million. Thus, receivables appear to be growing at a faster pace than are payables, indicating possible collection problems and potential cash flow problems in the future. Another interesting aspect about receivables is that the allowance for uncollectibles only increased by 9 percent from 2010 to 2011, while outstanding accounts receivables increased by almost 30 percent. It is possible that the $500,000 bad debt expense reported on Avery’s income statement is not sufficient to cover the company’s uncollectibles and adding to this allowance would further reduce Avery’s reported net income. Avery also appeared to cut back on its pension contribution. While an accrued pension expense of $10 million was recognized for 2011, the company only paid $8 million to the fund. This combination increased the company’s unfunded pension liability to $7.339 million, an increase of 37 percent. Such a cutback may also indicate potential solvency problems. The issuance of a common stock dividend in 2011 is also troublesome. It appears to have been issued in lieu of a common stock cash dividend. The 10 percent dividend totaled a common stock issuance of 350,000 shares because 3.5 million common shares (5 million – 2 million – 1.5 million + 1 million + 1 million) were outstanding at the time of the declaration. The issuance had no effect on the assets or liabilities of the company, and it had essentially no effect on the wealth levels of the shareholders. It may have been a ploy by management to encourage the shareholders to believe that they were actually receiving something of value. It seems that the acquisition of Buckeye Corporation was a very risky transaction for Avery. The company paid $50 million in cash and issued 1 million common (treasury) shares with a total value of $15 million. Goodwill of $15 million was recognized on the transaction, indicating that the market value of Buckeye’s property, plant, and equipment was $50 million. This transaction drained the company of much of its cash and further diluted the common share value. Perhaps management believes that Buckeye can generate future profits capable of rescuing what appears to be a company with serious financial difficulties.

Returning to Tennenden’s letter to the shareholders and summarizing: 1. In 2011 Avery did not demonstrate strong earning power. 2. The inventory write-off is not considered “extraordinary.”


3. The 2011 acquisition of Buckeye was financed primarily by a preferred stock (debt?) issuance and the issuance of 1 million shares of common stock, not cash from operations. 4. The shareholders have not prospered and, in fact, received only a stock dividend in 2011. 5. The company’s debt/equity ratio is grossly understated. 6. Persistent earnings are not sufficient to cover the company’s annual interest payments.

Questions 1. Net income increased from $5,367,000 in 2010 to $7,049,000 in 2011 while EPS decreased from $3.58 in 2010 to $1.83 in 2011. Even without the extraordinary loss of $1.30 in 2011, EPS from continuing operations was only $3.13, still less than EPS in 2010. The reason for the drop is that the number of shares outstanding increased in 2011. Avery executives exercised options for 1,000,000 shares. Also, in December 2011, Avery purchased Buckeye Corporation for cash and 1,000,000 shares of stock. Finally, Avery declared and issued a 10% stock dividend on the outstanding shares. Between 12/31/10 and 12/31/11, the outstanding shares increased by 2,350,000. Number of Outstanding Shares Balance, 12/31/09 Less: Treasury shares—note 9 Treasury shares—note 9 Balance, 12/31/10 Options exercised—note 9 Buckeye acquisition—note 4 10% Stock dividend Balance, 12/31/11

5,000,000 (2,000,000) (1,500,000)a 1,500,000 1,000,000 1,000,000 3,500,000 350,000 3,850,000

a $18,000,000/$12 share = 1,500,000 shares

2. As outlined above, the 10% stock dividend resulted in the issuance of 350,000 shares. Stock dividends do not affect assets or liabilities. This stock dividend resulted in a decrease to retained earnings of $5,250,000 and an increase in contributed capital of $5,250,000. Thus, total stockholder’s equity is also unchanged. 3. In January 2009, the company issued $5,000,000 in bonds for $4,385,500 resulting in an effective market yield of 10%. The call feature of the bond allows the bonds to be called beginning in 2012 for 2% above the face value or $5,100,000 (note 6). With a current market rate of 8%, the bonds have a market value of $5,000,000 (par value, since the stated rate is also 8%). Therefore, since Avery can buy its own bonds back in the open market for $5,000,000, it is not wise to exercise the call option for $5,100,000. Also, since the company may be facing solvency difficulties, it may be unwise to use its cash for this purpose. 4. Based on Avery’s consolidated balance sheet at 12/31/11, Avery has total liabilities of $88,758,000 and total equity of $126,827,000. This results in a debt/equity ratio of 0.70. The ratio is not an accurate measure of the company’s actual debt position because it does not include a number of other debt-like events.


The company’s debt/equity ratio could be grossly understated. First, Avery makes $10 million in lease payments each year. Footnote (8) suggests that the lease contract may meet the criteria for a capitalized lease. If we assume a 10 percent discount rate and that these lease payments are required for the remainder of the firm’s life, a $100 million ($10,000/.10) liability could be added to Avery’s balance sheet. Second, the characteristics of the preferred stock issuance appear to resemble debt in many ways, suggesting that an additional $50 million liability could be added to the company’s balance sheet. Third, the bond payable liability is undervalued on the balance sheet because interest rates have fallen and the market value of the liability ($50 million) is greater than the book value ($48.264 million). Revaluing the liability would add another $1.736 million of debt to the balance sheet. Finally, Avery owns a 30 percent interest in a highly levered financial institution, Spartan Savings, and some would argue that adding 30 percent of Spartan’s assets and liabilities to Avery’s balance sheet would further increase the company’s debt/equity ratio. The capital structures of financial institutions are often comprised of 80 to 90 percent debt. In conclusion, Avery reports liabilities of $88.758 million as of the end of 2011. That number could be increased by $151 million or more, which, in turn, would increase the debt/equity ratio from 0.70 to nearly 2.0. It is also not likely that Avery will exercise its options to call its outstanding bonds. First, the call price ($5.1 million = $1,020  5,000 bonds) exceeds the market value of the debt ($5.0 million = $1,000  5,000 bonds), indicating that it would be economically unwise to call the bonds. Second, the company appears to be facing solvency problems, and it may not wish to use its cash for this purpose. 5. The footnotes disclose sales of $115,000,000 in 2011. This represents sales earned, and since all sales are made on credit, it does not necessarily measure cash collected from customers. The actual amount of cash collected from customers can be calculated as follows: Beginning A/R $35,000 + Sales $115,000 – Drop in Unearned Revenue $8,000 – Ending A/R $45,000 = $97,000

Likewise, the amount of cash paid to suppliers for inventory can be calculated as follows: Cost of goods sold Plus: Increase in inventory (before write-down) Purchases

$60,500,000 15,000,000 $75,500,000

Purchases Less: Increase in accounts payable Cash paid to suppliers

$75,500,000 7,800,000 $67,700,000

6. While current assets have increased due to inventory and accounts receivable, it is not necessarily a good sign. Increases in inventory may mean sluggish sales and high inventory storage costs. The inventory write-down in 2011 (income statement) is a signal that Avery is not selling its inventory in a timely manner, resulting in obsolescence. Increases in accounts receivable may signal poor collections from customers and/or a change in the credit approval criteria. The failure to turn over inventory quickly and collect cash from customers ties up funds that could be reinvested into the company. 7. 2011 revenues consist of sales that have fallen from 2010, affiliate income, a gain on the sale of equipment, and revenue from special services (note 1). The latter three sources of income accounted for almost $16,000,000 of Avery’s revenue in 2011 but are not expected to persist in the future as part of the primary operations.


The income from affiliate will not exist in 2012, because Avery sold its interest in Spartan Savings in December 2011 (note 4). Likewise, the gain on the equipment sale and special services revenue are not Avery’s primary sources of revenue. However, by aggregating these items in revenues, Avery is suggesting that they all are usual and frequent. Only the sales revenue is usual and frequent and may persist into the future. However, if the trend continues, sales revenue may not persist in the future either. 8. Purchase price (note 4) Less: Goodwill purchased (balance sheet) Value of Buckeye’s PPE acquired

$65,000,000

($50

million

cash

&

$15

15,000,000 $50,000,000

(primarily PPE)

9. Cash (statement of cash flows) 20,000,000 Accumulated Depreciation ? Equipment (note 3) Gain on Sale of Equipment (note 1)

20,000,000 6,000,000

Accumulated Depreciation must equal $6,000,000. Thus, the book value of the equipment was $14,000,000 ($20,000,000 – $6,000,000). 10. Pension expense Less: Increase in unfunded pension liability Cash contribution

$10,000,000 2,000,000 $ 8,000,000

million

stock)


CASE 5: ZENITH CREATIONS Date: January 2012 Re: Evaluation of Zenith’s financial condition Zenith Services is a potential acquisition candidate of several companies. Offers from buyers will reflect their assessments of Zenith’s financial condition and performance. So that Zenith can evaluate offers received and to understand how these users may be evaluating Zenith, I have prepared (1) an evaluation of Zenith’s financial condition and performance and (2) an analysis of the financial statement effects of several proposed actions. Financial Condition Zenith Services’ 2011 current ratio of 2.15 has declined slightly from 2.94 in 2010 but remains strong. A greater drop occurred in the quick ratio, from 1.95 in 2010 to 1.13 in 2011. Part of this decline is due to the use of cash in 2011 to retire part of a note, and to purchase $15,000,000 in equipment. Given Zenith’s other obligations, it can clearly better utilize its liquid assets. For example, as of the end of 2011, Zenith reports payables and other accrual balances of almost twice that at the end of 2010. At the same time, Zenith is reporting sufficient cash balances to settle these liabilities. While it may be important to settle some obligations, potential buyers of Zenith are also likely to require a much stronger cash position. The size of Zenith’s liabilities is growing faster than its equity, resulting in large debt/equity ratios. Despite the retirement of half a note issuance, the net loss experienced in 2011 has caused its debt/equity ratio to increase from 0.83 in 2010 to 0.90 in 2011. This means that Zenith is financed with approximately equal amounts of debt and equity. However, this ratio may grossly understate Zenith’s debt position. First, Zenith has an operating lease for $8,000,000 each year. Capitalized at a 10% rate results in an additional $80,000,000 of debt. If these leases were capitalized, the debt/equity ratio would approach 2.0. Also, in 2011 Zenith issued net $15,000,000 of preferred stock (500,000 shares at $30 each), which resembles debt but is not classified as debt on the balance sheet. Finally, due to a falling interest rate, the market value of its bond payable is actually higher than stated on the balance sheet. In addition, the debt/equity ratio is even higher because Zenith chose to buy back some of its own stock, thus decreasing total stockholders’ equity. Most of these treasury shares have not been subsequently reissued. Clearly, Zenith is carrying too much debt relative to its equity and current profitability. Another part of the increase in the debt/equity ratio was due to an increase in the pension liability by $2,000,000. This growth in the pension liability indicates that Zenith may be cutting back on its pension funding to conserve cash and improve its liquidity position. The ability of Zenith to turn over its receivables is particularly strong. In 2010 its receivables turnover was 5.58 and rose to 7.21 in 2011. This ratio indicates that it takes an average of only 50 days to collect on sales. Receivables appear to be stable, and there are no signs of significant collection problems. The 50% increase in bad debts and the need to write off an uncollectible account suggest only limited problems with granting credit to customers. Zenith may have a problem with the quick sale of inventory. Its inventory turnover of 1.93 in 2010 and 2.04 in 2011 is satisfactory but needs to improve. A turnover ratio of 2.0 indicates that it takes 182 days to turn over inventory. Inventory is not turning over as quickly as it should be. The need to write off obsolete inventory is further evidence that Zenith carries too much inventory. Without a change in its inventory or sales policies, it is likely that future inventory write-offs are probable.


With the exception of inventory turnover, Zenith’s immediate liquidity is strong. However, its solvency is dangerously poor due to a large amount of liabilities. One area in which it can utilize its liquid assets is in the reduction of its liabilities. Performance Reported net income has dropped significantly from $19,529,000 in 2010 to a loss of $11,510,000 in 2011. One of the largest contributors to this drop was the fall in operating revenue of $16,000,000, including a decline in sales of $10,000,000. While the special services revenue and the loss on the sale of equipment (note 1) may not persist into the future, the decline in sales indicates that Zenith Services’ primary source of revenue and earnings may not persist into the future. The other primary contributor to the decline in net income in 2011 is the restructuring charge of $15,000,000. While this charge is not extraordinary, it is not part of the normal operations and should not persist. In addition, users may positively view the charge if the restructuring activities contribute to improved operations. Other contributors to the net loss are an extraordinary loss on the extinguishment of part of a note and an inventory write-down of $2,000,000. While the inventory write-down may occur in the future, the note retirement is not expected to occur repeatedly in the future. While much of the net loss is due in part to nonrecurring items, earning power has dropped off significantly. Without the extraordinary loss and the restructuring charge, net income would have been a positive $8,914 and EPS $0.84. This still results in a return on assets of only 0.05 and a return on equity of only 0.10, down from 0.14 and 0.28 in 2010, respectively. Income from operations fell 58% between 2010 and 2011. Zenith’s operating income return on assets fell from 18.3% to 7.1%. As a result of its poor performance Zenith’s times-interest-earned ratio fell from a strong 6.77 in 2010 to a low 2.20 in 2011. Unless earnings improve, Zenith may have trouble meeting its interest obligations. Also, despite the net loss, Zenith paid a stock dividend. Some users may view this dividend in lieu of a cash dividend as a ploy to appear as a healthy company issuing dividends. Stock dividends, however, do not add to the wealth of the investor. As they dilute potential ownership and reduce retained earnings, potential investors may, in fact, not view the stock dividends favorably. Also, Zenith’s investment in Lyon Real Estate through the issuance of common stock significantly diluted the common share value and has not generated significant returns. A $200 loss and only $700 in dividends in 2011 result in less than a 7% return on the cash investment of $10,000. It is important that Zenith Services’ income statement disclose sales revenue separate from the other sources of revenue and expense (income from affiliate, loss on sale of equipment, special services), particularly if Zenith wants users to focus on those items expected to persist in the future. This would highlight the sales and expenses expected to persist into the future. Earnings quality may be weak. First, Zenith has experienced LIFO liquidations in the past three years. LIFO liquidations result in inflated profits. Sales are not actually greater; rather, the costs of the inventory liquidated are lower than the inventory costs in prior years. While a preliminary analysis suggests Zenith’s liquidity is strong, examination of its solvency and earnings power indicates otherwise. Overall, Zenith is unhealthy because of its debt position. Also, its weakening profitability is contributing to a declining ability to pay its debt obligations or secure additional financing.


Management Actions Zenith’s management is considering a number of actions, some of which significantly affect the financial statements and assessments of financial health. Below are discussed the effects of each of these actions. (1)

Purchase treasury stock at current market price.

The purchase of treasury stock requires the use of cash. While Zenith has sufficient cash available, it would be wiser to use the cash to pay off outstanding payables. On the other hand, if Zenith anticipates improving its health and its stock price, it may be able to secure additional cash by purchasing stock now at a low price and reselling it later for a higher price. The effect of a purchase on the financial statements would be to reduce cash and stockholders’ equity. By reducing current assets and equity, the current ratio will fall and the debt/equity ratio will increase. Earnings per share will increase (if earnings are positive) and decrease (if earnings are negative) as the number of outstanding shares decreases. Further, if retained earnings are declining, further net losses may place Zenith in a deficit retained earnings situation. Outsiders may wonder whether treasury stock was purchased in an attempt to drive up the stock price or increase earnings per share. The purchase will not improve the financial health of Zenith, and the cash reserves may be more wisely used to help improve Zenith’s financial health.

(2)

Write off a relative large uncollectible accounts receivable.

An accounts receivable write-off does not affect net accounts receivable, total assets, or income. The effect is only to remove the account from the allowance and receivable accounts. Outsiders shouldn’t notice a significant difference due to the write-off. (3)

Issue a 20% stock dividend.

A 20% stock dividend would amount to 2,311,000 shares and reduce retained earnings by market value of the shares. Since retained earnings is only $23,488,000 as of 12/31/11, a stock dividend would reduce retained earnings to zero if the current market price is much over $8 per share. The stock dividend does not change assets, liabilities, or total stockholder’s equity. Thus, ratios will not be affected. Retained earnings is effectively capitalized. Unless current shareholders are demanding a dividend, Zenith should save its retained earnings as much as possible, particularly if future net losses are anticipated. Investors’ wealth levels do not benefit from most stock dividends. Also, outsiders may question the reason for the stock dividend, given Zenith’s declining health, and view it as a ploy to convince investors of the financial health of the company.


(4)

Redeem the remaining notes payable for $23,200,000.

Currently, notes payable has a book value of $19,665,000. If Zenith redeems the note for $23,200,000, it will realize an extraordinary loss of $3,535,000. The cash paid will contribute to a decreased current ratio. However, the retirement of the debt will reduce the debt/equity ratio to a more attractive 0.71 ($82,185,000 – $19,665,000)/($91,648,000 – $3,535,000). Because the loss is extraordinary, it will not affect income from continuing operations. However, the overall return on assets will worsen due to the loss on the retirement. While the reduced debt/equity ratio improves Zenith’s flexibility to take advantage of future opportunities and users would not expect the resulting extraordinary loss to persist in the future, the reduction in cash and overall profitability may suggest that it is economically unwise to redeem the note right now.

(5)

Sell the real estate received in the acquisition of Lyon Real Estate for $12,000,000.

When Zenith acquired Lyon Real Estate, it acquired real estate worth approximately $5,010,000. Price Goodwill Value of real estate

$25,010,000 20,000,000 $ 5,010,000

Thus, if Zenith sells the real estate for $12,000,000, it will realize a gain of $6,990,000. By increasing equity by $6,990,000 (before taxes), the gain will reduce the debt/equity ratio. At the same time, the cash received will improve the current ratio. Unlike the debt retirement action, the sale of the real estate does not result in an extraordinary item. The sale of real estate is not a primary operation of Zenith, but it is also not usual and infrequent. Thus, users are more likely to perceive the gain from this sale as more persistent than the gain from the retirement of debt. Given the small return currently being earned on this investment, the sale may be a wise action.

(6)

Change the inventory cost flow assumption from LIFO to FIFO.

A change in the accounting method from LIFO to FIFO requires retroactive adjustments to the comparative balance sheets. This retroactive restatement will result in an increase in retained earnings for the lower income than was reported in the past, due to the LIFO assumption relative to using the FIFO assumption. By increasing retained earnings, debt/equity ratios in the restated years will decrease. Depending on how other firms in the same industry treat inventory, outsiders may not favorably view the change. However, they also may view the change as window-dressing and poorly evaluate Zenith’s earnings quality.


CASE 6: PIERCE AND SNOWDEN

MEMO To: Recommendation Team Re: Analysis of Wellington Mart and Wagner Stores To assess whether Pierce and Snowden should extend credit to Wellington Mart and Wagner, I have analyzed their financial statements along three dimensions—solvency, earning power and persistence, and earnings quality. Solvency position

Current ratio Quick ratio Debt/equity Receivables turnover

Wellington Mart 2011 2010 0.47 1.09 0.39 0.58 0.80 0.62 9.85

Wagner Stores 2011 2010 1.97 3.43 0.82 1.80 1.80 1.51 8.65

Wellington Mart is experiencing decreasing liquidity and solvency. Both its current and quick ratios have fallen significantly between 2010 and 2011. As of the end of 2011, Wellington’s current and quick ratios are a dangerously low 0.47 and 0.39, respectively. A large contributor to the fall in the liquidity is Wellington’s acquisition of Marilyn Real Estate. Marilyn cost $80,000,000, $50,000,000 of which was in cash. Also in 2011, Wellington made plant and equipment acquisitions of $23,000,000. Altogether Wellington made significant drains on its cash, given its already weak liquidity and solvency positions. At the same time, its total debt/equity ratio has increased from 0.62 to 0.80 in 2011. Part of the increase is due to the issuance of a new note for $13,900,000. While the $6,200,000 increase in debt is due to its adoption of a defined pension plan, the ratios may, in fact, be grossly understated. First, Wellington has an operating lease, which requires annual payments of $12,000,000. Given that it has an option to buy the facilities at the end of the lease term for 25% of the market value, this lease may be more properly classified as a capital lease. At a discount rate of 10%, an additional $120,000,000 in debt would be included on Wellington’s balance sheet. However, it appears that Wellington management may be using some discretion to classify the lease in order to avoid violating a bond covenant to keep its liability/asset ratio below 0.60. Also, Wellington has $50,000,000 of outstanding preferred stock, which resembles debt but is not classified as debt on the balance sheet. Finally, Wellington may be subject to certain environmental liabilities in the future. These also are not yet reflected on the balance sheet as future obligations. On the positive side, due to an increasing interest rate, the market value of its bond payable is actually lower than stated on the balance sheet. If Wellington calls these bonds, it will realize an extraordinary gain on the retirement of the bond. The ability of Wellington to turn over its receivables is particularly strong. With a turnover ratio of 9.85, it takes, on average, only 37 days to collect on sales. While receivables are growing, the allowances may be understated. In 2011, bad debts expense was $1,550,000, while the allowance only increased $650, indicating possible write-offs of additional uncollectible accounts.


Also, despite its net loss, Wellington paid a stock dividend. This dividend in lieu of a cash dividend may be a ploy to appear as a healthy company issuing dividends. Stock dividends, however, do not add to the wealth of the investor. As they dilute potential ownership and reduce retained earnings, potential investors may, in fact, not view the stock dividends favorably. Wagner Stores is also experiencing a decreasing current ratio. At the end of 2010, its current ratio was a strong 3.43. At the end of 2011, the ratio had fallen to 1.97. Also, its quick ratio has fallen from 1.80 to a much weaker 0.82. These ratios are still strong and may only reflect a decision to better use its assets. One reason for the strong ratios is that Wagner issued debt and new common stock for a total of $20,800,000, so that the purchase of additional property, plant, and equipment for $34,500,000 did not severely drain its liquid assets. Due to the additional bond issuance, the debt/equity ratio of Wagner also increased slightly from 1.51 in 2010 to 1.80 in 2011. Unlike Wellington, Wagner’s liabilities include capital leases of $4,771,000. In addition, however, Wagner also has operating lease payments of $6,200,000 per year. If capitalized at a discount rate of 10%, an additional $62,000,000 in debt would be included on Wagner’s balance sheet. Also, unlike Wellington, Wagner has only $5,000,000 in preferred stock, which may have features and obligations similar to debt. On the positive side, due to an increasing interest rate, the market value of its bond payable is actually lower than stated on the balance sheet. Like Wellington, if Wagner calls these bonds, it will realize an extraordinary gain on the retirement of the bond. Like Wellington, Wagner maintains a strong receivables turnover ratio of 8.65. Wagner does not separately report on its allowance accounts. However, there are no signs that Wagner has collection problems. Unlike Wellington, Wagner was able to pay a cash dividend of $2,800,000, or 35% of net income. Summary: Given its declining liquidity and solvency and potential additional liabilities, I rate Wellington’s solvency position a 3. Wellington appears to be increasing its debt, spending its cash on investments not immediately necessary for improving its financial position, and not significantly extinguishing its current obligations. Since it is not yet in default, I did not believe a 1 or 2 rating appropriate. Also, since its receivables turnover is strong, obtaining cash from its operations does not seem to be a problem yet. In contrast to Wellington, Wagner has a stronger solvency position, although it is weakening slightly. Wagner has a strong balance sheet, a strong receivables turnover, and spends its cash more conservatively than does Wellington. Wagner needs to be careful to not overload itself with debt. Based on these strengths, I give Wagner an 8 on solvency.

Earning Power and Persistence


Inventory turnover ROA ROE Return on sales Times interest earned (based on operating income [loss])

Wellington Mart 9.00 –0.04 –0.10 –0.07 1.56

Wagner Stores 1.93 0.09 0.26 0.11 8.91

In 2011, Wellington experienced a significant net loss of $10,878,000. A large contributor to this loss was a restructuring charge of $9,000,000. While this charge is not extraordinary, it is not part of the normal operations and is not expected to persist in the future. In addition, users may positively view the charge if the restructuring activities contribute to improved operations. The loss would have been even greater, however, without other sources of income or gains included in other revenues and expenses on the income statement. Wellington realized a $2,000,000 gain from the sale of securities, $2,500,000 in affiliate income, and $1,500,000 from a translation adjustment. There is reason to be concerned because none of these items is essential to Wellington’s operations or can be expected to persist in the future. Based on their income statement, it appears as if Wellington’s selling and administrative expenses are contributing to the loss situation. There is not evidence that these significant expenses are not expected to persist in the future. The loss of $1,600,000 is more appropriately classified as a normal recurring part of operations. Given that the net loss would have been even greater without some of these items unrelated to the normal business, earning power is extremely poor. A return on assets of –0.04 suggests that Wellington is not utilizing its assets effectively at all. Although interest expense is not excessive, the loss on operations alone results in a negative times-interest-earned ratio. Clearly, Wellington will have trouble meeting its interest obligations. Wagner has a good level of earning power. All three earning power ratios (ROA, ROE, and EPS) are strong. In addition, the return on sales and times-interest-earned ratio suggests Wagner is financially strong and is utilizing its assets effectively. Note that Wagner has profitability on operating income of $13,250,000, versus $3,622 for Wellington on almost twice as much sales The only apparent weakness is Wagner’s inventory turnover of 1.93. A turnover of 2 means that it is taking 182 days to get rid of the inventory. This long period is resulting in inventory obsolescence and unnecessary write-off losses. Note that like Wellington, Wagner has an “other revenue and expense items” on the income statement. Wagner appears to put a number of significant and important expenses in this item. For example, interest expense, loss on inventory write-down, and loss from discontinued operations should all be disclosed separately. The interest and the inventory write-down are part of normal recurring operations and should be included in operating income. The loss on the discontinued equipment, while not part of recurring operations, should be separately disclosed, given its magnitude of $3,200,000. If these items had been properly classified, operating income would have been lower and users would realize that the $3,200,000 loss from the discontinued equipment is not expected to persist. Based on their income statement, it appears that Wagner’s expenses, including general operating and depreciation expense, are reasonable. Like Wellington, disaggregation of the general operating expenses may provide information. This would help users decide whether these expenses are expected to persist in the future or not.


Finally, Wagner’s equity investments appear to be performing well also. In 2011, Wagner earned $1,500,000 on an investment totaling about $10,000,000 for a return of about 15%. Summary: Given its poor operating performance, Wellington does not appear to have good earning power or persistence of positive income sources. Based on these factors, I rate Wellington a 2 on earning power and persistence. Wagner earned positive income in 2011 mostly due to its primary operating activities. Also, some of the losses or expenses are not expected to persist in the future. For these reasons, Wagner earns a 7. Earnings Quality Wellington has presented its income statement and made decisions that favor the presentation of its performance. First, hidden within “other revenues and expenses” is the inventory write-down of $1,600,000. This is an ordinary part of business and should be reported in the operations section. Second, selling and administrative expenses are a significant $45,000,000 but are not described further. Clearly, this expense may contain some questionable items. Third, any of the expense due to current-period pension expense should be included as a component of normal operating income. Fourth, Wellington may have reclassified a security to trading in order to recognize the unrealized gain. Wagner also appears to have good earnings quality. Like Wellington, some items are hidden in the “other revenue and expense” line that would be more appropriately disclosed as a component of income from operations (interest expense, inventory write-down). Also, like Wellington, general operating expenses may contain estimated expenses that hurt earning quality. Unlike Wellington, trading securities do not appear to be managed and goodwill is not a significant drain on net income. Note that Wagner uses a FIFO inventory cost flow assumption; Wellington uses a LIFO assumption. The result is a lower income for Wellington. However, the footnotes indicate that price levels are relatively stable so that this difference is not significant. Summary: Wellington’s earnings quality is not overly inflated by items at the discretion of management. Estimated expenses do not appear to be understated, and except for a single gain on a security, some unrealized gains and losses, and a translation gain, the reported loss amount appears to be of good quality, earning it a rating of 6. There is the matter of the contingent liability related to possible environmental cleanup bill. At this point it is not clear as to the magnitude of the cost and therefore it is difficult to assess its impact on the quality of earnings of Wellington. Wagner does not appear to have any artificially high amounts or have write-off earnings. For this reason, I rate Wagner’s earning quality an 8.


Turn static files into dynamic content formats.

Create a flipbook
Issuu converts static files into: digital portfolios, online yearbooks, online catalogs, digital photo albums and more. Sign up and create your flipbook.